<<

Boundary value problems for elliptic partial differential equations Mourad Choulli

To cite this version:

Mourad Choulli. Boundary value problems for elliptic partial differential equations. Master. France. 2019. ￿hal-02406106￿

HAL Id: hal-02406106 https://hal.archives-ouvertes.fr/hal-02406106 Submitted on 12 Dec 2019

HAL is a multi-disciplinary open access L’archive ouverte pluridisciplinaire HAL, est archive for the deposit and dissemination of sci- destinée au dépôt et à la diffusion de documents entific research documents, whether they are pub- scientifiques de niveau recherche, publiés ou non, lished or not. The documents may come from émanant des établissements d’enseignement et de teaching and research institutions in France or recherche français ou étrangers, des laboratoires abroad, or from public or private research centers. publics ou privés. Mourad Choulli

Boundary value problems for elliptic partial differential equations

Preface

This course is intended as an introduction to the analysis of elliptic partial differen- tial equations. The objective is to provide a large overview of the different aspects of elliptic partial differential equations and their modern treatment. Besides vari- ational and Schauder methods we study the unique continuation property and the stability for Cauchy problems. The derivation of the unique continuation property and the stability for Cauchy problems relies on a Carleman inequality. This inequal- ity is efficient to establish three-ball type inequalities which are the main tool in the continuation argument. We know that historically a central role in the analysis of partial differential equa- tions is played by their fundamental solutions. We added an appendix dealing with the construction of a by the so-called Levi parametrix method. We tried as much as possible to render this course self-contained. Moreover each chapter contains many exercices and problems. We have provided detailed solutions of these exercises and problems. The most parts of this course consist in an enhanced version of courses given by the author in both undergraduate and graduate levels during several years. Remarks and comments that can help to improve this course are welcome.

3

Contents

1 Sobolev spaces ...... 7 1.1 Lp spaces ...... 7 1.2 Approximation by smooth functions ...... 10 1.3 Weak ...... 15 1.4 W k,p spaces ...... 17 1.5 Extension and trace operators ...... 23 1.6 Imbedding Theorems ...... 31 1.7 Exercises and problems ...... 38 References ...... 44

2 Variational solutions ...... 45 2.1 Stampacchia’s theorem and Lax-Milgram lemma ...... 45 2.2 Elements of the spectral theory of compact operators ...... 48 2.3 Variational solutions for model problems ...... 60 2.3.1 Variational solutions ...... 60 2.3.2 H2-regularity of variational solutions ...... 64 2.3.3 Maximum principle ...... 72 2.3.4 Uniqueness of continuation across a non characteristic hypersurface ...... 74 2.4 General elliptic operators in divergence form ...... 81 2.4.1 Weak maximum principle ...... 82 2.4.2 The Dirichlet problem ...... 84 2.4.3 Harnack inequalities ...... 87 2.5 Exercises and problems ...... 98 References ...... 104

3 Classical solutions ...... 105 3.1 Holder¨ spaces ...... 105 3.2 Equivalent semi-norms on Holder¨ spaces ...... 111 3.3 Maximum principle ...... 114 3.4 Some estimates for Harmonic functions ...... 121

5 6 Contents

3.5 Interior Schauder estimates ...... 126 3.6 The Dirichlet problem in a ball ...... 130 3.7 Dirichlet problem on a bounded domain ...... 133 3.8 Exercises and problems ...... 136 References ...... 145

4 Classical inequalities, Cauchy problems and unique continuation . . . . 147 4.1 Classical inequalities for harmonic functions ...... 147 4.2 Two-dimensional Cauchy problems ...... 155 4.3 A Carleman inequality for a family of operators ...... 160 4.4 Three-ball inequalities ...... 163 4.5 Stability of the Cauchy problem ...... 168 4.6 Exercises and problems ...... 178 References ...... 184

Solutions of Exercises and Problems ...... 185

A Building a fundamental solution by the parametrix method ...... 237 A.1 Functions defined by singular ...... 237 A.2 Weakly singular operators ...... 245 A.3 Canonical parametrix ...... 251 A.4 Fundamental solution ...... 258 References ...... 266

Index ...... 267 Chapter 1 Sobolev spaces

We define in this chapter the W k,p-spaces when k is non negative integer and 1 ≤ p < ∞ and we collect their main properties. We adopt the same approach as in the books by H. Brezis´ [2] and M. Willem [13]. That is we get around the theory of distributions. We precisely use the notion of weak which in fact coincide with the derivative in the distributional sense. We refer the interested reader to the books by L. Hormander¨ [3] and C. Zuily [14] for a self-contained presentation of the theory of distributions. The most results and proofs of this chapter are borrowed from [2, 13].

1.1 Lp spaces

n In this chapter Ω denotes an open subset of R . As usual, we identify a measur- able f on Ω by its equivalence class consisting of functions that are equal almost everywhere to f . For simplicity convenience the essential supremum and the essential minimum are simply denoted respectively by the supremum and the minimum. n Let dx be the Lebesgue measure on R . Define  Z  1 L (Ω) = f : Ω → C measurable and | f (x)|dx < ∞ . Ω

Theorem 1.1. (Lebesgue’s dominated convergence theorem) Let ( fm) be a sequence in L1(Ω) satisfying (a) fm(x) → f (x) a.e. x ∈ Ω, 1 (b) there exists g ∈ L (Ω) so that for any m | fm(x)| ≤ g(x) a.e. x ∈ Ω. Then f ∈ L1(Ω) and Z | fm − f |dx → 0. Ω

7 8 1 Sobolev spaces

1 Theorem 1.2. (Fubini’s theorem) Let f ∈ L (Ω1 ×Ω2), where Ωi is an open subset n of R i , i = 1,2. Then Z 1 1 f (x,·) ∈ L (Ω2) a.e. x ∈ Ω1, f (·,y)dy ∈ L (Ω1) Ω2 and Z 1 1 f (·,y) ∈ L (Ω1) a.e. y ∈ Ω2, f (x,·)dx ∈ L (Ω2). Ω1 Moreover Z Z Z Z Z dx f (x,y)dy = dy f (x,y)dx = f (x,y)dxdy. Ω1 Ω2 Ω2 Ω1 Ω1×Ω2 Recall that Lp(Ω), p ≥ 1, denotes the Banach space of measurable functions f on Ω satisfying | f |p ∈ L1(Ω). We endow Lp(Ω) with its natural norm

Z 1/p p k f kp;Ω = k f kLp(Ω) = | f | dx . Ω

Let L∞(Ω) denotes the Banach space of bounded measurable functions f on Ω. This space equipped with the norm

k f k∞;Ω = k f kL∞(Ω) = sup| f |.

When there is no confusion k f kLp(Ω) is simply denoted by k f kp. The conjugate exponent of p, 1 < p < ∞, is given by the formula p p0 = . p − 1 Note that we have the identity 1 1 + = 1. p p0 If p = 1 (resp. p = ∞) we set p0 = ∞ (resp. p0 = 1). The following inequalities will be very useful in the rest of this text.

0 Proposition 1.1. (Holder’s¨ inequality) Let f ∈ Lp(Ω) and g ∈ Lp (Ω), where 1 ≤ p ≤ ∞. Then f g ∈ L1(Ω) and Z | f g|dx ≤ k f kpkgkp0 . Ω Proposition 1.2. (Generalized Holder’s¨ inequality) Fix k ≥ 2 an integer. Let 1 < p j < ∞, 1 ≤ j ≤ k so that 1 1 + ... + = 1. p1 pk

p j k 1 If u j ∈ L (Ω), 1 ≤ j ≤ k, then ∏ j=1 u j ∈ L (Ω) and 1.1 Lp spaces 9

Z k k ∏ |u j|dx ≤ ∏ ku jkp j . Ω j=1 j=1

Proposition 1.3. (Interpolation inequality) Let 1 ≤ p < q < r < ∞ and 0 < λ < 1 given by 1 1 − λ λ = + . q p r If u ∈ Lp(Ω) ∩ Lr(Ω) then u ∈ Lq(Ω) and

1−λ λ kukq ≤ kukp kukr .

n As usual if E ⊂ R is a measurable set, its measure is denoted by |E|. Proposition 1.4. Let 1 ≤ p < q < ∞ and assume that |Ω| < ∞. If u ∈ Lq(Ω) then u ∈ Lp(Ω) and 1 − 1 kukp ≤ |Ω| p q kukq.

We collect in the following theorem the main properties of the Lp spaces.

Theorem 1.3. (i) Lp(Ω) is reflexive for any 1 < p < ∞. (ii) Lp(Ω) is separable for any 1 ≤ p < ∞. (iii) Let 1 ≤ p < ∞ and u ∈ [Lp(Ω)]0, the dual of Lp(Ω). Then there exists a unique p0 gu ∈ L (Ω) so Z p hu, f i = u( f ) = f gudx for any f ∈ L (Ω). Ω Furthermore 0 kuk[Lp(Ω)]0 = kgukLp (Ω).

Theorem 1.3 (iii) says that the mapping u 7→ gu is an isometric isomorphism 0 between [Lp(Ω)]0 and Lp (Ω). Therefore, we always identify in the sequel the dual 0 of Lp(Ω) by Lp (Ω). The following theorem gives sufficient conditions for a subset of Lp(Ω) to be relatively compact.

Theorem 1.4. (M. Riesz-Frechet-Kolmogorov)´ Let F ⊂ Lp(Ω), 1 ≤ p < ∞, admit- ting the following properties.

(i) sup f ∈F k f kLp(Ω) < ∞. (ii) For any ω b Ω, we have

lim kτh f − f kLp(ω) = 0, h→0 where τh f (x) = f (x − h), x ∈ ω, provided that |h| sufficiently small. (iii) For any ε > 0, there exists ω b Ω so that, for every f ∈ F , k f kLp(Ω\ω) < ε. Then F is relatively compact in Lp(Ω). 10 1 Sobolev spaces

We end the present section by a density result, where Cc(Ω) denotes the space of continuous functions with compact supported in Ω. p Theorem 1.5. Cc(Ω) is dense in L (Ω) for any 1 ≤ p < ∞.

1.2 Approximation by smooth functions

p Recall that Lloc(Ω), 1 ≤ p ≤ ∞, is the space of measurable functions f : Ω → R p satisfying f χK ∈ L (Ω), for every K b Ω. Here χK is the characteristic function of p K. We usually say that Lloc(Ω) is the space of locally p-integrable functions on Ω Define the space of test functions by

∞ D(Ω) = {ϕ ∈ C (Ω); supp(ϕ) b Ω}.

∂ n We use the notation ∂i = and, if α = (α1,...,αn) ∈ is a multi-index, we ∂xi N set α α1 αn ∂ = ∂1 ...∂n . Proposition 1.5. The function f defined by

 e1/x, x < 0, f (x) = 0, x ≥ 0, belongs to C∞(R). Proof. We show, by using an induction with respect to the integer m, that

(m) (m) 1/x f (0) = 0, f (x) = Pm (1/x)e , x < 0, (1.1) where Pm is a polynomial. Clearly, the claim is true for m = 0. Assume then that (1.1) holds for some m. Then

f (m)(x) − f (m)(0) P (1/x)e1/x lim = lim m = 0, x<0, x→0 x x<0, x→0 x showing that f (m+1)(0) = 0. Finally, we have, for x < 0,

1 f (m+1)(x) = − P (1/x) + P0 (1/x)e1/x x2 m m 1/x = Pm+1 (1/x)e .

The proof is then complete. ut

We call a sequence of mollifiers any sequence of functions (ρm) satisfying Z n 0 ≤ ρm ∈ D(R ), supp(ρm) ⊂ B(0,1/m), ρmdx = 1. Rn 1.2 Approximation by smooth functions 11

Let  2 c−1e1/(|x| −1), |x| < 1, ρ(x) = 0, |x| ≥ 1, where Z 2 c = e1/(|x| −1)dx. B(0,1)

It is straightforward to check that (ρm) defined by

n n ρm = m ρ(mx), x ∈ R , is a sequence of mollifiers. 1 n Let f ∈ Lloc(Ω) and ϕ ∈ Cc(R ) satisfying supp(ϕ) ⊂ B(0,1/m) with m ≥ 1. Define the convolution ϕ ∗ f on

Ωm = {x ∈ Ω; dist(x,∂Ω) > 1/m} by Z Z ϕ ∗ f (x) = ϕ(x − y) f (y)dy = ϕ(y) f (x − y)dy. Ω B(0,1/m)

1 n Proposition 1.6. If f ∈ Lloc(Ω) and ϕ ∈ D(R ) is so that supp(ϕ) ⊂ B(0,1/m), ∞ n for m ≥ 1, then ϕ ∗ f ∈ C (Ωm) and, for every α ∈ N ,

∂ α (ϕ ∗ f ) = (∂ α ϕ) ∗ f .

Proof. Assume first that |α| = ∑αi = 1. Let x ∈ Ωm and r > 0 such that B(x,r) ⊂ Ωm. Whence, ω = B(x,r + 1/m) b Ω and, for 0 < |ε| < r, ϕ ∗ f (x + εα) − ϕ ∗ f (x) Z ϕ(x + εα − y) − ϕ(x − y) = f (y)dy. ε ω ε But ϕ(x + εα − y) − ϕ(x − y) lim = ∂ α ϕ(x − y) ε→0 ε and ϕ(x + εα − y) − ϕ(x − y) sup < ∞. y∈Ω, 0<|ε|

Define, for |y| < 1/m, the translation operator τy as follows

1 τy : f ∈ Lloc(Ω) → τy f : τy f (x) = f (x − y), x ∈ Ωm. 12 1 Sobolev spaces

Lemma 1.1. Let ω b Ω. (i) If f ∈ C(Ω) then we have, for sufficiently large m,

sup|ρm ∗ f (x) − f (x)| ≤ sup sup τy f (x) − f (x) . x∈ω |y|<1/m x∈ω

p (ii) If f ∈ Lloc(Ω), 1 ≤ p < ∞ then we have, for sufficiently large m,

kρm ∗ f − f kLp(ω) ≤ sup kτy f − f kLp(ω). |y|<1/m

Proof. Note first that ω ⊂ Ωm provided that m is sufficiently large. Let f ∈ C(Ω). As Z ρm(y)dy = 1, B(0,1/m) we obtain

Z |ρm ∗ f (x) − f (x)| = ρm(y)( f (x − y) − f (x))dy 1 B(0, m )

≤ sup sup|τy f (x) − f (x)|. 1 x∈ω |y|< m That is (i) holds. p Next, let f ∈ Lloc(Ω), 1 ≤ p < ∞. We have by Holder’s¨ inequality, for any x ∈ ω,

Z |ρm ∗ f (x) − f (x)| = ρm(y)[ f (x − y) − f (x)]dy B(0,1/m)

Z 1 1 0 ≤ ρm(y) p ρm(y) p [ f (x − y) − f (x)]dy B(0,1/m) Z 1/p p ≤ ρm(y)| f (x − y) − f (x)| dy . B(0,1/m)

We get by applying Fubini’s theorem

Z p Z Z p ρm ∗ f (x) − f (x) dx ≤ dx ρm(y) f (x − y) − f (x) dy ω ω B(0,1/m) Z Z p = dy ρm(y) f (x − y) − f (x) dx B(0,1/m) ω Z Z p ≤ ρm(y)dy sup | f (x − z) − f (x)| dx B(0,1/m) |z|<1/m ω Z ≤ sup | f (x − z) − f (x)|pdx. |z|<1/m ω

Assertion (ii) then follows. ut 1.2 Approximation by smooth functions 13

Lemma 1.2. (Continuity of translation operator) Let ω b Ω. (i) If f ∈ C(Ω) then

lim sup τy f (x) − f (x) = 0. y→0 x∈ω p (ii) If f ∈ Lloc(Ω), 1 ≤ p < ∞, then

lim kτy f − f kLp(ω) = 0. y→0

Proof. Let f ∈ C(Ω) and ω˜ be so that ω b ω˜ b Ω. Then (i) follows from the fact that f is uniformly continuous on ω˜ . p Next, let f ∈ Lloc(Ω), 1 ≤ p < ∞ and ε > 0. According to Theorem 1.5, we find g ∈ Cc(ω˜ ) so that k f − gkLp(ω˜ ) ≤ ε. By (i), there exists 0 < δ < dist(ω,ω˜ ) so that

sup|τyg(x) − g(x)| ≤ ε for any |y| < δ. x∈ω

Thus we have, for |y| < δ,

kτy f − f kLp(ω) ≤ kτy f − τygkLp(ω) + kτyg − gkLp(ω) + kg − f kLp(ω) 1/p ≤ 2kg − f kLp(ω) + |ω| sup τyg(x) − g(x) e x∈ω ≤ (2 + |ω|1/p)ε.

Hence (ii) follows because ε > 0 is arbitrary. ut

A combination of Lemmas 1.1 and 1.2 yields the following regularization theo- rem.

Theorem 1.6. (i) If f ∈ C(Ω) then ρm ∗ f converges to f uniformly in any compact subset of Ω. p p (ii) If u ∈ Lloc(Ω), 1 ≤ p < ∞, then ρm ∗ f converges to f in Lloc(Ω). 1 Theorem 1.7. (Cancellation theorem) Let f ∈ Lloc(Ω) satisfying Z f ϕdx = 0 for all ϕ ∈ D(Ω). Ω Then f = 0 a.e. in Ω.

Proof. We have in particular ρm ∗ f = 0, for every m. The proof is completed by applying Theorem 1.6 (ii). ut

Theorem 1.8. D(Ω) is dense Lp(Ω) for any 1 ≤ p < ∞. p p Proof. Let f ∈ L (Ω) and ε > 0. Since Cc(Ω) is dense L (Ω) by Theorem 1.5, we find g ∈ Cc(Ω) so that 14 1 Sobolev spaces

kg − f kp ≤ ε/2.

As supp(ρm ∗g) ⊂ B(0,1/m)+supp(g) (see Exercise 1.5), there exists a compact set K ⊂ Ω such that supp(ρm ∗ g) ⊂ K if m ≥ m0, for some integer m0. By Proposition p 1.6, ρm ∗ g ∈ D(Ω), and from Theorem 1.6, ρm ∗ g converges to g in L (Ω) (note that g has compact ). Hence, we find k ≥ m0 so that

kρk ∗ g − gkp ≤ ε/2, from which we deduce that k(ρk ∗ g) − f kp ≤ ε. ut

Here is another application of the regularization theorem

Lemma 1.3. Let K ⊂ Ω be compact. There exists ϕ ∈ D(Ω) so that 0 ≤ ϕ ≤ 1 and ϕ = 1 in K.

Proof. Set, for r > 0,

n Kr = {x ∈ R ; dist(x,K) ≤ r}.

Let ε > 0 so that K2ε ⊂ Ω. Let g be the function which is equal to 1 in Kε and n equal to 0 in R \ Kε . Fix m > 1/ε. Then the function ϕ = ρm ∗ g takes its values n between 0 and 1, it is equal to 1 in K and equal to 0 in R \ K2ε . Finally, ϕ ∈ D(Ω) by Proposition 1.6. ut

n Theorem 1.9. (Partition of unity) Let K be a compact subset of R and Ω1,...,Ωk n k n be open subsets of R so that K ⊂ ∪ j=1Ω j. Then there exist ψ0,ψ1,...ψk ∈ D(R ) so that k (a) 0 ≤ ψ j ≤ 1, 0 ≤ j ≤ k, ∑ j=0 ψ j = 1, n (b) supp(ψ0) ⊂ R \ K, ψ j ∈ D(Ω j), 1 ≤ j ≤ k. Proof. By Borel-Lebesgue’s theorem K can be covered by finite number of balls B(xm,rm), 1 ≤ m ≤ `, such that, for any m, B(xm,2rm) ⊂ Ω j(m) for some 1 ≤ j(m) ≤ k. If 1 ≤ j ≤ k then set

` ` [ [ [ Kj = B(xm,2rm), K0 = B(xm,rm), Ω = B(xm,2rm). j(m)= j m=1 m=1

According to Lemma 1.3, there exists, where 1 ≤ j ≤ k, ϕ j ∈ D(Ω j) satisfying 0 ≤ ϕ j ≤ 1 and ϕ j = 1 sur Kj. There exists also ϕ ∈ D(Ω) satisfying 0 ≤ ϕ ≤ 1 and ϕ = 1 in K0. Take then ϕ0 = 1 − ϕ and, for 0 ≤ j ≤ k,

ϕ j ψ j = k . ∑i=0 ϕi

Then it is straightforward to check that ψ0,ψ1,...ψk have the expected properties. ut 1.3 Weak derivatives 15 1.3 Weak derivatives

The notion of weak derivative consists in generalizing the derivative in classical 1 sense by means of an formula. Precisely if f ,g ∈ Lloc(Ω) and n α ∈ N , we say that g = ∂ α f in the weak sense provided that Z Z f ∂ α ϕdx = (−1)|α| gϕdx for any ϕ ∈ D(Ω). Ω Ω Note that by the cancellation theorem ∂ α f is uniquely determined. In light of to this definition we can state the following result. 1 Lemma 1.4. (Closing lemma) If ( fm) converges to f in Lloc(Ω) and (gm) converge 1 α α vers g in Lloc(Ω) and if gm = ∂ fm in the weak sense, for any m, then g = ∂ f in the weak sense.

1 n α Proposition 1.7. Let f ,g ∈ Lloc(Ω) and α ∈ N . If g = ∂ f in the weak sense then in Ωm = {x ∈ Ω; dist(x,∂Ω) > 1/m} we have α ∂ (ρm ∗ f ) = ρm ∗ g. ∞ Proof. As ρm ∗ f ∈ C (Ωm) by Proposition 1.6, we have, for x ∈ Ωm, Z α α ∂ (ρm ∗ f )(x) = ∂x ρm(x − y) f (y)dy Ω Z |α| α = (−1) ∂y ρm(x − y) f (y)dy. Ω Whence Z α 2|α| ∂ (ρm ∗ f )(x) = (−1) ρm(x − y)g(y)dy = ρm ∗ g(x) Ω as expected. ut Lemma 1.5. Let f ,g ∈ C(Ω) and |α| = 1. If f has a compact support and g = ∂ α f in the classical sense then Z gdx = 0. Ω Proof. We may assume, without loss of generality, that α = (0,...,0,1). As supp( f ) n− is compact, it is contained in R 1×]a,b[ for some finite interval ]a,b[. We naturally extend f and g by 0 outside Ω. We still denote by f and g these extensions. Using 0 n−1 the notation x = (x ,xn) ∈ R × R, we get Z Z b 0 0 0 0 g(x ,xn)dxn = g(x ,xn)dxn = f (x ,a) − f (x ,b) = 0. R a Whence, we obtain by applying Fubini’s theorem 16 1 Sobolev spaces Z Z Z Z 0 gdx = gdx = dx gdxn = 0. Ω Rn Rn−1 R This completes the proof. ut Theorem 1.10. (Du Bois-Reymond lemma) Let f ,g ∈ C(Ω) and |α| = 1. Then g = ∂ α f in the weak sense if and only if g = ∂ α f in the classical sense. Proof. If g = ∂ α f in the weak sense then, according to Proposition 1.7, we have in the classical sense α ∂ (ρm ∗ f ) = ρm ∗ g. Hence Z ε Z ε α ρm ∗ g(x +tα)dt = ∂ (ρm ∗ f )(x +tα)dt = ρm ∗ f (x + εα) − ρm ∗ f (x). 0 0 That is we have Z ε ρm ∗ f (x + εα) = ρm ∗ f (x) + ρm ∗ g(x +tα)dt. (1.2) 0

From the regularization theorem ρm ∗ f (resp. ρm ∗ g) tends to f (resp. g) uniformly in any compact subset of Ω, as m → ∞. We get by passing to the limit in (1.2)

Z ε f (x + εα) = f (x) + g(x +tα)dt. 0 Whence, g = ∂ α f in the classical sense. Conversely, if g = ∂ α f in the classical sense then by virtue of Lemma 1.5 we have Z Z 0 = ∂ α ( f ϕ)dx = ( f ∂ α ϕ + gϕ)dx for every ϕ ∈ D(Ω). Ω Ω Thus, g = ∂ α f in the weak sense. ut We close this section by an example. Let, for −n < θ ≤ 1,

θ 2 θ/2 f (x) = |x| and fε (x) = (|x| + ε) , ε > 0.

n n The function fε is continuous in R and therefore it is measurable in R . As ( fε ) converges a.e. (in fact everywhere except at 0), we deduce that f is measurable. On the other hand, for any R > 0, we have by passing to polar coordinates

Z Z Z R |x|θ dx = dσ rn+θ−1dr < ∞, B(0,R) Sn−1 0

n− n where S 1 is the unit sphere of R . We have, when θ ≤ 0, 0 ≤ fε ≤ f , and since fε converges a.e. to f , we may apply Lebesgue’s dominated convergence theorem. Therefore, ( fε ) converges to f 1 n in Lloc(R ). We proceed similarly in the case θ ≥ 0. We note that fε ≤ f1, 0 < ε ≤ 1, 1 n and we conclude that we still have the convergence of ( fε ) to f in Lloc(R ). 1.4 W k,p spaces 17

∞ n We get by simple computations that fε ∈ C (R ) and

2 θ−2 ∂ j fε = θx j(|x| + ε) 2 .

θ−1 1 θ−2 Hence |∂ j fε | ≤ θ|x| and then ∂ j fε converges in Lloc(R) to g = θx j|x| if θ > 1 − n. In light of the closing lemma, we obtain that g = ∂ j f in the weak sense.

1.4 W k,p spaces

If k ≥ 1 is an integer and 1 ≤ p < ∞, we define the Sobolev W k,p(Ω) by

W k,p(Ω) = {u ∈ Lp(Ω); ∂ α u ∈ Lp(Ω) for any |α| ≤ k}.

Here ∂ α u is understood as the derivative in the weak sense. We endow this space with the norm

1/p Z ! α p kukW k,p(Ω) = kukk,p = ∑ |D u| dx . |α|≤k Ω

The space Hk(Ω) = W k,2(Ω) is equipped with the scalar product

α α (u|v)Hk(Ω) = ∑ (∂ u|∂ v)L2(Ω). |α|≤k

k,p Define also the local Wloc (Ω) as follows

k,p p k,p Wloc (Ω) = {u ∈ Lloc(Ω); u|ω ∈ W (ω, for each ω b Ω}.

k,p We say that the sequence (um) converges to u in Wloc (Ω) if, for every ω b Ω, we have kum − ukW k,p(ω) → 0, as m → ∞. p d Consider L (Ω,R ) that we equip with its natural norm

1/p d Z ! p k(vs)kp = ∑ |vs| dx . s=1 Ω

Let Γ = {1,...,d} × Ω and define on

L = {u : Γ → R; u(s,·) ∈ Cc(Ω), 1 ≤ s ≤ d} the elementary integral 18 1 Sobolev spaces

Z d Z udµ = ∑ u(s,x)dx. Γ s=1 Ω p d Then it is not difficult to check that L (Ω,R ) is isometrically isomorphic to Lp(Γ ,dµ). An extension of Theorem 1.3 to a measure space with positive σ-finite measure yields the following result.

p d Lemma 1.6. Any closed subspace of L (Ω,R ) is a separable Banach space. We get by applying Riesz’s representation theorem1 to Lp(Γ , µ) the following lemma.

p d 0 Lemma 1.7. Let 1 < p < ∞ and Φ ∈ [L (Ω,R )] . Then there exists a unique ( fs) ∈ p0 d L (Ω,R ) so that d Z hΦ,(vs)i = ∑ fsvsdx. s=1 Ω

Moreover k( fs)kp0 = kΦk.

Let 1 ≤ p < ∞, k ≥ 1 an integer and d = ∑|α|≤k 1. Then the mapping

k,p p d α A : W (Ω) → L (Ω,R ) : u → Au = (∂ u)|α|≤k is isometric, i.e. kAukp = kukk,p.

Theorem 1.11. For 1 ≤ p < ∞ and k ≥ 1 an integer, the Sobolev space W k,p(Ω) is separable Banach space.

k,p p d Proof. According to the closing lemma, F = A(W (Ω)) is closed in L (Ω,R ). The theorem follows then from Lemma 1.6. ut 0 Theorem 1.12. Let k ≥ 1 be an integer and 1 < p < ∞. For any Φ ∈ W k,p(Ω) , p0 d there exists a unique ( fα ) ∈ L (Ω,R ) so that k( fα )kp0 = kΦk and

α k,p hΦ,ui = Φ(u) = ∑ fα D u, for all u ∈ W (Ω). |α|≤k

1 Riesz’s representation theorem Let (X,dµ) be a measure space, where dµ is positive σ-finite p 0 p0 measure. For each Φ ∈ [L (X, µ)] , there exists a unique gΦ ∈ L (X, µ) so that Z p Φ( f ) = hΦ, f i = f gΦ dµ for any f ∈ L (X,dµ). X

Furthermore kΦk = kgΦ kp0 . 1.4 W k,p spaces 19

Proof. By Hahn-Banach’s extension theorem2, there exists a continuous linear form p d extending Φ to L (Ω,R ) without increasing its norm. The theorem follows then from Lemma 1.7. ut

Let k ≥ 1 be an integer and 1 ≤ p < ∞. Denote the closure of D(Ω) in W k,p(Ω) k,p k,2 k by W0 (Ω). The space W0 (Ω) is usually denoted by H0 (Ω). Let W −k,p(Ω) be the space of continuous linear forms given as follows Z α Φ : D(Ω) → R : u → ∑ gα ∂ udx, |α|≤k Ω

k,p p0 d when D(Ω) is seen as a subspace of W (Ω) and where (gα ) ∈ L (Ω,R ). We endow W −k,p(Ω) with its natural quotient norm

kΦkW −k,p(Ω) = kΦk−k,p ( Z ) α =inf k(gα )kp0 ; hΦ,ui = ∑ gα ∂ udx for any u ∈ D(Ω) . |α|≤k Ω

The space W −k,2(Ω) is usually denoted by H−k(Ω).

Theorem 1.13. For any integer k ≥ 1 and 1 ≤ p < ∞, the space W −k,p(Ω) is iso- h k,p i0 metrically isomorphic to W0 (Ω) .

−k,p p0 d Proof. If Φ ∈ W (Ω) then there exists (gα ) ∈ L (Ω,R ) so that Z α hΦ,ui = ∑ gα ∂ udx, for any u ∈ D(Ω). |α|≤k Ω

Hence |hΦ,ui| ≤ k(gα )kp0 kukk,p for any u ∈ D(Ω). k,p We can extend uniquely Φ to W0 (Ω) by density. We still denote this exten- sion by Φ. Moreover kΦkh k,p i0 ≤ k(gα )kp0 in such a way that kΦkh k,p i0 ≤ W0 (Ω) W0 (Ω) kΦk −k,p . W0 (Ω)

2 Hahn-Banach’s extension theorem. Let V be a real normed vector space with norm k·k. Let V0 be a subspace of V and let Φ0 : V0 → R be a continuous linear form with norm

kΦ0k 0 = sup Φ0(x). V0 x∈V0, kxk≤1

0 Then there exists Φ ∈ V extending Φ0 so that

kΦk 0 = kΦ0k 0 . V V0 20 1 Sobolev spaces

h k,p i0 Conversely, let Φ ∈ W0 (Ω) . Then by Hahn-Banach’s extension theorem Φ has an extension, still denoted by Φ, to W k,p(Ω) that does not increase its norm. But p d from Theorem 1.12 there exists (gα ) ∈ L (Ω,R ) so that k(gα )kp0 = kΦkh k,p i0 W0 (Ω) and Z α hΦ,ui = ∑ gα D udx for all u ∈ D(Ω). |α|≤k Ω Whence, Φ ∈ W −k,p(Ω) and

kΦkW −k,p(Ω) ≤ k(gα )kp0 = kΦkh k,p i0 . W0 (Ω)

This completes the proof. ut Next, we extend some classical rules of differential calculus to weak derivatives.

1,1 1 Proposition 1.8. (Derivative of a product) if u ∈ Wloc (Ω) and f ∈ C (Ω) then f u ∈ 1,1 Wloc (Ω) and ∂ j( f u) = f ∂ ju + ∂ j f u.

Proof. If um = ρm ∗ u then we have in the classical sense

∂ j( f um) = f ∂ jum + ∂ j f um.

1 By the regularization theorem, um → u and ∂ jum = ρm ∗∂ ju → ∂ ju in Lloc(Ω). Thus

f um → f u, ∂ j( f um) = f ∂ jum + ∂ j f um → f ∂ ju + ∂ j f u

1 in Lloc(Ω). The expected result follows then from the closing lemma. ut This proposition will be used to prove the following result.

1,p n 1,p n Theorem 1.14. If 1 ≤ p < ∞ then W0 (R ) = W (R ). n ,p n Proof. It is sufficient to prove that D(R ) is dense in W 1 (R ). We use truncation and regularization procedures. We fix θ ∈ C∞(R) satisfying 0 ≤ θ ≤ 1 and

θ(t) = 1, t ≤ 1, θ(t) = 0, t ≥ 2.

We define a truncation sequence by setting

n θm(x) = θ (|x|/m), x ∈ R .

,p n Let u ∈ W 1 (R ). The formula giving the weak derivative of a product shows that 1,p n um = θmu ∈W (R ). With the help of Lebesgue’s dominated convergence theorem 1,p n  one can check that um converges to u in W (R ) θm converges a.e. to u because α θm tends to 1 and we have |θmu| ≤ |u|. On the other hand, supp(∂ θm) ⊂ {m ≤ |x| ≤ α α α  2m} and ∂ um = θmD u + D θmu . This construction guarantees that the support of um is contained in B(0,2m). 1.4 W k,p spaces 21

We now proceed to regularization. From the previous step we need only to con- ,p n n sider u ∈ W 1 (R ) with compact support. Let K be a compact subset of de R so ∞ n that, for any m, the support of um = ρm ∗ u is contained in K. As um ∈ C (R ) by n Proposition 1.6, we deduce that um belongs to D(R ). We have from the regulariza- tion theorem p n um → u, ∂ jum = ρm ∗ ∂ ju → ∂ ju in L (R ). The proof is then complete. ut

n Let Ω and ω be two open subsets of R . Recall that f : ω → Ω is a diffeo- morphism if f is bijective, it is continuously differentiable and satisfies Jf (x) = det(∂ j fi(x)) 6= 0, for every x ∈ ω. 1 The following result follows from the density of Cc(Ω) in L (Ω) and the classical formula of change of variable for smooth functions.

1 1 Theorem 1.15. If u ∈ L (Ω) then (u ◦ f )|Jf | ∈ L (ω) and Z Z u( f (x))|Jf (x)|dx = u(y)dy. ω Ω Proposition 1.9. (Change of variable formula) Let ω and Ω be two open subsets of n 1,1 1,1 R and let f : ω → Ω be a diffeomorphism. If u ∈ Wloc (Ω) then u ◦ f ∈ Wloc (ω) and n ∂ j(u ◦ f ) = ∑ (∂ku ◦ f )∂ j fk. k=1

Proof. Let um = ρm ∗ u and v ∈ D(ω). We have according to the definition of weak derivatives Z Z n (um ◦ f )(y)∂ jv(y)dy = − ∑ (∂kum ◦ f )(y)∂ j fk(y)v(y)dy. ω ω k=1

If g = f −1 then Theorem 1.15 yields Z Z (um ◦ f )(y)∂ jv(y)dy = um(x)(∂ jv ◦ g)(x)|det(Jg(x))|dx ω Ω Z n = − ∑ ∂kum(x)(∂ j fk ◦ g)(x)(v ◦ g)(x)|det(Jg(x))|dx Ω k=1 Z n = ∑ (∂kum ◦ f )(y)∂ j fk(y)v(y)dy. (1.3) ω k=1 Now, we have by the regularization theorem

1 um → u, ∂ jum → ∂ ju in Lloc(ω).

We pass to the limit, when m → ∞, in the second and the third members of (1.3). We get 22 1 Sobolev spaces Z u(x)(∂ jv ◦ g)(x)|det(Jg(x))|dx Ω Z n = − ∑ ∂ku(x)(∂ j fk ◦ g)(x)(v ◦ g)(x)|det(Jg(x))|dx Ω k=1 and hence Z Z n (u ◦ f )(y)∂ jv(y)dy = − ∑ (∂ku ◦ f )(y)∂ j fk(y)v(y)dy ω ω k=1 by Theorem 1.15. The proposition is then proved. ut 1 1,1 Proposition 1.10. (Derivative of a composition) Let f ∈ C (R) and u ∈ Wloc (Ω). If 0 1,1 M = sup| f | < ∞ then f ◦ u ∈ Wloc (Ω) and 0  ∂ j( f ◦ u) = f ◦ u ∂ ju.

Proof. If um = ρm ∗ u then we have in the classical sense

0 ∂ j( f ◦ um) = f ◦ um∂ jum.

So, by the regularization theorem, we obtain

1 um → u, ∂ jum → ∂ ju in Lloc(Ω).  If ω b Ω we get subtracting if necessary a subsequence we may assume that the convergence holds also almost everywhere in ω again from the regularization the- orem Z Z |( f ◦ um) − ( f ◦ u)|dx ≤ M |um − u|dx → 0, ω ω

Z 0  0  | f ◦ um ∂ jum − f ◦ u ∂ ju|dx ω Z Z 0  0  ≤ M |∂ jum − ∂ ju|dx + | f ◦ um − f ◦ u ||∂ ju|dx → 0. ω ω Hence 0  0  1 f ◦ um → f ◦ u, f ◦ um ∂ jum → f ◦ u ∂ ju in Lloc(Ω) from Lebesgue’s dominated convergence theorem. The proof is then completed by using the closing lemma. ut 1,1 + − 1,1 Corollary 1.1. If u ∈ Wloc (Ω) then u ,u ,|u| ∈ Wloc (Ω) and   ∂ ju in {u > 0}, ∂ j|u| = −∂ ju in {u < 0},  0, in {u = 0}.

2 2 1/2 Proof. Let, for ε > 0, fε (t) = (t + ε ) and 1.5 Extension and trace operators 23   ∂ ju in {u > 0}, v = −∂ ju in {u < 0},  0 in {u = 0}.

We get by using Proposition 1.10 u ∂ j( fε ◦ u) = ∂ ju. (u2 + ε2)1/2 Hence 1 fε ◦ u → |u|, ∂ j( fε ◦ u) → v in Lloc(Ω)

and ∂ j|u| = v according to the closing lemma. Finally, for completing the proof we observe that 2u+ = |u| + u and 2u− = |u| − u. ut

1.5 Extension and trace operators

We start by extension operators using the simple idea of reflexion. If ω is an open n− subset of R 1 and 0 < δ ≤ +∞, we let

Q = ω×] − δ,+δ[, Q+ = ω×]0,δ[.

For an arbitrary u : Q+ → R, we define σu and τu on Q by

 0 0 u(x ,xn) if xn > 0, σu(x ,xn) = 0 u(x ,−xn) if xn < 0,

and  0 0 u(x ,xn) if xn > 0, τu(x ,xn) = 0 −u(x ,−xn) if xn < 0.

1,p Lemma 1.8. (Extension by reflexion) Let 1 ≤ p < ∞. If u ∈ W (Q+) then σu ∈ W 1,p(Q) and

1/p 1/p k uk p ≤ kuk p , k uk 1,p ≤ kuk 1,p . σ L (Q) 2 L (Q+) σ W (Q) 2 W (Q+) Proof. We first prove that

∂ jσu = σ∂ ju, 1 ≤ j ≤ n − 1.

If v ∈ D(Q), we have Z Z 0 0 σu∂ jvdx = uD jwdx, with w(x ,xn) = v(x,xn) + v(x ,−xn). (1.4) Q Q+

Fix η ∈ C ∞(R) satisfying 24 1 Sobolev spaces

 0 if t < 1 , η(t) = 2 1 if t > 1, and set ηm = η(mt). 0 As ηm(xn)w(x ,xn) ∈ D(Q+), we obtain Z Z Z uηmD jw = u∂ j(ηmw) = − ∂ juηmw. Q+ Q+ Q+ Il light of Lebesgue’s dominated convergence theorem, we can pass to the limit, when m tends to infinity, in the first and third terms. We obtain Z Z Z u∂ jwdx = − ∂ juwdx = − σ∂ juvdx Q+ Q+ Q which, combined with (1.4), entails Z Z σu∂ jvdx = − σ∂ juvdx. Q Q That is we have ∂ j(σu) = σ∂ ju, 1 ≤ j ≤ n − 1. (1.5)

Next, we prove that ∂n(σu) = τ∂nu. For this purpose, we note that Z Z 0 0 σu∂nvdx = u∂nwdx˜ , withw ˜(x ,xn) = v(x,xn) − v(x ,−xn). (1.6) Q Q+

0 0 Asw ˜(x ,0) = 0, there exists C0 > 0 so that |w˜(x ,xn)| ≤ C0|xn| in Q+. Using the fact 0 that ηm(xn)w˜(x ,xn) ∈ D(Q+), we find Z Z u∂n(ηmw˜)dx = − ∂nuηmwdx˜ . Q+ Q+ But 0 ∂n(ηmw˜) = ηmDnw˜ + mη (mxn)w˜. 0 Let C1 = kη k∞. We get, observing thatw ˜ has a compact support, that there exists a compact subset K of ω so that

Z Z 0 mη (mxn)uwdx˜ ≤ C0C1m |u|xndx Q+ K×]0,1/m[ Z ≤ C0C1 |u|dx → 0. K×]0,1/m[

We get by applying again Lebesgue’s dominated convergence theorem Z Z Z u∂nwdx˜ = − ∂nuwdx˜ = − τ∂nuvdx. Q+ Q+ Q 1.5 Extension and trace operators 25

This identity together with (1.6) imply Z Z Z σu∂nvdx = ∂nuwdx˜ = − τ∂nuvdx. Q Q+ Q In other words, we demonstrated that

∂n(σu) = τ∂nu. (1.7)

Finally, identities (1.5) and (1.7) yield the expected result. ut

We use in the sequel the following notations

n D(Ω) = {u|Ω ; u ∈ D(R )}, n 0 n 0 n−1 R+ = {(x ,xn) ∈ R ; x ∈ R , xn > 0}.

n Lemma 1.9. (Trace inequality) Let 1 ≤ p < ∞. We have, for u ∈ D(R+), Z 0 p 0 p−1 |u(x ,0)| dx ≤ pkuk p n k∂nuk p n . L (R ) L (R+) Rn−1 + n Proof. Consider first the case 1 < p < ∞. Let u ∈ D(R+). As u has compact support, 0 n−1 0 0 for each x ∈ R , we find yn = yn(x ) so that u(x ,yn) = 0, yn ≥ yn. Whence Z y 0 p n 0 p−1 0 |u(x ,0)| = − p|u(x ,xn)| ∂nu(x ,xn)dxn 0 and hence Z ∞ 0 p 0 p−1 0 |u(x ,0)| ≤ p|u(x ,xn)| |∂nu(x ,xn)|dxn. 0 We get by applying Fubini’s theorem and then Holder’s¨ inequality Z Z 0 p 0 p−1 |u(x ,0)| dx ≤ p |u| |∂nu|dx n−1 n R R+ 1 1 Z  0 Z  p (p−1)p0 p p ≤ p |u| dx |∂nu| dx n n R+ R+ 1 1 Z 1− p Z  p p p ≤ p |u| dx |∂nu| dx , n n R+ R+ which yields the expected inequality. The proof in the case p = 1 is quite similar to that of the case 1 < p < ∞. ut

Proposition 1.11. Let 1 ≤ p < ∞. There exists a unique linear bounded operator

1,p n p n−1 γ0 : W (R+) → L (R )

n satisfying γ0u = u(·,0), for each u ∈ D(R+). 26 1 Sobolev spaces

n Proof. If u ∈ D(R+), we set γ0u = u(·,0). From Lemma 1.9, we have

1 p kγ uk p n−1 ≤ p kuk 1,p n . 0 L (R ) W (R+)

n We deduce, using the theorem of extension by reflexion and the density of D(R ) 1,p n n 1,p n in W (R ) (a consequence of Theorem 1.14), that D(R+) is dense W (R+). We complete the proof by extending γ0 by density. ut

1,p n Proposition 1.12. (Integration by parts) Let 1 ≤ p < ∞. If u ∈ W (R+) and v ∈ n D(R+) then Z Z Z 0 v∂nudx = − ∂nvudx − γ0vγ0udx n n n−1 R+ R+ R and Z Z v∂ judx = − ∂ jvudx, 1 ≤ j ≤ n − 1. n n R+ R+

n Proof. Assume first that u ∈ D(R+). The classical integration by parts formula 0 n− yields, for each x ∈ R 1, Z +∞ Z +∞ 0 0 0 0 v(x ,xn)∂nu(x ,xn)dxn = − u(x ,xn)∂nv(x ,xn)dxn 0 0 − u(x0,0)v(x0,0).

We then obtain by applying Fubini’s theorem Z Z Z 0 0 0 v∂nudx = − ∂nvudx − v(x ,0)u(x ,0)dx , n n n−1 R+ R+ R that we write in the form Z Z Z 0 v∂nudx = − ∂nvudx − γ0vγ0udx . (1.8) n n n−1 R+ R+ R

n 1,p n We know from the proof of Proposition 1.11 that D(R+) is dense in W (R+). So 1,p n n if u ∈ W (R+) then we may find a sequence (um) in D(R+) that converges to u 1,p n 1,p n in W (R+). This and the fact that γ0 is a bounded operator from W (R+) into p n−1 p n−1 L (R ) entail that γ0um converges to γ0u in L (R ). We obtain from (1.8) Z Z Z 0 v∂numdx = − ∂nvumdx − γ0vγ0umdx . n n n−1 R+ R+ R We then pass to the limit, when m → ∞, to get the first formula. The second formula can be established analogously. ut

n n Hereafter, if u is a function defined on R+ then its extension to R by 0 is denoted by u. 1.5 Extension and trace operators 27

1,p n Proposition 1.13. Let 1 ≤ p < ∞ and u ∈ W (R+). The following assertions are equivalent. 1,p n (i) u ∈ W0 (R+). (ii) γ0u = 0. 1,p n (iii) u ∈ W (R ) and ∂ ju = ∂ ju, 1 ≤ j ≤ n.

1,p n n Proof. If u ∈ W0 (R+) then there exits a sequence (um) in D(R+) converging to 1,p n p n−1 u in W (R+). Therefore γ0um → γ0u dans L (R ). But γ0um = 0, for each m. Whence, γ0u = 0. That is (i) implies (ii). n If γ0u = 0 then by Proposition 1.12 we have, for every v ∈ D(R ), Z Z v∂ judx = ∂ jvudx, 1 ≤ j ≤ n. Rn Rn In other words, we proved that (ii) implies (iii). Assume finally that (iii) holds. If (θm) is the truncation sequence introduced in 1,p n the proof of Theorem 1.14 then the sequence um = θmu converges to u in W (R ) n and um has its support contained in B(0,2m) ∩ R+. We are then reduced to consider n the case where additionally u has a compact support in R+. Let ym = (0,...,0,1/m) and vm = τym u. Since ∂ jvm = τym ∂ ju, the continuity of 1,p n translation operators guarantees that vm → u in W (R+). That is we are lead to n the case where u has a compact support in R+. Therefore, we may find a compact n ∞ n subset K of R+ so that, for each m, supp(ρm ∗ u) ⊂ K. As wm = ρm ∗ u ∈ C (R+), n we have wm ∈ D(R+). According to the regularization theorem, wn tends to u in 1,p n 1,p n W (R+). In consequence, u ∈ W0 (R+) and then (iii) entails (i). This completes the proof. ut

n Prior to considering extension and trace theorems for an arbitrary domain of R , we introduce the definition of an open subset of class Ck. We say that an open subset n k Ω of R is of class C if, for each x ∈ Γ = ∂Ω, we can find a neighborhood U of n n− k x in R , an open subset ω of R 1, ψ ∈ C (ω) 3 and δ > 0 so that, modulo a rigid transform,

U = {(y0,ψ(y0) +t); y0 ∈ ω, |t| < δ}, Ω ∩U = {(y0,ψ(y0) +t); y0 ∈ ω, 0 < t < δ}, Γ ∩U = {(y0,ψ(y0)); y0 ∈ ω}.

In other words, an open subset Ω is of class Ck if any point of its boundary admits a neighborhood U so that U ∩ Ω coincide with the epigraph of a function of class Ck. We leave to the reader to check that, with the aid of the implicit function theorem, n k the above definition of an open subset of R of class C is equivalent to the following one: let

3 k k n Recall that C (ω) = {u|ω ; u ∈ C (R )}. 28 1 Sobolev spaces

0 n 0 Q = {x = (x ,xn) ∈ R ; |x | < 1 and |xn| < 1}, n Q+ = Q ∩ R+, 0 n 0 Q0 = {x = (x ,xn) ∈ R ; |x | < 1 and xn = 0}.

Ω is said of class Ck, k ≥ 1 is an integer if, for each x ∈ Γ , there exists a neighbor- n hood U of x in R and a bijective mapping Φ : Q → U satisfying

k −1 k Φ ∈ C (Q), Φ ∈ C (U), Φ(Q+) = U ∩ Ω, Φ(Q0) = U ∩Γ .

If the open subset Ω is of class Ck and has bounded boundary then there exist n (think to the compactness of Γ ) a finite number of open subsets of R , U1,...,U`, n−1 open subsets of R , ω1,...,ω`, functions ψ1,...ψ` and positive real numbers δ1,...,δ` satisfying all the conditions of the preceding definition and are so that

` [ Γ ⊂ Uj. j=1

∞ n By the theorem of partition of unity, there exist φ0,...,φ` ∈ C (R ) satisfying ` (i) 0 ≤ φ j ≤ 1, 0 ≤ j ≤ `, ∑ j=0 φ j = 1, n (ii) supp(φ0) ⊂ R \Γ , φ j ∈ D(Uj), j = 1,...,`. Theorem 1.16. (Extension theorem) Let 1 ≤ p < ∞ and let Ω be an open subset of n R of class C1 with bounded boundary. There exists a bounded operator

1,p 1,p n P : W (Ω) → W (R )

so that Pu|Ω = u. Proof. We use the notations that we introduced above in the definition of Ω of class Ck with bounded boundary. Fix u ∈ W 1,p(Ω) and 1 ≤ j ≤ `. From the change of variable formula, we have

0 0  1,p u y ,ψ j(y ) +t ∈ W (ω j×]0,δ j[).

The reflexion extension lemma then entails

0 0  1,p u y ,ψ j(y ) + |t| ∈ W (ω j×] − δ j,δ j[).

Thus 0  0 0 0  1,p v j y ,yn = u y ,ψ j(y ) + |yn − ψ j(y )| ∈ W (Uj). For 1 ≤ j ≤ `, we can easily check that

kv k 1,p ≤ C kuk 1,p , j W (Uj) 0 W (Ω∩Uj)

where the constant C0 > 0 is independent of u. Let (φ j) the partition of unity defined as above. Set U0 = Ω, v0 = u and, for 0 ≤ j ≤ `, let 1.5 Extension and trace operators 29  φ j(x)v j(x), x ∈ Uj, u j(x) = n 0, x ∈ R \Uj. 1,p n By the formula of the derivative of a product, we obtain that u j ∈ W (R ) and

ku k 1,p n ≤ C kuk 1,p , j W (R ) 1 W (Ω)

where the constant C1 > 0 is independent of u. Define ` 1,p n  Pu = ∑ u j ∈ W (R ) . j=0 Then there exists a constant C, independent of u, so that

kPuk 1,p n ≤ Ckuk 1,p . W (R ) W (Ω) Furthermore, we have

` Pu(x) = ∑ ψ j(x)u(x) = u(x), x ∈ Ω, j=0

as expected. ut n Remark 1.1. In the case where Ω is a cube of R (which is not of class C1), the extension operator can easily be constructed by using extensions by reflexion and a localization argument. Theorem 1.17. (Density theorem) Let 1 ≤ p < ∞ and Ω be an open subset of class C1 with bounded boundary. Then D(Ω) is dense W 1,p(Ω). 1,p n Proof. Let u ∈W (Ω). From Theorem 1.14, there exists a sequence (vm) in D(R ) 1,p n 1,p converging to Pu in W (R ). Therefore, um = vm|Ω tends to u in W (Ω). ut Let Ω be a bounded open subset of class C1 with boundary Γ . For u ∈ C(Γ ), the formula Z ` Z q 0 0 0 2 0 u(γ)dγ = ∑ (φ ju)(y ,ψ j(y )) 1 + |∇ψ j(y )| dy Γ j=1 ω j defines an elementary integral. Theorem 1.18. (Trace theorem) Let Ω be a domain of class C1 with bounded boundary Γ . For 1 ≤ p < ∞, there exists a unique bounded operator

1,p p γ0 : W (Ω) → L (Γ )

so that γ0u = u|Γ if u ∈ D(Ω).

Proof. Fix u ∈ D(Ω) and 1 ≤ j ≤ `. There exists ϕ j ∈ D(Uj) so that 0 ≤ ϕ j ≤ 1 and ϕ j = 1 in supp(φj). With the help of the change of variable formula and the formula of the derivative of a product, we get 30 1 Sobolev spaces

0  0 0  1,p v j y ,t = (ϕ ju) y ,ψ j(y ) +t ∈ W (ω j×]0,δ j[).

As v j has a compact support in ω j × [0,δ j[, Proposition 1.11 implies Z 0 p 0 p |v j(y ,0)| dy ≤ C0kv jk 1,p ≤ C1kuk 1,p . W (ω j×]0,δ j[) W (Ω) ω j

That is we proved, where we set γ0u = u|Γ ,

kγ0ukLp(Γ ) ≤ CkukW 1,p(Ω).

We end up the proof by noting that D(Ω) is dense in W 1,p(Ω) (Theorem 1.17). ut

Let Ω be a open subset of class C1 with bounded boundary Γ . Define the unit exterior normal vector to Γ at γ ∈ Γ ∩Uj by

0 (∇ψ j(y ),−1) ν(γ) = . p 0 2 1 + |∇ψ j(y )|

Theorem 1.19. (Divergence theorem) Let Ω an open bounded set of class C1 with , n boundary Γ . If V ∈ W 1 1(Ω,R ) then Z Z divVdx = γ0V · νdγ. Ω Γ Proof. Follows from the classical divergence theorem, which is valid when V ∈ n n , n C1 Ω,R , and the density of C1 Ω,R in W 1 1 (Ω,R ). ut

Proposition 1.14. Let Ω an open bounded set of class C1, 1 ≤ p < ∞ and u ∈ W 1,p(Ω). The following assertions are equivalent. 1,p (i) u ∈ W0 (Ω). (ii) γ0u = 0. (iii) There exits a constant C > 0 so that

Z n 0 uDiϕdx ≤ CkϕkLp (Ω), ϕ ∈ D(R ), 1 ≤ i ≤ n. Ω

1,p n (iv) u ∈ W (R ) and ∂iu = ∂iu, 1 ≤ i ≤ n, where, as before, u denotes the extension of u by 0 outside Ω.

1,p 1,p Proof. (i) implies (ii): if u ∈ W0 (Ω) then u is the limit in W0 (Ω) of a sequence 1,p p (um) of elements of D(Ω). As γ0 is continuous from W0 (Ω) into L (Γ ) and γ0um = 0, we deduce immediately that γ0u = 0. n (ii) implies (iii): if ϕ ∈ D(R ) and 1 ≤ i ≤ n then the divergence theorem yields 1.6 Imbedding Theorems 31 Z Z Z u∂iϕdx = − ∂iuϕdx + γ0(uϕ)νidγ Ω Ω Γ Z = − ∂iuϕ. Ω

That is we have (iii) with C = k∇ukLp(Ω,Rn). n (iii) implies (iv): for ϕ ∈ D(R ) and 1 ≤ i ≤ n, we have

Z Z u∂iϕdx = u∂iϕdx ≤ Ckϕk p0 n . L (R ) Rn Ω

p n This and Riesz’s representation theorem show that there exists gi ∈ L (R ) so that Z Z u∂iϕdx = giϕdx. Rn Rn 1,p n Thus ∂iu = gi and then u ∈ W (R ). Finally, from the identities Z Z Z Z u∂iϕdx = − ∂iuϕdx = − ∂iuϕdx = − ∂iuϕdx Rn Rn Ω Rn

we get ∂iu = ∂iu. (iv) implies (i): by using local cards and partition of unity, we are reduced to the n case Ω = R+. The result follows then from Proposition 1.13. ut

1.6 Imbedding Theorems

Let us first explain briefly how to use an homogeneity argument to get an informa- tion on the validity of a certain inequality. Assume then that we can find a constant n C > 0 and 1 ≤ q < ∞ so that, for any u ∈ D(R ),

kukLq(Rn) ≤ Ck∇ukLp(Rn,Rn).

We get by substituting u by uλ (x) = u(λx), λ > 0,

n n 1+ q − p kukLq(Rn) ≤ Cλ k∇ukLp(Rn,Rn). This implies that we must have necessarily p < n and np q = p∗ = . n − p

Lemma 1.10. () For 1 ≤ p < n, there exists a constant c = n c(p,n) > 0 so that, for every u ∈ D(R ),

kuk p∗ n ≤ ck∇uk p n n . L (R ) L (R ,R ) 32 1 Sobolev spaces

n Proof. We prove by induction in n that, for any u ∈ D(R ),

n 1/n kukn/(n−1) ≤ ∏ k∂ juk1 . (1.9) j=1

If n = 2, we have

Z x1 Z x2 u(x) = u(x1,x2) = ∂1u(t,x2)dt = ∂2u(x1,s)ds. −∞ −∞ Whence Z x Z x 2 1 2 |u(x)| ≤ |∂1u(t,x2)|dt |∂2u(x1,s)|ds −∞ −∞ Z Z ≤ |∂1u(t,x2)|dt |∂2u(x1,s)|ds. R R

Integrating side by side each member of the preceding inequality over R2. We obtain

kuk2 ≤ k∂1uk1k∂2uk1.

n+ Assume now that (1.9) is valid until some n ≥ 2. If u ∈ D(R 1) then, for any t ∈ R,

Z (n−1)/n n Z 1/n n/(n−1) |u(x,t)| dx ≤ |∂ ju(x,t)|dx . n ∏ n R j=1 R

We find by applying generalized Holder’s¨ inequality

Z Z (n−1)/n n n/(n−1) 1/n dt |u(x,t)| dx ≤ k∂ juk . (1.10) n n ∏ 1 R R j=1

R t On the other hand, since u(x,t) = −∞ ∂n+1u(x,s)ds, we have

Z 1/n (n+1)/n |u(x,t)| ≤ ∂n+1u(x,s)ds |u(x,t)|. R Holder’s¨ inequality then yields

Z Z (n−1)/n (n+1)/n 1/n n/(n−1) |u(x,t)| dx ≤ k∂n+1uk1 |u(x,t)| dx . Rn Rn

Integrating over R with respect to t and using (1.10) in order to obtain

n+1 (n+1)/n 1/n kuk(n+1)/n ≤ ∏ k∂ juk1 . j=1

That is 1.6 Imbedding Theorems 33

n 1/(n+1) kuk(n+1)/n ≤ ∏ k∂ juk1 . j=1

n Fix u ∈ D(R ) and λ > 1. Inequality (1.9) applied to |u|λ and Holder’s¨ inequality give n λ λ−1 1/n kukλn/(n−1) ≤ λkuk(λ−1)p0 ∏ kD jukp . j=1

λ λ−1  Note that ∂ j|u| = λ|u| ∂ ju . The choice of λ satisfying

λn/(n − 1) = (λ − 1)p0 yields n 1/n kukp∗ ≤ λ ∏ k∂ jukp ≤ ck∇ukp, j=1 which is the expected inequality. ut

Lemma 1.11. (Morrey inequality) Let n < p < ∞ and λ = 1 − n/p. There exists a n n constant c = c(p,n) > 0 so that, for every u ∈ D(R ) and any x,y ∈ R , we have

λ |u(x) − u(y)| ≤ c|x − y| k∇ukLp(Rn,Rn),

kuk ≤ ckuk 1,p n . ∞ W (R ) Proof. Let Q be a cube containing 0 and having each side parallel to axes and is of n length r. Let u ∈ D(R ). For x ∈ Q, we have

Z 1 u(x) − u(0) = ∇u(tx) · xdt. 0 Hence Z 1 n n Z 1 |u(x) − u(0)| ≤ ∑ |∂ ju(tx)||x j|dt ≤ r ∑ |∂ ju(tx)|dt. 0 j=1 j=1 0 If 1 Z m(u,Q) = u(x)dx, |Q| Q we get by integrating the last inequality over Q

r Z n Z 1 |m(u,Q) − u(0)| ≤ dx ∑ |∂ ju(tx)|dt |Q| Q j=1 0 1 Z 1 n Z ≤ n−1 dt ∑ |∂ ju(tx)|dx r 0 j=1 Q 1 Z 1 n Z dy ≤ n−1 dt ∑ |∂ ju(y)| n . r 0 j=1 tQ t 34 1 Sobolev spaces

Observe that, as Q is convex, we have tQ = tQ+(1−t){0} ⊂ Q. We obtain then by applying Holder’s¨ inequality

0 n Z 1 (tr)n/p nrλ p n p n |m(u,Q) − u(0)| ≤ n−1 k∇ukL (Q) n dt = k∇ukL (Q) . r 0 t λ n By making a translation, we can substitute 0 by an arbitrary x ∈ R in such a way that nrλ |m(u,Q) − u(x)| ≤ k∇uk p n . (1.11) λ L (Q) We find by taking r = 1 in this inequality n |u(x)| ≤ |m(u,Q)| + k∇uk p n ≤ C kuk 1,p ≤ C kuk 1,p n . λ L (Q) 0 W (Q) 0 W (R ) n Let x,y ∈ R . Then r = 2|x − y| in (1.11) gives |u(x) − u(y)| ≤ |m(u,Q) − u(x)| + |m(u,Q) − u(y)| 1+λ n2 λ ≤ |x − y| k∇uk p n λ L (Q) λ ≤ C1|x − y| kDukLp(Rn). The proof is then complete. ut

n n Define C0(R ) = {u ∈ C(R ); u(x) → 0 as |x| → +∞} and

n C0(Ω) = {u|Ω ; u ∈ C0(R )}.

n Theorem 1.20. (Sobolev imbedding theorem) Let Ω be an open subset of R of class C1 with bounded boundary. (i) If 1 ≤ p < n and if p ≤ q ≤ p∗ then W 1,p(Ω) ⊂ Lq(Ω) and the imbedding is continuous. 1,p (ii) If n < p < ∞ and λ = 1−n/p then W (Ω) ⊂ C0(Ω), the imbedding is contin- uous and there exists a constant c = c(p,n) > 0 so that, for every u ∈ W 1,p(Ω) and any x,y ∈ Ω, we have

λ |u(x) − u(y)| ≤ c|x − y| kukW 1,p(Ω).

1,p n Proof. Let 1 ≤ p < n and u ∈ W (R ). By Theorem 1.14, we find a sequence (um) n ,p n in D(R ) converging to u in W 1 (R ). Sobolev’s inequality then gives

kum − u`kLp∗(Rn) ≤ ck∇(um − u`)kLp(Rn)n . p∗ n p n Hence (um) is a Cauchy sequence L (R ). As um → u in L (R ), we deduce that p∗ n um → u in L (R ). Therefore

kukLp∗(Rn) ≤ ck∇ukLp(Rn,Rn). 1.6 Imbedding Theorems 35

Let P be the extension operator corresponding to Ω and v ∈ W 1,p(Ω). Then

kvkLp∗(Ω) ≤ kPvkLp∗(Rn) ≤ ck∇PvkLp(Rn)n ≤ c0kvkW 1,p(Ω).

If p ≤ q ≤ p∗, we define 0 ≤ λ ≤ 1 by

1 1 − λ λ = + q p p∗ and we apply the interpolation inequality in Proposition 1.3. We conclude that

1−λ λ λ kvk q ≤ kvk kvk ∗ ≤ c kvk 1,p . L (Ω) Lp(Ω) Lp (Ω) 0 W (Ω)

1,p n We proceed similarly for the case p > n. If u ∈ W (R ), we pick (um) a se- n ,p n n quence in D(R ) converging to u in W 1 (R ) and a.e. in R . We apply the Morrey inequality to um and pass then to the limit, as m → ∞. We obtain

λ n |u(x) − u(y)| ≤ c|x − y| k∇ukLp(Rn), a.e. x,y ∈ R . (1.12)

Now, substituting if necessary u by a continuous representative4, we may assume n n that u ∈ C0(R ) and the last inequality holds for any x,y ∈ R . We end up the proof by using, as in the previous case, the extension operator P corresponding to Ω. ut

One obtains by applying recursively Theorem 1.20 the following corollary.

n Corollary 1.2. Let Ω be an open subset of R of class C1 with bounded boundary. (i) If 1 ≤ p < n/m and if p ≤ q ≤ p∗ = np/(n − mp) then W m,p(Ω) ⊂ Lq(Ω) and the imbedding is continuous. m,p k (ii) If n/m < p < ∞,W (Ω) ⊂ C0(Ω), where k = [m − n/p],

k α C0(Ω) = {u; ∂ u ∈ C0(Ω) for each α ∈ N so that |α| ≤ k} and the embedding is continuous. In addition, if m − n/p in non integer, there exists a constant c = c(p,n,m) > 0 so that, for every u ∈ W m,p(Ω) and any x,y ∈ Ω,

α α λ |∂ u(x) − ∂ u(y)| ≤ c|x − y| kukW m,p(Ω) for all |α| = k, with λ = m − n/p − [m − n/p].

Prior to stating the Rellich-Kondrachov imbedding theorem, we prove the fol- lowing lemma. n Lemma 1.12. Let Ω be an open subset of R of class C1 with bounded boundary Γ , 1,1 ω b Ω and u ∈ W (Ω). For |y| < dist(ω,Γ ), we have

u − u ≤ |y|k uk 1 n . τy L1(ω) ∇ L (Ω,R ) n n n n 4 If A is a negligible set of R so (1.12) holds for any x,y ∈ R \ A then, as R \ A is dense in R , n u|Rn\A admits a unique continuous extension to R . 36 1 Sobolev spaces

Proof. As D(Ω) is dense in W 1,1(Ω), it is enough to prove the lemma when u ∈ D(Ω). In that case we have

Z 1 Z 1 τyu(x) − u(x) = ∇u(x −ty) · ydt ≤ |y| |∇u(x −ty)|dt. 0 0

Thus, where |y| < dist(ω,Γ ),

Z Z 1 kτyu − ukL1(ω) ≤ |y| dx |∇u(x −ty)|dt ω 0 Z 1 Z ≤ |y| dt |∇u(x −ty)|dx 0 ω Z 1 Z ≤ |y| dt |∇y(z)|dz 0 ω−ty

≤ |y|k uk 1 n ∇ L (Ω,R ) and hence the expected inequality holds. ut Theorem 1.21. (Rellich-Kondrachov imbedding theorem) Let Ω be a bounded open n subset of R of class C1. (a) If 1 ≤ p < n and if 1 ≤ q < p∗ then W 1,p(Ω) ⊂ Lq(Ω) and the imbedding is compact. (b) If n < p < ∞ then W 1,p(Ω) ⊂ C(Ω) and the imbedding is compact. Proof. (a) Let 1 ≤ p < n. We are going to show that B, the unit ball of W 1,p(Ω), satisfies the assumption of Theorem 1.4 in Lq(Ω) provided that 1 ≤ q < p∗. This will implies that B will be relatively compact in Lq(Ω). (i) From Theorem 1.20 and Proposition 1.4, we derive that

1/q−1/p∗ ∗ kukLq(Ω) ≤ kukLp (Ω)|Ω| ≤ C, for any u ∈ B.

(ii) Let ω b Ω and define 0 ≤ λ < 1 so that 1 1 − λ λ = + . q 1 p∗

If |y| < dist(ω,Γ ), the interpolation inequality in Proposition 1.3 and Lemma 1.12 yield, for every u ∈ B,

1−λ λ ∗ τyu − u Lq(ω) ≤ τyu − u L1(ω) τyu − u Lp (ω) λ 1−λ 1−λ   ≤ |y| k∇uk 2kuk p∗ L1(Ω,Rn) L (ω) ≤ c|y|1−λ , where we used that

1−1/p k uk 1 n ≤ k uk p n | | . ∇ L (Ω,R ) ∇ L (Ω,R ) Ω 1.6 Imbedding Theorems 37

5 (iii) Let ε > 0. There exists ω b Ω such that

1/q−1/p∗ 1/q−1/p∗ ∗ kukLq(Ω\ω) ≤ kukLp (Ω\ω)|Ω \ ω| ≤ c|Ω \ ω| ≤ ε. (b) Let p > n. We have from the Sobolev imbedding theorem

kukC(Ω) ≤ ckukW 1,p(Ω) for any u ∈ B, and λ λ |u(x) − u(y)| ≤ ckukW 1,p(Ω)|x − y| ≤ c|x − y| , for any x, y ∈ Ω and u ∈ B. This means that B satisfies the assumptions of Ascoli- Arzela’s theorem6. Whence B is relatively compact in C(Ω). ut

Theorem 1.22. (Poincare’s´ inequality) Let 1 ≤ p < ∞. If there exists an isometry n n n−1 1,p A : R → R so that A(Ω) ⊂ R ×]0,a[ then, for any u ∈ W0 (Ω), a kuk p ≤ k∇uk p n . L (Ω) 2 L (Ω,R ) Proof. Fix 1 < p < ∞. If u ∈ D(]0,a[) then Holder’s¨ inequality implies

0 1 Z a a1/p Z a 1/p |v(x)| ≤ |v0(x)|dx ≤ |v0(x)|pdx . 2 0 2 0 Hence Z a p/p0 Z a p Z a p a 0 p a 0 p |v(x)| dx ≤ p a |v (x)| dx = p |v (x)| dx. 0 2 0 2 0 According to our assumption on Ω, we may assume, without loss of generality, that n− Ω is of the form Ω = R 1×]0,a[. So, for u ∈ D(Ω), we deduce from the last inequality and Fubini’s theorem that Z Z Z a p 0 0 p |u| dx = dx |u(x ,xn)| dxn Ω Rn−1 0 p Z Z a p Z a 0 0 p a p ≤ p dx |∂nu(x ,xn)| dxn = p |∂nu| . 2 Rn−1 0 2 Ω 1,p The expected inequality follows by using that D(Ω) is dense in W0 (Ω).

5 One can take ω of the form

ω = Ωm = {x ∈ Ω; dist(x,Γ ) > 1/m}.

6 Ascoli-Arzela’s theorem. Let K = (K,d) be a compact metric space and let F be a bounded subset of C(K). Assume that F is uniformly equicontinuous, i.e. for any ε > 0, there exists δ > 0 so that d(x,y) < δ =⇒ |u(x) − u(y)| < ε, for any u ∈ F . Then F is relatively compact in C(K). 38 1 Sobolev spaces

The case p = 1 can be established analogously. ut

s n We close this chapter by some comments. The Sobolev spaces H (R ), s ∈ R, can be defined using the Fourier transform. With the help of local cards and a partition s n of unity, one can build the H spaces on a submanifold of R . The Sobolev spaces s n H (Ω), where Ω is an open subset of R , can constructed by interpolation from Hk(Ω) spaces. The reader is referred to the book by J.-L. Lions and E. Magenes [8] for more details. More generally we define the fractional order Sobolev space W s,p(Ω), 0 < s < 1 and 1 ≤ p < ∞, as follows

 | f (x) − f (y)|  W s,p(Ω) = f ∈ Lp(Ω); ∈ Lp(Ω × Ω) . |x − y|s+n/p

Observe that W s,p(Ω) can be seen as an interpolated space between W 1,p(Ω) and Lp(Ω). For arbitrary non integer s > 1, we set n o W s,p(Ω) = f ∈ W k,p(Ω); ∂ α f ∈ W t,p(Ω) for any |α| = k , where k = [s] is the integer part of s and t = s − [s]. Again, using a partition of unity and local cards, one can define W s,p spaces on a n submanifold of R . The reader can find in the book by P. Grisvard [2, Chapter 1] a detailed study of the W s,p spaces.

1.7 Exercises and problems

n 1.1. Let 1 ≤ p < q < ∞ and Ω be an open subset of R satisfying |Ω| < ∞. Show that if u ∈ Lq(Ω) then u ∈ Lp(Ω) and

1/p−1/q kukp ≤ |Ω| kukq.

1.2. (Generalized Holder’s¨ inequality) Let 1 < p j < ∞, 1 ≤ j ≤ k, so that 1 1 + ... = 1 p1 pk

p j k 1 and let u j ∈ L (Ω), 1 ≤ j ≤ k. Prove that ∏ j=1 u j ∈ L (Ω) and

Z k k ∏ |u j|dx ≤ ∏ ku jkp j . Ω j=1 j=1

1.3. (Interpolation inequality) Let 1 ≤ p < q < r < ∞ and 0 < λ < 1 given by 1.7 Exercises and problems 39 1 1 − λ λ = + . q p r

Prove that if u ∈ Lp(Ω) ∩ Lr(Ω) then u ∈ Lq(Ω) and

1−λ λ kukq ≤ kukp kukr .

n 1.4. (Support of measurable function) Let Ω be an open subset of R and f : Ω → R be a measurable function. Assume that there exists a family (ωi)i∈I of open subsets of Ω so that, for each i ∈ I, f = 0 a.e. in ωi. Set ω = ∪i∈Iωi. Prove that there exists a countable set J ⊂ I such that ω = ∪i∈Jωi. Conclude then that f = 0 a.e. in ω. If ω is the union of all open subset of Ω in which f = 0 a.e., the closed set supp( f ) = Ω \ ω is called the support of the measurable function f .

n p n 1.5. Let f ∈ L1(R ) and g ∈ L (R ), 1 ≤ p ≤ ∞. n n a) Prove that y 7→ f (x − y)g(y) is absolutely integrable in R , a.e. x ∈ R . Define the convolution product of f and g by Z ( f ∗ g)(x) = f (x − y)g(y)dy. Rn p n Show that f ∗ g ∈ L (R ) and

k f ∗ gkp ≤ k f k1kgkp.

Hint: consider first the case p = 1 for which we can apply Tonelli’s theorem7. In a second step, reduce the case 1 < p < ∞ to that of p = 1. b) Demonstrate that supp( f ∗ g) ⊂ supp( f ) + supp(g).

1.6. Prove that the function xα admits a weak derivative belonging to L2(]0,1[) if and only if α > 1/2.

n 1.7. Let Ω be an open bounded R so that there exists a sequence (Ωi)1≤i≤k of open k subsets that are pairwise disjoint and Ω = ∪i=1Ωi. Assume moreover that, for each 1 i, Ωi is piecewise of class C . Set n o C1 (Ω,(Ω ) ) = u : Ω → ; u ∈ C1(Ω ), 1 ≤ i ≤ k . pie i 1≤i≤k R |Ωi i

1 (a) Prove that any function from C(Ω) ∩Cpie (Ω,(Ωi)1≤i≤k) admits a weak deriva- tive belonging to L2(Ω). 1 (b) Does an arbitrary function from Cpie (Ω,(Ωi)1≤i≤k) admits a weak derivative belonging to L2(Ω)?

7 n Tonelli’s theorem For i = 1,2, let Ωi be an open subset of R i . Let h : Ω1 × Ω2 → R be a measurable function so that R |h(x,y)|dy < ∞, a.e. x ∈ Ω , and R dxR |h(x,y)|dy < ∞. Then Ω2 1 Ω1 Ω2 1 h ∈ L (Ω1 × Ω2). 40 1 Sobolev spaces

n 1.8. Denote by Br the ball of R with center 0 and radius r. α 1  (a) Let n = 2. Prove that the function u(x) = |ln|x|| belongs to H B1/2 for 0 < α < 1/2, but its is unbounded. −β 1 (b) Assume that n ≥ 3. Show that the function u(x) = |x| is in H (B1) provided that 0 < β < (n − 2)/2, but its is unbounded.

n 1.9. Let B be the unit ball of R . ,p ,p (a) Compute the values of α 6= 0 for which |x|α ∈W 1 (B) (resp. |x|α ∈W 1 (R\B)). (b) Show that x/|x| ∈ W 1,p(B)n if and only if p < n.

1.10. Let a, b ∈ R and u ∈ H1(]a,b[). a) For x, y ∈]a,b[, show that

Z b u(x)2 + u(y)2 − 2u(x)u(y) ≤ (b − a) u0(x)2dx. a b) Integrate with respect to x and then with respect to y to deduce that there exists a constant c > 0 so that

Z b Z b Z b 2! u(x)2dx ≤ c u0(x)2dx + u(x)dx . a a a

1 p−1 1.11. (a) Let v ∈ Cc (R) and G(s) = |s| s, s ∈ R, with 1 ≤ p < ∞. Let w = G(v) 1  R x 0 ∈ Cc (R) . Use the relation w(x) = −∞ w (t)dt to show that

1/p 1/p0 0 1/p 1/e 1/p0 0 1/p |v(x)| ≤ p kvkp kv kp ≤ e kvkp kv kp 1/e 0 ≤ e (kvkp + kv kp).

(b) Deduce that there exists a constant c ≥ 0 so that, for any 1 ≤ p < ∞ and any ,p u ∈ W 1 (R), we have kuk ∞ ≤ ckuk 1,p . L (R) W (R) 1.12. Let n ≥ 2 and 1 ≤ p < n. Set p∗ = np/(n − p) and q = p(n − 1)/(n − p). n Demonstrate that, for every u ∈ D(R ), Z 0 q 0 q−1 |u(x ,0)| dx ≤ qkukp∗ k∂nukp. Rn−1

1.13. Let Ω be the open subset of R2 given by 0 < x < 1 and 0 < y < xβ with β > 2. Let v(x) = xα . Prove that v ∈ H1(Ω) if and only if 2α + β > 1 ; while v ∈ L2(∂Ω) if and only if 2α > −1. Conclude.

1.14. (a) Prove the Caffarelli-Kohn-Nirenberg’s theorem: let 1 < p < ∞ and α +n > n 0. Then, for any u ∈ D(R ), Z p Z p Z p α p p α p p α+p |u| |x| dx ≤ p |x · ∇u| |x| dx ≤ p |∇u| |x| dx. Rn (α + n) Rn (α + n) Rn 1.7 Exercises and problems 41

Hint: Regularize |x|α to show that div(|x|α x) = (α + n)|x|α in the weak sense. ,p n (b) Deduce the Hardy’s inequality: let 1 < p < n. Then, for every u ∈ W 1 (R ), we p n have u/|x| ∈ L (R ) and

u p ≤ k∇ukp. |x| p n − p

n ,p n 1.15. (a) Let u ∈ L1(R ) and v ∈ W 1 (R ) with 1 ≤ p ≤ ∞. Prove that

1,p n u ∗ v ∈ W (R ) and ∂i(u ∗ v) = u ∗ ∂iv, 1 ≤ i ≤ n.

n Let Ω be an open subset of R . For a function u defined on Ω, denote by u its extension by 0 outside Ω, i.e.

 u(x) if x ∈ Ω, u(x) = n 0 if x ∈ R \ Ω.

1,p (b) Let u ∈ W (Ω), um = ρm ∗ u, ω b Ω and ϕ ∈ D(Ω) satisfying 0 ≤ ϕ ≤ 1 and ϕ = 1 in a neighborhood of ω. (i) Show the following claims:

ρm ∗ ϕu = ρm ∗ u in ω if m sufficiently large. p n ∂i(ρm ∗ ϕu) → ϕ∂iu + ∂iϕu in L (R ). Deduce that p ∂i(ρm ∗ u) → ∂iu in L (ω). (ii) Prove the Friedrichs’s theorem: let u ∈ W 1,p(Ω) with 1 ≤ p < ∞. Then there n exists a sequence (um) in D(R ) so that, for any ω b Ω, p p n um|Ω → u in L (Ω), ∇um|ω → ∇u|ω in L (ω) .

(c) Let u,v ∈ W 1,p(Ω) ∩ L∞(Ω) with 1 ≤ p ≤ ∞. Show that uv ∈ W 1,p(Ω) ∩ L∞(Ω) and ∂i(uv) = ∂iuv + u∂iv, 1 ≤ i ≤ n.

1,p 1,p 1.16. In this exercise we only use the definition of W0 (Ω), that is W0 (Ω) is the ,p n closure of D(Ω) in W 1 (Ω). Let Ω be an open subset of R of class C1 having bounded boundary Γ and 1 ≤ p < ∞. Let G ∈ C1(R) so that |G(t)| ≤ t, t ∈ R, and G(t) = 0 if |t| ≤ 1, G(t) = t if |t| ≥ 2.

1,p 1,p (a) Let u ∈ W (Ω). If u has a compact support, show then that u ∈ W0 (Ω). (b) Let u ∈ W 1,p(Ω) ∩C(Ω) satisfying u = 0 on Γ . (i) Assume that u has a compact support and set um = G(mu)/m. Check that um ∈ 1,p 1,p W (Ω), um → u in W (Ω) and

supp(um) ⊂ {x ∈ Ω; |u(x)| ≥ 1/m}. 42 1 Sobolev spaces

1,p 1,p Deduce that um ∈ W0 (Ω) (and hence u ∈ W0 (Ω)). (ii) Show that the result in (i) still holds without the assumption that u has a compact support. (c) Prove that

1,p 1,p 8 W0 (Ω) ∩C(Ω) = {u ∈ W (Ω) ∩C(Ω); γ0u = 0} , where γ0 is the trace operator introduced in Theorem 1.18.

1.17. (a) Let ϕ ∈ D(]0,1[). Prove the following inequalities:

Z 1/2 |ϕ(x)|2 ≤ x |ϕ0(t)|2dt, x ∈ [0,1/2], 0 Z 1 |ϕ(x)|2 ≤ (1 − x) |ϕ0(t)|2dt, x ∈ [1/2,1]. 1/2

Deduce then that Z 1 Z 1 2 1 0 2 1 |u(x)| dx ≤ |u (x)| dx, for any u ∈ H0 (]0,1[). 0 8 0 Set ( ) R 1 |u(x)|2dx C = sup 0 ; u ∈ H1(]0,1[), u 6= 0 R 1 0 2 0 0 |u (x)| dx and consider the boundary value problem, where f ∈ L2(]0,1[),

 −u00(x) − ku(x) = f (x), x ∈]0,1[, (1.13) u(0) = u(1) = 0.

2 1 A solution of (1.13) is a function u ∈ H (]0,1[)∩H0 (]0,1[) satisfying the first iden- tity in (1.13) a.e. x ∈]0,1[. (b) Prove that if kC < 1 then (1.13) has at most one solution. Hint: as a first step, we can show that if u is a solution of (1.12) then, for every ϕ ∈ D(]0,1[),

Z 1 Z 1 Z 1 u0(x)ϕ0(x)dx − k u(x)ϕ(x)dx = f (x)ϕ(x)dx. 0 0 0 (c) Compute the non trivial solutions, for k 6= 0, of the boundary value problem

u00(x) + ku(x) = 0, x ∈]0,1[ and u(0) = u(1) = 0, and deduce from it that

8 We have in fact (see Proposition 1.14)

1,p 1,p W0 (Ω) = {u ∈ W (Ω); γ0u = 0}. 1.7 Exercises and problems 43 1 1 ≤ C ≤ . π2 8 1.18. In this exercise, I =]a,b[ is an interval and 1 ≤ p ≤ ∞. 1 (a) Let g ∈ Lloc(I). Fix c ∈ I and set Z x f (x) = g(t)dt, x ∈ I. c

(i) Prove that f ∈ C(I). Hint: use Lebesgue’s dominated convergence theorem. (ii) Let ϕ ∈ D(I). With the aid of the identity

Z Z c Z c Z b Z x f ϕ0dx = − dx g(t)ϕ0(x)dt + dx g(t)ϕ0(x)dt, I a x c c and Fubini’s theorem, demonstrate that Z Z f ϕ0dx = − gϕdx. I I

(b) Let u ∈ W 1,p(I). Use (a) to show that there existsu ˜ ∈ C(I) so that u = u˜ a.e. in I and Z y u˜(y) − u˜(x) = u0(t)dt for all x,y ∈ I. x (c) Show that, if I is bounded and 1 < p ≤ ∞, then the imbedding W 1,p(I) ,→ C(I) is compact. Hint: use Arzela-Ascoli’s theorem. n 1.19. (Poincare-Wirtinger´ inequality) Let Ω be a bounded domain of R . Prove that there exists a constant C > 0, only depending on Ω, so that, for every u ∈ H1(Ω), we have ku − uk 2 ≤ Ck uk 2 n , L (Ω) ∇ L (Ω,R ) where 1 Z u = u(x)dx. |Ω| Ω Hint: Show first that it is enough to establish the above inequality when u = 0. Proceed then by contradiction. / n 1.20. (The space H1 2(Γ )) Let Ω be a bounded domain of R of class C1 and recall 1 2 that the trace operator γ0 : H (Ω) → L (Γ ), defined by γ0(u) = u|Γ , u ∈ D(Ω), is bounded. Set 1/2 1  H (Γ ) = γ0 H (Ω) . Define on H1/2(Γ ) the quotient norm

1 1/2 kvkH1/2(Γ ) = min{kukH1(Ω); u ∈ H (Ω) and γ0(u) = v}, v ∈ H (Γ ).

1/2 1 Prove that, for any v ∈ H (Γ ), there exists a unique uv ∈ H (Ω) so that kvkH1/2(Γ ) = kuvkH1(Ω). 44 1 Sobolev spaces References

1. Brezis´ H. : Analyse fonctionnelle, theorie´ et applications, 2eme` tirage. Masson, Paris, 1987. 2. Grisvard, P. : Elliptic problems in nonsmooth domains. Pitman, Boston, 1985. 3.H ormander¨ L. : The analysis of linear partial I, 2nd ed.. Springer, Berlin, 1990. 4. Lions J.-L. et Magenes E. : Problemes` aux limites non homogenes` et applications, Vol. I. Dunod, Paris,1968. 5. Willem M. : Analyse fonctionnelle el´ ementaire.´ Cassini, Paris, 2003. 6. Zuily C.: El´ ements´ de distributions et equations´ aux deriv´ ees´ partielles. Dunod, Paris, 2002. Chapter 2 Variational solutions

This chapter is mainly devoted to study existence and uniqueness of variational so- lutions of elliptic partial differential equations. It contains also some classical prop- erties of weak solutions of elliptic equations. Amongst the properties we establish, there are the maximum principle, Harnack inequalities and the unique continuation across a non characteristic hypersurface. This chapter can be completed by the following classical textbooks [1, 2, 5, 4, 5, 6, 7, 8, 10, 11, 12, 13, 14]. We just quote these few references, but of course there are many other excellent textbooks dealing with the analysis of elliptic partial differential equations. Section 2.1, Section 2.2, Subsections 2.3.2 and 2.3.3 are largely inspired by the book of Brezis´ [2]. Subsections 2.4.1 and 2.4.2 were based on the book of Gilbarg- Trudinger, while Subsection 2.4.3 was prepared from both the book of Benilan´ [1] and Gilbarg-Trudinger [5].

2.1 Stampacchia’s theorem and Lax-Milgram lemma

We recall the projection theorem on closed convex set of a .

Theorem 2.1. Let H be a real Hilbert space with scalar product (·|·), and let K be a closed nonempty convex subset of H. (i) For any u ∈ H, there exists a unique PKu ∈ K so that

ku − PKuk = minku − vk, v∈K where k · k is the norm associated to the scalar product (·|·). Moreover PKu is char- acterized by

PKu ∈ K and (u − PKu|v − PKu) ≤ 0 for any v ∈ K.

45 46 2 Variational solutions

PKu is called the projection of u on K. (ii) PK : u ∈ H → PKu ∈ H is a contractive operator, i.e.

kPKu − PKvk ≤ ku − vk for all u,v ∈ H.

The projection on a closed subspace is characterized by the following proposi- tion. Proposition 2.1. Let H be as in Theorem 2.1 and let E be a closed subspace of H. If u ∈ H then PE u is characterized by

PE u ∈ E and (u − PE u,v) = 0 any v ∈ E.

Furthermore, PE is a linear operator.

We recall that a bilinear form a : H × H → R is continuous if and only if there exists a constant C > 0 so that

|a(u,v)| ≤ Ckukkvk for any u,v ∈ H.

The bilinear form a is said coercive if we can find α > 0 with the property that

a(u,u) ≥ αkuk2 for every u ∈ H.

Theorem 2.2. (Stampacchia’s theorem) Let H be a real Hilbert space with scalar product (·|·). Let a be a coercive and continuous bilinear form on H × H and K be a closed convex subset of H. For every Φ ∈ H0, there exists a unique u ∈ K so that

a(u,v − u) ≥ Φ(v − u) for any v ∈ K. (2.1)

Moreover, if a symmetric then u is characterized by

1 1  u ∈ K and a(u,u) − Φ(u) = min a(v,v) − Φ(v)i . 2 v∈K 2

Proof. By Riesz-Frechet’s´ representation theorem1, we find a unique f ∈ H so that

hΦ,vi = ( f ,v) for any v ∈ H.

On the other hand, for arbitrary fixed u ∈ H, the mapping v → a(u,v) is linear contin- uous form on H. Therefore, again by Riesz-Frechet’s´ representation theorem, there exists Au ∈ H such that a(u,v) = (Au,v) for every v ∈ H. It is then not hard to check that A is a linear operator from H into H and satisfies

1 Riesz-Frechet’s´ representation theorem. Let H be a real Hilbert space with scalar product (·|·). If Φ ∈ H0 then there exists a unique f ∈ H so that

Φ(v) = ( f ,v) for any v ∈ H.

Furthermore, kΦk = k f k. 2.1 Stampacchia’s theorem and Lax-Milgram lemma 47

kAuk ≤ Ckuk, for any u ∈ H, (2.2) and (Au,u) ≥ αkuk2, for any u ∈ H. (2.3) In consequence, the problem (2.1) is reduced to find u ∈ K so that

(Au,v − u) ≥ ( f ,v − u), for any v ∈ K. (2.4)

Let θ > 0 be a constant that we will fix later in the proof and note that inequality (2.4) is equivalent to the following one.

([θ f − θAu + u] − u,v − u) ≤ 0 for any v ∈ K. (2.5)

That is u = PK(θ f − θAu + u). For v ∈ K, set Tv = PK(θ f − θAv + v). Then we are reduced to show that T has a unique fixed point. We have, according to Theorem 2.1,

kTv − Twk ≤ k(v − w) − θ(Av − Aw)k for all v,w ∈ K.

Hence

kTv − Twk2 ≤ kv − wk2 − 2θ(Av − Aw,v − w) + θ 2kAv − Awk2 ≤ kv − wk2(1 − 2θα + θ 2C2), where we used (2.2) and (2.3). The value of θ that minimize 1 − 2θα + θ 2C2 is equal α/C2. We have for this choice of θ

kTv − Twk ≤ kkv − wk for all v,w ∈ K.

Here k = (1 − α2/C2)1/2 < 1. By Banach’s fixed point theorem2, T has a unique fixed point u ∈ K, that is u = Tu. Assume next that a symmetric. Then (u,v) → a(u,v) defines a new scalar product on H and the associated norm a(u,u)1/2 is equivalent to the initial norm on H. Therefore H is a Hilbert space with respect to this new norm. Once again, in light of Riesz-Frechet’s´ theorem, we find f ∈ H so that

2 Banach’s fixed point theorem. On a complete metric space M with metric d, let T : M → M be so that there exists a constant 0 < k < 1 with the property that

d(Mx,My) ≤ kd(x,y) for any x,y ∈ M.

Then T has a unique fixed point u, i.e. u = Tu. 48 2 Variational solutions

hΦ,vi = a(g,v) for any v ∈ H.

As a consequence of this, (2.1) takes the form

a(g − u,v − u) ≤ 0 for any v ∈ H. (2.6)

Then, according to Theorem 2.1, (2.6) is reduced to find u ∈ K satisfying

mina(g − v,g − v)1/2, v∈K which is the same as minimizing, over K, a(g − v,g − v) or equivalently a(v,v)/2 − a(g,v). ut

As an immediate consequence of Theorem 2.2, we have the following corollary.

Corollary 2.1. (Lax-Milgram’s lemma) Let H be a real Hilbert space with scalar product (·|·) and let a be a bilinear continuous and coercive form on H × H. Then, for any Φ ∈ H0, there exists a unique u ∈ H so that

a(u,v) = Φ(v) for every v ∈ H.

Furthermore, if a is symmetric then u is characterized by

1 1  a(u,u) − Φ(u) = min a(u,u) − Φ(u) . 2 v∈H 2

2.2 Elements of the spectral theory of compact operators

In this section, E, F are two Banach spaces and U denotes the unit ball of E. An operator A ∈ L (E,F) is said compact whenever A(U) is relatively compact. The subset of L (E,F) consisting in compact operators is denoted by K (E,F). For simplicity convenience we set K (E) = K (E,E).

Theorem 2.3. K (E,F) is a closed subspace of L (E,F).

Proof. Note first that is not difficult to see that K (E,F) is a subspace of L (E,F). Next, let A ∈ L (E,F) be the limit in L (E,F) of a sequence (Ak) in K (E,F). For an arbitrary ε > 0, we are going to show that A(U) can be covered by finite number of ball B(yi,ε) in F, implying, since F is complete, that A(U) is relatively compact. Fix k so that kAk − Ak < ε/2. As Ak(U) is relatively compact, Ak(U) ⊂ ∪i∈IB( fi,ε/2), where I is finite. We deduce from this that A(U) ⊂ ∪i∈IB( fi,ε). ut

We say that A ∈ L (E,F) is of finite rank if dimR(A) < ∞, where R(A) is the range of A. Observing that finite rank operators are compact, we get immediately from Theorem 2.3 the following corollary. 2.2 Elements of the spectral theory of compact operators 49

Corollary 2.2. If A ∈ L (E,F) is the limit in L (E,F) of finite rank operators, then A ∈ K (E,F).

It is straightforward to check that the composition of bounded operator and com- pact operator is again a . We have precisely the following result.

Proposition 2.2. Let G be another Banach space. If A ∈ L (E,F) and B ∈ K (F,G) or A ∈ K (E,F) and B ∈ L (F,G), then BA ∈ K (E,G).

Define the adjoint of an operator A ∈ L (E,F) as the unique operator A∗ ∈ L (F0,E0) given by the relation

hv|Aui = hA∗v|ui for any u ∈ E and v ∈ F0.

Here h·|·i denotes the duality pairing both between E and E0 and between F and F0.

Theorem 2.4. A ∈ K (E,F) if and only if A∗ ∈ K (F0,E0).

Proof. Assume that A ∈ K (E,F). If U0 is the unit ball of F0, we are going to prove ∗ 0 0 0 that A (U ) is relatively compact in E . We pick a sequence (vm) in U and we show ∗ that (A (vm)) admits a convergent subsequence. Consider then the compact metric space M = A(U) and K ⊂ C(M) given by

K = {ϕm; ϕm(x) = hvm|xi, x ∈ M}.

It is easy to check that K possesses the assumption of Arzela-Ascoli’s theorem. Hence, we can subtract from (ϕm) a subsequence (ϕk) converging to ϕ ∈ C(M). We have in particular

sup |hvk|Aui − ϕ(Au)| → 0 when k → +∞, u∈U from which we obtain

sup |hvk|Aui − hv`|Aui| → 0 if k,` → +∞. u∈U

∗ ∗ 0 ∗ That is kA vk − A v`k → 0 as k,` → +∞. Whence, since E is complete, (A vk) converges in E0. Conversely, assume that A∗ ∈ K (F0,E0). From the previous step, we obtain that A∗∗ ∈ K (E00,F00) and consequently A∗∗(U) is relatively compact in F00. But A(U) = A∗∗(U) and F is identified isomophicly and isometrically to a subspace of F00. Thus A(U) is relatively compact in F. ut

Recall that the kernel of A ∈ L (E,F) is defined by

N(A) = {u ∈ E; Au = 0}.

For X ⊂ E, we use in the sequel the notation 50 2 Variational solutions

X⊥ = {ϕ ∈ E0; hϕ|ui = 0 for any u ∈ X}.

Similarly, if Φ ⊂ E0 we set

Φ⊥ = {u ∈ E; hϕ|ui = 0 for any ϕ ∈ Φ}.

Theorem 2.5. (Fredholm’s alternative) Let A ∈ K (E). Then the following asser- tions hold. (a) N(I − A) is of finite dimension. (b) R(I − A) = N(I − A∗)⊥ and therefore R(I − A) is closed. (c) N(I − A) = {0} if and only if R(I − A) = E. (d) dim(I − A) = dim(I − A∗). Fredholm’s alternative theorem is useful tool to solve the equation

u − Au = f . (2.7)

Theorem 2.5 says that we have the following alternative. • For any f ∈ E, (2.7) has unique solution, • or else u − Au = 0 admits p linearly independent solutions and in that case (2.7) is solvable if and only f satisfies p orthogonality conditions, which means precisely that f ∈ N(I − A∗)⊥. The following theorem will be used in the proof of Fredholm’s alternative. Theorem 2.6. (Riesz’s theorem) If U is compact then E is of finite dimension. We need to introduce the notion of topological supplement. Let G be a closed subspace of E. We say that a subspace L of E is a topological supplement of G if L is closed, G ∩ L = {0} and G + L = E. In that case any z ∈ E has a unique decomposition z = x + y with x ∈ G and y ∈ L. One can check that the projectors z → x et z → y define linear bounded operators. We know that any subspace of finite dimension or finite co-dimension admits a topological supplement. On the other hand one can check that any closed subspace of Hilbert space possesses a topological supplement. We shall need also the following result in the proof of Theorem 2.5.

Theorem 2.7. Let A ∈ L (E,F). The following assertions are equivalent. (a) R(A) is closed. (b) R(A∗) is closed. (c) R(A) = N(A∗)⊥. (d) R(A∗) = N(A)⊥.

Proof (of Theorem 2.5). (a) If E1 = N(I − A) and if U1 is the unit ball of E1, then U1 ⊂ A(U). Whence E1 is of finite dimension by Riesz’s theorem. (b) Let ( fm) be a sequence in R(I − A) with fm = um − Aum, for each m, that converges to f ∈ E. We want to check that f ∈ R(I − A). To this end, let dm = dist(um,N(I −A)). As N(I −A) is of finite dimension, there exists vm ∈ N(I −A) so that dm = kum − vmk. Note that we have 2.2 Elements of the spectral theory of compact operators 51

fm = (um − vm) − A(um − vm). (2.8)

We claim that kum − vmk is bounded. Otherwise, (um − vm) would admit a subse- quence (uk − vk) so that kuk − vkk → ∞ as k → ∞. Let

uk − vk wk = . kuk − vkk

From (2.8), wk −Awk → 0, as k → ∞. Since A is compact, subtracting again if neces- sary a subsequence, we may assume that Awk → z. Hence wm → z and z ∈ N(I −A). On the other hand, we have

dist(uk,N(I − A)) dist(wk,N(I − A)) = = 1. kuk − vkk

Passing to the limit, as k → ∞, to deduce that dist(z,N(I − A)) = 1 which is im- possible. That is we proved that kum − vmk is bounded and using once more that A is compact to conclude that there exists a subsequence (A(uk − vk)) converging to h ∈ E. This and (2.8) entail that uk − vk → f + h. Whence, if g = f + h then g − Ag = f . In other words, we proved that f ∈ R(I − A) and hence R(I − A) is closed. We deduce from Theorem 2.7 that

R(I − A) = N(I − A∗)⊥ and R(I − A∗) = N(I − A)⊥.

(c) We first prove that N(I − A) = {0} implies that R(I − A) = E. We proceed again by contradiction. To this end, we assume that

E1 = R(I − A) 6= E.

We have that E1 is a Banach space and A(E1) ⊂ E1. Hence, A|E1 ∈ K (E1) and E2 = (I − A)(E1) is a closed subspace of E1. Moreover E2 6= E1 because I − A is m injective. Let Em = (I − A) (E). Then (Em) is a sequence of strictly decreasing 3 closed subspaces. By Riesz’s lemma we find a sequence (um) satisfying um ∈ Em, kumk = 1 and dist(um,Em+1) ≥ 1/2. Thus

Au` − Aum = −(u` − Au`) + (um − Aum) + (u` − um).

If ` > m, we have E`+1 ⊂ E` ⊂ Em+1 ⊂ Em and consequently

−(u` − Au`) + (um − Aum) + u` ∈ Em+1.

Whence, kAu` − Aumk ≥ 1/2 which is impossible since A is compact. We end up concluding that R(I − A) = E. Conversely, if R(I − A) = E then Theorem 2.7 allows us to get that N(I − A∗) = R(I − A)⊥ = {0}. But since A∗ ∈ K (E0) the preceding result is applicable when A

3 Riesz’s lemma Let M be a closed subspace of E so that M 6= E. Then, for any ε > 0, there exists u ∈ E satisfying kuk = 1 and dist(u,M) ≥ 1 − ε. 52 2 Variational solutions is substituted by A∗. That is we have R(I − A∗) = E0. We get by applying one more time Theorem 2.7 that N(I − A) = R(I − A∗)⊥ = {0}. (d) Set d = dimN(I − A) and d∗ = dimN(I − A∗). Let us first show that d∗ ≤ d. We proceed once again by contradiction. Assume then that d < d∗. As N(I − A) is of finite dimension, it admits a topological supplement in E. This yields that there exists a bounded projector P from E into N(I − A). On the other hand, R(I − A) = N(I −A∗)⊥ has finite co-dimension d∗ and hence R(I −A) admits in E a topological supplement, denoted by L, of dimension d∗. As d < d∗ there exists an injective linear map Λ : N(I −A) → L which is non surjective. If B = A+ΛP then B ∈ K (E) because ΛP is of finite rank. Next, we prove that N(I − B) = {0}. If

0 = u − Bu = (u − Au) −ΛPu then

(I − A)u = 0 and ΛPu = 0 (because (I − A)u ∈ R(I − A) and ΛPu ∈ L).

Thus u = 0. We then apply (c) to B to conclude that R(I −B) = E. This not possible since there exists f ∈ L, f 6∈ R(Λ) ; the equation u−Bu = f does not have a solution Indeed, if it has a solution u then, as previously, we should have (I −A)u = ΛPu+ f . But (I − A)u ∈ R(I − A) and ΛPu + f ∈ L. Hence ΛPu + f = 0 because L is a topological supplement of R(I − A). That is, f = −ΛPu ∈ R(Λ) which is absurd. In other words d∗ ≤ d. This result applied to A∗, yields

dimN(I − A∗∗) ≤ dim(I − A∗) ≤ dimN(I − A).

We get by using N(I −A∗∗) ⊃ N(I −A) that d = d∗ note that A∗∗ is an extension of A : E ⊂ E00 → E00. ut

Define the resolvent set of A ∈ L (E) by

ρ(A) = {λ ∈ R; (A − λI) is bijective from E onto E}.4 5

The spectrum of σ(A) is the complement of the resolvent set, i.e. σ(A) = R \ ρ(A). We say that λ is an eigenvalue of A and write λ ∈ ev(A) if N(A − λI) 6= {0}. The subspace N(A − λI) is called the eigenspace associated to λ. We make the following remarks. • If λ ∈ ρ(A) then (A − λI)−1 ∈ L (E). • We have ev(A) ⊂ σ(A). Apart the case dim(E) < ∞ for which we have ev(A) = σ(A), the inclusion is in general strict. Indeed, one can find λ so that N(A − λI) = {0} and R(A − λI) 6= E. This is for instance the case when E = `2 and Au = (0,u1,...,um,...) when u = (u1,...,um,...) (the right shift operator).

4 For simplicity convenience we considered the resolvent set as a subset of R. But the more appro- priate framework should be to consider the resolvent set as a subset of C. 5 Note that if A − λI is bijective, then (A − λI)−1 ∈ L (E) by de Banach’s theorem. Banach’s theorem. Any bijective bounded operator between two Banach spaces admits a bounded inverse. 2.2 Elements of the spectral theory of compact operators 53

Proposition 2.3. σ(A) is compact with σ(A) ⊂ [−kAk,kAk], where kAk denotes the norm of A in L (E).

Proof. Let λ ∈ R so that |λ| > kAk. From Banach’s fixed point theorem, for any f ∈ E, there exists a unique u ∈ E so that u = (1/λ)(Au− f ), that is (A−λI)u = f . Hence A − λI is bijective and consequently σ(A) ⊂ [−kAk,kAk]. Next we show that ρ(A) is open. This will imply that σ(A) = R\ρ(A) is closed. Let λ ∈ ρ(A). If f ∈ E, solving the problem

Au − µu = f is equivalent to find a solution of the equation

u = (A − λI)−1((µ − λ)u + f ).

We deduce by applying once again Banach’s fixed point theorem that the last equa- tion has a unique solution whenever

|µ − λ|k(A − λI)−1k < 1.

That is  1 1  λ − ,λ + ⊂ ρ(A). k(A − λI)−1k k(A − λI)−1k This completes the proof. ut

Proposition 2.4. Let A ∈ K (E) with dim(E) = ∞. Then 0 ∈ σ(A) and σ(A)\{0} = ev(A) \{0}.

Proof. If 0 does not belong to σ(A), then A would be bijective and hence I = AA−1 would be compact. This would imply that U is compact and then, by Theorem 2.6, E would be of finite dimension. This leads to the expected contradiction. Let λ ∈ σ(A) \{0}. If λ is not an eigenvalue of A, we would have N(A − λI) = {0} and hence R(A−λI) = E by Fredholm’s alternative. That is we would have λ ∈ ρ(A) (by Banach’s theorem) contradicting the fact that λ belongs to the spectrum of A. ut

We now give a more precise description of the spectrum of compact operators. Prior to doing that we prove the following lemma.

Lemma 2.1. Let A ∈ K (E) and let (λm) be a sequence of distinct reals numbers so that λm → λ and λm ∈ σ(A) \{0}, for any m. Then λ = 0. In other words, the elements of σ(A) \{0} are isolated.

Proof. We know from Proposition 1.4 that λm ∈ ev(A). Let then em ∈ E, em 6= 0 so that (A − λmI)em = 0. Define Em = span{e1,...,em}. Let us prove Em ⊂ Em+1 strictly for each m. To this end, it is enough to check that e1,...,em are linearly independent. We proceed by induction in m. Assume that the result is true for some m m and that em+1 = ∑i=1 αiei. We have then 54 2 Variational solutions

m m Aem+1 = ∑ λiαiei = ∑ λm+1αiei. i=1 i=1

Hence αi(λi − λm+1) = 0, 1 ≤ i ≤ m. As λi’s are distinct, we derive that αi = 0, 1 ≤ i ≤ m, and consequently Em ⊂ Em+1 strictly for each m. On the other hand, it is clear that (A − λmI)Em ⊂ Em−1. By Riesz’s lemma we can find a sequence (um) satisfying um ∈ Em, kumk = 1 and dist(um,Em−1) ≥ 1/2, for each m ≥ 2. Let 2 ≤ m < ` in such a way that

Em−1 ⊂ Em ⊂ E`−1 ⊂ E`.

We have

Au` Aum Au` − λ`u` Aum − λmum − = − + u` − um λ` λm λ` λm

≥ dist(u`,E`−1) ≥ 1/2.

If λm → λ 6= 0 we get a contradiction since, by compactness, (Aum) admits a con- vergent subsequence. ut

Theorem 2.8. Let A ∈ K (E) with dim(E) = ∞. Then σ(A) = {0}, or else σ(A) \ {0} is finite, or else σ(A) \{0} consists in a sequence converging to 0. Proof. For m ≥ 1, σ(A) ∩ {λ ∈ R; |λ| ≥ 1/m} is empty or else it is finite. Otherwise, it would contain an infinite distinct points entailing, as σ(A) is compact, that this set has an accumulation point in σ(A) which contradicts Lemma 2.1. If the case where σ(A) \{0} consists in infinite points, we can order these points in a sequence converging to 0. ut

Our next objective is to provide a spectral decomposition of self-adjoint com- pact operators. We suppose then that E = H is a Hilbert space, with scalar product (·|·), and A ∈ L (H). Identifying H with its dual H0, we may consider that A∗ is an element of L (H). In this case, we say that A is self-adjoint if A∗ = A, i.e.

(Au|v) = (u|Av) for any u,v ∈ H.

Proposition 2.5. Let A ∈ L (H) be a self-adjoint operator and set

m = inf (Au|u) and M = sup (Au|u). u∈H, kuk=1 u∈H, kuk=1

Then σ(A) ⊂ [m,M] and m,M ∈ σ(A).

Proof. If λ > M then λ ∈ ρ(A). Indeed, from (Au|u) ≤ Mkuk2 for any u ∈ H, we get

([λI − A]u,u) ≥ (λ − M)kuk2 = αkuk2 for all u ∈ H, with α = λ − M > 0. 2.2 Elements of the spectral theory of compact operators 55

Hence λI − A is bijective according to Lax-Milgram’s lemma. Next, we show that M ∈ σ(A). We proceed by contradiction. We assume then that M ∈ ρ(A). In that case the symmetric continuous bilinear form a(u,v) = ([MI − A]u|v) defines a new scalar product on H. Whence we obtain by applying Cauchy- Schwarz’s inequality

|([MI − A]u|v)| ≤ ([MI − A]u|u)1/2([MI − A]v|v)1/2 for every u,v ∈ H.

In particular, for any u ∈ H, we have

kMu − Auk = sup |([MI − A]u|v)| ≤ C([MI − A]u,u)1/2. (2.9) v∈H, kvk=1

Let (uk) be a sequence satisfying kukk = 1 and (Auk|uk) → M. We deduce from −1 (2.9) that kMuk − Aukk converges to 0 and hence uk = (MI − A) (MI − A)uk → 0 which is impossible because kukk = 1. This yields the expected contradiction. The proof for m is obtained by substituting A by −A. ut

Corollary 2.3. If A is self-adjoint and σ(A) = {0} then A = 0. Proof. We have from Proposition 2.5 that (Au|u) = 0, for any u ∈ H, and hence

2(Au|v) = (A(u + v)|u + v) − (Au|u) − (Av,v) = 0 for any u,v ∈ H.

Whence A = 0. ut

The final result concerning the abstract spectral theory is a fundamental result showing that in a separable Hilbert space we can diagonalize any self-adjoint com- pact operator.

Theorem 2.9. Let H be a separable Hilbert space and let A ∈ K (H) be self-adjoint. Then H admits a Hilbertian basis consisting of eigenvectors of A.

Proof. Let (λm)m≥1 be the sequence of distinct eigenvalues of A except 0. Set λ0 = 0, E0 = N(A) and Em = N(A − λmI), m ≥ 1. Then

0 ≤ dim(E0) ≤ ∞ and 0 < dim(Em) < ∞, m ≥ 1.

Let us prove that H is the Hilbertian sum of (Em)m≥0. We first note that the sub- spaces Em are pairwise orthogonal. Indeed, if u ∈ Em and v ∈ E`, m 6= `, then, since Au = λmu and Av = λ`v, we have

(Au|v) = λm(u|v) = (u,Av) = λ`(u|v) implying (u,v) = 0. Next, we show that the subspace spanned by (Em)m≥0, denoted by F, is dense in H. Clearly A(F) ⊂ F and therefore A(F⊥) ⊂ F⊥. To see this, we observe that ⊥ if u ∈ F and v ∈ F then (Au|v) = (u|Av). The operator B = A|F⊥ is then self- adjoint and compact. Let claim that σ(B) = {0}. Since otherwise we would find 56 2 Variational solutions

λ ∈ σ(B) \{0}, that is λ ∈ ev(B). Whence there exists u ∈ F⊥, u 6= 0, satisfying ⊥ Bu = λu and consequently λ is equal to one of the eigenvalues λm and u ∈ F ∩Em. Thus u = 0 which leads to a contradiction. We conclude by applying Corollary 2.3 that B = 0 and then F⊥ ⊂ N(A) ⊂ F. We deduce that F⊥ = {0} which means exactly that F is dense in H. Finally, in each Em we choose a Hilbertian basis consisting of eigenvectors of A. The union of all these eigenvectors form a Hilbertian basis of H consisting in eigenvectors of A. ut

The rest of this section is inspired by [9, Section 6.2]. To apply the abstract spectral theory to elliptic boundary value problems we need to formulate such spectral problems, via the variational formulation, in an abstract way involving bilinear forms. In such general framework we consider V and H two infinite dimensional Hilbert spaces with V continuously and densely imbedded in H. The norm and the scalar product on H are denoted by (·|·) and | · |, while the norm on V is denoted by k · k. As V in continuously imbedded in H, there exists a c > 0 so that

|v| ≤ ckvk for any v ∈ V. (2.10)

Let a : V ×V → R be a continuous symmetric bilinear form and consider the spectral problem : find the values of λ ∈ R so that there exists u ∈V, u 6= 0, satisfying the equation a(u,v) = λ(u|v) for any v ∈ V. (2.11) We assume in addition that a is V-elliptic, i.e. there exists a constant α > 0 such that a(v,v) ≥ αkvk2 for every v ∈ V, Define A ∈ L (H,V) by

a(Au,v) = (u|v), for any v ∈ V. (2.12)

This definition has a sense since, noting that for any u ∈ H the linear form v → Φ(v) = (u,v) is continuous on V by (2.10), the problem (2.12) admits a unique solution Au ∈ V according to Lax-Milgram’s Lemma (here V is endowed with scalar product given by a). It is clear that A defines a linear map from H into V. On the other hand (2.10) yields

(u,v) kΦk = sup ≤ c|u| kvk=1 kvk in such a way that 1 c kAuk ≤ kΦk ≤ |u|, α α where α is the V-ellipticity constant of a. 2.2 Elements of the spectral theory of compact operators 57

The advantage in introducing the operator A is that (2.11) can be converted into the following spectral problem : find u ∈ V, u 6= 0, so that

u = λAu˜ . (2.13)

Here A˜ = AI, I being the canonical imbedding of V into H. Lemma 2.2. If I the canonical imbedding of V into H is compact and the bilinear form a is V-elliptic, then A˜ ∈ K (V). This lemma follows readily from Proposition 2.2 because A˜ = AI with I ∈ K (V,H) and A ∈ L (H,V). Lemma 2.3. If the bilinear form a is continuous, symmetric and V-elliptic, then A˜ ∈ L (V) is self-adjoint when V is endowed with the scalar product a(·,·). Moreover, A is positive in the sense that a(Av˜ ,v) > 0 for any v ∈ V, v 6= 0. Proof. Using that a is symmetric, we obtain from (2.12) that, for all u, v ∈ V,

a(Au˜ |v) = (u|v) = (v|u) = a(u,Av˜ ), showing that A˜ is self-adjoint whenever V is endowed with the scalar product a(·,·). The positivity of A˜ follows from the fact that a(Av˜ ,v) = |v|2 > 0 for any v ∈ V, v 6= 0. ut Theorem 2.10. Assume that V is compactly imbedded in H and the bilinear form a is continuous, symmetric and V-elliptic. Then the eigenvalues of (2.11) form an non decreasing sequence converging to ∞:

0 < λ1 ≤ λ2 ≤ ... ≤ λm ≤ ... and there exists an orthonormal Hilbertian basis of H consisting of eigenvectors wm so that, for any m ≥ 1,

a(wm,v) = λm(wm|v) for any v ∈ V.

−1/2 Moreover, the sequence (λm wm) form an orthonormal Hilbertian basis of V for the scalar product a(·,·). Proof. By Lemma 2.3, A˜ is self-adjoint and positive. We apply then Theorem 2.9 to deduce that the spectrum of A˜ consists in a sequence (µm) of non increasing of positive reals numbers converging to 0, and there exists an orthonormal Hilbertian basis of V for the scalar product a(·,·) consisting in eigenvectors vm so that

Av˜ m = µmvm. (2.14)

We deduce that the eigenvalues of (2.11) are given by 1 λm = . µm 58 2 Variational solutions

We get from (2.12) and (2.14)

a(vm,v) = λma(Ave m,v) = λm(vm|v) for any v ∈ V.

1/2 Let wm = λm vm. We check that (wm) form an Hilbertian orthonormal basis of H. We first observe that, from (2.12), we have

1 λ` (wm|w`) = a(wm,w`) = a(vm,v`) = δm`. λm λm

On the other hand, if u ∈ H satisfies (u|wm) = 0, for any m ≥ 1, then (u|v) = 0 for any v ∈ V because (vm) form an Hilbertian orthonormal basis of V. Next, using the fact that V is dense in H to deduce that u = 0 and hence (wm) form an orthonormal basis of H. ut

Remark 2.1. We can weaken the V-ellipticity condition in Theorem 2.10. Precisely, we can substitute the V-ellipticity condition by the following one : there exist α > 0 and λ ∈ R so that

a(v,v) + λ|v|2 ≥ αkvk2 for any v ∈ V.

In that case, (u,v) → a(u,v) + λ(u|v) possesses the assumptions of Theorem 2.10.

Under the assumptions and notations of Theorem 2.10, as (wm) is an orthonormal basis of H, we have, for any u ∈ H,

u = ∑ (u,wm)wm (2.15) m≥1 and 2 2 |u| = ∑ (u,wm) . (2.16) m≥1 −1/2 Also, as (vm) = (λm wm) is an orthonormal basis of V endowed with the scalar product a(·,·), we have for each v ∈ V

2 −1 2 a(v,v) = ∑ a(v,vm) = ∑ λm a(v,wm) . m≥1 m≥1

Hence we get from (2.12)

2 a(v,v) = ∑ λm(v|wm) . (2.17) m≥1

We now give a characterization of the eigenvalues. Define the Rayleigh quotient by a(v,v) R(v) = , v ∈ V, v 6= 0. (2.18) |v|2 We obtain from (2.17) 2.2 Elements of the spectral theory of compact operators 59

R(wm) = λm, m ≥ 1. (2.19)

For v = ∑i≥1 αiwi, a non zero element in V, we have by virtue of (2.19)

2 ∑i≥1 λiαi R(v) = 2 ≥ λ1. ∑i≥1 αi We deduce from this the following characterization of the first eigenvalue:

λ1 = min R(v). v∈V, v6=0

⊥ Let Vm be the subspace of V spanned by the eigenvectors w1,...,wm and let Vm the orthogonal of Vm in V with respect to the scalar product a(·,·), i.e.

⊥ Vm = {v ∈ V; a(v,wi) = 0, 1 ≤ i ≤ m}.

Note that we have also

⊥ Vm = {v ∈ V; (v|wi) = 0, 1 ≤ i ≤ m}.

⊥ If v = ∑i≥1 αiwi ∈ Vm−1 then αi = 0, 1 ≤ i ≤ m − 1. Thus,

2 ∑i≥m λiαi R(v) = 2 ≥ λm, ∑i≥m αi which entails in light of (2.19)

λm = min R(v). (2.20) ⊥ v∈Vm−1, v6=0

In fact we have another useful characterization of the eigenvalues λm as shows the following theorem.

Theorem 2.11. (Min-max principle) Under the assumption of Theorem 2.10, we have λm = min max R(v), Em∈Vm v∈Em, v6=0 where Vm is the set of all subspaces Em of V of dimension m.

m Proof. If Em = Vm then, for v = ∑i=1 αiwi 6= 0, we have

m 2 ∑i=1 λiαi R(v) = m 2 ≤ λm, ∑i=1 αi which implies by virtue of (2.20)

max R(v) = λm. v∈Vm, v6=0 60 2 Variational solutions

We claim that λm ≤ max R(v) v∈Em, v6=0 for any Em ∈ Vm. Indeed, we can choose v ∈ Em, v 6= 0 in such a way that (v|wi) = 0, ⊥ 1 ≤ i ≤ m − 1, i.e. v ∈ Em ∩ Vm−1. We obtain from (2.20) that λm ≤ R(v). This completes the proof. ut

2.3 Variational solutions for model problems

2.3.1 Variational solutions

We show by a simple model how to build a variational formulation associated to a boundary value problem. n Let Ω be a bounded domain of R of class C1 with boundary Γ . If f ∈ L2(Ω) we consider the problem of finding a function u defined on Ω and satisfying boundary value problem −∆u = f in Ω, (2.21) u = 0 on Γ . (2.22) Assume that (2.21)-(2.22) admits a solution u ∈ H2(Ω). We multiply each side of (2.21) by ϕ ∈ D(Ω) and then we integrate over Ω. We obtain Z Z − ∆uϕdx = f ϕdx. Ω Ω But the divergence theorem yields Z Z Z div(ϕ∇u)dx = ∆uϕdx + ∇u · ∇ϕdx = 0. Ω Ω Ω

Hence Z Z ∇u · ∇ϕdx = f ϕdx. Ω Ω 1 As D(Ω) is dense H0 (Ω), we deduce that Z Z 1 ∇u · ∇vdx = f vdx for any v ∈ H0 (Ω). (2.23) Ω Ω

1 1 On the other hand, (2.22) implies that u ∈ H0 (Ω), where we used that H0 (Ω) is characterized by 1 1 6 H0 (Ω) = {u ∈ H (Ω); u|Γ = 0}. Let us then substitute (2.21) and (2.22) by the following problem :

6 u|Γ stands for the trace of u on Γ in the sense of Theorem 1.18. 2.3 Variational solutions for model problems 61

2 1 for f ∈ L (Ω), find u ∈ H0 (Ω) satisfying (2.23). This reformulation of the Dirichlet problem (2.21) and (2.22) is usually called the variational formulation associated to this boundary value problem. Hence, any solution in H2(Ω) of (2.21) and (2.22) is a solution of (2.23). Conversely, if u ∈ 1 2 H0 (Ω) is a solution of (2.23) belongs to H (Ω) then we have in particular Z Z ∇u · ∇ϕdx = f ϕdx for any ϕ ∈ D(Ω). Ω Ω We get from the definition of weak derivatives Z Z −∆uϕdx = f ϕdx, for any ϕ ∈ D(Ω), Ω Ω and hence −∆u = f by the cancellation theorem. Therefore u is the solution of (2.21) and (2.22). Consider now the Neumann boundary value problem

−∆u + u = f in Ω, (2.24)

∂ν u = 0 on Γ , (2.25) where ∂ν u = ∇u · ν denotes the derivative along the unit exterior normal vector ν. If u ∈ H2(Ω) is a solution of (2.24) and (2.25), we multiply each side of (2.24) by v ∈ H1(Ω) and then we integrate over Ω. In light of the boundary condition (2.25), the divergence theorem gives Z Z Z ∇u · ∇vdx + uvdx = f vdx for any v ∈ H1(Ω). (2.26) Ω Ω Ω As before we substitute the boundary value problem (2.24) and (2.25) by the fol- lowing one: for f ∈ L2(Ω) find u ∈ H1(Ω) satisfying (2.26). Conversely, if u ∈ H1(Ω) is a solution of (2.26) then Z Z Z ∇u · ∇ϕdx + uvdx = f ϕdx for any ϕ ∈ D(Ω). Ω Ω Ω

If we admit that u, the solution of this variational problem (2.26), belongs to H2(Ω) then as in the Dirichlet case we prove that u satisfies (2.24). On the other hand, we get by choosing ϕ ∈ D(Ω) in (2.26) Z ∂ν uϕ = 0, for any ϕ ∈ D(Ω), Γ

n 2 and admitting also that D(Γ ) = {ψ = ϕ|Γ ; ϕ ∈ D(R )} is dense in L (Γ ), we obtain 62 2 Variational solutions Z 2 ∂ν uw = 0, for any w ∈ L (Γ ), Γ implying that (2.25) holds. We now give a general framework that generalize the previous two examples. Let n Ω be a bounded domain of R and let V be a closed subspace of H1(Ω) satisfying

1 1 H0 (Ω) ⊆ V ⊆ H (Ω).

Therefore V is Hilbert space when its is endowed with the norm of H1(Ω). i j ∞ ∞ Pick a ∈ L (Ω), 1 ≤ i, j ≤ n, a0 ∈ L (Ω) and set

Z ( n ) i j a(u,v) = ∑ a ∂ ju∂iv + a0uv dx. Ω i, j=1

If A = (ai j) then the last identity takes the form Z a(u,v) = (A ∇u · ∇v + a0uv)dx. Ω Simple computations show

|a(u,v)| ≤ CkukH1(Ω)kvkH1(Ω),

i j where C = maxi j ka kL∞(Ω) + ka0kL∞(Ω). That is the bilinear form a is continuous on H1(Ω) × H1(Ω). Assume moreover that the following ellipticity condition holds : there exists α > 0 so that 2 n (A ξ,ξ) ≥ α|ξ| a.e. in Ω, for any ξ ∈ R .

We also assume that there exists α0 > 0 so that

a0 ≥ α0 a.e. in Ω.

Under these assumptions, we have

2 2 2 a(u,u) ≥ αk∇uk2 + α0kuk2 ≥ min(α,α0)kukV , in such a way that a is V-elliptic. Pick f ∈ L2(Ω) and set Z Φ(v) = f vdx. Ω The linear form v → Φ(v) is continuous in L2(Ω) and therefore it is also continuous in V. As a is continuous and V-elliptic, we find by applying Lax-Milgram’s lemma a unique u ∈ V satisfying

a(u,v) = Φ(v) for any v ∈ V. (2.27) 2.3 Variational solutions for model problems 63

Let us interpret the problem we just solved. We make the extra assumption that the solution of (2.27) belongs to H2(Ω). In light of the definition of weak deriva- tives, we have Z (Lu − f )ϕ = 0 for any ϕ ∈ D(Ω). Ω Here L is the differential operator with variable coefficients which is given as follows

n i j Lw = − ∑ ∂i(a ∂ jw) + a0w. i, j=1

Then the cancellation theorem yields

Lu = f a.e. in Ω.

Summing up we get that the solution (2.27) satisfies the following conditions:

u ∈ V, (2.28)

Lu = f a.e. in Ω, (2.29) Z a(u,v) = Luvdx for any v ∈ V. (2.30) Ω Conversely, we see immediately that if u is a solution of (2.28)-(2.30) then u is also a solution of (2.27). In other words, the solution of (2.27) is characterized by (2.28)-(2.30). We return back to the two examples we discussed in the beginning of this 1 subsection. We choose V = H0 (Ω) endowed with the equivalent norm kwkV = k wk 2 n ´ ∇ L (Ω,R ) (follows from Poincare’s inequality). Then we have that (2.23) ad- 1 1 mits a unique solution u ∈ H0 (Ω). Similarly, taking V = H (Ω) equipped with the the norm of H1(Ω), we get that (2.26) has a unique solution u ∈ H1(Ω). We end this subsection by a spectral problem associated to the L: Lu = λu. In addition of the previous assumptions on a, we assume that the matrix A (x) = (ai j(x)) is symmetric for a.e. x ∈ Ω. We obtain by applying Theorem 2.10 a non 2 decreasing sequence (λm), λm → ∞, and an orthonormal basis (wm) of L (Ω), wn ∈ V for each m, consisting in eigenvectors so that

a(wm,v) = λm(wm|v) for any v ∈ V. (2.31)

Here (·|·) is the usual scalar product of L2(Ω). As we have done for (2.27), we show that the solution of (2.31) is characterized by 64 2 Variational solutions

wm ∈ V,

Lwm = λmwm a.e. in Ω, Z a(wm,v) = Lwmvdx for any v ∈ V. Ω

2.3.2 H2-regularity of variational solutions

We limit our study to the Dirichlet problem. The following proposition will be useful in the sequel. n p Proposition 2.6. Let Ω be an open subset R , 1 < p ≤ ∞ and u ∈ L (Ω). The fol- lowing properties are equivalent. (i) u ∈ W 1,p(Ω). (ii) There exits a constant C > 0 so that

Z 0 u∂iϕdx ≤ CkϕkLp (Ω) for any ϕ ∈ D(Ω), i = 1,...n. Ω

n (iii) There exists a constant C > 0 so that, for any ω b Ω and any h ∈ R with |h| < dist(ω,Ω c), we have

kτhu − ukLp(ω) ≤ C|h|.

Furthermore, we can take C = k∇ukLp(Ω)n in (ii) and (iii). Proof. It is straightforward to check that (i) entails (ii). Let us prove that (ii) implies (i). By assumption the linear form Z Φ : D(Ω) → u∂iϕ Ω

0 is continuous when D(Ω) is endowed with the norm of Lp (Ω). Therefore, we can 0 extend Φ by density to a continuous linear form, still denoted by Φ, on Lp (Ω). p From Riesz’s representation theorem there exists vi ∈ L (Ω) so that Z hΦ,ϕi = viϕdx for any ϕ ∈ D(Ω), 1 ≤ i ≤ n. Ω

Whence u ∈ W 1,p(Ω). n Next, we proceed to the proof of (i) entails (iii). Let us first consider u ∈ D(R ). n If h ∈ R then Z 1 u(x + h) − u(x) = ∇u(x +th) · hdt. 0 Hence Z 1 p p p |τhu(x) − u(x)| ≤ |h| |∇u(x +th)| dt 0 2.3 Variational solutions for model problems 65

and then Z Z Z 1 p p p |τhu(x) − u(x)| dx ≤ |h| dx |∇u(x +th)| dt ω ω 0 Z 1 Z ≤ |h|p dt |∇u(x +th)|pdx 0 ω Z 1 Z ≤ |h|p dt |∇u(y)|pdy. 0 ω+th

c 0 0 Fix |h| < dist(ω,Ω ). Then there exists ω b Ω such that ω + th ⊂ ω for any t ∈ [0,1]. Thus Z p p p kτhu − ukLp(ω) ≤ |h| |∇u| dx. (2.32) ω0 1,p If u ∈ W (Ω), p 6= ∞, by Friedrichs’s theorem (see Exercise 1.5), there exists (um) n p p n a sequence in D(R ) so that um → u in L (Ω) and ∇um → ∇u dans L (ω,R ) for any ω b Ω. Apply (2.32) to um and pass to the limit, when m → ∞ to get (iii). When p = ∞, we apply the case p < ∞ and then we pass to the limit when p → ∞. We complete the proof by showing that (iii) implies (ii). Take then ω so that n c supp(ϕ) ⊂ ω b Ω. Let h ∈ R satisfying |h| < dist(ω,Ω ). Then (iii) yields

Z 0 (τhu − u)ϕdx ≤ C|h|kϕkLp (Ω). Ω

But Z Z (τhu − u)ϕdx = u(τ−hϕ − ϕ)dx. Ω Ω Thus Z   τ−hϕ − ϕ 0 u dx ≤ CkϕkLp (Ω). Ω h

We derive then (ii) by choosing h = tei , t ∈ R, and passing to the limit as t goes to 0. ut

We use the following definition of an open set of class Ck (see comments in Chapter 1 for an equivalent definition). Define

n 0 n R+ = {x = (x ,xn) ∈ R ; xn > 0}, 0 n 0 Q = {x = (x ,xn) ∈ R ; |x | < 1 and |xn| < 1}, n Q+ = Q ∩ R+, 0 n 0 Q0 = {x = (x ,xn) ∈ R ; |x | < 1 and xn = 0}.

Recall that Ω is of class Ck, k ≥ 1 is an integer, if for any x ∈ Γ = ∂Ω there exists n U a neighborhood of x dans R and a bijective mapping H : Q → U so that

k −1 k H ∈ C (Q), H ∈ C (U), H(Q+) = U ∩ Ω, H(Q0) = U ∩Γ . 66 2 Variational solutions

n Theorem 2.12. Let Ω be an open subset of R of class C2 with bounded boundary n 2 1 Γ or else Ω = R+. For f ∈ L (Ω), let u ∈ H0 (Ω) satisfies Z Z Z 1 ∇u · ∇v + uv = f v for any v ∈ H0 (Ω). (2.33) Ω Ω Ω

Then u ∈ H2(Ω) and kukH2(Ω) ≤ Ck f kL2(Ω), where the constant C only depends on Ω.

n Proof. The proof consists in several steps. We first consider the case Ω = R and n 2 then the case Ω = R+. For the general case the H interior regularity is obtained n from the case Ω = R while the regularity at the boundary is deduced from that of n the case Ω = R+ by using local cards and a partition of unity. n n • The case Ω = R . For h ∈ R , h 6= 0, set τ u − u d u = h . h |h|

That is τ u(x) − u(x) (d u)(x) = h . h |h|

We find by taking in (2.33) v = d−hdhu Z Z Z 2 2 |∇dhu| dx + |dhu| dx = f d−hdhu. Rn Rn Rn Note that we have use the fact that Z Z d−hw1w2dx = w1dhw2dx. Rn Rn Therefore 2 kd uk ≤ k f k 2 n kd d uk 2 n . (2.34) h H1(Rn) L (R ) −h h L (R ) On the other hand,

1 n kd vk 2 n ≤ k vk 2 n n v ∈ H ( ). −h L (R ) ∇ L (R ,R ) for all R (2.35) To see this, we recall (see Proposition 2.6) that

n n kd vk 2 ≤ k vk 2 n n h ∈ , −h L (ω) ∇ L (R ,R ) for all ω b R and R Hence (2.35) follows. By obtain by combining (2.34) and (2.35)

2 kd uk ≤ k f k 2 n kd uk 1 n h H1(Rn) L (R ) h H (R ) and consequently 2.3 Variational solutions for model problems 67

kd uk 1 n ≤ k f k 2 n . h H (R ) L (R ) In particular, kd uk 2 n ≤ k f k 2 n , j = ,...,n. h∂ j L (R ) L (R ) 1 1 n 2 n By Proposition 2.6 we deduce that ∂ ju ∈ H (R ) and then u ∈ H (R ). n • The case Ω = R+. We use again translations but in the present case only in the n− tangential directions. That is directions of the form h ∈ R 1 ×{0}. In this case, we say that h is parallel to the boundary and we write h k Γ . Note that

1 1 u ∈ H0 (Ω) =⇒ τhu ∈ H0 (Ω) for any h k Γ ,

1 which means that H0 (Ω) is invariant under tangential translations. Let h k Γ . We get by taking v = d−h(dhu) in (2.33) Z Z Z 2 2 |∇dhu| dx + |dhu| dx = f d−hdhu. Ω Ω Ω That is 2 kdhukH1(Ω) ≤ k f kL2(Ω)kd−hdhukL2(Ω). (2.36) We shall need the following lemma. Lemma 2.4. We have

kd vk2 ≤ k∇vk2 for any v ∈ H1(Ω) and h k Γ . h L2(Ω) L2(Ω,Rn)

n Proof. If v ∈ D(R+), using that Ω + th = Ω for any t ∈ R and h k Γ , we have similarly to the proof of Proposition 2.6

kd uk2 ≤ k∇uk2 for any h k Γ . h L2(Ω) L2(Ω,Rn)

n 1 The expected inequality follows since D(R+) is dense in H (Ω) (see the proof of Proposition 1.11). ut In light of the inequality in Lemma 2.4 and (2.36), it follows

2 kdhukH1(Ω) ≤ k f kL2(Ω) for any h k Γ . (2.37)

Let 1 ≤ j ≤ n, 1 ≤ k ≤ n − 1, h = |h|ek and ϕ ∈ D(Ω). We have Z Z dh(∂ ju)ϕdx = − ud−h(∂ jϕ)dx Ω Ω and by (2.37) we obtain

Z dh(∂ ju)ϕdx ≤ k f kL2(Ω)kϕkL2(Ω). Ω We then get by passing to the limit, when h → 0, 68 2 Variational solutions Z 2 u∂ jkϕdx ≤ k f kL2(Ω)kϕkL2(Ω), 1 ≤ j ≤ n, 1 ≤ k ≤ n − 1. (2.38) Ω Let us finally prove that

Z 2 u∂n ϕdx ≤ k f kL2(Ω)kϕkL2(Ω) for any ϕ ∈ D(Ω). (2.39) Ω We deduce directly from equation (2.33) and (2.38) that

Z n−1 Z Z 2 2 u∂n ϕdx ≤ ∑ u∂i ϕdx + ( f − u)ϕdx Ω i=1 Ω Ω

≤ Ck f kL2(Ω)kϕkL2(Ω). Inequalities (2.38) and (2.39) give

Z u∂ jkϕdx ≤ Ck f kL2(Ω)kϕkL2(Ω), 1 ≤ j,k ≤ n, for all ϕ ∈ D(Ω). Ω

Whence u ∈ H2(Ω). Note that, by Hahn-Banach’s extension theorem and Riesz- 2 Frechet’s´ representation theorem, there exist f jk ∈ L (Ω) so that Z Z u∂ jkϕdx = f jkϕ for any ϕ ∈ D(Ω). Ω Ω • General case. For simplicity convenience, we assume that Ω is bounded. As Ω is 2 n of class C , there exist Ui, 1 ≤ i ≤ k, an open subset of R and a bijective mapping Hi : Q → Ui so that

2 −1 2 Hi ∈ C (Q), Hi ∈ C (Ui), Hi(Q+) = Ui ∩ Ω, Hi(Q0) = Ui ∩Γ (2.40)

Sk and Γ ⊂ i=1 Ui. Let θ0,...θk be a partition of unity so that

θi ∈ D(Ui), 1 ≤ i ≤ k ∞ n n θ0 ∈ C (R ), supp(θ0) ⊂ R \Γ k n 0 ≤ θi ≤ 1, 0 ≤ i ≤ k and ∑ θi = 1 in R . i=0

Since Ω is bounded, we have θ0|Ω ∈ D(Ω). k 2 Write u = ∑i=0 θiu and let us first check that θ0u ∈ H (Ω) (interior regularity). 1 n As θ0|Ω ∈ D(Ω), θ0u extended by 0 outside Ω belongs to H (R ). Simple compu- n tations show that θ0u is the variational solution in R of the equation

−∆(θ0u) + θ0u = θ0 f − 2∇θ0 · ∇u − (∆θ0)u = g, 2.3 Variational solutions for model problems 69

2 n n 2 n with g ∈ L (R ). We get by applying the case Ω = R that θ0u ∈ H (R ) and   k uk 2 n ≤ C k f k 2 + kuk 1 . θ0 H (R ) L (Ω) H (Ω)

Thus k uk 2 n ≤ Ck f k 2 θ0 H (R ) L (Ω)

because kukH1(Ω) ≤ k f kL2(Ω) by (2.33). 2 Next, we prove that θiu ∈ H (Ω), 1 ≤ i ≤ k (boundary regularity). Fix i, 1 ≤ i ≤ k, and, for simplicity convenience, we use the notations θ = θi, H = Hi et U = Ui, where Hi and Ui are the same as in (2.40). We write x = H(y) and then y = −1 1 H (x) = J(x). Since θ ∈ D(U) and v = θu ∈ H0 (Ω ∩U), we easily check that v is the variational solution in Ω ∩U of the equation

−∆v + v = θ f − 2∇θ · ∇u − (∆θ)u = g,

2 n where g ∈ L (R ) and kgkL2(Ω∩U) ≤ Ck f kL2(Ω). Precisely, we have Z Z 1 ∇v · ∇ϕdx = gϕdx, for any ϕ ∈ H0 (Ω ∩U). (2.41) Ω∩U Ω∩U

We now make a change of variable in order to transform v|Ω∩U to a function defined on Q+. For doing that, we set

w(y) = v(H(y)), y ∈ Q+,

or equivalently w(J(x)) = v(x), x ∈ Ω ∩U. We use the following lemma, whose proof is given later, to convert (2.41) to a vari- ational problem in Q+. 1 Lemma 2.5. Under the notations above, we have w ∈ H0 (Q+) and n Z Z k` 1 ∑ a ∂kw∂`ψdy = g˜ψdy for all ψ ∈ H0 (Q+), (2.42) k,`=1 Q+ Q+

2 k` 1 where g˜ = (g ◦ H)|Jac(H)| ∈ L (Q+) and the functions a ∈ C (Q+) satisfy the ellipticity condition

n k` 2 n ∑ a (y)ξkξl ≥ α|ξ| , for any y ∈ Q+, ξ ∈ R , k,`=1

for some constant α > 0. 2 w ∈ H (Q ) kwk 2 ≤ Ckgk 2 We prove that + and H (Q+) ˜ L (Ω). This will imply that 2 2 θu ∈ H (Ω ∩U) and then θu ∈ H (Ω). Moreover, kθukH2(Ω) ≤ Ck f kL2(Ω). 70 2 Variational solutions

n As in the case Ω = R+ we use tangential translations. We choose in (2.42) ψ =  0 0 d−h(dhw) with h k Q0 recall that supp(w) ⊂ {(x ,xn); |x | < 1−δ, 0 < xn < 1−δ} for some δ > 0. We get Z Z k` ∑ dh(a ∂kw)∂`(dhw)dx = g˜d−h(dhw)dx. k,` Q+ Q+

But Z gd˜ (d w)dx ≤ kg˜k 2 k∇(d w)k 2 n −h h L (Q+) h L (Q+,R ) Q+ by Lemma 2.4. Hence Z k` d (a ∂ w)∂ (d w)dx ≤ kg˜k 2 k∇(d w)k 2 n . (2.43) ∑ h k ` h L (Q+) h L (Q+,R ) k,` Q+

On the other hand,

k` k` k` dh(a ∂kw)(y) = a (y + h)∂k(dhw)(y) + (dha )(y)∂kw(y) and hence Z kl 2 dh(a ∂kw)∂`(dhw)dx ≥ αk∇(dhw)k 2 n (2.44) ∑ L (Q+,R ) k,` Q+

−Ckwk 1 k∇(d w)k 2 n . H (Q+) h L (Q+,R ) A combination of (2.43) and (2.44) yields

k ( w)k 2 n ≤ C(kwk 1 + kgk 2 ), ∇ dh L (Q+) H (Q+) ˜ L (Q+)

kwk 1 ≤ Ckgk 2 and since H (Q+) ˜ L (Q+) (a consequence of (2.42)), we have

k∇(d w)k 2 n ≤ Ckg˜k 2 . (2.45) h L (Q+,R ) L (Q+) n Similarly to the case Ω = R+, we deduce from (2.45)

Z w dx ≤ Ckgk 2 k k 2 ∂k ∂`ψ ˜ L (Q+) ψ L (Q+) (2.46) Q+ 1 for any ψ ∈ Cc (Q+), (k,`) 6= (n,n).

2 w ∈ H (Q ) kwk 2 ≤ Ckgk 2 To complete the proof of + and H (Q+) ˜ L (Q+) it is sufficient to check that Z 1 w dx ≤ Ckgk 2 k k 2 ∈ C (Q ). ∂n ∂nψ ˜ L (Q+) ψ L (Q+) for any ψ c + (2.47) Q+

1 nn We apply (2.42) in which we substitute ψ ∈ Cc (Q+) by ψ/a , where we note that ann > α > 0. Whence 2.3 Variational solutions for model problems 71

Z Z " # nn  ψ  geψ k`  ψ  a ∂nw∂n nn dx = nn − ∑ a ∂kw∂` nn dx. Q+ a Q+ a (k,`)6=(n,n) a

Then nn Z Z ∂na ∂nw∂nψdx = nn ∂nwψdx Q+ Q+ a Z Z g˜ψ k` ψ  k` ψ  + nn dx + ∑ ∂kw∂la nn dx − ∑ ∂kw∂` a nn dx. Q+ a (k,`)6=(n,n) a (k,`)6=(n,n) Q+ a

This identity together with (2.46), in which we substituted ψ by ak`ψ/ann, yield

Z w dx ≤ C(kwk 1 + kgk 2 k k 2 . ∂n ∂nψ H (Q+) ˜ L (Q+) ψ L (Q+) Q+ This gives (2.47). ut

1 Proof (of Lemma 2.5). Let ψ ∈ H0 (Q+) and set ϕ(x) = ψ(J(x)), x ∈ Ω ∩U. Then 1 ϕ ∈ H0 (Ω ∩U) and

∂ jv(x) = ∑∂kw(J(x))∂ jJk(x), ∂ jϕ(x) = ∑∂`ψ(J(x))∂ jJ`(x). k ` Thus,

Z Z n ∇v(x) · ∇ϕ(x)dx = ∑ ∂ jJk(x)∂ jJ`(x)∂kw(J(x))∂`ψ(J(x))dx Ω∩U Ω∩U j,k,`=1 Z n = ∑ ∂ jJk(H(y))∂ jJ`(H(y))∂kw(y)∂`ψ(y)|JacH(y)|dy Q+ j,k,`=1 by the classical formula of change of variable. Therefore

Z Z n k` ∇v(x) · ∇ϕ(x)dx = ∑ a ∂kw(y)∂`ψ(y)dy, (2.48) Ω∩U Q+ k,`=1 where n k` a (y) = ∑ ∂ jJk(H(y))∂ jJ`(H(y))|JacH(y)|. j=1 k` 1 Note that a ∈ C (Q+) and the ellipticity condition fulfills

2 n n n k` min a (y)ξkξ` = min |JacH(y)| ∂ jJk(H(y))ξk ≥ α, n ∑ n ∑ ∑ ξ∈R , |ξ|=1 k,`=1 ξ∈R , |ξ|=1 j=1 k=1 with α > 0, because the Jacobian matrices JacH and JacJ are non singular. On the other hand, we have 72 2 Variational solutions Z Z g(x)ϕ(x)dx = g(H(y))ψ(y)|JacH(y)|dy. (2.49) Ω∩U Q+ A combination of (2.48), (2.49) and (2.41) then implies (2.42) and completes the proof. ut

2.3.3 Maximum principle

n In this subsection, Ω is an arbitrary open subset of R . As we have mentioned in Chapter 1, we use infΩ f et supΩ f respectively for infessΩ f and supessΩ f . Theorem 2.13. Let f ∈ L2(Ω) and u ∈ H1(Ω) ∩C(Ω) so that Z Z Z 1 ∇u · ∇ϕ + uϕ = f ϕ for any ϕ ∈ H0 (Ω). (2.50) Ω Ω Ω Then min{infu,inf f } ≤ u(x) ≤ max{supu,sup f } for any x ∈ Ω. Γ Ω Γ Ω Remark 2.2. When Ω is of class C1 the condition u ∈ C(Ω) is unnecessary because 2 in that case the trace u|Γ is well defined as an element of L (Γ ). Also, when u ∈ 1 H0 (Ω) we can drop the condition u ∈ C(Ω) (we refer to the maximum principle in the next section). Proof (of Theorem 2.13). We use Stampacchia’s truncation method. Fix G ∈ C1(R) so that 0 (i) |G (s)| ≤ M for any s ∈ R, (ii) G is increasing in ]0,+∞[, (iii) G(s) = 0 for any s ≤ 0. Let K = max{supΓ u,supΩ f }. If K = +∞ there is nothing to prove. Assume then that K < +∞ and let v = G(u − K). We consider separately two cases : (a) |Ω| < ∞ and (b) |Ω| = ∞. In case (a), we apply Proposition 1.10 to f (t) = G(t − K) − G(−K) to deduce that v ∈ H1(Ω). On 1 the other hand, v ∈ H0 (Ω) because v ∈ C(Ω) and v = 0 on Γ . We obtain by taking v as test function in (2.50) Z Z Z G0(u − K)|∇u|2dx + G(u − K)udx = f G(u − K)dx Ω Ω Ω and hence Z Z Z G0(u − K)|∇u|2dx + G(u − K)(u − K)dx = ( f − K)G(u − K)dx. Ω Ω Ω But f − K ≤ 0, G(u − K) ≥ 0 and G0(u − K) ≥ 0. Whence Z G(u − K)(u − K)dx ≤ 0. Ω 2.3 Variational solutions for model problems 73

The assumptions on G yield tG(t) ≥ 0 for any t ∈ R. Then the last inequality implies (u − K)G(u − K) = 0 a.e. in Ω and consequently u ≤ K a.e. in Ω. We complete the proof of (a) by repeating the precedent analysis with u substi- tuted by −u. We now proceed to the proof of case (b). Note that in this case we have nec- essarily K ≥ 0. Otherwise K < 0 would imply that | f (x)| ≥ K a.e. in Ω because f (x) ≤ K = −|K| a.e. in Ω which is impossible since f ∈ L2(Ω) and |Ω| = ∞. Fix K˜ > K and set v = G(u − K˜). Similarly to the precedent case, we check that 1 1 v ∈ H (Ω) ∩C(Ω) and v = 0 on Γ . In particular, v ∈ H0 (Ω). Taking v as test func- tion in (2.50), we obtain Z Z Z G0(u − K˜)|∇u|2dx + G(u − K˜)udx = f G(u − K˜)dx. (2.51) Ω Ω Ω But, as −K˜ < 0, we have G(−K˜) = 0 and hence

G(u − K˜) = G(u − K˜) − G(−K˜) ≤ M|u|.

On the other hand, Z Z Z Z KG˜ (u − K˜)dx = KG˜ (u − K˜)dx ≤ M K˜|u| ≤ K0 |u|2, Ω [u≥K˜] [u≥K˜] Ω where u ≥ K˜ = {x ∈ Ω; u(x) ≥ K˜}. Therefore G(u − K˜) ∈ L1(Ω). We get from (2.51) that Z Z (u − K˜)G(u − K˜)dx ≤ ( f − K˜)G(u − K˜)dx ≤ 0. Ω Ω Thus u ≤ K˜ a.e. in Ω and then u ≤ K a.e. in Ω because K˜ > K was chosen arbitrarily. ut

Corollary 2.4. Let f ∈ L2(Ω) and u ∈ H1(Ω) ∩C(Ω) satisfying (2.50). Then

(u ≥ 0 on Γ ) and ( f ≥ 0 in Ω) =⇒ (u ≥ 0 in Ω)  kukL∞(Ω) ≤ max kukL∞(Γ ),k f kL∞(Ω) .

In particular, kukL∞(Ω) ≤ kukL∞(Γ ) if f = 0 and kukL∞(Ω) ≤ k f kL∞(Ω) if u = 0 on Γ . 74 2 Variational solutions 2.3.4 Uniqueness of continuation across a non characteristic hypersurface

We first establish a Carleman inequality with a convex weight. In this subsection the gradient and the with respect to the vari- 0 n− 0 0 able x ∈ R 1 are denoted respectively by ∇ and ∆ . 0 0 n− Let θ = θ(x ), x ∈ R 1, be a C4 function defined on a neighborhood of the origin satisfying θ(0) = 0 and ∇0θ(0) = 0. Set 0 2 0 2 0 n−1 ϕ(x ,xn) = (xn − 1) + |x | , (x ,xn) ∈ R × R and consider the partial differential operator P = P0 + P1 with

0 0 2 2 0 0 P0 = ∆ + 1 + |∇ θ| ∂n + 2∇ θ · ∇ ∂n.

n Let P1 be a first order partial differential operator of the variable x ∈ R , i.e. n i P1 = ∑ b (x)∂i + c(x), i=1 where bi and c are measurable and bounded functions in a neighborhood of the origin.

n Theorem 2.14. There exists a neighborhood U of 0 in R and two constants τ0 > 0 and C > 0 so that Z  Z Z  e2τϕ (Pu)2dx ≥ C τ e2τϕ |∇u|2dx + τ3 e2τϕ u2dx (2.52) U U U

2 for all τ ≥ τ0 and u ∈ H0 (U ). τϕ −τϕ Proof. Introduce the operator L = e P0e . Then straightforward computations show that i j 2 L = ∑a ∂i j − 2τB · ∇ + τc1 + τ c2, i, j where  I ∇0θ  (ai j) = n−1 I denotes the identity matrix of n−1, t ∇0θ 1 + |∇0θ|2 n−1 R

 0 0  ∇ ϕ + ∂nϕ∇ θ B = 0 2 0 0 , ∂nϕ(1 + |∇ θ| ) + ∇ θ · ∇ ϕ

0 0 2 2 c1 = −∆ ϕ − 1 + |∇ θ| ∂n ϕ, 0 2 0 2 2 0 0 c2 = |∇ ϕ| + 1 + |∇ θ| (∂nϕ) + 2∂nϕ∇ ϕ · ∇ θ. 2.3 Variational solutions for model problems 75

It is not hard to check that the formal adjoint of L si given by 7

n n n ∗ i j 2 i j i j 2 L = ∑ a ∂i j + 2 ∑ (∂ia )∂ j + 2τB · ∇ + ∑ ∂i ja + 2τdivB + τc1 + τ c2. i, j=1 i, j=1 i, j=1

Therefore, the self-adjoint and skew-adjoint parts8 L+ and L− of L are respectively given as follows

n n n + i j 2 i j 1 2 i j 2 L = ∑ a ∂i j + ∑ ∂ia ∂ j + ∑ ∂i ja + τdivB + τc1 + τ c2, i, j=1 i, j=1 2 i, j=1

n n − i j 1 2 i j L = − ∑ ∂ia ∂ j − 2τB · ∇ − τdivB − ∑ ∂i ja . i, j=1 2 i, j=1

For τ > 0, we introduce the new notations

n + + + + i j B = (B1 ,...,Bn ), B j = ∑ ∂ia , i=1 − − − − 1 i j Bτ = (Bτ,1,...,Bτ,n), Bτ, j = −2B j − ∑∂ia , τ i n + 1 1 2 i j cτ = c2 + (c1 + divB) − 2 ∑ ∂i ja , τ 2τ i, j=1 n − 1 2 i j cτ = −divB − ∑ ∂i ja . 2τ i, j=1

With these new notations, L+ and L− take the form

n + i j 2 + 2 + L = ∑ a ∂i j + B · ∇ + τ cτ , i, j=1 − − − L = τBτ · ∇ + τcτ . Pick v an arbitrary C∞ function with a compact support in a neighborhood of 0. If [·,·] denotes the usual commutator 9 we have the following decomposition

7 Recall that the formal adjoint of L, denoted by L∗, is determined according to the relation Z Z Luvdx = uL∗vdx for all C∞ compactly supported functions u and v. 8 Recall that X is self-adjoint if X = X∗ ; it is skew-adjoint if X = −X∗. 9 If X and Y are two operators then [X,Y] = XY −YX. 76 2 Variational solutions

Z 6 + − v[L ,L ]v = ∑ Ii, i=1 where

Z " n # i j 2 − I1 = τ v ∑ a ∂i j,Bτ · ∇ vdx, i, j=1 Z " n # i j 2 − I2 = τ v ∑ a ∂i j,cτ vdx, i, j=1 Z  + −  I3 = τ v B · ∇,Bτ · ∇ vdx, Z  + − I4 = τ v B · ∇,cτ vdx, Z 3  + −  I5 = τ v cτ ,Bτ · ∇ vdx, Z 3  + − I6 = τ v cτ ,cτ vdx.

We have I6 = 0. (2.53) Until the end of this proof C denotes a generic constant independent of τ. + − + − As [B · ∇,cτ ] = B · ∇cτ , we obtain Z 2 I4 ≥ −τ v dx if τ  1. (2.54)

Here and henceforth the notation τ  1 means that τ is sufficiently large. For I2, we get by making integrations by parts that Z 2 2 2 I2 ≥ −C τ v + |∇v| dx if τ  1. (2.55)

Making once again integrations by parts, we find Z Z  + −  + − − + I3 = τ v B · ∇,Bτ · ∇ vdx = τ v B · ∇Bτ − Bτ · ∇B · ∇vdx τ Z = B+ · ∇B− − B− · ∇B+ · ∇v2dx 2 τ τ τ Z = − divB+ · ∇B− − B− · DB+v2dx. 2 τ τ

Hence Z 2 I3 ≥ −Cτ v dx if τ  1. (2.56)

− + 0 For I5, we first compute −Bτ · ∇cτ near (x ,xn) = 0. We obtain 2.3 Variational solutions for model problems 77   − + 1 −B · ∇c 0 = 16 + O if τ  1. τ τ |(x ,xn)=0 τ

Since Z Z 3  + −  3 − + 2 I5 = τ v cτ ,Bτ · ∇ vdx = −τ Bτ · ∇cτ v , we deduce Z 3 2 I5 ≥ Cτ v dx if τ  1 and v has a small support. (2.57)

Next, we estimate I1. In light of the identity

" n # n n n i j 2 − i j 2 − i j − 2 − i j 2 ∑ a ∂i j,Bτ · ∇ = ∑ a ∂i jBτ,k∂k + 2 ∑ a ∂iBτ,k∂k j − ∑ Bτ,k∂ka ∂i j i, j=1 i, j,k=1 i, j,k=1 i, j,k=1 we can split I1 into three terms : I1 = τ(J1 + J2 + J3), where

Z n i j 2 − J1 = ∑ a ∂i jBτ,k∂kvvdx, i, j,k=1 Z n i j − J2 = 2 ∑ a ∂iBτ,k∂k jvvdx, i, j,k=1 Z n − i j 2 J3 = − ∑ Bτ,k∂ka ∂i jvvdx. i, j,k=1

We have Z n Z n i j 2 − 1  i j 2 −  2 J1 = ∑ a ∂i jBτ,k∂kvvdx = − ∑ ∂k a ∂i jBτ,k v dx. i, j,k=1 2 i, j,k=1

Thus Z 2 J1 ≥ −C v dx if τ  1. (2.58)

For J2, we find Z n i j − 2 J2 = 2 ∑ a ∂iBτ,k∂k jvvdx i, j,k=1 Z n Z  i j −  i j − = −2 ∑ ∂ j a ∂iBτ,k ∂kvvdx − 2 ∑ a ∂iBτ,k∂kv∂ jv i, j,k=1 i, j,k Z Z n 2 i j − ≥ −C v dx − 2 ∑ a ∂iBτ,k∂kv∂ jvdx. i, j,k=1

On the other hand, we easily check that 78 2 Variational solutions n   i j − 1 ∑ a ∂iBτ,k = −4δ jk + O i=1 τ and consequently

 Z Z  2 2 J2 ≥ C − v dx + |∇v| dx if τ  1 and v with a small support. (2.59)

Proceeding similarly we obtain

Z Z n 2 − i j J3 ≥ −C v dx + ∑ Bτ,k∂ka ∂iv∂ jvdx. i, j,k=1

But we can show in a straightforward manner that

n   − 1 ∑ Bτ,k∂kai j = O k=1 τ in a neighborhood of 0. Whence

Z  1 Z J ≥ −C v2dx+O |∇v|2dx if τ  1 and v with a small support. (2.60) 3 τ

Inequalities (2.58) to (2.60) imply

 Z Z  2 2 I1 ≥ Cτ − v dx + |∇v| dx . (2.61)

In light of inequalities (2.53) to (2.57) and (2.61) there exists U a neighborhood of 0, two constants τ0 > 0 and C > 0 so that Z  Z Z   + − 2 3 2 v L ,L vdx ≥ C τ |∇v| dx + τ v dx , τ ≥ τ0 and v ∈ D(U ). U U Using Z Z Z Z Z (Lv)2dx = L+v2 dx + L−v2 dx + L+vL−vdx + L−vL+vdx U U U U U Z Z Z ≥ L+vL−vdx + L−vL+vdx = L+L−v − L−L+vvdx U U U Z ≥ vL+,L−vdx, U we find Z  Z Z  2 2 3 2 (Lv) ≥ C τ |∇v| dx + τ v dx , τ ≥ τ0 and v ∈ D(U ). (2.62) U U 2.3 Variational solutions for model problems 79

Let v = eτϕ u, u ∈ D(U ). We have eτϕ ∇u = ∇v − τv∇ϕ and hence

e2τϕ |∇u|2 ≤ 2|∇v|2 + τ2v2|∇ϕ|2 ≤ C |∇v|2 + τ2v2.

τϕ As e P0u = Lv, we deduce from (2.62) Z  Z Z  2τϕ 2 2τϕ 2 3 2τϕ 2 e (P0u) ≥C τ e |∇u| + τ e u dx, (2.63) U U

τ ≥ τ0 and u ∈ D(U ).

Finally from Z Z Z 2τϕ 2 1 2τϕ 2 2τϕ 2 e (Pu) dx ≥ e (P0u) dx − e (P1u) dx U 2 U U Z Z Z  1 2τϕ 2 2τϕ 2 2τϕ 2 ≥ e (P0u) −C e |∇u| dx + e u dx 2 U U U and (2.63) it follows, changing τ0 if necessay, that Z  Z Z  e2τϕ (Pu)2dx ≥ C τ e2τϕ |∇u|2dx + τ3 e2τϕ u2 , U U U

τ ≥ τ0 and u ∈ D(U ).

2 We get the expected inequality by using that D(U ) is dense in H0 (U ). ut Let P be an elliptic operator of the form

n i P = ∆ + ∑ b (x)∂i + c(x), i=1

i n where the functions b and c are bounded measurable on an open subset D of R . 4 Theorem 2.15. Let ψ ∈ C (D) and x0 ∈ D so that ∇ψ(x0) 6= 0. There exits V a 2 neighborhood of x0 in D so that, if u ∈ H (V ), Pu = 0 and u = 0 in {x ∈ V , ψ(x) < 0}, then u = 0 in V .

Proof. Making a translation if necessary we may assume that x0 = 0. Also, changing 0 the coordinate system, we are reduced to ∇ ψ(0) = 0 and ∂nψ(0) 6= 0. With the help of the implicit function theorem the equation ψ(x) = 0 in a neighborhood of 0 is 0 4 equivalent to xn = µ(x ) in a neighborhood of 0, where µ is a C function defined in a neighborhood of 0 and satisfies µ(0) = 0, ∇0µ(0) = 0. Observe that these transformations leave invariant the principal part P because the Laplace operator is invariant under orthogonal transformations. We now make the following change of variable

0 0 0 0 2 0 0 (x ,xn) → (x ,xn − µ(x ) + |x | ) = (x ,xn + θ(x )), 80 2 Variational solutions which transform P into P, where P has the same form as in the beginning of this subsection. Note also that under this change of variable supp(u) ⊂ {x; ψ(x) ≥ 0} is trans- 0 2 formed into supp(u) ⊂ {x; xn ≥ |x | }. Recall that 0 2 0 2 ϕ(x ,xn) = (xn − 1) + |x | and define E+ by

0 0 2 E+ = {(x ,xn); 0 ≤ xn < 1 et xn ≥ |x | }.

It is not hard to see that

0 0 E+\{0} ⊂ {(x ,xn); ϕ(x ,xn) < ϕ(0,0) = 1}.

Let U be as in Theorem 2.14. Reducing U if necessary we may assume that

0 0 U ∩ E+ ⊂ {(x ,xn); ϕ(x ,xn) ≤ ϕ(0,0)}.

2 Let u ∈ H (U ) satisfying Pu = 0 and supp(u) ⊂ E+. Let w ∈ D(U ), w = 1 in a 2 neighborhood U0 ⊂ U of the origin and set v = wu. As v ∈ H0 (U ), Theorem 2.14 implies Z Z e2τϕ (Pv)2dx = e2τϕ (Pv)2dx U U \U0 Z = e2τϕ (Pv)2dx E+∩(U \U0) Z ≥ Cτ3 e2τϕ v2dx if τ  1. (2.64) E+∩U Let ε > 0 so that

0 0 E+ ∩ (U \U0) ⊂ {(x ,xn); ϕ(x ,xn) < ϕ(0,0) − ε}, and V ⊂ U a neighborhood of the origin chosen in such a way that

0 0 ε E+ ∩ V ⊂ {(x ,xn); ϕ(x ,xn) ≥ ϕ(0,0) − }. 2 Then (2.64) entails

Z Z −ετ Z 2 2 e 2 v = v ≤ 3 (Pv) if τ  1. V E+∩V Cτ E+∩(U \U0)

Whence v = 0 in V and hence u = 0 in V too. ut

Theorem 2.15 can be used to obtain a global uniqueness of continuation for the operator P from an interior data. 2.4 General elliptic operators in divergence form 81

Theorem 2.16. Assume that Ω is connected. Let u ∈ H2(Ω) satisfying Pu = 0. Let ω be a nonempty open subset of Ω. If u = 0 in ω then u is identically equal to zero.

Proof. Let Ω0 the greatest open set in which u vanishes (a.e.). We claim that Ω0 = Ω (modulo a set of zero measure). Otherwise, Ω\Ω0 would be nonempty. As Ω is connected Ω ∩ ∂Ω0 would be also nonempty. Fix then x1 ∈ Ω ∩ ∂Ω0 and assume that B(x1,3r) ⊂ Ω, where B(x1,3r) is the ball with center x1 and radius 3r. Let K = ∂(B(x1,3r)∩Ω0), x2 ∈ B(x1,r)∩Ω0 and d = dist(x2,K)(≤ r). We easily check that d = dist(x2,∂Ω0). Therefore B(x2,d) is contained in Ω0 and ∂Ω0 ∩ B(x2,d) is nonempty. We choose then x0 in ∂Ω0 ∩∂B(x2,d). Apply Theorem 2.15 with ψ(x) = 2 2 |x−x2| −d in order to get that there exists V a neighborhood of x0 such that u = 0 in V (a.e.). But this contradicts the maximality of Ω0. ut

We apply Theorem 2.16 to obtain the uniqueness of continuation from Cauchy data.

Corollary 2.5. Assume that Ω is a bounded domain of class C2 with boundary Γ . Let Σ an nonempty open subset of Γ . Let u ∈ H2(Ω) satisfying Pu = 0 and u = 10 ∂ν u = 0 on Σ . Then u is identically equal to zero.

Proof. Let B be a ball centered at a point of Σ so that B ∩Γ = B ∩ Σ. The condition u = ∂ν u = 0 on Σ entails that the extension by 0 of u in B \ Ω, still denoted by u, is in H2(B ∪ Ω) 11. Now as u vanishes on B \ Ω, we get from Theorem 2.16 that u is identically equal to zero. ut

2.4 General elliptic operators in divergence form

n In this section Ω is an open bounded domain of R . Consider the differential oper- ator in divergence form with measurable bounded coefficients given by

n n ! n i j i i Lu = − ∑ ∂i ∑ a ∂ ju + c u + ∑ d ∂iu + du. (2.65) i=1 j=1 i=1

In all of this section L is assumed to be elliptic in the sense that there exists a constant λ > 0 so that n i j 2 n ∑ a (x)ξiξ j ≥ λ|ξ| a.e. x ∈ Ω, for anyξ ∈ R . (2.66) i, j=1

Assume moreover that there exist two constants Λ > 0 and µ > 0 so that

10 2 u and ∂ν u exist, in the trace sense, as elements of L (Γ ). 11 This fact can be proved by using divergence theorem and the definition of weak derivatives. 82 2 Variational solutions

n n ∑ |ai j(x)|2 ≤ Λ 2, λ −2 ∑ |ci(x)|2 + |di(x)|2 (2.67) i, j=1 i=1 + λ −1|d(x)| ≤ µ2 a.e. x ∈ Ω.

We associate to L the bilinear form n n i j i i  L (u,v) = ∑ a ∂ ju∂iv + ∑ c u∂iv + d ∂iuv + duv. i, j=1 i=1

For f ∈ L2(Ω), consider the equation

Lu = f in Ω. (2.68)

1,1 We say that u ∈ Wloc (Ω) is a of (2.68) if Z Z L (u,v)dx = f vdx, v ∈ D(Ω). Ω Ω

1,1 u ∈ Wloc (Ω) is said a sub-solution (resp. super-solution) of (2.68) whenever Z Z L (u,v)dx ≤ (resp. ≥) f vdx, v ∈ D(Ω), v ≥ 0. Ω Ω

2.4.1 Weak maximum principle

1 + 1 12 Let u ∈ H (Ω). We say that u ≤ 0 on Γ = ∂Ω if u = max(u,0) ∈ H0 (Ω) . If u, v ∈ H1(Ω) the notation u ≤ v on Γ will mean that u − v ≤ 0 on Γ . 1 Let u ∈ H (Ω). Define then supΓ u as follows

supu = inf{k ∈ R; u ≤ k on Γ }. Γ

n i 13 1 Theorem 2.17. Under the assumption d + ∑i=1 ∂ic ≥ 0 if u ∈ H (Ω) is a sub- solution of Lu = 0 in Ω then supu ≤ supu+. Ω Γ

+ ∗ Proof. We seek a contradiction by assuming that ` = supΓ u < k = supΩ u. We have as u is a sub-solution of Lu = 0 in Ω

12 Observe that u+ ∈ H1(Ω) follows from Corollary 1.1. 13 This condition is to be understood in the following sense

Z n ! i dϕ − ∑ c ∂iϕ dx ≥ 0 for any ϕ ∈ D(Ω), ϕ ≥ 0. Ω i=1 2.4 General elliptic operators in divergence form 83

Z Z " n n # i j i i L (u,v)dx = ∑ a ∂ ju∂iv + ∑ d − c ∂iuv dx Ω Ω i, j=1 i=1 Z " n # i + ∑ c ∂i(uv) + duv dx ≤ 0, v ∈ D(Ω), v ≥ 0. Ω i=1

1 By density the previous inequality still holds for any v ∈ H0 (Ω), v ≥ 0. 1 n i Let v ∈ H0 (Ω) so that v ≥ 0 and uv ≥ 0. Since d +∑i=1 ∂ic ≥ 0 we deduce from the last inequality

Z " n n # Z " n # i j i i i ∑ a ∂ ju∂iv + ∑ d − c ∂iuv dx ≤ − ∑ c ∂i(uv) + duv dx ≤ 0. Ω i, j=1 i=1 Ω i=1

That is Z n Z n i j i i ∑ a ∂ ju∂ivdx ≤ ∑ c − d ∂iuvdx. Ω i, j=1 Ω i=1 This implies Z n Z i j ∑ a ∂ ju∂ivdx ≤ C v|∇u|dx, (2.69) Ω i, j=1 Ω ∞ where the constant C only depends on the L -norms of ci and di. ∗ Let (km) be a non decreasing sequence so that ` ≤ km < k for every m and km ∗ + converges to k . Then (2.69) with v = vm = (u − km) gives

Z n Z i j ∑ a ∂ jvm∂ivmdx ≤ C vm|∇vm|dx, (2.70) Ω i, j=1 Ω where we used that ∇vm = χ[u>km]∇u. If Am = supp(|∇vm|) then (2.66) and (2.70) entail Z 2 C | v | dx ≤ kv k 2 k v k 2 n ∇ m m L (Am) ∇ m L (Ω) Ω λ and hence C k∇vmk 2 n ≤ kvmk 2 . (2.71) L (Ω) λ L (Am) But (see Lemma 1.10)

1/n 1/n kv k 2 ≤ |A | kv k 2n/(n−2) ≤ K(n)|A | k∇v k 2 n if n > 2, m L (Am) m n L (Am) m m L (Am,R ) and

1/2 kv k 2 ≤ K(2)k∇v k 1 n ≤ K(2)|A | k∇v k 2 n if n = 2, m L (Am) m L (Am) m m L (Am,R ) where the constant K(n) only depends on n. These two inequalities together with (2.71) yield 1/n kv k 2 ≤ K|A | kv k 2 m L (Ω) m m L (Am) 84 2 Variational solutions

K = CK(n)/ kv k 2 6= m with λ. By assumption, m L (Am) 0 for any . Therefore

−n |Am| ≥ K for any m. (2.72)

−n In particular, if A = ∩mAm then |A| = lim|Am| ≥ K and

supp(|∇u|) ⊃ A. (2.73)

On the other hand, if Bm = [u ≥ km] we have

∗ |[u = k ]| = | ∩m Bm| = lim|Bm|. m

∗ 14 But Am ⊂ Bm for any m. Hence [u = k ] ⊃ A and then ∇u = 0 in A . We get the expected contradiction by comparing this with (2.73). ut

We have as an immediate consequence of Theorem 2.17 the following uniqueness result.

1 Corollary 2.6. Assume that the assumptions of Theorem 2.17 hold. If u ∈ H0 (Ω) is a weak solution of Lu = 0 in Ω then u = 0.

2.4.2 The Dirichlet problem

We aim in this section to prove the following existence result.

n i 1 Theorem 2.18. Assume that d + ∑i=1 ∂ic ≥ 0. For any ϕ ∈ H (Ω) and g, fi ∈ L2(Ω), i = 1,...,n, the generalized Dirichlet problem

 n Lu = g + ∑i=1 ∂i fi in Ω, u = ϕ on Γ , admits a unique solution u ∈ H1(Ω).

Proof. We first reduce the initial problem to a problem with zero boundary condi- 1 tion. Set w = u − ϕ. Then clearly w ∈ H0 (Ω) and in light of (2.65)

n ˆ Lw = gˆ + ∑ ∂i fi, i=1 where

14 Note that ∇u = 0 in any set where u is constant. This an immediate consequence of Corollary 1.1. 2.4 General elliptic operators in divergence form 85

n i 2 gˆ = g − ∑ d ∂iϕ − dϕ ∈ L (Ω), i=1 n ˆ i j i 2 fi = fi + ∑ a ∂ jϕ + c ϕ ∈ L (Ω), i = 1,...,n. j=1

1 Introduce the notations H = H0 (Ω), g = (g, f1,..., fn) and

Z n ! F(v) = gv + ∑ fi∂iv dx, v ∈ H . Ω i=1

Then we have F ∈ H ∗ because

|F(v)| ≤ kgk 2 n+1 kvk 1 , v ∈ H . L (Ω,R ) H0 (Ω)

We now establish that L + σ(·|·) is continuous and coercive for some σ > 0. Prior to doing that we prove the following lemma.

Lemma 2.6. λ Z Z L (u,u) ≥ |∇u|2dx − λ µ2 u2dx. (2.74) 2 Ω Ω Proof. We have from (2.65) that

Z n n ! i j i i 2 L (u,u) = ∑ a ∂iu∂ ju + ∑ c − d u∂iu − du dx. Ω i, j=1 i=1

From the elementary inequality 1 αβ ≤ εα2 + β 2, 4ε we deduce that

i λ 2 1 i 2 2 |c ||∂iu||u| ≤ |∂iu| + |c | |u| , 4 λ i λ 2 1 i 2 2 |d ||∂iu||u| ≤ |∂iu| + |d | |u| . 4 λ These two inequalities, (2.66) and (2.67) imply

Z  λ  L (u,u) ≥ λ|∇u|2 − |∇u|2 − λν2u2 dx Ω 2 λ Z Z = |∇u|2dx − λν2 u2dx. 2 Ω Ω This proves the lemma. ut 86 2 Variational solutions

For σ ∈ R, let Lσ given by Lσ u = Lu + σu. According to Lemma 2.6, Lσ , the bilinear form associated to Lσ , is coercive provided that σ is sufficiently large. Next, we consider the operator I : H → H ∗ defined by Z Iu(v) = uvdx, v ∈ H . Ω Lemma 2.7. The operator I is compact.

2 Proof. Write I = I1I2, where I2 is the canonical imbedding of H into L (Ω) and 2 ∗ I1 : L (Ω) → H is given by Z 2 I1u(v) = uvdx, v ∈ L (Ω). Ω

By Sobolev imbedding theorems I2 is compact and since I1 is bounded I is also compact. ut

Fix σ0 so that Lσ0 is coercive in the Hilbert space H . Note that the equation Lu = F, for u ∈ H , is equivalent to the following one

Lσ0 + σ0Iu = F.

∗ But Lσ0 is an isomorphism from H onto H Lax-Milgram’s lemma. Therefore, the last equation is equivalent to the following one

u + L−1Iu = L−1F. (2.75) σ0 σ0 σ0

The operator T = − L−1I is compact by Lemma 2.7. In consequence, the existence σ0 σ0 of u ∈ H satisfying

n Lu = g + ∑ ∂i fi in Ω, u = 0 on ∂Ω i=1 is guaranteed by Fredholm’s alternative whenever the equation

Lu = 0 in Ω, u = 0 on ∂Ω has only u = 0 as a solution. This is true by Corollary 2.6. ut

We now describe the spectrum of the operator L. We can check in a straightfor- ward manner that L∗, the formal adjoint of L, is given by

n n ! n ∗ i j j i L u = − ∑ ∂ j ∑ a ∂iu + d u + ∑ c ∂iu + du. j=1 i=1 i=1

∗ 1 ∗ As L (u,v) = L (v,u) for any u, v ∈ H = H0 (Ω) we derive that L is also the adjoint of L in the Hilbert space H , and the same is valid if L is substituted by Lσ . 2.4 General elliptic operators in divergence form 87

The argument we used in the proof of Theorem 2.18 enables us to claim that the solvability of the equation Lσ u = F is equivalent to the solvability of the following one u + ( − )L−1Iu = L−1F. Since T ∗, the adjoint of the operator T = ( − σ0 σ σ0 σ0 σ σ σ0 )L−1I, is given by T ∗ = ( − )(L∗ )−1I, we can apply Fredholm’s alternative in σ σ0 σ σ0 σ σ0 order to obtain the following result.

Theorem 2.19. There exists a countable set Σ ⊂ R with no accumulation point so ∗ n that if σ 6∈ Σ the Dirichlet problem Lσ u (resp. Lσ u) = g + ∑i=1 ∂i fi in Ω, u = ϕ on 2 1 Γ admits a unique solution provided that g, fi ∈ L (Ω) and ϕ ∈ H (Ω). Let σ ∈ Σ, ∗ then the subspace of solutions of the homogenous equation Lσ u (resp. Lσ u) = 0, n u = 0 on Γ is of finite dimension (> 0), and the equation Lσ u = g + ∑i=1 ∂i fi in Ω, u = ϕ on Γ has a solution if and only if

n Z  i  i j i  ∑ g − d ∂iϕ − dϕ + σϕ v − fi + a ∂ jϕ + c ϕ ∂iv dx = 0 i, j=1 Ω

∗ n i for any v satisfying Lσ v = 0, v = 0 on Γ . Finally, if d + ∑i=1 ∑∂ic ≥ 0 then Σ ⊂ (−∞,0).

2.4.3 Harnack inequalities

We first prove a Harnack inequality for sub-solutions. 1 + Theorem 2.20. Let u ∈ Hloc(Ω) be a sub-solution of Lu = 0 in Ω. Then u ∈ ∞ Lloc(Ω) and, for any compact subset K of Ω, 0 < r < dist(K,Γ ) and p > 1, we have n/p + u ≤ [C(r, p)] ku kLp(K+B(0,r)) a.e. in K, (2.76) where  1 p4 C(r, p) = C 1 + . r (p − 1)2 Here the constant C only depends on n and the L∞-norm of the coefficients of λ −1L. Proof. The proof consists in three steps. In this proof C is a generic constant only depending on n and the L∞-norm of the coefficients of λ −1L. First step. Pick K a compact subset of Ω, 0 < r < dist(K,Γ ) and q > 1. We are 0 going to prove that if u+ ∈ Lq(K + B(0,r)) then u+ ∈ Ln q(K), and

+ 1/q + ku kLn0q(K) ≤ [C (r,q)] ku kLq(K+B(0,r)). (2.77)

Here and henceforth, n0 = n/(n − 1) is the conjugate exponent of n. Let dist(·,Ω\(K + B(0,r)) η = . (2.78) dist(·,Ω\(K + B(0,r)) + dist(·,K) 88 2 Variational solutions

It is not hard to check that η is Lipschitz continuous with Lipschitz constant equal to , n 1/r, supp(η) ⊂ K + B(0,r), 0 ≤ η ≤ 1 and η = 1 in K. In particular, η ∈ W 1 ∞(R ). Let θ be a Borelian function defined on R so that θ = 0 in ] − ∞,0], θ > 0 in ]0,+∞[ and set Z u v = η2 θ(s)ds. −∞ We have v ∈ H1(Ω), supp(v) ⊂ K + B(0,r) and v ≥ 0. The function v is then the limit of a sequence of (vk) belonging to D(Ω), vk ≥ 0 for any k. As u is a sub- solution of Lu = 0 in Ω, we have Z L (u,vk)dx ≤ 0 for all k. Ω We get by passing to the limit, when k → ∞, Z L (u,v)dx ≤ 0. Ω On the other hand, elementary computations give

n 2 i j L (u,v) = ∑ θ(u)η a ∂ ju∂iu i, j=1 n " ! # i j j j 1/2 + ∑ v1 2∑a ∂iη + d η + c ηv2 ηθ (u)∂ ju j=1 i " n # i 2 + ∑ 2c η∂iη + dη v3, i=1 with Z u Z u −1/2 1/2 v1 = θ (u) θ(s)ds, v2 = uθ (u), v3 = u θ(s)ds. −∞ −∞ Hence

Z n Z n " n ! # 2 i j i j j j 1/2 ∑ θ(u)η a ∂ ju∂iudx ≤ − ∑ v1 2 ∑ a ∂iη + d η + c ηv2 ηθ (u)∂ judx Ω i, j=1 Ω j=1 i=1 Z " n # i 2 − ∑ 2c η∂iη + dη v3dx. Ω i=1

This together with the convexity inequality |AB| ≤ (a/2)A2 +(1/2a)B2, a > 0, entail 2.4 General elliptic operators in divergence form 89 Z n Z 2 i j λ 2 2 ∑ θ(u)η a ∂ ju∂iudx ≤ θ(u)η |∇u| Ω i, j=1 2 Ω 2 Z n " ! # 1 i j j j + ∑ v1 2∑a ∂iη + d η + c ηv2 dx 2λ Ω j=1 i Z " n # i 2 − ∑ 2c η∂iη + dη v3dx. Ω i=1

But (see (2.66))

Z n Z 2 i j 2 2 ∑ θ(u)η a ∂ ju∂iudx ≥ λ θ(u)η |∇u| dx. Ω i, j=1 Ω

Whence

2 Z Z n " n ! # λ 2 2 1 i j j j θ(u)η |∇u| dx ≤ ∑ v1 2 ∑ a ∂iη + d η + c ηv2 dx 2 Ω 2λ Ω j=1 i=1 Z " n # i 2 − ∑ 2c η∂iη + dη v3dx. (2.79) Ω i=1

Introduce the auxiliary function Z u w = θ(s)1/2ds. −∞ Then |∇w|2 = θ(u)|∇u|2, (2.80) and from Cauchy-Schwarz’s inequality, we have

Z u 2 Z u 2 1/2 w = θ(s) ds ≤ u θ(s)ds = v3. (2.81) 0 0

On the other hand, if v4 = v1 + v2 then

2 2 v4 ≥ 4v1v2 = 4v3 ≥ 4w . (2.82)

Using estimates (2.79) to (2.81), the fact that 0 ≤ η ≤ 1 and |∇η| ≤ 1/r in order to deduce  1 kη∇wk 2 ≤ C 1 + kv k 2 . (2.83) L (Ω) r 4 L (K+B(0,r)) But ∇(ηw) = η∇w + w∇η. Hence 90 2 Variational solutions

k|∇(ηw)|kL2(Ω) = k|∇(ηw)|kL2(K+B(0,r)) 1 ≤ kη|∇w|k 2 + kwk 2 . L (K+B(0,r)) r L (K+B(0,r)) This and (2.82) imply 1 k|∇(ηw)|k 2 ≤ kη|∇w|k 2 + kv k 2 . L (Ω) L (K+B(0,r)) 2r 4 L (K+B(0,r)) In light of (2.83), the last inequality yields

 1 k|∇(ηw)|k 2 ≤ C 1 + kv k 2 . (2.84) L (Ω) r 4 L (K+B(0,r))

1,1 n Taking into account that ηw ∈ W0 (R ), we get from Lemma 1.10

2 2 2 kηwk 0 = k(ηw) k 0 ≤ Ck∇(ηw) k 1 ≤ 2Ckηwk 2 k∇wk 2 n , L2n (Ω) Ln (Ω) L (Ω) L (Ω) L (Ω) for some constant C = C(n). This inequality together with (2.82) and (2.84) imply

  11/2 kwk 0 ≤ C 1 + kv k 2 . (2.85) L2n (K) r 4 L (K+B(0,r))

2 q−2 We complete the proof of this first step by choosing θ(s) = χ{s>0}q s /4. Then

q2 w = (u+)q/2 and v = (u+)q/2. 4 2(q − 1)

0 We deduce from (2.85) that if u+ ∈ Lq(K +B(0,r)) then u+ ∈ Lqn (K +B(0,r)) and

   4 1/q + 1 q + ku k 0 ≤ C 1 + ku k q . (2.86) Lqn (K) r (q − 1)2 L (K+B(0,r))

+ p + 2 Second step. We claim that u ∈ Lloc(Ω) for any p > 1. Indeed, as u ∈ Lloc(Ω), + 2n0 we have that u ∈ Lloc(Ω) by (2.86). We iterate k times (2.86) to obtain is a simple 2(n0)k p manner that u ∈ Lloc (Ω) for any positive integer k and hence u ∈ Lloc(Ω) for every p > 1. Third step. We employ Moser’s iterative method to establish (2.76). Let K be a given compact subset of Ω, 0 < r < dist(K,Γ ), p > 1, and set r ρ = , r = ρ + ... + ρ = r(1 − 1/2k) and q = p(n0)k. k 2k k 1 k k

Fix a positive integer k and define the sequence of compacts Ki by 2.4 General elliptic operators in divergence form 91

Kk = K,...,Ki−1 = Ki + B(0,ρi),...,K0 = K + B(0,r).

We have in light of (2.86)

+ 1/q + ku k q ≤ C( ,q ) i−1 ku k q L i (Ki) ρi i−1 L i−1 (Ki−1) and then

k ! + 1/qi−1 + ku k q ≤ [C( ,q )] ku k p L k (K) ∏ ρi i−1 L (K+B(0,rk)) i=1 k ! 1/qi−1 + ≤ ∏[C(ρi,qi−1)] ku kLp(K+B(0,r)). i=1

On the other hand, elementary computations show

0 1−i 0 1−i 1/qi−1 (n ) /p 0 (i−1)(n ) /p [C(ρi,qi−1)] ≤ [C(r, p)] (2n ) .

Therefore k 1/qi−1 αk 0 βk ∏C(ρi,qi−1) ≤ [C(r, p)] (2n ) , i=1 with n n(n − 1) α = (1 − (n0)−k) and β = (1 + (k − 1)(n0)−k − k(n0)1−k). k p k p

Thus + αk 0 βk + ku kLqk (K) ≤ C(r, p) (2n ) ku kLp(K+B(0,r)). (2.87) We obtain by passing to the limit in (2.87), when k tends to ∞,

+ + n/p + ku kL∞(K) = lim ku kLqk (K) ≤ C(r, p) ku kLp(K+B(0,r)) k→+∞ as expected. ut We are now going to prove the following Harnak inequality for non negative super-solutions. 1 Theorem 2.21. Let u ∈ Hloc(Ω) be a super-solution of Lu = 0 in Ω. Let x0 ∈ Ω, 0 0 < r ≤ r0 < dist(x0,Γ )/4, p < n and assume that u is non negative in B(x0,4r). Then n/p p kukL (B(x0,2r)) ≤ Cr u a.e. in B(x0,r), where the constant C only depends on the L∞-norm of the coefficients of λ −1L, n, p, r and r0. Proof. The proof consists in three steps. First step. Fix ε > 0 and let 0 < r ≤ r0 . Let K be a compact subset of Ω with K ⊂ B(x0,3r). In the sequel, η is the function defined in (2.78). Set uε = u + ε and 92 2 Variational solutions

η2 v = α+1 with α > −1. (α + 1)uε As we have done in the first step of the proof of Theorem 2.20, we get by using L (u,v) ≥ 0

λ Z η2|∇u|2 α+2 dx 2 Ω uε " ! #2 Z 1 n 1 n u ≤ ai j + di − ci dx ∑ / 2 ∑ ∂iη η / + η Ω 2λ α 2 α 2 1 j=1 (α + 1)uε i=1 uε Z n ! u η i + α+1 2 ∑ c ∂iη + dη dx. Ω uε α + 1 i=1

When α = 0 we use the fact that 0 ≤ u/uε ≤ 1 a.e. in Ω to deduce from the last inequality

η|∇u|   ≤ C k k 2 + k k 2 n . η L (Ω) ∇η L (Ω,R ) (2.88) uε L2(Ω) If α > 0, noting that u 1 u 1 ≤ and ≤ , α+1 uα α/2+1 α/2 uε ε uε uε we easily check that

|∇u| η|∇u|  1 1 ≤ ≤ C 1 + . α/2+1 α/2+1 r α/2 uε L2(K) uε L2(Ω) uε L2(K+B(0,r)) But ! ∇u 2 1 = − ∇ . α/2+1 α α/2 uε uε Hence ! 1  1 1 ∇ ≤ Cα 1 + . α/2 r α/2 uε L2(K) uε L2(K+B(0,r)) This and Sobolev imbedding theorems yield similarly to the first step of the proof of Theorem 2.2015 that

15 Observe that 2/α 1 1 = . α/2 uε α L (·) uε L2(·) 2.4 General elliptic operators in divergence form 93

   1/α 1 1 1 ≤ C 1 + α . (2.89) uε Lαn0 (K) r uε Lα (K+B(0,r))

Finally, if −1 < α < 0 we prove in a similar manner to that used to establish (2.89) that   1 1/β ku k 0 ≤ C 1 + β ku k , (2.90) ε Lβn (K) r ε Lβ (K+B(0,r)) with β = −α. We apply Moser’s iterative scheme to (2.89) for completing this step . We obtain

   n/α 1 1 1 ≤ C 1 + α , α > 0. (2.91) uε L∞(K) r uε Lα (K+B(0,r))

Second step. Let x0 ∈ Ω and 0 < r ≤ r0 < dist(x0,Γ )/4. For simplicity conve- n nience, we use in this step B(r) instead of B(x0,r). Let BR be a ball of R with radius R. If R ≤ r we apply (2.88) with K = BR ∩ B(3r) and r substituted by R (note that 16 ∇uε = ∇u and η = 1 in K) . We obtain, with 1 Z w = lnuε − lnuε , |B(3r)| B(3r)

  1 1/2 k∇wk 2 n ≤ C 1 + |B ∩ B(3r) + B(0,R)| L (BR∩B(3r)) R R  1  ≤ C 1 + |B |1/2 R 2R 1 ≤ C(1 + r ) |B |1/2 0 R 2R ≤ MRn/2−1.

Here and until the end of this step, M is a generic constant only depending on the ∞ −1 L -norm of the coefficients of λ L, n and r0. Hence

1/2 k∇wk 1 n ≤ |B ∩ B(3r)| k∇wk 2 n L (BR∩B(3r),R ) R L (BR∩B(3r),R ) ≤ MRn−1.

If R > r then an application of (2.88) with K = BR ∩ B(3r) gives

16 Remark that η in (2.88) is built from K (see formula (2.78)). 94 2 Variational solutions   1 1/2 k∇wk 2 n ≤ C 1 + |B ∩ B(3r) + B(0,r)| L (BR∩B(3r),R ) r R 1 ≤ C(1 + r ) |B |1/2 0 r 4r ≤ Mrn/2−1 and hence n−1 n−1 k∇wk 1 n ≤ Mr ≤ MR . L (BR∩B(3r),R ) That is we proved

n−1 n k∇wk 1 n ≤ MR for any ball B of . L (BR∩B(3r),R ) R R This inequality together with n Theorem 2.22. 17(F. John - L. Nirenberg) Let ω be an open convex subset of R . 1,1 R Let f ∈ W (ω) satisfying ω f = 0. Assume that there exists a constant A > 0 so that n−1 n k| f |k 1 ≤ AR for any ball B of . ∇ L (ω∩BR) R R

Then there exist σ0 > 0 and D > 0, only depending on n, such that Z eσ| f |/Adx ≤ Ddiam(ω)n, ω

−n where σ = σ0|ω|diam(ω) . yield Z eq|w|dx ≤ Mrn. (2.92) B(3r) On the other hand, Z Z eq|w|dx ≥ e−qwdx B(3r) B(3r) Z Z −qlnuε +qm qm −q ≥ e dx = e uε dx, B(3r) B(3r) where 1 Z m = lnuε dx. |B(3r)| B(3r) But by Jensen’s inequality18, ln being concave,

17 A proof can be found in [5, Theorem 7.21 in page 166]. 18 Jensen’s inequality. Let (M ,A , µ) a probability space and ϕ a convex function from R into R. For any µ-integrable and real-valued function f , we have Z  Z ϕ f dµ ≤ ϕ ◦ f dµ. M M 2.4 General elliptic operators in divergence form 95

  1 R q ln 1 R uqdx 1 Z qm |B(3r)| B(3r) lnuε dx |B(3r)| B(3r) ε q e = e ≥ e = uε dx. |B(3r)| B(3r)

Thus Z Z Z  q|w| 1 q −q e dx ≥ uε dx uε dx B(3r) |B(3r)| B(3r) B(3r) and hence

Z 1/q  1/q q|w| 1 1 e dx ≥ kuε kLq(B(3r)) . B(3r) |B(3r)| uε Lq(B(3r)) In light of (2.92), this inequality implies

1 2n/q kuε kLq(B(3r)) ≤ Cr . (2.93) uε Lq(B(3r))

Third step. Let 0 < p < n0. We write (2.90) with β = p/n0, K = B(2r) and r substituted by r/k, k is a positive integer. We get

0   k  p n /p ku k p ≤ C 1 + ku k 0 . ε L (B(2r)) r n0 ε Lp/n (B(2r+r/k))

We have similarly

0 2   k  p (n ) /p ku k p ≤ C 1 + ku k 0 2 . ε L (B(2r+r/k)) r (n0)2 ε Lp/(n ) (B(2r+2r/k))

Hence

0   k  p n /p ku k p ≤ C 1 + ε L (B(2r)) r n0 0 2   k  p (n ) /p × C 1 + ku k 0 2 . r (n0)2 ε Lp/(n ) (B(2r+2r/k))

This inequality yields in a straightforward manner

0 i ! k   k  p (n ) /p ku k p ≤ C 1 + ku k 0 k ε L (B(2r)) ∏ 0 i ε Lp/(n ) ((B(3r)) i=1 r (n ) −(∑k (n0)i)/p ≤ Mr i=1 ku k 0 k ε Lp/(n ) ((B(3r)) n(1−(n0)k)/p ≤ Mr ku k 0 k . ε Lp/(n ) ((B(3r))

If we choose k so that p ≤ q(n0)k then, from Holder’s¨ inequality, we obtain 96 2 Variational solutions

(q(n0)k−p)/(qp) ku k 0 k ≤ |B(3r)| ku k q , ε Lp/(n ) ((B(3r)) ε L ((B(3r)) from which we deduce

n/p−n/q kuε kLp(B(2r)) ≤ Mr kuε kLq((B(3r)). (2.94)

We now combine (2.91) (with α = q and K = B(r)), (2.93) and (2.94) in order to get 1 n/p kuε kLp(B(2r)) ≤ Mr uε L∞(B(r)) and hence n/p kuε kLp(B(2r)) ≤ Mr uε a.e. in B(r). (2.95) The expected inequality follows by passing to the limit, when ε goes to 0, in (2.95). ut

As we have done before, for simplicity convenience, we use respectively sup and inf instead of supess et infess. The following Harnack’s inequality for positive solution follows readily from Theorems 2.20 and 2.24.

1 Theorem 2.23. Let u ∈ Hloc(Ω) be a positive weak solution of Lu = 0 in Ω. Then for any x0 ∈ Ω and 0 < r ≤ r0 < dist(x0,Γ )/4, we have

sup u ≤ C inf u. (2.96) B(x ,r) B(x0,r) 0

∞ −1 where the constant C only depends on the L -norm of the coefficients of λ L, n, r0 and dist(x0,Γ ) − r0. We deduce from this theorem the following result.

1 Corollary 2.7. Assume that Ω is connected. Let u ∈ Hloc(Ω) be a non negative weak solution of Lu = 0 in Ω. Then, for any compact subset K of Ω, we have

supu ≤ C infu. K K where the constant C only depends on the L∞-norm of the coefficients of λ −1L, n and dist(K,Γ ) 19.

Proof. Using the connexity of Ω and the compactness of K, we find N balls B(xi,4ri) contained in Ω, (B(xi,ri)) cover K and

B(xi,ri) ∩ B(xi+1,ri+1) 6= /0, i = 1,...,N − 1.

19 Recall that dist(K,Γ ) = inf{d(x,y); (x,y) ∈ K ×Γ }. 2.4 General elliptic operators in divergence form 97

Let k and ` be two indices so that

supu = sup u and infu = inf u. K K∩B(x ,r ) K K∩B(xk,rk) ` `

Changing the order of the elements of the sequence (xi) if necessary, we may assume that k ≤ `. If k = ` the conclusion is straightforward from (2.96). When k < ` we have, once again from (2.96),

sup u ≤ sup u ≤ C inf u ≤ C sup u. B(x ,r ) K∩B(xk,rk) B(xk,rk) k k K∩B(xk+1,rk+1)

We get by iterating these inequalities

sup u ≤ C`−k+1 inf u ≤ C`−k+1 inf u. B(x ,r ) K∩B(x ,r ) K∩B(xk,rk) ` ` ` `

This completes the proof. ut

We end this subsection by showing how one can use Harnak’s inequality in The- orem 2.24 for non negative super-solutions in order to obtain a strong maximum principle for weak solutions.

1 Theorem 2.24. Let u ∈ Hloc(Ω) be a weak super-solution of Lu = 0 in Ω. Assume that Ω is connected and one of the following two assumptions is satisfied. n i (i) d = −∑i=1 ∂ic . n i (ii) d + ∑i=1 ∂ic ≥ 0 and supΩ u ≥ 0. Then u is constant in Ω or else, for any compact subset K of Ω, we have

supu < supu. K Ω

Proof. Let M = supΩ u. If M = +∞ then the conclusion is straightforward because + ∞ u ∈ Lloc(Ω). Assume now that M < +∞ and let w = M −u. If w vanishes then u is constant. If 1 w does not vanish then clearly w ∈ Hloc(Ω), w ≥ 0 and

Z Z Z n ! i L (w,v)dx = L (M,v)dx = M d + ∑ c ∂i vdx ≥ 0, v ∈ D(Ω), v ≥ 0, Ω Ω Ω i=1

if condition (i) or (ii) holds. In other words, w is a non negative super-solution of Lu = 0 in Ω. We proceed by contradiction by assuming that there exist a compact subset K of Ω so that infK w = 0. Let V be the greatest open set containing K in which w vanishes (a.e.). As Ω is connected and w is non identically equal to zero, Ω ∩ ∂V is nonempty. Therefore, we find a ball B(y,r) ⊂ V so that B(y,2r) ⊂ Ω and ∂B(y,r) ∩ ∂V 6= /0.We conclude by applying Theorem 2.20 that w vanishes on V ∪ B(y,2r) (a.e.). But this contradicts the definition of V and completes the proof. ut 98 2 Variational solutions 2.5 Exercises and problems

2.1. Show that in infinite dimensional Banach space a linear compact operator is never invertible with bounded inverse.

2 2.2. Recall that ` is the Hilbert space of real sequence x = (xm)m≥1 so that 2 ∑m≥1 xm < ∞. This space in endowed with the scalar product hx|yi = ∑m≥1 xmym. Let (am)m≥1 be a real bounded sequence satisfying supm≥1 |am| ≤ C < ∞. Define the linear operator A : `2 → `2 by Ax = (amxm)m≥1. Prove that A is bounded and it is compact if and only if limm→+∞ am = 0. 2.3. Let H = L2(0,1) and let A be the linear operator from H into H defined by (A f )(x) = (x2 + 1) f (x). Check that A is bounded, positive definite i.e. (A f , f ) > 0 for any f ∈ H, f 6= 0, where (·|·) is the scalar product of H and self-adjoint, but it is non compact. Show that A has no eigenvalues. Prove finally that A−λI is invertible with bounded inverse if and only if λ 6∈ [1,2].

2.4. Let E, F be two Banach spaces and A ∈ L (E,F). Assume that E is separable and reflexive. Prove that A is compact if and only if any sequence (xn) in E so that (xn) converges weakly to x ∈ E then the sequence (Axn) converges strongly to Ax. 2.5. Let (X, µ) et (Y,ν) be two measure spaces and k ∈ L2(X ×Y, µ × ν). For f ∈ L2(X, µ), set Z (A f )(y) = k(x,y) f (x)dµ(x). X Show that A is linear bounded operator from L2(X, µ) into L2(Y,ν). We say that A is a kernel operator with kernel k. Prove that if L2(X, µ) is separable this is for instance the case for L2(Ω,dx) n  with Ω an open subset of R and dx is the Lebesgue measure then A is compact. Hint: use Exercise 2.4.

n 2.6. Let Ω be an open subset of R of class C1 with boundary Γ and consider the boundary value problem for the Laplace operator with Neumann boundary condi- tion, where f ∈ C(Ω),

−∆u = f in Ω, ∂ν u = 0 on Γ . (2.97)

Let u ∈ C2(Ω). Show that u is a solution of (2.97) if and only if u satisfies Z Z ∇u · ∇vdx = f vdx for any v ∈ C1(Ω). Ω Ω

Deduce that a necessary condition ensuring the existence of a solution u ∈ C2(Ω) R of (2.97) is Ω f dx = 0. n 2.7. Let Ω be an open bounded subset of R with boundary Γ , f ∈ L2(Ω) and V ∈ n C1(Ω,R ) satisfying div(V) = 0. Demonstrate that there exists a unique variational solution of the following convection-diffusion boundary value problem 2.5 Exercises and problems 99

−∆u +V · ∇u = f in Ω, u = 0 on Γ . (2.98)

n 2.8. Let Ω be a bounded open subset of R of class C1 with boundary Γ . a) Prove that there exist a constant C > 0 so that

  1 kvk 2 ≤ C kvk 2 + k vk 2 n v ∈ H ( ). L (Ω) L (Γ ) ∇ L (Ω,R ) for any Ω

Hint: proceed by contradiction. b) Let f ∈ L2(Ω) and g ∈ L2(Γ ). Prove the existence and uniqueness of a variational solution of the boundary value problem Laplace operator with Fourier boundary condition : −∆u = f in Ω, ∂ν u + u = g on Γ . (2.99) 2.9. Compute the eigenvalues and the eigenfunctions of the Laplace operator with Dirichlet boundary condition in the case where Ω =]0,1[ Note that if ϕ is an eigen- function, then by the elliptic regularity ϕ belongs to C∞([0,1]). Prove by using the spectral decomposition of this operator, that the series ∑k≥1 ak sin(kπx) converges 2 2 1 2 2 in L (0,1) if and only if ∑ak < ∞ and in H (0,1) if and only if ∑k≥1 k ak < ∞.

2.10. Consider the cube Ω =]0,`1[×...×]0,`n[, where `i > 0, 1 ≤ i ≤ n. Compute the eigenvalues and the eigenfunctions of the Laplace operator on Ω with Dirichlet boundary condition. Hint: use the method of separation of variables.

n 2.11. Let Ω a bounded domain of R . Recall the Poincare’s´ inequality : there exists 1 a constant C > 0 so that for any u ∈ H0 (Ω) we have Z Z u2dx ≤ C |∇u|2dx. (2.100) Ω Ω

Prove that the best constant in (2.100) is exactly 1/λ1, where λ1 is the first eigen- value of the Laplace operator on Ω with Dirichlet boundary condition. 2.12. We consider the eigenvalue problem for the Schrodinger¨ operator with a quadratic potential Q(x) = Ax · x, where A is a symmetric positive definite matrix a model of harmonic oscillator

n −∆u + Qu = λu in R . (2.101)

n Let H = L2(R ) and define

1 n 2 n V = {v ∈ H (R ) so that |x|v(x) ∈ L (R )}. (a) Prove that V is a Hilbert space when it is endowed with the scalar product Z Z 2 hu,viV = ∇u · ∇vdx + |x| uvdx. Rn Rn

Hint: if Bi = B(0,i), i = 1,2, we can first establish that there exists a constant C > 0 so that, for any u ∈ V, we have 100 2 Variational solutions Z Z Z  u2dx ≤ C |∇u|2dx + u2dx . B2 B2 B2\B1 (b) Show that the imbedding of V into H is compact and deduce from it that there exists a nondecreasing sequence (λk) of real numbers converging to ∞ and a Hilber- 2 n tian basis (ϕk) of L (R ) consisting respectively of eigenvalues and eigenfunctions associated to the boundary value problem (2.101).

2.13. Let 0 < β < 2π and

2 Ω = {(x,y) ∈ R ; x = r cosθ, y = r sinθ, 0 < r < 1, 0 < θ < β}. Consider the boundary value problem

∆u = 0 in Ω u = u0 on Γ = ∂Ω, (2.102) where  0 if θ = 0,β, u (x,y) = v (r,θ) = 0 0 sin(θπ/β) if r = 1. Use the method of separation of variables to find the explicit solution of (2.102). Then show that this solution belongs to H2(Ω) if and only if β < π.

n 1 2.14. Let Ω be a bounded domain of R of class C with ∂Ω = Γ1 ∪ Γ2, where Γ1 and Γ2 are disjoint and closed. By mimicking the proof of Theorem 2.19, prove that there exists a unique 1 2 bounded operator t0 : H (Ω) → L (Γ1) so that t0w = w|Γ1 if w ∈ D(Ω), and a bounded operator 2 2 2 (t1,t2) : H (Ω) → L (Γ1) × L (Γ2) so that (t1,t2)w = (∂ν w|Γ1 ,∂ν w|Γ2 ) if w ∈ D(Ω). We use in the sequel the notations w|Γ1 and ∂ν w|Γ2 respectively instead of t0w and t2w. Define the vector space

1 V = {u ∈ H (Ω); w|Γ1 = 0}.

(a) (i) Show that V is a closed subspace of H1(Ω). (ii) Demonstrate that there exists a constant C > 0 so that

kwk 2 ≤ Ck wk 2 n w ∈ V. L (Ω) ∇ L (Ω,R ) for all

Pick f ∈ L2(Ω) and consider the boundary value problem   −∆u = f in Ω, u|Γ1 = 0, (2.103)  ∂ν u|Γ2 = 0 and the variational problem : find u ∈ V satisfying 2.5 Exercises and problems 101 Z Z ∇u · ∇vdx = f vdx, for any v ∈ V. (2.104) Ω Ω

(b) Prove that u ∈ V ∩ H2(Ω) is a solution of (2.103) if and only if u is a so- 20 2 lution of (2.104) we can admit that D(Γ2) is dense in L (Γ2) and note that  {w ∈ D(Ω); w|Γ1 = 0} is dense in V . (c) Show that (2.104) admits a unique solution u ∈ V. Consider next the following spectral problem : find the values µ ∈ R for which there exists a solution u ∈ V, u 6= 0, of the problem Z Z ∇u · ∇vdx = µ uvdx for any v ∈ V. (2.105) Ω Ω (d) Show that the eigenvalues of (2.105) consists in a non deceasing sequence con- verging to +∞ : 0 < µ1 ≤ µ2 ≤ ... ≤ µm ≤ ... 2 and there exists a Hilbetian basis (wn) of L (Ω) consisting in eigenfunctions so that Z Z ∇wm · ∇vdx = µm wmvdx for any v ∈ V. Ω Ω

(e) Denote by (λm)m≥1 the sequence of eigenvalues of the spectral problem : find the 1 values of λ ∈ R for which there exists a solution u ∈ H0 (Ω), u 6= 0, of the problem Z Z 1 ∇u · ∇vdx = λ uvdx for all v ∈ H0 (Ω). Ω Ω Check that we have µm ≤ λm, for every m ≥ 1.

2.15. (Holder¨ regularity of weak solutions) Let L be the divergence form differential operator n n ! n i j i i Lu = − ∑ ∂i ∑ a ∂ ju + c u + ∑ d ∂iu + du. i=1 j=1 i=1 Assume that L has bounded coefficients and that conditions (2.66) and (2.67) hold. n i 1 We make additionally the assumption d = −∑i=1 ∂ic . Let u ∈ Hloc(Ω) be a weak solution of Lu = 0 in Ω. Let x0 ∈ Ω and 0 < r0 < dist(x0,Γ )/4. For 0 < r ≤ r0, define

m(r) = inf u, M(r) = supu and ω(r) = M(r) − m(r), B(r) B(r) where B(r) denotes the ball of center x0 and radius r. a) Prove that there exists a constant C > 0, only depending on the L∞-norm of the −1 coefficients of λ L, n and r0, so that

20 n Recall that D(Γ2) = {v = u|Γ2 ; u ∈ D(R )}. 102 2 Variational solutions Z (u − m(4r)) ≤ Crn(m(r) − m(4r)), B(2r) Z (M(4r) − u) ≤ Crn(M(4r) − M(r)). B(2r)

Deduce that C − 1 ω(r) ≤ γω(4r) with γ = . C b) Establish the inequality r r ω(r) ≤ γkω(4r ), 0 < r ≤ 0 , 0 4k 4k−1 and deduce from it that ω(r) ≤ Mrα for some constants M > 0 and α > 0. 2.16. (Weak form of Kato’s inequality) Consider the divergence form operator

n n ! n i j i i Lu = − ∑ ∂i ∑ a ∂ ju + c u + ∑ d ∂iu + du i=1 j=1 i=1 with bounded coefficients and a = (ai j) is positive definite a.e. in Ω. 1 1 Let f ∈ Lloc(Ω) and u ∈ Hloc(Ω) be a weak subsolution of Lu = f . Prove that + 1 v = u ∈ Hloc(Ω) is a weak sub-solution of of Lv = (χ{u>0} + µχ{u=0}) f in Ω for any µ ∈ [0,1]. Hint: one can use first as a test function ϕε = φθ(u/ε), with ε > 0, 0 φ ∈ D(Ω), φ ≥ 0 and θ ∈ C 1(R) satisfying θ ≥ 0, θ(0) = µ, θ = 0 on ] − ∞,−1], θ = 1 on [1,+∞[. Pass then to the limit when ε goes to 0. 2.17. Consider the boundary value problem

 −∆u = F(u) + f in Ω, (2.106) u = 0 on Γ , where F : R → R is continuous and non increasing function satisfying : there exist a > 0, b > 0 so that |F(s)| ≤ a + b|s| for any s ∈ R. 2 1 Let f ∈ L (Ω). We say that u ∈ H0 (Ω) is a variational solution of (2.106) if Z Z Z 1 ∇u · ∇vdx = F(u)vdx + f vdx for all v ∈ H0 (Ω). Ω Ω Ω

1 a) Prove that if u ∈ H0 (Ω) is a variational solution of (2.105) then there exists a constant C > 0, only depending on Ω, F(0) and k f k2, so that

k∇uk2 ≤ C.

Hint: use that s(F(s) − F(0)) ≤ 0 for any s, which is a consequence of the fact that F is non increasing. b) Show that (2.106) has at most one variational solution. Introduce the mapping T : L2(Ω)×[0,1] → L2(Ω) : (w,λ) → T(w,λ) = u, where u is the variational solution of the boundary value problem 2.5 Exercises and problems 103

 −∆u = λ(F(w) + f ) in Ω, u = 0 on Γ . c) Check that T is compact. Then prove with the help of Leray-Schauder’s fixed point theorem21 that T(·,1) possesses a fixed point. Note that a fixed point of T(·,1) is a solution of the variational problem (2.106).

21 Leray-Schauder’s fixed point theorem. Let X be a Banach space and let T : X × [0,1] → X be a compact mapping (i.e. T is continuous and sends the bounded sets of X × [0,1] into relatively compact sets of X). If T(·,0) = 0 and if {x ∈ X; x = T(x,λ) for some λ ∈ [0,1]} is bounded then T(·,1) admits a fixed point. 104 2 Variational solutions References

1.B enilan´ Ph. : L’operateur´ de Laplace : Analyse mathematique´ et calcul numerique,´ R. Dautray et J.-L. Lions eds, Ch. II. Masson, Paris, 1983. 2. Brezis´ H. : Analyse fonctionnelle, theorie´ et applications, 2eme` tirage. Masson, Paris, 1987. 3. Gilbarg D. and Trudinger N. S. : Elliptic partial differential equations of second order, 2nd ed.. Springer, Berlin, 1983. 4. John F. : Partial differential equations. Fourth edition. Springer, New York, 1991. 5. Jost, J.: Partial differential equations. Second edition. Springer, New York, 2007. 6. Han, Q., Lin, F.-H. : Elliptic partial differential equations. American Mathematical Society, Providence, RI, 1997. 7. Ladyzhenskaja O. A. and Ural’tzeva N. N. : Linear and quasilinear elliptic equations. Nauka, Moscow (1964) in Russian ; English translation: Academic Press, New York, 1968 ; 2nd Russian ed., 1973. 8. Lions J.-L. et Magenes E. : Problemes` aux limites non homogenes` et applications, Vol. I. Dunod, Paris, 1968. 9. Raviart P.-A. et Thomas J.-M.: Introduction a` l’analyse numerique´ des equations´ aux deriv´ ees´ partielles. Masson, Paris, 1983. 10. Renardy, M. Rogers, R. C. : An introduction to partial differential equations. Springer, New York, 1993. 11. Sauvigny F. : Partial differential equations. 1. Foundations and integral representations. With consideration of lectures by E. Heinz. Translated and expanded from the 2004 German origi- nal. Springer, Berlin, 2006. 12. Sauvigny F. : Partial differential equations. 2. Functional analytic methods. With consider- ation of lectures by E. Heinz. Second revised and enlarged edition of the 2006 translation. Springer, London, 2012. 13. Willem M. : Analyse fonctionnelle el´ ementaire.´ Cassini, Paris, 2003. 14. Zuily C.: El´ ements´ de distributions et equations´ aux deriv´ ees´ partielles. Dunod, Paris, 2002. Chapter 3 Classical solutions

In this chapter we show existence and uniqueness of classical solutions of elliptic partial differential equations under Dirichlet boundary condition. The approach is based only on interior Schauder estimates without any use of boundary estimates. The original ideas are due to J. H. Michael [2] with an improvement by D. Gilbarg and N. S. Trudinger [5, Section 6.5, p. 112]. The content of this chapter is largely inspired by the lecture notes of a course given by M. V. Safonov at the university of Minesotta during the academic years 2003 and 2004.

3.1 Holder¨ spaces

n k Let Ω be a domain of R , n ≥ 1. As usual, for k ∈ N, C (Ω) denotes the space of continuous functions u in Ω together with their derivatives ∂ `u, |`| ≤ k, where

` `1 `n ∂ = ∂1 ...∂n if ` = (`1,...,`n). We set for convenience ∂ 0u = u. Introduce the notations

` |u|0 = |u|0;Ω = sup|u|, [u]k,0 = [u]k,0;Ω = max|∂ u|0;Ω . (3.1) Ω |`|=k

k, k Define C 0(Ω), where k ∈ N, as the subset of functions u ∈ C (Ω) satisfying

k |u|k = |u|k,0 = |u|k,0;Ω = ∑[u] j,0;Ω < ∞. (3.2) j=0

k,0 It is not hard to check that C (Ω) endowed with the norm | · |k is a Banach space. Let 0 < α ≤ 1. We say that u is Holder¨ continuous, with exponent α, in Ω if the quantity

105 106 3 Classical solutions |u(x) − u(y)| [u] = [u] = α α;Ω sup α (3.3) x,y∈Ω, x6=y |x − y| is finite. Set then ` [u]k,α = [u]k,α;Ω = max[∂ u]α;Ω . (3.4) |`|=k

k, Define the Holder¨ space C α (Ω), k ∈ N and 0 < α ≤ 1, as the Banach space of functions u ∈ Ck(Ω) with finite norm

|u|k,α = |u|k,α;Ω = |u|k,0;Ω + [u]k,α;Ω . (3.5)

We define in a similar manner the Holder¨ Ck,α (Ω). We use for simplicity convenience Cα instead of C0,α , 0 < α < 1. Let u, v ∈ Cα (Ω) with 0 < α ≤ 1. Using the elementary inequality

|u(x)v(x) − u(y)v(y)| ≤ |u(x)||v(x) − v(y)| + |v(y)||u(x) − u(y)|,

(3.1) and (3.3) we easily obtain

[uv]α ≤ |u|0[v]α + |v|0[u]α . (3.6)

k+ , k, Also, if k ∈ N and u ∈ C 1 0(Ω) ∩C 1(Ω) then it is straightforward to check that

[u]k+1,0;Ω ≤ [u]k,1;Ω . (3.7)

n We now establish other inequalities when Ω = Br = {x ∈ R ; |x − x0| < r}, the n ball of center x0 ∈ R and radius r > 0. k+1,0 Let u ∈ C (Br), |`| = k and x, y ∈ Br. Then an application of the mean-value theorem yields ` ` |∂ u(x) − ∂ u(y)| ≤ C|x − y|[u]k+1,0;Br , where the constant C only depends on n. This inequality combined with (3.3) and k+1,0 (3.4) implies, for any u ∈ C (Br), k ∈ N and 0 < α ≤ 1,

1−α [u]k,α;Br ≤ C(n)r [u]k+1,0;Br . (3.8)

k,0 Lemma 3.1. Let u ∈C (Br). Then, for any ball Bρ = Bρ (x) ⊂ Br, ρ > 0 and |`| = k, there exists y ∈ Bρ so that

2k k |∂ `u(y)| ≤ |u| . (3.9) ρ 0,Br

Proof. Let h = ρ/k, |`| = k and consider the operator

` `1 `2 `n δh u(x) = δh,1δh,2 ...δh,n with 3.1 Holder¨ spaces 107

u(x + he j) − u(x) δ u(x) = , h, j h n where (e1,e2,...,en) is the canonical basis of R . Using the mean-value theorem, ` ` we can easily check that there exists y ∈ Bρ so that δh u(x) = ∂ u(y). Inequality (3.9) is then a straightforward consequence of the following estimate 2 |δ u(x)| ≤ |u| . h, j h 0 The proof is then complete. ut

We now prove the following interpolation inequality.

k,α Theorem 3.1. Let j, k ∈ N and 0 ≤ α,β ≤ 1 so that j+β < k+α. Let u ∈ C (Br). Then, for any ε > 0, we have

j+β k+α r [u] j,β;Br ≤ εr [u]k,α;Br +C(ε)|u|0;Br , (3.10) with C(ε) = C(ε,n,k,α,β).

Proof. Making the transformation x → (x − x0)/r we may assume that x0 = 0 and r = 1. We distinguish four cases, where ε > 0 is fixed. (a) The case j = k and 0 = β < α: Fix z ∈ B1, |`| = k and ρ ∈ (0,1). Let x ∈ B1 so that z ∈ Bρ = Bρ (x) ⊂ B1. By Lemma 3.1, there exists y ∈ Bρ so that

|∂ `u(z)| ≤ |∂ `u(z) − ∂ `u(y)| + |∂ `u(y)| 2k k ≤ |z − y|α [u] + |u| k,α ρ 0 2k k ≤ (2ρ)α [u] + |u| . k,α ρ 0

As z ∈ B1 and |`| = k are arbitrary, we conclude that

2k k [u] ≤ (2ρ)α [u] + |u| , 0 < ρ < 1. (3.11) k,0 k,α ρ 0

The expected inequality follows by taking ρ = min(1,ε1/α )/2. (b) The case j = k and 0 < β < α: from definition (3.4), we find |`| = k and x, y ∈ B1 such that

1 |∂ `u(x) − ∂ `u(y)| [u] ≤ ≤ |x − y|α−β [u] . (3.12) 2 k,β |x − y|β k,α

1/(α−β) If |x − y| ≤ (ε/2) we have [u]k,β ≤ ε[u]k,α and (3.10) is satisfied. Otherwise, the first inequality in (3.12) yields 108 3 Classical solutions

−β [u]k,β ≤ 4|x − y| [u]k,0 ≤ C0[u]k,0,

β/(α−β) where C0 = C0(ε) = 4(ε/2) . The last inequality and (3.11) with ρ = 1/α min(1,(ε/C0) )/2 entails (3.10). (c) The case j < k and 0 < α: we find by applying (a) with ε = 1, α = 1 and j in place of k [u] j,0 ≤ [u] j+1,0 +C|u|0.

On the other hand, for 0 < β ≤ 1, [u] j,β ≤ C[u] j+1,0 by (3.8). Hence, in any case (i.e. 0 ≤ β ≤ 1) we have the estimate

[u] j,β ≤ C([u] j+1,0 + |u|0), and by iteration, we get  [u] j,β ≤ C0 [u]k,0 + |u|0 , where C0 = C0(n,k). As before this inequality yields (3.10). (d) The case α = 0: since j + β < α + k = k, we have j ≤ k − 1 and 0 ≤ β ≤ 1. The three preceding cases with α = 1 give

[u] j,β ≤ ε[u]k−1,1 +C(ε)|u|0 (3.13) for all ε > 0. But, from (3.8), we have [u]k−1,1 ≤ C[u]k,0. In light of this estimate, (3.10) is deduced from (3.13) with ε is substituted by ε/C. ut m k,α Corollary 3.1. Let k ∈ N, 0 < α ≤ 1 and (u ) a bounded sequence in C (Br). m Assume that (u (x)) converges for any x ∈ Br. Then

m k,α m u = lim u ∈ C (Br) and |u|k,α ≤ A = sup|u |k,α . (3.14) m→+∞ m

m j,β Furthermore, (u ) converges to u in C (Br) if j + β < k + α. k,α Proof. As before we can assume that r = 1. Since B1 is convex, C (B1) is clearly 0,α continuously imbedded in C (B1). In light of Ascoli-Arzela’s threorem, we deduce m 0 that (u ) converges to u in C (B1). Pick j +β < k +α, ε0 > 0 and set ε = ε0/(4A). Then (3.10) applied to um1 − um2 entails

m1 m2 m1 m2 m1 m2 [u − u ] j,β ≤ ε[u − u ]k,α + N(ε)|u − u |0 ε ≤ 0 +C |um1 − um2 | , 2 0 0 m with C0 =C0(ε0) does not depend of m1 and m2. But the sequence (u ) is convergent 0 in C (B1). Whence, there exists m0 = m0(ε0) an integer so that ε C |um1 − um2 | < 0 for m , m > m . 0 0 2 1 2 0 Thus m1 m2 [u − u ] j,β ≤ ε0 for m1, m2 > m0. 3.1 Holder¨ spaces 109

m j,β In other words, (u ) is a Cauchy sequence in C (B1) and consequently it con- j,β m verges to u in C (B1). We finally note that the estimate |u|k,α ≤ A = supm |u |k,α is straightforward. ut

We need introducing weighted Holder¨ spaces. For k ∈ N, 0 < α ≤ 1, γ ∈ R and u ∈ Ck(Ω) set (γ) (γ) k+α+γ [u]k,α;Ω = [u]k,α = sup d (x)[u]k,α;B(x), (3.15) x∈Ω with 1 d(x) = dist(x,Γ ), B(x) = B (x). (3.16) 2 d(x) Here Γ = ∂Ω. Denote by Ck;γ (Ω) = Ck,0;γ (Ω) the Banach space of functions u ∈ Ck(Ω) with finite norm k (γ) (γ) (γ) kukk,0 = kukk,0;Ω = ∑[u] j,0;Ω . (3.17) j=0 Define the weighted Holder¨ space Ck,α;γ (Ω) as the Banach space of functions u ∈ Ck(Ω) having finite norm

(γ) (γ) (γ) (γ) kukk,α = kukk,α;Ω = kukk,0;Ω + [u]k,α;Ω . (3.18)

k,α k,α;γ Lemma 3.2. Let Ω ⊂ B2r, r ≥ 1 and γ ≥ 0. Then C (Ω) ⊂ C (Ω) and

(γ) k+α+γ kukk,α;Ω ≤ r |u|k,α;Ω . (3.19)

Proof. Follows from (3.15) by observing that d(x) ≤ r. ut

, , Lemma 3.3. Let β, γ ∈ R, 0 < α ≤ 1, u ∈ C0 α;β (Ω) and v ∈ C0 α;γ (Ω). Then

(γ+β) (β) (γ) (γ) (β) [uv]0,α ≤ [u]0,0 [v]0,α + [v]0,0[u]0,α (3.20) and (γ+β) (β) (γ) kuvk0,α ≤ kuk0,α kvk0,α . (3.21) Proof. We have from (3.6)

β+γ β γ γ β d [uv]α;B ≤ d |u|0;B · d [v]α;B + d |v|0;B · d [u]α;B,

where d = d(x) and B = B(x) are given by (3.16). We get then (3.20) by multiplying each side of the last inequality by dα and then taking the sup in x ∈ Ω. We get similarly from |uv|0 ≤ |u|0|v|0

(γ+β) (β) (γ) |uv|0,0 ≤ |u|0,0 |v|0,0. This inequality and (3.20) entail (3.21). ut 110 3 Classical solutions

We show by proceeding as in the preceding proof

(γ) (γ) [u]k,α ≤ C(n)[u]k+1,0, 0 < α ≤ 1, (3.22)

for all u ∈ Ck+1,0;γ (Ω). The following identity follows from the definition of Holder¨ weighted norms:

` (γ+ j) (γ) max[∂ u]k− j,α = [u]k,α , 0 ≤ j ≤ k, 0 ≤ α ≤ 1, (3.23) |`|= j

for all u ∈ Ck,α;γ (Ω). Theorem 3.1 can be used to obtain an interpolation inequality for the weighted k,α;γ Holder¨ spaces C (Ω). In Theorem 3.1, with r = d = d(x), Br = B(x), we multiply each side of (3.10) by dγ and then we take the sup in x ∈ Ω. We get the following result.

Theorem 3.2. Let j, k ∈ N and 0 ≤ α,β ≤ 1 so that j + β < k + α. Let u ∈ k, n C α;γ (Ω), where Ω is bounded domain of R and γ ∈ R. We have, for any ε > 0,

(γ) (γ) (γ) [u] j,β;Ω ≤ ε[u]k,α;Ω +C(ε)|u|0,0;Ω , (3.24)

with C(ε) = C(ε,n,k,α,β).

m k, Corollary 3.2. Let k ∈ N, 0 < α ≤ 1 and let (u ) be a sequence of C α;γ (Ω). As- sume that (um(x)) converges for any x ∈ Ω. Then

m k,α;γ (γ) m (γ) u = lim u ∈ C (Ω) and kukk,α ≤ A = supku kk,α . (3.25) m→+∞ m

This corollary is immediate from Corollary 3.1 applied to the balls Br = B(x) ⊂ Ω. Recall that the norm of C0;γ (Ω) is given by

(γ) γ [u]0,0 = supd (x)sup|u|. (3.26) Ω B(x)

Let us compare this norm with the following one

(γ) (γ) γ kuk = kukΩ = supdx |u(x)|, dx = dist(x,Γ ) = 2d(x). (3.27) Ω We shall use the following elementary inequality 1 d < d < 2d for all x ∈ Ω, y ∈ B(x) = B (x). (3.28) 2 y x y d(x)

(γ) (γ) 0;γ Lemma 3.4. The norms [u]0,0 and kuk are equivalent on C (Ω). We have pre- cisely −γ (γ) (γ) |γ|−γ (γ) 2 kuk ≤ [u]0,0 ≤ 2 kuk . (3.29) 3.2 Equivalent semi-norms on Holder¨ spaces 111

Proof. The first inequality of (3.29) follows from

−γ γ γ γ 2 dx |u(x)| = d (x)|u(x)| ≤ d (x)sup|u|. B(x)

On the other hand, (3.27) and (3.28) imply

−γ (γ) |γ| −γ (γ) |γ|−γ −γ (γ) |u(y)| ≤ dy kuk ≤ 2 dx kuk = 2 d (x)kuk , for all y ∈ B(x), and hence the second inequality in (3.29) holds. ut

Remark 3.1. In the classical Schauder interior estimates (see [5, Chapter 6]), the (γ) notation [u]k,α is used for

|∂ `u(x) − ∂ `u(y)| A = max sup dk+α+γ = sup k+α+γ [u] , (3.30) x,y α δ k,α;Ωδ |`|=k x,y∈Ω |x − y| δ>0 where 0 < α ≤ 1, k + α + γ ≥ 0, dx,y = min(dx,dy) and

Ωδ = {x ∈ Ω; dx = dist(x,Γ ) > δ}. (3.31)

(γ) One can prove that when Ω is a Lipschitz domain the semi-norm [u]k,α given by (3.15) and that given by (3.30) are equivalent whenever k + α + γ ≥ 0. We have in particular (−k−α) C[u]k,α ≤ [u]k,α;Ω ≤ [u]k,α , (3.32) where the constant C only depends on k, α and Ω. In the case where k + α + γ < 0, we have A < ∞ only for polynomials of degree (γ) at most equal to k ; while [u]k,α < ∞ for larger class of functions u. If for instance k+1 k + α + γ < 0 ≤ k + 1 + γ and u ∈ C (B1), then by (3.22) we have

(γ) (γ) [u]k,α ≤ C[u]k+1,0 ≤ C[u]k+1. (3.33)

3.2 Equivalent semi-norms on Holder¨ spaces

n Let Ω be a bounded domain of R and k ∈ N. Denote by Pk the set of all poly- nomials of degree less or equal to k. The Taylor polynomial, of degree k at y ∈ R, corresponding to the smooth function u is given as follows

` ∂ u(y) ` Ty,ku(x) = ∑ (x − y) ∈ Pk. (3.34) |`|≤k `! 112 3 Classical solutions

Lemma 3.5. Let u ∈ Ck,α (Ω), 0 < α ≤ 1. Then, for any x, y ∈ Ω so that [x,y]1 ⊂ Ω, we have k+α |u(x) − Ty,ku(x)| ≤ C(n)[u]k,α |x − y| . (3.35) Proof. By Taylor’s formula there exists ξ ∈ [x,y] so that

` ∂ u(ξ) ` u(x) = Ty,k−1u(x) + ∑ (x − y) . |`|=k `!

Therefore

`u( ) − `u(y) ∂ ξ ∂ ` u(x) − Ty,ku(x) = ∑ (x − y) |l|=k `!

≤ C(n)max ∂ `u(ξ) − ∂ `u(y) |x − y|k. |`|=k

But from (3.4)

` ` α α max ∂ u(ξ) − ∂ u(y) ≤ [u]k,α |ξ − y| ≤ [u]k,α |x − y| . |`|=k

Then result follows. ut k,α Corollary 3.3. Let k ∈ N, 0 < α ≤ 1 and u ∈ C (Bρ ), where Bρ = Bρ (x0). Then

k+α Ek[u;Bρ ] = inf sup|u − p| ≤ C(n)[u]k,α ρ . (3.36) p∈Pk Bρ

k,α Lemma 3.6. Let k ∈ N, 0 < α ≤ 1 and u ∈ C (Bρ ), where Bρ = Bρ (x0). Then, for any ε > 0, we have

−α ` −k−α ρ maxoscBρ ∂ u ≤ ε[u]k,α;Bρ +C(ε)ρ Ek[u;Bρ ], (3.37) |`|=k where C(ε) = C(ε,n,k,α) is a constant and oscX f = supX f − infX f . Proof. Noting that osc f ≤ 2sup| f |, (3.10) with r = ρ, j = k and β = 0 gives

1 −α ` −α −k−α ρ maxoscBρ ∂ u ≤ ρ [u]k,0;Bρ ≤ ε[u]k,α;Bρ +C(ε)ρ sup|u|. 2 |`|=k Bρ

In this inequality we substitute u by u − p, p ∈ Pk. We obtain

1 −α ` −α −k−α ρ maxoscBρ ∂ u ≤ ρ [u]k,0;Bρ ≤ ε[u]k,α;Bρ +C(ε)ρ sup|u − p|. 2 |`|=k Bρ

In the right hand side of the last inequality we take the infimum over p ∈ Pk and then we substitute ε by ε/2. The expected inequality then follows. ut 1 Here [x,y] = {z = tx + (1 −t)y; 0 ≤ t ≤ 1}. 3.2 Equivalent semi-norms on Holder¨ spaces 113

k Theorem 3.3. Let k ∈ N, 0 < α ≤ 1, γ ∈ R so that k + α + γ ≥ 0 and u ∈ C (Ω) (γ) with finite semi-norm [u]k,α (see (3.15)). Set

(γ) (γ) k+α+γ −k−α Mk,α = Mk,α [u;Ω] = sup d (x) sup ρ Ek[u,Bρ (x)], (3.38) x∈Ω ρ∈(0,d(x)]

(γ) with d(x) = dist(x,Γ )/2 and Ek is defined by (3.36). Then the semi-norms [u]k,α and (γ) Mk,α are equivalent. Precisely, we have

(γ) (γ) (γ) C1[u]k,α ≤ Mk,α ≤ C2[u]k,α . (3.39)

Here C1 = C1(n,k,α,γ) > 0 and C2 = C2(n) > 0 are two constants.

Proof. By Corollary 3.3, for any x ∈ Ω and ρ ∈ (0,d(x)], we have

k+α+γ −k−α k+α+γ (γ) d (x)ρ Ek[u,Bρ (x)] ≤ Cd (x)[u]k,α;Bρ (x) ≤ C[u]k,α;Ω , implying the second inequality in (3.39). To prove the first inequality in (3.39), we fix x0 ∈ Ω, d = d(x0) = dist(x0,Γ )/2, |`| = k and x, y ∈ Bd(x0) so that

|∂ `u(x) − ∂ `u(y)| A = [u](γ) ≤ 2dk+α+γ . (3.40) k,α |x − y|α

d d We distinguish two cases: (a) ρ = |x − y| < 2 and (b) ρ = |x − y| ≥ 2 . For case (a), it is not hard to see that x and y belong to a ball Bρ (z) ⊂ B(x0) = Bd(x0). As d/2 ≤ d(z), we deduce by using (3.2)

k+α+γ −α ` A ≤ C0d (z)ρ oscBρ (z)∂ u, (3.41) with C0 = C0(k,α,γ). This inequality is also trivially satisfied in case (b) for z = x0, 1+α ρ = d and C0 = 2 . Hence, in any case (3.41) holds for 0 < ρ ≤ d(z). Using (γ) Lemma 3.6, the definition of A and Mk,α , in order to obtain, for any ε > 0,

k+α+γ k+α+γ −k−α A ≤ C0εd (z)[u]k,α;B(z) +C(ε)d (z)ρ Ek[u,Bρ (z)] (γ) ≤ C0εA +C(ε)Mk,α .

(γ) Upon taking ε = 1/(2C0), the expected inequality follows, i.e. C1A ≤ Mk,α . ut With some simplifications we prove analogously to the preceding theorem the following result.

k n Theorem 3.4. Let k ∈ N, 0 < α ≤ 1 and u ∈ C (R ) having finite semi-norm [u]k,α;Rn (see (3.4)). Let 114 3 Classical solutions

n −k−α Mk,α = Mk,α [u;R ] = sup ρ Ek[u,Bρ (x)]. (3.42) x∈Rn, ρ>0

The semi-norms [u]k,α and Mk,α are then equivalent. Precisely, we have

C1[u]k,α ≤ Mk,α ≤ C2[u]k,α , (3.43) where C1 = C1(n) > 0 and C2 = C2(n,k,α) > 0 are two constant.

3.3 Maximum principle

n In this section Ω is again an open subset of R . We aim to derive some properties of the following linear partial differential operator satisfying the assumptions listed below. n n i j 2 i 2 Lu = ∑ a ∂i ju + ∑ b ∂iu + cu, u ∈ C (Ω). (3.44) i, j=1 i=1 Here ai j, bi et c are continuous functions. We assume in the sequel that the following assumptions are satisfied on a domain varying eventually. (a) (Ellipticity condition) The matrix A = (ai j) is symmetric and there exists a constant ν ∈ (0,1] so that

n 2 i j −1 2 n ν|ξ| ≤ ∑ a (x)ξiξ j ≤ ν |ξ| , x ∈ Ω, ξ ∈ R . (3.45) i, j=1

(b) c ≤ 0 and there exists a constant K ≥ 0 so that

n ∑ |bi(x)| ≤ K, x ∈ Ω. (3.46) i=1 By classical result from linear algebra, as A is symmetric, there exists an orthog- onal matrix P so that P∗AP = Λ, where Λ is the diagonal matrix whose diagonal elements consists in the eigenvalues λ 1,...λ n of A (note that P∗ = P−1). As an n orthogonal transformation leave invariant the Euclidian scalar product on R , for η = P∗ξ, we have ξ = Pη, |ξ| = |η| and

n n i j ∗ ∗ ∗ ∗ k 2 ∑ a ξiξ j = ξ Aξ = η P APη = η Λη = ∑ λ ηk . i, j=1 k=1

n k − Hence (3.45) is valid for any ξ ∈ R if and only if λ ∈ [ν,ν 1], for any k = 1,2,...,n. Using these facts, it is not hard to deduce from (3.45) that 3.3 Maximum principle 115

aii ≥ ν, |ai j| ≤ ν−1, i, j = 1,2,...,n, 2 (3.47)

n n i j ii 2 n 3 ∑ a ξiξ j ≤ ∑ a |ξ| , ξ ∈ R . (3.48) i, j=1 i=1 Also, the identity P∗AP = Λ implies A = PΛP∗ and then

n i j k k k a = ∑ λ ξi ξ j , (3.49) k=1

∗ where ξ1,...ξn are the column vectors of P .

∞ Lemma 3.7. Fix r > 0. Then there exists v0 ∈ C (Br),Br = Br(0), so that

Lv0 ≤ −1 in Br. (3.50)

Furthermore,

0 < v0 ≤ C0 = C0(ν,K,r) in Br, v0 = 0 on ∂Br. (3.51)

Proof. Consider the function cosh(λ|x|), λ > 0 (of class C∞). Then

−1 ∂i cosh(λ|x|) = λ sinh(λ|x|)ξi, with ξ = |x| x, 2 2 −1 ∂i j cosh(λ|x|) = λ cosh(λ|x|)ξiξ j + λ|x| sinh(λ|x|)(δi j − ξiξ j).

Noting that |ξ| = 1, sinht < cosht and using (3.45) to (3.48), we obtain

n n ! 2 (L − c)cosh(λ|x|) = ∑ ai j∂i j + ∑ bi∂i cosh(λ|x|) i, j=1 i=1 n n 2 ≥ λ cosh(λ|x|) ∑ ai jξiξ j + λ sinh(λ|x|) ∑ biξi i, j=1 i=1 ≥ cosh(λ|x|)(λ 2ν − λK) ≥ λ(λν − K) ≥ 1,

2 ii k i j −1 To get a ≥ ν it is enough to take ξ = (δi ) in (3.45). While to prove |a | ≤ ν we proceed as follows: if we take in (3.45), where k and ` are given, ξi = 0 for i 6= k and i 6= `, then

kk 2 `` 2 k` −1 2 a ξk + a ξl + 2a ξkξ` ≤ ν |ξ| and hence (aii ≥ ν for each i) kl −1 2 2a ξkξl ≤ ν |ξ| . The result follows for ξ = √1 sgn(ak`) and ξ = √1 . k 2 ` 2 n ∗ 3 For ξ ∈ R and η = P ξ, we have

n n n n n i j k 2 k 2 k 2 2 kk 2 ∑ a ξiξ j = ∑ λ ηk ≤ ∑ λ |η| = ∑ λ |ξ| = Tr(A)|ξ| = ∑ a |ξ| . i, j=1 k=1 k=1 k=1 k=1 116 3 Classical solutions for a well chosen λ = λ(ν,K). Set v0 = cosh(λr)−cosh(λ|x|). Clearly, v0 satisfies to (3.51) with C0 = cosh(λr). On the other hand, as c ≤ 0, we have

Lv0 ≤ (L − c)v0 = −(L − c)cosh(λ|x|) ≤ −1 in Br.

That is v0 satisfies also to (3.50). ut

Theorem 3.5. (Weak maximum principle) Assume that c = 0. Let u ∈ C2(Ω)∩C(Ω) satisfying Lu ≥ 0 (Lu ≤ 0) in Ω. Then   supu = supu infu = infu . (3.52) Ω Γ Ω Γ

Here Γ = ∂Ω.

Proof. We first claim that if Lu > 0 in Ω then u can not attain it maximum at a point in Ω. Otherwise, we would find x0 ∈ Ω (where u attains its maximum) so that ∂iu(x0) = 0, 1 ≤ i ≤ n, and

n 2 n ∑ ∂i ju(x0)ξiξ j ≤ 0 for any ξ ∈ R . i, j=1

This inequality together with (3.49) entail

i j 2 k 2 k k Lu(x0) = ∑a ∂i ju(x0) = ∑λ ∑∂i ju(x0)ξi ξ j ≤ 0. i j k i j

But this contradicts the fact that Lu > 0 in Ω. In the case Lu ≥ 0, let v0 be the function in Lemma 3.7 with Br ⊃ Ω. We have then, for all ε > 0, L(u − εv0) ≥ ε > 0. Whence

sup(u − εv0) = sup(u − εv0) Ω Γ from the preceding case. Upon passing to the limit as ε goes to zero, we end up getting supu = supu. Ω Γ The proof is then complete. ut

+ An application of Theorem 3.5 to L0u = Lu−cu ≥ −cu ≥ 0 in Ω = {u > 0} ⊂ Ω gives, where u± = max(±u,0), the following corollary.

Corollary 3.4. Let u ∈ C2(Ω) ∩C(Ω) satisfies Lu ≥ 0 (Lu ≤ 0) in Ω. Then   supu+ = supu+ supu− = supu− . (3.53) Ω Γ Ω Γ 3.3 Maximum principle 117

This corollary applied to u − v yields the following result.

Theorem 3.6. (Comparison principle) Let u, v ∈ C2(Ω) ∩C(Ω) so that Lu ≥ Lv in Ω and u ≤ v on Γ . Then u ≤ v in Ω. In particular, Lu = Lv in Ω and u = v on Γ imply that u = v in Ω.

A consequence of this theorem is

Theorem 3.7. Let f ∈ C(Ω) and u ∈ C2(Ω) ∩C(Ω) so that Lu = f in Ω. Then

sup|u| ≤ sup|u| +C0 sup| f |, (3.54) Ω Γ Ω

where C0 = C0(ν,K,diam(Ω)) > 0 is a constant.

Proof. We have Ω ⊂ Br for some ball Br = Br(x0), r = diam(Ω). Let

v(x) = sup|u| + sup| f |v0(x − x0), Γ Ω

where v0 is as in Lemma 3.7. We find by using (3.50) and (3.51)

Lv(x) ≤ sup| f |Lv0(x − x0) ≤ −sup| f | ≤ ± f (x) = ±Lu(x), Ω Ω

and v ≥ |u| on Γ . Hence |u| ≤ v in Ω by the comparison principle. Therefore (3.54) is satisfied with the same constant C0 as in (3.51). ut

Let us now introduce the notion of sub-solution and super-solution. A function w ∈ C(Ω) is said a sub-solution (resp. super-solution) of Lu = f in Ω if for any ball 2 B b Ω and any function v ∈ C (B) ∩C(B) such that Lv < f (resp. Lv > f ) in B, the inequality v ≥ w (resp. v ≤ w) on ∂B implies v > w (resp. v < w) in B. Observe that w is a sub-solution of Lu = f if and only if −w is a super-solution of Lu = f . Therefore it is enough to consider sub-solutions.

Remark 3.2. If ai j, bi, c, f belong to C(Ω) then any sub-solution w ∈C2(Ω) satisfies Lw ≥ f in Ω. Otherwise, we would find a ball B b Ω so that Lw < f in B. The choice of v = w contradicts then the definition of a sub-solution. The converse is also true without the continuity condition on ai j, bi, c and f . Indeed, let w ∈ C2(Ω) so that 2 Lw ≥ f in Ω. Let B b Ω a ball and v ∈ C (B) ∩C(B) such that Lv < f and v ≥ w on ∂B. As L(w − v) > 0, the function w − v can not attain, by Theorem 3.5, its maximum at a point in Ω. Therefore

w − v < sup(w − v) ≤ 0 in B. ∂B

Lemma 3.8. Let w ∈ C(Ω) be a sub-solution of Lu = f in Ω. Then, for any v ∈ C2(Ω) ∩C(Ω) so that Lv < f in Ω, the inequality v ≥ w on Γ implies v > w in Ω. If Lv ≤ f in Ω and v ≥ w on Γ then v ≥ w in Ω. 118 3 Classical solutions

Proof. We proceed by contradiction. Assume then that there exists v ∈ C2(Ω) ∩ C(Ω) so that Lv < f in Ω, v ≥ w on Γ and that v > w in Ω does not hold. Hence, we find y ∈ Ω so that

0 ≥ −µ = min(v − w) = v(y) − w(y). (3.55) Ω

For a ball Br = Br(y) b Ω, we have

L(v + µ) = Lv + cµ ≤ Lv < f in Br, v + µ ≥ w on ∂Br.

But w is a sub-solution. Then v + µ > w in Br and hence v(y) + µ > w(y). This inequality contradicts (3.55) and consequently v > w in Ω. In the case Lv ≤ f in Ω, consider the function v0 given by the Lemma 3.7, which ε is defined in Br = Br(0) ⊃ Ω. Thus the function v = v + εv0, ε > 0, satisfies

Lvε ≤ f − ε < f in Ω, vε ≥ v ≥ w on Γ .

Therefore vε > w in Ω by the preceding case. We get v ≥ w in Ω, upon passing to the limit as ε goes to 0. ut

Theorem 3.8. Let w ∈ C(Ω) and assume that, for any y ∈ Ω, there exists a sub- solution wy of Lu = f in a ball By so that

y y y y y ∈ B b Ω, w ≤ w in B , w (y) = w(y). (3.56) Then w is a sub-solution of Lu = f in Ω.

2 Proof. If the result is not true we would find a ball B b Ω and v ∈ C (B) ∩C(B) so that Lv < f in B and v ≥ w on ∂B ; but the inequality v > w does not hold in B. Thus, there exists y ∈ B so that

0 ≥ −µ = min(v − w) = v(y) − w(y). (3.57) B

y Fix a ball Br = Br(y) b B ∩ B . Then L(v + µ) = Lv + cµ ≤ Lv in By, v + µ ≥ w ≥ wy on ∂By

y y y and v + µ > w in Br (because w is a sub-solution of Lu = f in B c Br). In par- ticular, v(y) + µ > wy(y) = w(y), which contradicts (3.57) and completes the proof. ut

Corollary 3.5. Let w1, w2 be two sub-solutions of Lu = f in the respective domains Ω1, Ω2. Assume that

w1 ≥ w2 on Ω1 ∩ ∂Ω2 and w2 ≥ w1 on Ω2 ∩ ∂Ω1. (3.58)

Then w defined in Ω1 ∪ Ω2 by 3.3 Maximum principle 119   w1(x), x ∈ Ω1\Ω2, w(x) = w2(x), x ∈ Ω2\Ω1, (3.59)  max(w1(x),w2(w)), x ∈ Ω1 ∩ Ω2, belongs to C(Ω) and it is a sub-solution of Lu = f in Ω.

We now use this corollary to construct special sub-solutions of the equation

β−2 Lu = d in Ω, with d = dx = dist(x,∂Ω), 0 < β < 3. (3.60)

We first consider the particular case Ω = BR = BR(0), with R > 0 fixed. In the sequel, d = R − |x|, x ∈ BR.

Lemma 3.9. Fix 0 < β < 1 and R > 0. Then, for any Ω = BR = BR(0), there exists a sub-solution w ∈ C(Ω) of the equation (3.60) so that w = 0 on Γ and

β 0 ≥ w ≥ −C1d in Ω, (3.61) where C1 = C1(ν,K,β,R) is a constant.

β β−1 Proof. Since d = R − |x|, we have ∂id = −βd ξi and

2 β β−2 −1 β−1 ∂i jd = β(β − 1)d ξiξ j + β|x| d (ξiξ j − δi j), with ξ = |x|−1x. As |ξ| = 1, we get from (3.45) to (3.48)

n n ! β i j 2 i β Ld ≤ ∑ a ∂i j + ∑ b ∂i d i, j=1 i=1 n n β−2 i j β−1 i ≤ β(β − 1)d ∑ a ξiξ j − βd ∑ b ξi i, j=1 i=1 ≤ βdβ−2[(β − 1)ν + Kd], where we used that β − 1 < 0. In light of the last inequality, we can choose two constants β0 > 0 and δ0 ∈ (0,R/2), depending on ν , K, β and R, in such a way that

β β−2 Ld < −β0d , 0 < d < 2δ0. (3.62)

∞ On the other hand, there exists, by Lemma 3.7, v0 ∈ C (BR) so that

Lv0 ≤ −1, 0 ≤ v0 ≤ C0 (3.63) in Ω, for some constant C0 = C0(ν,K,R). Set β β β−2 w1 = −C1d , w2 = −C1δ0 − δ0 v0, (3.64) −1 β −1 −2  with C1 = max β0 ,(2 − 1) δ0 C0 , and decompose Ω as follows 120 3 Classical solutions

Ω = Ω1 ∪ Ω2, with Ω1 = Ω ∩ {d < 2δ0}, Ω2 = Ω ∩ {d > δ0}. (3.65)

We obtain from (3.62) and (3.63)

β−2 β−2 β−2 β−2 Lw1 ≥ C1β0d ≥ d in Ω1, Lw2 ≥ δ0 ≥ d in Ω2, β−2 w1 − w2 = δ0 v0 ≥ 0 on Ω1 ∩ ∂Ω2 = {d = δ0}, β β β−2 β−2 w2 − w1 ≥ C1(2 − 1)δ0 − δ0 v0 ≥ δ0 (C0 − v0) ≥ 0 in Ω\Ω1.

β−2 Therefore w1 and w2 satisfy the assumptions of Corollary 3.5 with f = d . Whence, w defined by (3.59) is a sub-solution of Lu = dβ−2 in Ω. Note that w2 ≥ w1 in Ω\Ω1 yields

β 0 ≥ w ≥ w1 ≥ −C1d in Ω.

In other words, we proved (3.61). We end up the proof by remarking that w = w1 = 0 on Γ . ut

We next extend Lemma 3.9 to domains having the exterior sphere property. We say that Ω has the exterior sphere property if for any y ∈ ∂Ω there exists a ball n B = BR(z) ⊂ R \ Ω satisfying Ω ∩ B = {y}, where R > 0 does not depend on y. Theorem 3.9. Lemma 3.9 can be extended to a bounded domain of Ω possessing the exterior sphere property, with a constant C1 = C1(ν,K,β,R,diam(Ω)) in (3.61).

Proof. Assume that Ω has the exterior sphere property. Then

d = dx = dist(x,Γ ) = min(|x − z| − R), x ∈ Ω, z∈Z

n where Z = {z ∈ R ; dist(z,Ω) = R}. If h(x) = |x| − R then

β β β d = dx = minh (x − z). (3.66) z∈Z

We prove similarly to (3.62)

β β−2 Lh < −β0h , 0 < h < 2δ0, (3.67) with constants β0 > 0 and δ0 only depending on ν, K, β and R. y Fix y ∈ Ω1 = Ω ∩{d < 2δ0}, choose z ∈ Z such that h(y−z) = dy and set w (x) = −hβ (x − z). Then (3.67) is equivalent to

y β−2 Lw (x) > β0dx (3.68)

y y at x = y. By continuity, (3.68) remains true in a ball B so that y ∈ B b Ω1. Moreover

y β y y β w ≤ −d in B , w (y) = −dy . 3.4 Some estimates for Harmonic functions 121

β β−2 We conclude from Theorem 3.8 that −d is a sub-solution of Lu = β0d in Ω1. We can then substitute (3.62) by this inequality in the proof of Lemma 3.9. On the other hand, (3.63) holds in a Br ⊃ Ω, r = diam(Ω). The remaining part of the proof is identical to that of Lemma 3.9. ut

Corollary 3.6. Under the assumptions of Theorem 3.6, if u ∈ C2(Ω) ∩C(Ω) is a solution of Lu = f in Ω and u = 0 sur Γ then

(−β) (2−β) kukΩ ≤ C1k f kΩ , (3.69) where 0 < β < 1,C1 = C1(ν,K,β,R,diam(Ω)) is the constant in (3.61) and the norms k · k(γ) are defined in (3.27).

Proof. Set (2−β) 2−β A = k f kΩ = supdx | f (x)|. Let w be a sub-solution of Lu = dβ−2 in Ω given by Theorem 3.9. As Adβ−2 ≥ f , Aw is a sub-solution of Lu = f in Ω. We have u = w = 0 on Γ . Hence, u ≥ Aw ≥ β −C1Ad in Ω by Lemma 3.8 and (3.61). Since the inequalities above hold trivially when u is substituted by −u we get β that |u| ≤ C1Ad in Ω. That is, we have

(−β) −β kuk = supd |u| ≤ C1A Ω as expected. ut

3.4 Some estimates for Harmonic functions

n Let Ω be a domain of R . We say that u ∈ C2(Ω) is harmonic (sub-harmonic, super- harmonic) in Ω if it satisfies

∆u = 0 (≥ 0, ≤ 0) in Ω. (3.70)

In other words, (sub, super) harmonic functions are (sub, super) solutions of the Laplace equation ∆u = 0. Since div(∇u) = 0 in Ω, we have according to the diver- gence theorem Z Z Z ∂ν uds = ∇u · νds = ∆udx = 0 (3.71) ω ∂ω ω for any Lipschitz domain ω b Ω, where ν is the exterior unit normal vector field on ∂ω. A consequence (3.71) is the following mean-value theorem

Theorem 3.10. Let u ∈ C2(Ω) satisfies ∆u = 0 in Ω. Then, for any ball B = Br(x0) b Ω, we have 1 Z u(x0) = n−1 uds, (3.72) ωnr ∂B 122 3 Classical solutions

n/2 n−1 where ωn = 2π /Γ (n/2) is the Lebesgue measure of S . The proof of this theorem will be given in Chapter 4. We going to show that the mean-value theorem can be used to establish interior n regularity of harmonic functions. We fix ζ ∈ C∞(R ) depending only on |x| such that Z n n ζ ≥ 0 in R , ζ = 0 in R \ B1(0) and ζ(x)dx = 1. (3.73) Rn 1 Let u ∈ Lloc(Ω), ε > 0 and recall the definition Z Z u(ε)(x) = u(x − εy)ζ(y)dy = ε−n u(z)ζ(ε−1(x − z))dz, (3.74) B1(0) Ω for x ∈ Ωε = {x ∈ Ω; dx = dist(x,Γ ) > ε}.

(ε) ∞ Lemma 3.10. (a) We have, for any ε > 0, u ∈ C (Ωε ) and

(ε) ` (ε) −k [u ]k,0;Ωε = maxsup|∂ u | ≤ Cε sup|u|, k ∈ N, (3.75) |`|=k Ωε Ω where C = C(n,k) is a constant. (b) Assume that |u(x) − u(y)| ≤ ω(ε) for any x, y ∈ Ω such that |x − y| ≤ ε. Then

sup|u(ε) − u| ≤ ω(ε). (3.76) Ωε In particular, (ε) α sup|u − u| ≤ ε [u]α;Ω , 0 < α ≤ 1. (3.77) Ωε k ` (ε) ` (ε) (c) If u ∈ C (Ω) then ∂ u = (∂ u) in Ωε , for any |`| ≤ k. Moreover,

(ε) |u |k,α;Ωε ≤ |u|k,α;Ω , k ∈ N, 0 ≤ α ≤ 1. (3.78) In (3.77) and (3.78) we used the same notations as in (3.1) to (3.4).

Proof. This lemma is a consequence of (3.74). Indeed, we have, for |`| = k and x ∈ Ω , ε Z ∂ `u(ε)(x) = ε−n−k u(z)∂ `ζ(ε−1(x − z))dz Bε (x) and hence ` (ε) −n−k |∂ u (x)| ≤ ε mes(Bε (x))[ζ]k,0 sup|u|. Ω

Whence (3.75) follows with C = mes(B1)[ζ]k,0. While (3.76) is a consequence of Z (ε) u (x) − u(x) = [u(x − εy) − u(x)]ζ(y)dy, x ∈ Ωε . B1 Finally, (3.78) is immediate from the first inequality in (3.74). ut 3.4 Some estimates for Harmonic functions 123

(ε) Lemma 3.11. If u is harmonic in Ω then u = u in Ωε , ε > 0. Furthermore, u ∈ C∞(Ω) and k ` maxsupdx |∂ u| ≤ C sup|u|, k ∈ N, (3.79) |`|=k Ω Ω where C = C(n,k) is a constant.

Using the semi-norms defined in (3.15) and (3.16) we can rewrite (3.79) in the form (0) [u]k,0;Ω ≤ C sup|u|, k ∈ N. (3.80) Ω Proof. We have

Z Z 1 Z (ε) u (x) = u(x − εy)ζ(y)dy = dr u(x − εy)ζ(y)dsy, (3.81) B1 0 |y|=r

for x ∈ Ωε . As ζ is constant on {|y| = r} = ∂Br(0), we get from (3.72) Z Z n−1 u(x − εy)dsy = u(x)ωnr = u(x) dsy. |y|=r |y|=r

Hence, we find by using R ζ(y)dy = 1

Z 1 Z Z (ε) u (x) = u(x) dr ζ(y)dsy = u(x) ζ(y)dy = u(x), x ∈ Ωε . 0 |y|=r B1(0)

We complete the proof by noting that (3.79) is obtained by taking ε = dx = dist(x,Γ ) in (3.75). ut

The remaining part of this section is devoted to the Dirichlet problem in a ball, for an operator that can be deduced of the Laplace operator by an orthogonal trans- formation. Consider then an operator of the form

n 0 i j 2 L u = ∑ a ∂i ju, i, j=1

where ai j are constants, for each i, j and the matrix (ai j) is symmetric and posi- tive definite. By diagonalizing the matrix (ai j) one can easily check that L0 can be transformed into the Laplace operator by means of a change of variable. We study first the following Dirichlet problem

n 0 i j 2 L u = ∑ a ∂i ju = f in Br = Br(x0), u = ϕ on ∂Br, (3.82) i, j=1

when f and ϕ are polynomials. 124 3 Classical solutions

4 Lemma 3.12. Let f ∈ Pk and ϕ ∈ Pk+2 be given . Then (3.82) admits a unique 2 2 2 solution u in C (Br)∩C(Br). This solution takes the form u = ϕ +(r −|x| )g, with g ∈ Pk.

Proof. We may assume, without loss of generality, that r = 1 and x0 = 0. The uniqueness is contained in 3.6 (comparison principle). To prove the existence we set u = v + ϕ. Then (3.82) is transformed into the equation

0 L v = f0 in B1, v = 0 on ∂B1, (3.83)

0 with f0 = f − L ϕ ∈ Pk. Consider then the linear map

0 2  T : Pk → Pk : p → T p = L 1 − |x| p .

If T p = 0 then u = 1 − |x|2 p is a solution (3.82) with f = 0 and ϕ = 0. By unique- ness u = 0 and hence p = 0. Therefore T is injective, and since Pk is of finite di- mension, T is also surjective, i.e. TPk = Pk. Hence, Tg = f0, for some g ∈ Pk, and v = 1 − |x|2g is a solution of (3.83). ut

This lemma will be useful to extend the existence of solutions of (3.82) for f and ϕ in a larger class than polynomials.

m Lemma 3.13. Let 0 < α ≤ 1, γ ∈ R and let (u )m≥1 be a bounded sequence in 2,α;γ m (γ) C (Ω), i.e. ku k2,α ≤ A for any m, for some constant A > 0. Assume that the following limits exist

u(x) = lim um(x), f (x) = lim Lum(x) for any x ∈ Ω. m→∞ m→∞

2,α;γ (γ) Then u ∈ C (Ω), kuk2,α ≤ A and Lu = f in Ω.

m Proof. Fix x ∈ Ω. Let d = dist(x,Γ )/2 and B = Bd(x). As (u )m≥1 is bounded in C2,α;γ (Ω), it is also bounded in C2,α (B) and converges to u. By Corollary 3.1, we have u ∈ C2,α (B) and um → u in C2,α (B). Whence

Lu = lim Lum = f in Ω. m→∞ The rest of the proof is contained in Corollary 3.2. ut

Theorem 3.11. Let Br = Br(x0) and ϕ ∈ C(Br). Then the Dirichlet problem

n 0 i j 2 L u = ∑ a ∂i ju = 0 in Br, u = ϕ on ∂Br (3.84) i, j=1

∞ admits a unique solution u ∈ C (Br) ∩C(Br). Furthermore, estimates (3.79) and (3.80) hold with Ω = Br and C = C(n,ν,k).

4 Recall that Pk is the vector space of polynomials of degree less or equal to k. 3.4 Some estimates for Harmonic functions 125

Proof. From Stone-Weierstrass’s theorem, we find a sequences of polynomials (ϕm) so that |ϕ −ϕm| ≤ 1/m in Br. We conclude then, by applying Lemma 3.12, that there exists a sequence of polynomials (um) satisfying

0 L um = 0 in Br, um = ϕm on ∂Br. (3.85)

The comparison principle then yields 1 1 sup|ui − u j| ≤ sup|ϕi − ϕ j| ≤ + −→ 0 as i, j → ∞. Br ∂Br i j

That is (um) is a Cauchy sequence in C(Br). Therefore it converges to some u ∈ C(Br). As we have said above there exists a bijective linear map y = Ax transform- 0 ing L v(x) = 0 in Br to ∆v(y) = 0 in Ω = A(Br). Therefore, estimates (3.79) and (3.80) for v(y) in Ω produce analogous estimates for v(x) in Br, with a con- stant C = C(n,ν,k). In particular, these estimates are valid for each um with a constant C independent of m. Passing to the limit, as m goes to infinity, we get ∞ ` u = limum ∈ C (Br)-note that according to (3.79), (∂ um) is a Cauchy sequence for each `- satisfies the same estimates as um. We end up the proof by using Lemma 0 0 3.13, with γ = 0 and α = 1. We obtain L u = limL um = 0. ut

n n Corollary 3.7. Let r > 0, x0 ∈ R0 = {x ∈ R ; xn = 0} and set + + Br = Br (x0) = {x ∈ Br(x0); xn > 0}. (3.86)

 + n (a) For any ϕ ∈ C Br satisfying ϕ = 0 on Γ = R0 ∩Br(x0), the Dirichlet problem

+ + ∆u = 0 in Br , u = ϕ on ∂Br (3.87)

∞ +  + admits a unique solution u ∈ C (Br ∪Γ ) ∩C Br . Moreover,

−k [u]k,0;B+ ≤ C(n,k)r sup|u|, k ∈ N. (3.88) r/2 + Br

 + + (b) For any ϕ ∈ C Br , the equation ∆u = 0 in Br with boundary condition

+ ∂nu = 0 on Γ , u = ϕ on ∂Br \Γ (3.89)

∞ +  + admits a unique solution u ∈ C (Br ∪Γ )∩C Br and estimate (3.88) is satisfied.

Proof. (a) We extend ϕ, on ∂Br, to the odd function, still denoted by ϕ, defined by

+ ϕ(x1,...,xn−1,−xn) = −ϕ(x1,...,xn−1,xn), (x1,...,xn−1,xn) ∈ Br \Γ . 126 3 Classical solutions

The result follows then by using existence, uniqueness and estimates (3.79) and (3.80) for the solution of the Cauchy problem

∆u = 0 in Br, u = ϕ on ∂Br.

(b) We proceed similarly to (a) by using in that case the even extension of ϕ, still denoted by ϕ :

+ ϕ(x1,...,xn−1,−xn) = ϕ(x1,...,xn−1,xn), (x1,...,xn−1,xn) ∈ Br \Γ .

The proof is then complete. ut

3.5 Interior Schauder estimates

n Let Ω be a domain of R . As in the preceding section, n n i j 2 i L = ∑ a ∂i j + ∑ b ∂i + c, (3.90) i, j=1 i=1

We assume additionally to the assumptions in the preceding section that the coef- ficients of L satisfy the following regularity condition : there exist 0 < α < 1 and K1 > 0 so that i j (0) i (1) (2) maxka k , maxkb k , kck ≤ K1, (3.91) i, j 0,α i 0,α 0,α

(γ) where the norm k · k0,α is defined in (3.18).

, , + Theorem 3.12. For any γ ∈ R and u ∈ C2 α;γ (Ω), we have f = Lu ∈ C0 α;γ 2(Ω) and (γ+2) (γ) k f k0,α ≤ C1kuk2,α . (3.92)

Here C1 = C1(n,K1) is a constant. Proof. We have by (3.23)

` (γ+ j) (γ) k∂ ukk− j,α ≤ kukk,α , 0 ≤ j ≤ k, 0 < α ≤ 1. (3.93) This estimate, combined with (3.21) (Lemma 3.3) and (3.93), implies

i j 2 (γ+2) i j (0) 2 (γ+2) (γ) ka ∂i juk0,α ≤ ka k0,α k∂i juk0,α ≤ K1kuk2,α , for any i, j. In an analogous way, we have also in light of (3.21)

i (γ+2) i (1) (γ+1) (γ) (γ) kb ∂iuk0,α ≤ kb k0,α k∂iuk0,α ≤ K1kuk1,α ≤ Ckuk2,α , (γ+2) (2) (γ+2) (γ) kcuk0,α ≤ kck0,α kuk0,α ≤ Ckuk2,α , 3.5 Interior Schauder estimates 127 where C = C(n,K1) is a constant. The last three estimates yield (3.92). ut

Remark 3.3. Note that for the lower order terms we have in light of (3.22)

(γ+2) n i (γ) ∑ b ∂iu + cu ≤ N(n,K1)kuk2,0. (3.94) i=1 0,α We now give a result saying that for an operator with Holder¨ continuous coeffi- cients the norms of u and f = Lu in (3.92) are ”almost” equivalent.

, Theorem 3.13. We have, for any γ ∈ R and u ∈ C2 α;γ (Ω),

(γ)  (γ) (γ+2) kuk2,α ≤ C [u]0,0 + [ f ]0,α , (3.95)

5 where f = Lu and C = C(n,ν,K,K1,α,γ) is a constant.

Proof. In this proof C is generic constant only depending on n, ν, K, K1, α and γ. On the other hand, to simply the notations, we set

(γ) (γ) (γ+2) (γ+2) U2,α = [u]2,α , Uk = [u]k,0, Fα = [ f ]0,α , F0 = [ f ]0,0 . (3.96)

, i Let γ ∈ R and u ∈ C2 α;γ (Ω) be given. Assume first that b = 0 and c = 0 in Ω. Fix 1 1 y ∈ Ω, d = d = dist(y,Γ ), ρ ∈ (0,d] and ε ∈ (0, ]. y 2 2 Let r = ρ/ε. We distinguish two cases: (a) r ≤ d and (b) r > d. In case (a), let

n i j i j 0 i j 2 a0 = a (y), L = ∑ a0 ∂i j, ϕ = u − Ty,2u, i, j=1 and denote by v the solution of the problem

n 0 i j L v = ∑ a0 Di jv = 0 in Br = Br(y), v = ϕ on ∂Br. (3.97) i, j=1

∞ By Theorem 3.7 v ∈ C (Br) ∩C(Br) and

−3 −3 [v]3,0;Br/2 ≤ Cr sup|v| = Cr sup|ϕ|. Br ∂Br

As ρ = εr ≤ r/2, we apply first Corollary 3.3, (3.8) to v in Bρ and then apply Lemma 3.5 to u in Br. We then get

5 Here ν, α, K et K1 are the constants appearing in the assumptions on the coefficients of L. 128 3 Classical solutions

−2−α 1−α 1−α −3 ρ E2[v;Bρ ] ≤ Cρ [v]3,0;Br/2 ≤ Cρ r sup|ϕ| ∂Br 1−α α−1 1−α ≤ Cρ r [u]2,α;Br = Cε [u]2,α;Br .

(γ) Since r ≤ d, we obtain from the definition of U2,α = [u]2,α (see (3.15))

2+α+γ −2−α 1−α d ρ E2[v;Bρ ] ≤ Cε U2,α . (3.98)

Next, we estimate ϕ − v in Br. We have

n 0 0 i j 2 2 L (ϕ − v) = L ϕ = ∑ a0 [∂i ju(x) − ∂i ju(y)]. i, j=1

n i j 2 We get by noting that f = ∑i, j=1 a ∂i ju

n 0 0 h i j i j i 2 L (ϕ − v) = L (ϕ) = ∑ a0 − a (x) ∂i ju(x) + f (x) − f (y). (3.99) i, j=1

In view of notations (3.96), inequality (3.91) yields that we have, for any x ∈ Br = Br(y) ⊂ Bd,

i j i j i j i j α  i j −α α a − a (x) = a (y) − a (x) ≤ r a ≤ d r K1, 0 α;Br 2 −2−γ ∂i ju(x) ≤ [u]2,0;Br ≤ d U2, α −2−α−γ α | f (x) − f (y)| ≤ r [ f ]α;Br ≤ d r Fα .

These inequalities together with (3.99) entail

0 α |L (ϕ − v)| ≤ Ar in Br, (3.100) with −2−α−γ 2 A = d (n K1U2 + Fα ). (3.101) If Arα w(x) = r2 − |x − y|2 2nν then

0 α 0 L w ≤ −Ar ≤ −|L (ϕ − v)| in Br = Br(y), w = ϕ − v = 0 on ∂Br.

We deduce from this and the comparison principle A sup|ϕ − v| ≤ sup|ϕ − v| ≤ sup|w| = r2+α . Bρ Br Br 2nν

Since r = ρ/ε, (3.101) gives 3.5 Interior Schauder estimates 129

2+α+γ −2−α −2−α d ρ sup|ϕ − v| ≤ Cε (U2 + Fα ). (3.102) Bρ

On the other hand,

E2[u;Bρ ] ≤ E2[v;Bρ ] + E2[ϕ − v;Bρ ] ≤ E2[v;Bρ ] + sup|ϕ − v|. Bρ

This inequality together with (3.98) and (3.102) imply

2+α+γ −2−α 1−α −2−α d ρ E2[u;Bρ ] ≤ Cε U2,α +Cε (U2 + Fα ). (3.103)

We now consider case (b): r = ρ/ε > d. We have d2+α ρ−2−α < ε−2−α and

γ γ d E2[u;Bρ ] ≤ d sup|u| ≤ U0. Bρ

−2−α In consequence, the left hand side of (3.103) is less or equal to Cε U0 and hence (3.103) is satisfied for both cases (a) and (b). As y ∈ Ω and 0 < ρ ≤ d = d(y) can be chosen arbitrarily, we conclude

(γ) 1−α −2−α M2,α ≤ Cε U2,α +Cε (U2 +U1 +U0 + Fα ), (3.104)

(γ) for any ε > 0, where the semi-norm M2,α is defined in (3.38). By virtue of Theorem (γ) 3.3, this inequality still holds when M2,α is substituted by U2,α , i.e.

1−α 2−α U2,α ≤ Cε U2,α +Cε (U2 +U1 +U0 + Fα ).

1−α Thus, there exists ε = ε(n,ν,K,K1,α,γ) > 0 (take for instance 2Cε ≤ 1) for which U2,α ≤ C(U2 +U1 +U0 + Fα ). (3.105) In other words, we proved that (3.105) holds when bi = 0 et c = 0. For the general case, we write Lu = f in the form

n n i j 2 i ∑ a ∂i ju = f0 = f − ∑ b ∂iu − cu. i, j=1 i=1

It follows from the remark following Theorem 3.12

(2+γ) (2+γ) [ f0]0,α ≤ [ f ]0,α +C(U2 +U1 +U0). We have, according to (3.105) and the interpolation inequality in Theorem 3.2,

U2 +U1 ≤ εU2,α +C(ε)U0.

Whence (γ) kuk2,α = U2,α +U2 +U1 +U0 ≤ N(U0 + Fα ), 130 3 Classical solutions

which completes the proof. ut

We assume henceforward that Ω possesses the exterior sphere property with some R > 0.

Theorem 3.14. Let β ∈ (0,1) and u ∈ C2,α;−β (Ω). Then f = Lu ∈ C0,α;2−β (Ω) and

(2−β) (−β) (2−β) C1k f k0,α ≤ kuk2,α ≤ C2k f k0,α , (3.106)

with constants C1 = C1(n,K1) and C2 = C2(n,ν,K,K1,α,β,R,diam(Ω)).

Proof. The first inequality is contained in Theorem 3.12. Prior to proving the second (−β) (−β) inequality, we note that according to Lemma 3.4 the norms [w]0,0 and kwk are equivalent. On the other hand, we have by Corollary 3.6 the estimate

kuk(−β) ≤ Ck f k(−β).

We obtain then by applying Theorem 3.13 with γ = −β

(−β)  (2−β) (2−β) (2−β) kuk2,α ≤ C [ f ]0,0 + [ f ]0,α = Ck f k0,α ,

This proves the second inequality in (3.106). ut

Remark 3.4. The two inequalities in (3.106) show that the linear operator

L : u ∈ C2,α;−β (Ω) → f = Lu ∈ C0,α;2−β (Ω)

is bounded ; the mapping f → u = L−1 f defines also a bounded operator on L(C2,α;−β (Ω)) ⊂ C0,α;2−β (Ω). As 0 < β < 1, it is not hard to check that w ∈ 2,α;−β 2,α;−β 2,α C (Ω) vanishes on Γ . In other words, solving in C (Ω) ⊂ Cloc (Ω) ∩ C(Ω) the Dirichlet problem

Lu = f in Ω, u = 0 on Γ , (3.107)

for f ∈ Cα (Ω) ⊂ C0,α;2−β (Ω) is reduced to the surjectivity of L, i.e.

L(C2,α;−β (Ω)) = C0,α;2−β (Ω). (3.108)

We establish in the next section that (3.108) holds when Ω = Br. The general case will be discussed in the last section.

3.6 The Dirichlet problem in a ball

n In this section, the ball Br = Br(x0) ⊂ R and the constants α, β ∈ (0,1) are fixed. Consider then the Dirichlet problem 3.6 The Dirichlet problem in a ball 131

Lu = f in Br, u = ϕ on ∂Br. (3.109)

We assume that L fulfills the assumptions of the preceding section. We first consider the case ϕ = 0 and then the case ϕ ∈ C(Br).

0,α;2−β Theorem 3.15. For any f ∈ B2 = C (Br), there exists a unique u ∈ B1 = 2,α;−β C (Br) satisfying Lu = f in Br. In other words, L maps B1 onto B2, i.e. L(B1) = B2.

As we have mentioned in Remark 3.4, any function u ∈ B1 vanishes on ∂Br. Such u belongs to C(Br) and u = 0 sur ∂Br. Theorem 3.15 ensures the existence of α a solution of (3.109) when f ∈ B2 ⊃ C (Br) and ϕ = 0.

Proof. Note first that the uniqueness is a straightforward consequence of the com- parison principle. Before proceeding to the proof of the existence, we introduce some notations. The natural norms of B1 and B2 are denoted respectively by k · k1 and k · k2. Under these new notations, (3.106) takes the form

C1k f k2 ≤ kuk1 ≤ C2k f k2, f = Lu. (3.110)

The proof of existence consists in four steps. 1 First step. L = ∆ and f ∈ C (Br). By Stone-Weierstrass’s theorem, we find a m 1 sequence of polynomials ( f ) converging to f in C (Br). For each m, Lemma 3.12 m m m m guarantees the existence of u ∈ B1 so that ∆u = f . As 2−β > 0, f converges 0,α;2−β to f in C (Br). But, we have from (3.110)

m k m k ku − u k1 ≤ C2k f − f k2.

m Hence, (u ) is a Cauchy sequence in B1. Therefore, it converges to u ∈ B1 satisfy- ing ∆u = f . α Second step. L = ∆ and f ∈ C (Br) vanishing in a neighborhood of ∂Br. If ε > 0 is sufficiently small the regularization f (ε) of f given by (3.74) is well defined. By ε ε (ε) the first step, there exists u ∈ B1 satisfying ∆u = f . We get in light of (3.78) ε and (3.110) that (u ) is uniformly bounded in B1:

ε (ε) (ε) ku k1 ≤ C2k f k2 ≤ C| f |α;Br−ε ≤ C| f |α;Br .

(ε) On the other hand, f converges to f in C(Br) by (3.77). The comparison principle (Theorem 3.7) then entails

ε1 ε2 (ε1) (ε2) sup|u − u | ≤ C0 sup| f − f | → 0 when ε1, ε2 → 0. Br Br

ε Whence, there exists limε→0 u = u ∈ C(Br). An application of Lemma 3.13 yields ε (ε) u ∈ B1 and ∆u = lim∆u = lim f = f in Br. Third step. L = ∆ and f ∈ B2. Consider, for small ε > 0, the auxiliary function ε ∞ η ∈ C (Br) satisfying 132 3 Classical solutions

ε ε ε η = 0 in Br \ Br−ε , η = 1 in Br−3ε , et |∇η | ≤ C/ε, (3.111) where the constant C is independent on ε. It is not difficult to check that (3.111) ε (ε) holds for the regularization η = hε of the function hε given by hε = 1 in Br−2ε and h = 0 elsewhere. We obtain from (3.22) and (3.111)

ε (0) ε (0) kη k0,α ≤ Ckη k1,0 ≤ C(n), ε > 0. (3.112) Now, f ε = ηε f satisfies the assumptions of the second step and we have from in- equalities (3.21) and (3.112)

ε ε (2−β) ε (0) (2−β) k f k2 = kη f k0,α ≤ kη k0,α k f k0,α ≤ Ck f k2

ε ε ε for sufficiently small ε. By (3.110) the solutions u of ∆u = f are bounded in B1 and hence

ε β ε (−β) β ε β |u (x)| ≤ (r − |x|) ku k ≤ (r − |x|) ku k1 ≤ C(r − |x|) (3.113) for any x ∈ Br and small ε > 0. In order to study the convergence of uε , as ε → 0, we observe that

ε ε0 ε ε0 0 ∆(u − u ) = (η − η ) f = 0 in Br−3ε if 0 < ε < ε.

In light of (3.113), maximum principle’s yields

0 0 sup|uε − uε | = sup |uε − uε | ≤ Cεβ if 0 < ε0 < ε. Br Br\Br−3ε

ε Hence (u ) in bounded is B1 and converges in C(Br). We conclude similarly to the ε second step that u = limu ∈ B1 and ∆u = f . Fourth step. We use the continuity method to treat the general case. For 0 ≤ t ≤ 1, set Lt = ∆ +t(L−∆). We have in particular L0 = ∆ and L1 = L. The assumptions on the coefficients of L still valid for the coefficients of Lt with the same constants ν, K et K1. Consequently, the inequalities in (3.110) hold for f = Lt u, 0 ≤ t ≤ 1. From the third step, L0(B1) = B2. Assume that Ls(B1) = B2 for some s ∈ [0,1]. −1 Then again (3.110) tells us that Ls possesses a bounded inverse Ls : B2 → B1. For t ∈ [0,1] and f ∈ B2, Lt u = f is equivalent to the following equation

Lsu = f + (t − s)(L − ∆) and hence it is also equivalent to the following equation

−1 −1 u = Tu = Ls f + (t − s)Ls (L − ∆)u.

Once again (3.110) implies 3.7 Dirichlet problem on a bounded domain 133

−1 kTu − Tvk1 = |t − s|kLs (L − ∆)(u − v)k1 ≤ C2|t − s|k(L − ∆)(u − v)k2 ≤ 2C1C2|t − s|ku − vk1,

−1 for any u, v ∈ B1. Whence, if |t − s| < δ = (2C1C2) then T : B1 → B1 is strictly contractive. Therefore, there exists u ∈ B1 such that u = Tu or equivalently Lt u = f . As f is chosen arbitrary, we have Lt (B1) = B2 provided that |t −s| < δ. Dividing [0,1] into sub-intervals of length less or equal to δ, we deduce that Lt (B1) = B2 for any t ∈ [0,1]. We have, in particular for t = 1, that L(B1) = B2. This completes the proof. ut 2 Remark 3.5. If ϕ 6= 0, the existence of a solution of (3.109) in C (Br)∩C(Br) is not guaranteed. To see this, we consider the following one dimensional problem

u00 − x−2u = 0 in (0,1), u(0) = u(1) = 1, (3.114)

2 where (0,1) is considered as a ball Br = Br(x0), r = x0 = 1/2. If u ∈ C (Br)∩C(Br) then by (3.114) we have

1 u00 ∼ x−2, u0 ∼ x−1, u ∼ ln as x → 0+, x contradicting u(0) = 1. i j i α Theorem 3.16. Assume that a , b , c, f belong to C (Br) and ϕ ∈ C(Br). Then the 2,α;0 Dirichlet problem (3.109) admits a unique solution u ∈ C (Br) ∩C(Br). 3 Proof. Consider first the case ϕ ∈ C (Br). If u = v +ϕ then (3.109) is equivalent to the following equation

Lv = f0 in Br, v = 0 on ∂Br, (3.115)

α where f0 = f − Lϕ ∈ C (Br) ⊂ B2. The last theorem guarantees the solvability of 2,α;0 (3.115) and therefore the solvability of (3.109) in B1 ⊂ C (Br) ∩C(Br). For the general case we approximate ϕ ∈C(Br) by sequence of polynomials (ϕm) so that |ϕ − ϕm| ≤ 1/m in Br with m ≥ 1. As in Theorem 3.13, the solutions of the problems: Lum = f in Br, um = ϕm on ∂Br, define a sequence (um) converging in C(Br) to u ∈ C(Br). Once again Theorem 3.13 (0) with γ = 0 yields kumk2,α ≤ A, for any m, where the constant A is independent on 2,α;0 m. Lemma (3.13) then implies that u ∈ C (Br) and Lu = f in Br. ut

3.7 Dirichlet problem on a bounded domain

We extend the results of the preceding section to the Dirichlet problem on bounded domain. Consider then the boundary value problem 134 3 Classical solutions

Lu = f in Ω, u = ϕ on ∂Ω. (3.116) The assumptions on L are those of the preceding section. Furthermore, we as- sume that Ω has the exterior sphere property.

0,α;2−β Theorem 3.17. There exists, for any f ∈ B2 = C (Ω), a unique u ∈ B1 = 2,α;−β C (Ω) satisfying Lu = f in Ω. In other words, L sends B1 onto B2, i.e. L(B1) = B2.

0,α;2−β (2−β) Proof. Fix f ∈ B2 = C (Ω) and let A = k f k0,α;Ω . We prove the existence of a solution u ∈ B1 of Lu = f in Ω by using a variant of Perron’s method for sub-solutions. We split the proof in four steps. First step. We have, for x ∈ Ω,

d β−2 | f (x)| ≤ x [ f ](2−β) ≤ Adβ−2, with d = dist(x,Γ ). (3.117) 2 0,0 x x

β−2 There exists by Theorem 3.9 a sub-solution w ∈ C(Ω) of Lu = dx in Ω satisfying β 0 0 ≥ w ≥ −C1dx in Ω. We have according to estimate (3.117) that U = Aw and −U0 are respectively sub-solution and super-solution of Lu = f in Ω and

0 β 0 ≥ U (x) ≥ −C1Adx , x ∈ Ω, (3.118)

where C1 = C1(ν,K,β,R,diam(Ω)) is a constant. Second step. Starting from U0 we construct a sequence of sub-solutions (Uk) according to the following scheme : fix (y j) a dense sequence in Ω and consider a sequence of the form

(x1,x2,x3,...) = (y1,y1,y2,y1,y2,y3,...)

in such a way that each y j appears infinitely many times in the sequence (xk). Denote k k k k dk = dist(x ,Γ )/2 and B = B(x ) = Bdk (x ), k ≥ 1. By virtue of Theorem 3.16, there exists u ∈ C2,α;0(B1) ∩C(B1) a solution of the problem Lu1 = f in B1, u1 = U0 on ∂B1. As U0 is a sub-solution of Lu = f in Ω ⊃ B1, we have u1 ≥ U0 in B1. Define then the function U1 as follows

U1 = u1 on B1, U1 = U0 in Ω \ B1.

We have U1 ∈ C(Ω) and U1 ≥ U0 in Ω. Moreover, Corollary 3.5 guarantees that U1 is a sub-solution of Lu = f in Ω. Repeating this construction for k = 2,3,...: uk is the solution of the problem

Luk = f in Bk, uk = Uk−1 on ∂Bk 3.7 Dirichlet problem on a bounded domain 135

and define Uk by Uk = uk in Bk, Uk = Uk−1 in Ω \ Bk. We obtain in that manner a sequence

U0 ≤ U1 ≤ U2 ... ≤ Uk ≤ ... (3.119)

of sub-solution of Lu = f in Ω. As −U0 is a super-solution of Lu = f in Ω, we have also uk ≤ −U0 in Bk, Uk ≤ −U0 in Ω, k ≥ 1. (3.120) Hence, there exists u(x) = lim Uk(x), x ∈ Ω. (3.121) k→∞ 2,α Third step. We prove that u ∈ Cloc (Ω) and Lu = f in Ω. We get by using in- equalities (3.118) to (3.121)

k β |u(x)| ≤ sup|U (x)| ≤ C1Adx , x ∈ Ω. (3.122) k

j j j k Fix j ≥ 1, d = d(y ) = dist(y ,∂Ω)/2, B = Bd(y ) and choose (x i ) a sub-sequence of (xk) so that xki = y j for any i ≥ 1. Then Bki = B and the function uki ∈ C2,α;0(B)∩ C(B) satisfies Luki = f in B, for any i ≥ 1. We have (2) 2+α β β [ f ]0,α,B ≤ d [ f ]α;B ≤ d [ f ]0,α;B ≤ Ad , and from Theorem 3.13 with γ = 0, and (3.122) we obtain   ki (0) ki (2) β ku k2,α;B ≤ C sup|u | + [ f ]0,α;B ≤ CAd , i ≥ 1. B

We then get, by applying Lemma 3.13, u = limuki ∈ C2,α;0(B), Lu = f in B and

(0) β kuk2,α;B ≤ CAd . (3.123)

2,α But ∪ jB(y j) = Ω. Hence, u ∈ Cloc (Ω) and Lu = f in Ω. 2,α;−β Fourth step. We show in this last step that u ∈ B1 = C (Ω). We first note that (3.123) entails

2 dk[u] + d2+α [u] ≤ CAdβ , ∑ k,0;Bd/2 2,α;Bd/2 (3.124) k=0

j j where Bd/2 = Bd/2(y ) is the ball of radius d/2 = dist(y ,∂Ω)/4. We now evaluate the norm of u in C2,0;−β (Ω). By (3.15) and (3.16) we have, for any k ≥ 0, that there exist x ∈ Ω, |`| = k and x ∈ B(x )(= B (x )) so that 0 0 dx0 0 1 [u](−β) ≤ dk−β (x )|∂ `u(x)|. (3.125) 2 k,0;Ω 0 136 3 Classical solutions

As (y j) is dense in Ω, we find y j sufficiently close to x in such a way that x ∈ j j Bd/2(y ), d(x) ≤ 2d, where d = d(y ). Then inequalities (3.123) and (3.124) imply

(−β) k+1 k−β [u]k,0;Ω ≤ 2 d [u]k,0;B d ≤ CA, k ≥ 0. (3.126) 2

(−β) It remains to estimate [u]k,α;Ω . As previously, there exist x0 ∈ Ω, |`| = k and x,y ∈ B(x0) so that 1 |∂ `u(x) − ∂ `u(y)| [u](−β) ≤ d2+α−β (x ) . (3.127) 2 2,α;Ω 0 |x − y|α j j If |x − y| < d(x0)/4, we choose y such that x, y ∈ Bd/2(y ), d(x0) ≤ 2d. Then in- equalities (3.124) and (3.127) enable us obtaining the following estimate

(−β) 3+α 2+α−β [u]2,α;Ω ≤ 2 d [u]2,α;B d ≤ CA. 2

d(x0) Finally, if |x − y| ≥ 4 then (3.127) implies

(−β) α 2−β ` ` [u]2,α;Ω ≤ 2(4 )d (x0)|∂ u(x) − ∂ u(y)| ≤ 1+α d2−β (x )[u] ≤ 1+α [u](−β) ≤ CA. 4 0 2,0;B(x0) 4 2,0;Ω This completes the proof. ut

Theorem 3.18. Assume that ai j, bi, c, f belong to Cα (Ω) and ϕ ∈ C(Ω). Then Dirichlet problem (3.116) admits a unique solution u ∈ C2,α;0(Ω) ∩C(Ω).

Proof. Quite similar to that of Theorem 3.16 with Br substituted by Ω. ut

3.8 Exercises and problems

3.1. Let 0 < α ≤ 1 and f ∈ D(R). Set | f (x + h) − f (x)| | f (x + h) − 2 f (x) + f (x + h)| [ f ] = , [ f ]∗ = . α sup α α sup α x∈R, h6=0 |h| x∈R, h6=0 |h|

∗ ∗ Prove that [ f ]α ≤ 2[ f ]α and, for 0 < α < 1, [ f ]α ≤ C[ f ]α , where the constant C does not depend of α. Hint: consider the operators Th f (x) = f (x + h), I f (x) = f (x) and use the following identities 1 T − I = (T 2 − I) − (T 2 − I)2, T 2 = T . h 2 h h h 2h

0 n 3.2. (a) Let 0 < α < 1 and u ∈ Cc (R ) so that 3.8 Exercises and problems 137 |u(x) − u(y)| U = [u] = < α 0,α : sup α ∞ x6=y |x − y| and, for ε > 0, denote by u(ε) the regularization of u : Z u(ε)(x) = u(x − εy)ϕ(y)dy. Rn

` (ε) α−k n (i) Show that |∂ u | ≤ Cε Uα , for any ` ∈ N , |`| = k ≥ 1, where C = C(ϕ). ∂ (ε) α−1 (ii) If ∂ε = , prove that |∂ε u | ≤ Cε Uα , where C = C(n,ϕ) is a constant. R ∂ε Hint: use n (y) · ydy = −n. R ∇ϕ (b) Let u, v ∈ C0(R) with support in (−1,1) so that

Uα < ∞ and Vβ = [v]0,β < ∞, for some constants α, β ∈ (0,1). Set then w = u ∗ v. (i) Assume that α + β < 1. Show that [w]0,α+β < ∞. Hint: use the identity w = (1) (1) u ∗ v − w1 − w2, where

Z 1 Z 1 (ε) (ε) (ε) (ε) w1 = ∂ε u ∗ v dε, w2 = ∂ε v ∗ u dε. 0 0

1 0 (ii) If α +β > 1, demonstrate that w ∈ Cc (R) and [w ]0,α+β−1 < ∞. Hint: if w1 is as above then use the approximation

Z 1 (ε) (ε) w1,δ = w1 = ∂ε u ∗ v dε, 0 < δ < 1. δ n 3.3. Let Ω1 and Ω2 be two bounded open subsets of R so that Ω1 b Ω2. For k = 1, 2 2, let uk ∈ C (Ωk) ∩C(Ωk) satisfying

n i j 2 uk > 0, Luk = ∑ a ∂i juk = λkuk in Ωk, uk = 0 on ∂Ωk, i, j=1

i j i j where λk is a constant and the coefficients a = a (x) belong to C(Ω2) and satisfy

i j ji 2 i j −1 2 n a = a , ν|ξ| ≤ ∑a ξiξ j ≤ ν |ξ| for any ξ ∈ R , i, j for some constant ν ∈ (0,1]. Prove that λ1 < λ2 < 0. Hint: we can apply the maximum principle to the function v = u1/u2 in Ω1.

2 n 3.4. Let u ∈ C (B1), B1 = {x ∈ R ; |x| < 1}, so that u = 0 on ∂B1. Prove that Z Z u2dx ≤ C (∆u)2dx, B1 B1 138 3 Classical solutions the constant C only depends on the dimension n. Hint: use Poincare’s´ inequality Z Z 2 2 u dx ≤ C0(n) |∇u| dx. B1 B1

n 3.5. Let u, v ∈ C2(R ) be two harmonic functions so that

a b n u(tx) = t u(x), v(tx) = t v(x) for any x ∈ R and t > 0, with constants a 6= b. Establish the orthogonality relation Z uvds = 0. ∂B1(0)

3.6. Let f be a continuous and bounded function on R so that | f (t) − f (s)|  [ f ] = sup ; t,s ∈ t 6= s < ∞, α |t − s|α R where α ∈ (0,1) is a constant. Let u(x) = u(x1,x2) be the solution of the Laplace equation 2 ∆u = 0 in R+ = {x = (x1,x2); x2 > 0}, with boundary condition

u(x1,0) = f (x1,0), x1 ∈ R. Recall that u is explicitly given by the formula

1 Z x f (t) u(x ,x ) = 2 dt. 1 2 2 2 π R (x1 −t) + x2

2 a) Check that, for (x1,x2), (y1,x2) ∈ R+, we have  1 Z x  |u(x ,x ) − u(y ,x )| ≤ 2 dt [ f ] |x − y |α = [ f ] |x − y |α . 1 2 1 2 2 2 α 1 1 α 1 1 π R t + x2

2 b) If (y1,x2), (y1,y2) ∈ R+ then prove that  Z α  1 |s| α |u(y1,x2) − u(y1,y2)| ≤ 2 ds [ f ]α |x2 − y2| . π R s + 1 Hint: note that 1 Z f (x1 + sx2) u(x1,x2) = 2 ds. π R s + 1 c) Deduce that [u]α ≤ C[ f ]α , 3.8 Exercises and problems 139 the constant C only depends on α.

3.7. Fix p ∈ (0,1). a) Let q such that q − 2 = qp. Compute the constant c = c(n, p) for which v(x) = q n c|x| , x ∈ R , is a solution of the equation

p n ∆v = v in R .

n Let u ∈ C2(R ) satisfying

p n u > 0, ∆u = u in R . b) Check that maxu(x) = maxu(x) for any r > 0. |x|≤r |x|=1 c) Assume that there exists r > 0 so that u < v in ∂Br(0). Prove that u ≤ v in Ω = {v < u}∩Br(0) (an open subset containing 0). Obtain a contradiction by noting that

2 maxu(x) ≥ cr 1−p for any r > 0. |x|≤r

2 3.8. Let u ∈ C (B2(0)) so that

u > 0, ∆u = 0 in B2(0).

(a) Establish the estimate C sup |∇u| ≤ 0 sup u, for any B (x) ⊂ B (0), r 2r 2 Br(x) B2r(x) where C0 = C0(n) is a constant. Hint: deduce from Lemma 3.11 that

|∇u(x)| ≤ C1 sup u, for any x ∈ B1(0), B1/2(x) with a constant C1 = C1(n). (b) Prove the estimate

sup u ≤ C2u(x) for any x ∈ B1(0), B1/2(x) with C2 = C2(n) is a constant. (c) Conclude that we have the following estimate

sup |∇(lnu)| ≤ C, B1(0) for some constant C = C(n). 140 3 Classical solutions

n 3.9. Let Ω be a bounded domain of R with Lipschitz boundary. Prove the following interpolation inequality, where ε > 0 is arbitrary, Z Z Z 2 2 1 2 1 2 |∇u| dx ≤ ε (∆u) dx + u dx, u ∈ H0 (Ω) ∩ H (Ω). Ω Ω 4ε Ω

3.10. Let u be a in B = B(x0,1). Prove the gradient estimate   |∇u(x0)| ≤ n supu − u(x0) . B

Hint: as ∆u = 0 is invariant under rotation we may assume that |∇u(x0)| = −∂nu(x0). Apply the mean value theorem to the harmonic function M − u in Br = B(x0,r), 0 < r < 1, with M = supB u. Get then the estimate 1 Z |∂nu(x0)| ≤ n (M − u)dσ(x). ωnr ∂B

n 3.11. Let u ∈ C∞(R ) so that

α n ∆u(x) = 0, |u(x)| ≤ C|x| in R , where C and α are two positive constants. Prove that u is a polynomial of degree less or equal to [α].

n 3.12. We say that the bounded open set ω of R has the interior ball property at x0 ∈ ∂ω if there exists B ⊂ ω an open ball so that ∂B ∩ ∂ω = {x0}. One can prove n that any C2 bounded open subset of R has the interior ball property at each point of its boundary. n a) (Hopf’s lemma) Let Ω be a bounded subset of R admitting the interior ball 1 2 property at x0 ∈ ∂Ω, u ∈ C (Ω) ∩C (Ω) satisfying ∆u ≥ 0 in Ω and u(x0) > u(x), 0 0 0 for any x ∈ Ω. Let B = B(y,R) ⊂ Ω so that ∂B ∩ ∂Ω = {x0} and B = B (x0,R ), R0 < R. Set 2 2 r = |x − y|, D = B ∩ B0 v(x) = e−ρr − e−ρR . i) Prove that there exists ρ > 0 sufficiently large in such a way that ∆v > 0 in D. Fix this ρ. Verify then that, for a given ε > 0,

∆(u − u(x0) + εv) > 0 in D. ii) Show that, for sufficiently small ε, u − u(x0) + εv ≤ 0 in ∂D. Deduce that

∂ν u(x0) > 0.

n b) (Strong maximum principle) Let Ω be a domain of R and u ∈ C(Ω) ∩C2(Ω) satisfying ∆u ≥ 0. Prove the following claim : if u is non constant, then u can not attain its maximum at a point of Ω. Hint : Proceed by contradiction by using Hopf’s lemma and 3.8 Exercises and problems 141

6 Lemma 3.14. Let u ∈C(Ω) and set M = maxΩ u. Assume that F = {x ∈ Ω; u(x) = M} and Ω \F are nonempty. Then there exists an open ball B so that B ⊂ Ω,B∩F = /0 and ∂B ∩ F 6= /0. 3.13. Let K ≥ 0 be a constant. (a) Prove that the nonlinear equation

U00 + K|U0| + 1 = 0 in (−1,1), U(±1) = 0 admits a unique even solution U ∈ C2([−1,1]). (b) Let p ∈C([−1,1]) and |p(t)| ≤ K. Let u ∈C2([−1,1]) be a solution of the bound- ary value problem

u00 + pu0 + 1 = 0 in (−1,1), u(±1) = 0.

Check that 0 ≤ u ≤ U in (−1,1). Deduce that the nonlinear equation in (a) admits a unique solution.

2 2 3.14. Let Ω = {x = (x1,x2) ∈ R ; x1 > 0, x2 > 0} and let u ∈ C (Ω) be a solution of ∆u = 0 in Ω, u = 0 on ∂Ω satisfying |u(x)| ≤ c1 + c2|x| in Ω, where c1 et c2 are two positive constants. Prove that u is identically equal to zero. Hint: we can first extend u to R2 and then use the interior estimate of derivatives of harmonic functions in balls centered at the origin (see Lemma 3.11).

n   3.15. Let B˙ = {x ∈ R ; 0 < |x| < 1}, n ≥ 2, and let u ∈ C2 B˙ ∩C B be a harmonic function in B˙ so that u(x) = o(Γ (x)) when x → 0, where Γ is the fundamental

6 Proof. As Ω is connected, the boundary of F in Ω contains a least a point in Ω. Since F is closed in Ω there exits z ∈ F so that any neighborhood of z contains at least a point where u < M. Let δ > 0 such that B(z,3δ) ⊂ Ω and let x0 ∈ B(z,δ) so that u(x0) < M. Consider then the set

I = {λ ≥ 0; B(x0,λ) ⊂ {u < M}}.

(i) As u is continuous, I nonempty. 0 0 (ii) I is an interval because if λ ∈ I and λ ≤ λ then B(x0,λ ) ⊂ B(x0,λ). On the other hand, as z ∈ B(x0,δ) and u(z) = M, we have I ⊂ [0,δ]. (iii) For λ ∈ I and x ∈ B(x0,λ), we have

|x − z| ≤ |x − x0| + |z − x0| < λ + δ ≤ 2δ and hence B(x0,λ) ⊂ Ω for any λ ∈ I.

(iv) I is closed: let (λn) be a sequence in I converging to λ ∈ [0,δ]. If there exists n0 so λn0 ≥ λ then B(x0,λ) ⊂ B(x0,λn0 ) and hence λ ∈ I. Otherwise, we would have λn < λ, for any n. Let then x ∈ B(x0,λ). Since λn converges to λ, there exists an n0 so that |x − x0| < λn0 < λ. Whence, x ∈ {u < M}. We deduce then that B(x0,λ) ⊂ {u < M}. That is λ ∈ I. In conclusion, there exists R > 0 such that I = [0,R]. Then clearly B(x0,R) ⊂ {u < M}. We have also that ∂B(x0,R) ∩ F is non empty. Otherwise, a simple argument based on the continuity of u and the compactness of ∂B(x0,R) would imply that B(x0,R + ε) ⊂ {u < M}, for some ε > 0, and consequently R + ε ∈ I which is impossible. ut 142 3 Classical solutions solution of the operator −∆:  Γ (n/2) |x|2−n if n ≥ 3,  2(n−2)πn/2 Γ (x) =  1 − 2π ln|x| if n = 2.

Fix r ∈ (0,1) and denote by Br the ball with center 0 and radius r. Consider then ∞ v ∈ C (Br) ∩C(Br) the solution of the boundary value problem

∆v = 0 in Br, v = u on ∂Br.

(a) Prove that, for any ε > 0, there exists δ ∈ (0,r) so that

|u − v| ≤ εΓ in Bδ \{0}.

(b) Deduce that |u−v| ≤ εΓ in Br \Bδ . Hint: apply the comparison principle to both u and v± = v ± εΓ . (c) Conclude that u admits a harmonic extension in the whole unit ball B.

2 3.16. For r > 0, let Ωr = {x = (x1,x2) ∈ R ; |x2| < x1 < r} and, for 1 ≤ i, j ≤ 2, i j i j let a be continuous functions in R2 so that the matrix a = (a ) is symmetric and satisfies to the ellipticity condition

2 −1 2 2 ν|ξ| ≤ a(x)ξ · ξ ≤ ν |ξ| , for all x,ξ ∈ R , where ν ∈ (0,1] is a constant. Set n i j 2 L = ∑ a ∂i j. i, j=1

(a) Prove that there exists µ = µ(ν) et λ = λ(ν) for which v(x1,x2) = (1 − 2 µ x1) cosh(λx2) satisfies Lv ≥ 0, in (−1,1) × R. Until the end of this exercise, we fix µ and λ is such a way that the last inequality holds. 2 Let u ∈ C (Ω R), R > 0 is given, so that

Lu = 0 in ΩR, u = 0 on {|x2| = x1} ∩ ∂ΩR.

We want to prove that

2r 1+α Mr = sup|u| ≤ MR for 0 < 2r ≤ R, Ωr R where the constant α only depends on ν. (b) Check that is enough to prove that 3.8 Exercises and problems 143

−(1+α) Mr ≤ 2 M2r for 0 < 2r ≤ R.

Without loss of generality we may assume that r = 2 in the last inequality. Set then 1 U(x) = U(x ,x ) = M x ± u(x ,x ), in Ω = {|x | < x < 4}. 1 2 4 4 1 1 2 4 2 1 (c) If c = c(ν) = 1/(4cosh(3λ)), show that

0 U(x1,x1) ≥ cM4v(x1 − 2,x2), in Ω = {1 < x1 < 3,|x2| < x1}.

−(1+α) Then deduce that there exists α = α(ν) so that M2 ≤ 2 M4. 3.17. Consider the operator

n n i j 2 i L = ∑ a (x)∂i j + ∑ b (x)∂i, i, j=1 i=1 where the matrix a(x) = (ai j(x)) is symmetric and fulfills the ellipticity condition

2 −1 2 n ν|ξ| ≤ a(x)ξ · ξ ≤ ν |ξ| for all x,ξ ∈ R , with ν ∈ (0,1], and |bi(x)| ≤ K, 1 ≤ i ≤ n, for some constant K. Assume moreover that ai j and bi are 1-periodic. That is, ai j(x + z) = ai j(x) and bi(x + z) = bi(x), for n n any x ∈ R and z ∈ Z . n 2 (a) Let x0 ∈ R and set Br = B(x0,r). Prove that if u ∈ C (B4r) satisfies Lu = 0 in B4r then sup(M − u) ≤ C(M − u(x0)) with M ≥ supu, Br B4r where C = C(n,ν,K,r) is a constant. n n n Let u ∈ C2(R ) be a bounded solution of Lu = 0 in R . Fix z ∈ Z and let v(x) = n u(x + z) − u(x), x ∈ R . M = n v m = n v M > (b) If 1 supR and 1 infR , show that we cannot have neither 1 0, nor m1 < 0 (hence v is identically equal to zero and consequently u is 1-periodic). (c) Conclude that u is constant. Hint: as u is 1-periodic, it is enough to check that u is constant in the unit cube Q = [0,1)n. n 3.18. Let u be a harmonic function in the unit ball B1 = {x ∈ R ; |x| < 1}. (a) Use the analyticity of u in B1 to show that

u(x) = ∑ Pk(x), (3.128) k≥0

n in a neighborhood of 0 ∈ R where, for each k, Pk is a homogenous polynomial of k n degree k ; that is, P(λx) = λ Pk(x), for any λ ∈ R, x ∈ R and ∆Pk = 0. 2 (b) Prove that the polynomials Pk in (3.128) are two by two orthogonal in L (B1), i.e. Z PjPk = 0 if j 6= k. B1 144 3 Classical solutions

n (c) Let r ∈ (0,1), Br = {x ∈ R ; |x| < r} and let (p j) be a family of polynomials, p j of degree j, so that sup|u − p j| → 0 as j → +∞. Br (Note that such family exists by Stone-Weierstrass’s theorem). (i) Show that there exists h j, a harmonic polynomial of degree j, such that h j = p j on ∂Br. Deduce that sup|u − h j| → 0 as j → +∞. Br Hint: use the maximum principle. Therefore, ku − h k 2 → j → + . j L (Br) 0 as ∞

(ii) Let S j = P1 + ... + Pj be the orthogonal projection of u on E j, the subspace of harmonic polynomials of degree ≤ j. Check that

2 2 2 ku − h k 2 = ku − S k 2 + kS − h k 2 j L (Br) j L (Br) j j L (Br) and deduce that ku − S k 2 → j → + . j L (Br) 0 as ∞

(d) Demonstrate that the series in (3.128) converges uniformly in any ball Br, with r ∈ (0,1). Hint: use that any harmonic function h in Ω coincide with its regulariza- (ε) tion h in Ωε = {x ∈ Ω; dist(x,∂Ω) > ε}. References 145 References

1. Gilbarg D. and Trudinger N. S. : Elliptic partial differential equations of second order, 2nd ed.. Springer, Berlin, 1983. 2. Michael, J. H. : A general theory for linear elliptic partial differential equations. J. Differential Equations 23 (1) (1977), 1-29. 3. Safonov M.V. : Linear elliptic equations of second order, Lecture notes.

Chapter 4 Classical inequalities, Cauchy problems and unique continuation

The first part of this chapter is essentially dedicated to some classical inequalities for harmonic functions. We precisely establish three-ball, three-sphere and doubling inequalities for harmonic functions. We limited, for sake of clarity, ourselves to harmonic functions but these kind of inequalities are in fact true for general elliptic operators but the proofs are more involved in that case. We refer the reader to the paper by R. Brummelhuis [1] for the three-sphere inequality and to N. Garofalo and F.-H. Lin [3, 4] for the doubling inequality. The rest of this chapter is devoted to three-ball inequalities that we apply in various situation for establishing stability inequalities for Cauchy problems. Our results rely on a Carleman estimate for a family of elliptic operators depending on a parameter and a generalized Poincare-Wirtinger´ inequality. The results in this part improve substantially those in [2, Chapter 2].

4.1 Classical inequalities for harmonic functions

n In this section, Ω is a bounded domain of R (n ≥ 2) with boundary Γ and

H (Ω) = {u ∈ C2(Ω); ∆u = 0}.

Here ∆ is the usual Laplace operator acting as follows

n 2 2 ∆u = ∑ ∂i u, u ∈ C (Ω). i=1

The ball and the sphere of centrer ξ and radius r are respectively denoted by B(ξ,r) and S(ξ,r). The first result we prove is the following three-ball inequality

Theorem 4.1. We have, for any u ∈ H (Ω), ξ ∈ Ω and 0 < r1 < r2 < r3 < rξ = dist(ξ,Γ ),

147 148 4 Classical inequalities, Cauchy problems and unique continuation

α 1−α kuk 2 ≤ kuk 2 kuk 2 . L (B(ξ,r2)) L (B(ξ,r3)) L (B(ξ,r1))

Here α = (r2 − r1)/(r3 − r1).

Proof. For simplicity convenience, we use in this proof the following notations

S(r) = S(ξ,r), B(r) = B(ξ,r) and, where 0 < r < rξ , Z H(r) = u2(x)dS(x), S(r) Z K(r) = u2(x)dx. B(r)

As Z H(r) = u2(ξ + ry)rn−1dS(y), Sn−1 we get n − 1 Z H0(r) = H(r) + 2 u(ξ + ry)∇u(ξ + ry) · yrn−1ds(y) (4.1) r Sn−1 n − 1 Z = H(r) + 2 u(x)∂ν u(x)ds(x). r S(r)

But according to Green’s formula and taking into account that ∆u = 0 in Ω we have Z Z Z 2 u(x)∂ν u(x)ds(x) = ∆u(x)u(x)dx + |∇u(x)| dx (4.2) S(r) B(r) B(r) Z = |∇u(x)|2dx. B(r)

This in (4.1) yields n − 1 Z H0(r) = H(r) + 2 |∇u(x)|2dx ≥ 0. (4.3) r B(r)

Whence Z r Z r K(r) = H(ρ)dρ ≤ H(r)dr = rH(r). (4.4) 0 0 Define F(r) = lnK(r), 0 < r < rξ . We have H(r) H0(r)K(r) − H(r)2 F0(r) = and F00(r) = . K(r) H(r)2 We obtain in light of (4.4) 4.1 Classical inequalities for harmonic functions 149

rH0(r)H(r) − H(r)2 F00(r) ≥ . H(r)2

This and (4.3) imply Z 00 2r 2 F (r) ≥ n − 2 + 2 |∇u(x)| dx ≥ 0. H(r) B(r)

Therefore F is convex. As F is convex and r2 = αr3 + (1 − α)r1, we deduce that

α 1−α lnK(r2) = F(r2) ≤ αF(r3) + (1 − α)F(r1) = lnK(r3) + lnK(r1) α 1−α  = ln K(r3) K(r1) .

Thus α 1−α K(r2) ≤ K(r3) K(r1) , which leads immediately to the expected inequality. ut

We next establish the so-called doubling inequality.

Theorem 4.2. Let u ∈ H (Ω), ξ ∈ Ω and 0 < r < rξ = dist(ξ,Γ ). There exits a constant C > 0, depending on u, ξ and r, so that, for any 0 < r ≤ r/2, we have

kukL2(B(ξ,2r)) ≤ CkukL2(B(ξ,r)).

Proof. Let S(r), B(r), H and K be as in the preceding proof and set Z 2 D(r) = |∇u(x)| dx, 0 < r < rξ . B(r)

Taking into account that Z r Z D(r) = |∇u(ξ +ty)|2tn−1dS(y)dt, 0 Sn−1 we obtain Z D0(r) = |∇u(y)|2dS(y) S(r) implying 1 Z D0(r) = |∇u(y)|2(x − ξ) · ν(y)dS(y). r S(r) We get by applying the divergence theorem 1 Z D0(r) = div|∇u(x)|2(x − ξ)dx (4.5) r B(r) n Z 1 Z = |∇u(x)|2dx + ∇(|∇u(x)|2) · (x − ξ)dx. r B(r) r B(r) 150 4 Classical inequalities, Cauchy problems and unique continuation

On the other hand, we obtain by making an integration by parts Z Z  2 2 ∂ j (∂iu(x)) (x j − ξ j)dx = 2 ∂iu(x)∂i ju(x)(x j − ξ j)dx B(r) B(r) Z Z 2 = −2 ∂ii u(x)(x j − ξ j) − 2 ∂iu(x)∂ ju(x)δi jdx B(r) B(r) Z + 2 ∂iu(x)∂ ju(x)(x j − ξ j)νidS(x). S(r)

Hence Z Z Z 2 2 2 ∇(|∇u(x)| ) · (x − ξ)dx = −2 |∇u(x)| dx + 2r (∂ν u(x)) dS(x). B(r) B(r) S(r)

This in (4.5) yields n − 2 D0(r) = D(r) + 2L(r) (4.6) r with Z 2 L(r) = (∂ν u(x)) dS(x). S(r) Introduce now the so-called frequency function

rD(r) N(r) = . H(r)

Elementary computations show that

N0(r) 1 D0(r) H0(r) = + − . N(r) r D(r) H(r)

As ∆u = 0 we have ∆(u2) = 2|∇u|2. Applying Green’s formula we find out Z Z Z 2 2 2 |∇u(x)| dx = ∆(u )dx = 2 u(x)∂ν u(x)dS(x). B(r) B(r) S(r)

That is Z D(r) = u(x)∂ν u(x)dS(x). (4.7) S(r) (This is can be derived directly from (4.2)). We have from (4.3) H0(r) n − 1 D(r) = + 2 (4.8) H(r) r H(r) and (4.6) entails 4.1 Classical inequalities for harmonic functions 151

D0(r) n − 2 L(r) = + 2 . (4.9) D(r) r D(r) Therefore N0(r) L(r) D(r) L(r)H(r) − D(r)2 = 2 − 2 = 2 . (4.10) N(r) D(r) H(r) D(r)H(r) According to Cauchy-Schwarz’s inequality, (4.7) yields

D(r)2 ≤ L(r)H(r).

This in (4.10) entails N0(r) ≥ 0. In other words we proved that N is non-decreasing. Next, from  H(r)0 H0(r) n − 1 ln = − rn−1 H(r) r and (4.8) we deduce that

 H(r)0 D(r) N(r) ln = 2 = 2 . rn−1 H(r) r

Fix r ≤ rξ . Then, bearing in mind that N is non-decreasing, we get

 H(r)0 N(r) ln ≤ 2 , 0 < r ≤ r. rn−1 r

Thus, with 0 < r1 < r2 ≤ r,

0 n−1 Z r2  H(r) H(r )r rκ ln dr = ln 2 1 ≤ ln 2 , rn−1 n−1 rκ r1 H(r1)r2 1 where κ = 2N(r). In consequence

 κ+n−1 r2 H(r2) ≤ H(r1). r1 From this we get

1  κ+n−1 1  κ+n Z r2 Z r2 K(r2) = r2 H(sr2)ds ≤ r2 H(sr1)ds = K(r1). 0 r1 0 r1

The doubling inequality holds by taking r2 = 2r1 in the preceding inequality. That is we have, for any 0 < r ≤ 2−1r,

K(2r) ≤ CK(r). (4.11) 152 4 Classical inequalities, Cauchy problems and unique continuation

Here C = 2κ+n. ut

We say that u vanishes of infinite order at ξ if

N K(r) = O(r ), for any N ∈ N. (4.12) We have as a consequence of the doubling inequality in Theorem 4.2 the follow- ing strong unique continuation property for harmonic functions.

Corollary 4.1. If u ∈ H vanishes of infinite order at some ξ ∈ Ω then u = 0. Proof. In this proof K is as in the proof of Theorem 4.1. We get, for sufficiently small r, by applying recursively (4.11)

K(r) ≤ CK(2−1r) ≤ ...C`K(2−`r) = C`(2−`r)N[(2−`r)−NK(2−`r)].

` −` N Fix first ` and N0 so that C (2 r) remains bounded for any N ≥ N0. Whence, as (2−`r)−NK(2−`r) tends to zero as N converges to ∞, we obtain that K(r) = 0. That is u = 0 in B(r). The proof is completed by using Theorem 2.16 in Chapter 2. ut

The calculations we carried out in the proof of the preceding theorems can used to obtain a three-sphere inequality. Indeed, using

 H(r)0 N(r) ln = 2 , rn−1 r

N0(r) 1 D0(r) H0(r) = + − , N(r) r D(r) H(r) identities (4.8) and (4.9) to obtain by straightforward computations

 H(r)00 2 4(L(r)H(r) − D(r)2) ln = − + . rn−1 r2 rD(r)H(r)

But from the preceding proof, we observed that L(r)H(r) − D(r)2 ≥ 0. In conse- quence  H(r)00 ln ≥ 0, rn+1 showing in particular that H(r) r → ln rn+1 is convex. We can then state the following result.

Theorem 4.3. We have, for any u ∈ H (Ω), ξ ∈ Ω and 0 < r1 < r2 < r3 < rξ = dist(ξ,Γ ),

 (n+1)α  (n+1)(1−α) r2 r2 α 1−α kuk 2 ≤ kuk kuk , L (S(ξ,r2)) L2(S(ξ,r )) L2(S(ξ,r )) r3 r1 3 1 4.1 Classical inequalities for harmonic functions 153 with α = (r2 − r1)/(r3 − r1). We establish in the remaining part of this section the mean-value identities and their consequences.

Theorem 4.4. Let u ∈ H (Ω) and ξ ∈ Ω. Then we have for any 0 < r < rξ = dist(ξ,Γ ) 1 Z u(ξ) = u(x)dS(x), (4.13) |S(r)| S(r) n Z u(ξ) = u(x)dx. (4.14) |B(r)| B(r)

Here S(r) = S(ξ,r) and B(r) = B(ξ,r). Proof. Define 1 Z I(r) = u(x)dx, 0 < r < rξ . |S(r)| S(r) Since 1 Z I(r) = n−1 u(ξ + ry)dS(y), (4.15) |S | Sn−1 we have Z 0 1 I (r) = n−1 ∇u(ξ + ry) · ydS(y) |S | Sn−1 1 Z = n−1 ∂ν u(ξ + ry)dS(y) |S | Sn−1 1 Z = n−1 ∂ν u(x)dS(x). |S | Sn−1 We then get by applying then the divergence theorem 1 Z I0(r) = ∆u(x)dx = 0. |S(r)| B(r)

That is I is constant and from (4.15) we have that I can be extended by continuity at r = 0 by posing I(0) = u(ξ). Whence I(r) = u(ξ) or equivalently 1 Z u(ξ) = u(x)dx. |S(r)| S(r)

This is exactly (4.13). One gets using again (4.15)

Z r Z rn| n−1| u(ξ +ty)tn−1dtdS(y) = S u(ξ), 0 Sn−1 n from which we deduce 154 4 Classical inequalities, Cauchy problems and unique continuation n Z u(ξ) = u(x)dx. |B(r)| B(r) In other words, we proved (4.14). ut

We now apply the mean-value inequality (4.14) to obtains the strong maximum principle for harmonic functions. In the sequel

0 Hc(Ω) = H (Ω) ∩C (Ω).

Theorem 4.5. Let u ∈ Hc(Ω) be non constant. Then u can not achieve its maximum or its minimum at an interior point.

Proof. Set M = maxΩ u and

ΩM = {x ∈ Ω; u(x) = M}.

Note that ΩM is closed by the continuity of u. We claim that, as u is non constant, ΩM is empty. If ΩM was non empty then for ξ ∈ ΩM we have u(ξ) = M. Using one more time (4.14) in which we substitute u by u−M. We obtain, where 0 < r < rξ is fixed, n Z 0 = u(ξ) − M = (u(x) − M)dx ≤ 0. |B(r)| B(r)

Thus u−M = 0 in B(r) implying that ΩM would be also open. Therefore, we would have ΩM = Ω and consequently u is constant, which contradicts our assumption. The case of an interior minimum can be treated similarly. ut

We now apply again (4.14) to establish a Harnak type inequality for harmonic functions.

Theorem 4.6. Let 0 ≤ u ∈ H (Ω), ξ ∈ Ω and 0 < 4r < rξ = dist(ξ,Γ ). Then

maxu ≤ 3n minu B(r) B(r)

Proof. Pick x1,x2 ∈ B(r) so that

u(x1) = maxu and u(x2) = minu. B(r) B(r)

We obtain by applying twice (4.14) n Z n Z u(x1) = n n−1 u(x)dx ≤ n n−1 u(x)dx r |S | B(x1,r) r |S | B(2r) and n Z n Z u(x2) = n n n−1 u(x)dx ≥ n n n−1 u(x)dx. 3 r |S | B(x2,3r) 3 r |S | B(2r) n Whence u(x1) ≤ 3 u(x2) which means that 4.2 Two-dimensional Cauchy problems 155

maxu ≤ 3n minu. B(r) B(r)

This is the expected inequality. ut

4.2 Two-dimensional Cauchy problems

Let Ω be a bounded domain of R2 with Lipschitz boundary Γ . Fix ϕ ∈C2(Ω;R) and equip L2(Ω,eϕ dx) and L2(Γ ,eϕ dσ) respectively by the following scalar products Z ( f |g) = f (x)g(x)eϕ(x)dx. Ω Z h f |gi = f (σ)g(σ)eϕ(σ)dσ. Γ The norm associated to the scalar product (·|·) is simply denoted by k · k. Pick a1,a2 ∈ C and define the differential operator P = P(∂), where ∂ = (∂1,∂2), by P(∂) = a1∂1 + a2∂2. The formal adjoint of P is given by

∗ ∗ P = P (∂) = −P(∂ + ∂ϕ) = −a1(∂1 + ∂1ϕ) − a2(∂2 + ∂2ϕ).

The unit normal vector to Γ pointing outward Ω is denoted by

ν = ν(σ) = (ν1(σ),ν2(σ)).

Since Γ is Lipschitz ν(σ) is well defined for a.e. σ ∈ Γ . The unit tangent vector to Γ is given by τ(σ) = (−ν2(σ),ν1(σ)).

Let Q be the operator Q = a1ν1 + a2ν2. We obtain from Green’s formula

(Pu|v) = (u|P∗v) + hu|Qvi, u,v ∈ C1(Ω). (4.16)

We recall that [A,B] = AB−BA denotes the usual commutator of the operators A and B.

Lemma 4.1. We have, for any u ∈ C1(Ω),

2 ∗ 2 ∗ kPuk − kP uk = ℜ([P ,P]u|u) + ℜ (2iℑ(a1a2)∂τ − QPϕ)u|u . (4.17)

Proof. We get by applying twice (4.16)

kPuk2 = (Pu|Pu) = (u|P∗Pu) + hu|QPui 156 4 Classical inequalities, Cauchy problems and unique continuation and (PP∗u|u) = (P∗u|P∗u) + hP∗u|Qui = kP∗uk2 + hQP∗u|ui. Hence

kPuk2 = ℜ(P∗Pu|u) + ℜhQPu|ui, (4.18) kP∗uk2 = ℜ(PP∗u|u) − ℜhQP∗u|ui. (4.19)

We obtain by taking the difference side by side of (4.18) and (4.19)

kPuk2 − kP∗uk2 = ℜ([P∗,P]u|u) + ℜhRu|ui, with R = QP + QP∗. Since P∗ = −P − Pϕ,

R = 2iℑ(QP) − QPϕ = 2iℑ(a1a2)∂τ − QPϕ.

Consequently

2 ∗ 2 ∗ kPuk − kP uk = ℜ([P ,P]u|u) + ℜ (2iℑ(a1a2)∂τ − QPϕ)u|u as expected. ut

2 As usual, (x1,x2) ∈ R is identified with z = x1 + ix2 ∈ C. When P = ∂1 + i∂2 = 2∂z,

P = ∂1 − i∂2 = 2∂z, ∗ P = −2(∂z + ∂zϕ), ∗ [P ,P] = −4[∂z + ∂zϕ,∂z] = 4∂z∂zϕ = ∆ϕ,

QPϕ = (ν1 + iν2)(∂1ϕ − i∂2ϕ) = ∂ν ϕ − i∂τ ϕ.

In light of these identities, we have as a consequence of Lemma 4.1

Corollary 4.2. (1) We have, for any real-valued u ∈ C1(Ω),

2 2 4k∂zuk = 4k(∂z + ∂zϕ)uk + (∆ϕu|u) − h∂ν ϕu|ui. (4.20)

In particular, Z Z Z 2 ϕ 2 ϕ 2 ϕ (∆ϕ)u e dx ≤ |∇u| e dx + (∂ν ϕ)u e dσ. (4.21) Ω Ω Γ

(2) We have, for any real-valued u ∈ C2(Ω), Z Z Z 2 ϕ 2 ϕ 2 ϕ ∆ϕ|∇u| e dx ≤ |∆u| e dx + ∂ν ϕ|∇u| e dσ (4.22) Ω Ω Γ Z 2 ϕ + 2 (∆u(∂ν u) − (∂12u)J∇u · ν)e dσ. Γ 4.2 Two-dimensional Cauchy problems 157

 0 1  Here J = . 1 0 Proof. (1) is immediate from Lemma 4.1. To prove (2) we substitute in (4.19) u by ∂zu. We then get (4.22) since 1 1 ℜ(2i∂ ∂ u∂ u) = ℜ(2i∂ ∂ u∂ u) = ∆u(∂ u) − (∂ 2 )uJ∇u · ν. τ z z τ z z 2 ν 2 12 This completes the proof. ut

The sum, side by side, of inequalities (4.21) and (4.22) yields

Proposition 4.1. (Carleman inequality) We have, for any real-valued u ∈ C2(Ω), Z Z (∆ϕ)u2eϕ dx + (∆ϕ − 1)|∇u|2eϕ dx (4.23) Ω Ω Z Z 2 ϕ 2 2 ϕ ≤ |∆u| e dx + ∂ν ϕ(u + |∇u| )e dσ. Ω Γ Z 2 ϕ + 2 (∆u(∂ν u) − (∂12u)J∇u · ν)e dσ. Γ Fix 0 < α < 1. We assume in the remaining part of this section that Ω is of class 2,α C . Let γ be a closed subset of Γ with non empty interior so that Γ0 = Γ \ γ 6= /0.

2 Lemma 4.2. There exists ϕ0 ∈ C (Ω) possessing the properties:

∆ϕ0 = 0 in Ω, ϕ0 = 0 on Γ0, ∂ν ϕ0 < 0 on Γ 0, ϕ0 ≥ 0 on γ.

2,α Proof. Fix Γ˜0, an open subset of Γ so that Γ 0 ⊂ Γ˜0 and Γ \Γ˜ 0 6= /0.Pick χ ∈C (Γ ) non identically equal to zero, satisfying χ = 0 on Γ˜0 and χ ≥ 0 on γ. Since Ω is of 2,α 2,α class C by [5, Theorem 6.8, page 100] there exits a unique ϕ0 ∈ C (Ω) solving the BVP  ∆ϕ0 = 0 in Ω, ϕ0 = χ on Γ .

From the strong maximum principle ϕ0 > 0 in Ω (see Theorem 4.5) and, bearing in mind that ϕ0 = χ = 0 on Γ˜0, we have ∂ν ϕ0 < 0 on Γ˜0 ⊃ Γ 0 according to Hopf’s lemma (see Exercise 3.12). ut

Let Ψ(ρ) = |lnρ|−1/2 + ρ, ρ > 0, and Ψ(0) = 0.

Proposition 4.2. Let M > 0. There exists a constant C = C(M,Ω,γ) so that, for any 2 real-valued function u ∈ C (Ω) satisfying ∆u = 0 in Ω and kukC2(Ω) ≤ M, we have

Z 1/2 Z 1/2! (u2 + |∇u|2)dσ ≤ CΨ (u2 + |∇u|2)dσ . (4.24) Γ γ 158 4 Classical inequalities, Cauchy problems and unique continuation

Proof. For s > 0 let ϕ = ϕ1 + sϕ0, where ϕ0 is as in Lemma 4.2, and ϕ1 is the solution of the BVP  ∆ϕ1 = 2 in Ω, ϕ1 = 0 on Γ . Proposition 4.1 with that ϕ and an arbitrary u ∈ C2(Ω) satisfying ∆u = 0 in Ω and kukC2(Ω) ≤ M yields Z Z Z Z 2 ϕ 2 ϕ 2 2 ϕ ϕ 2 u e dx + |∇u| e dx ≤ ∂ν ϕ(u + |∇u| )e dσ + 2M |∇u|e dσ. Ω Ω Γ Γ

Therefore Z Z 2 2 ϕ ϕ 0 ≤ ∂ν ϕ(u + |∇u| )e dσ + 2M |∇u|e dσ. (4.25) Γ Γ

Let θ = min|∂ν ϕ0|, c = max|ϕ0|, c0 = max|∂ν ϕ0|, c1 = max|∂ν ϕ1| and Γ 0 Ω Γ Γ Z I = (u2 + |∇u|2)dσ, γ Z J = (u2 + |∇u|2)dσ. Γ0 Then (4.25) implies √ cs p cs p 2 0 ≤ (−sθ + c1)J + (sc0 + c1)e I + 2M |γ|e I + 2 |Γ0|M . (4.26)

In the rest of this proof, C = C(M,Ω,γ) and Cj = Cj(M,Ω,γ) are generic con- stants. When s ≥ s0 = 2c1/θ and I ≤ 1, we get from (4.26) √ 1 C J ≤ eC0s I + . (4.27) 1 s √ Let = min1,e−C0s0 /s . If ≤ = min( ,1) then there exists s ≥ s κ0 √ 0 I κ κ0 ∗ 0 c s so that s∗e 0 ∗ = 1/ I . Therefore s = s∗ in (4.27) gives

√ −1

J ≤ C ln I . (4.28)

In the case where I ≥ κ2, we have CM2 J ≤ CM2 ≤ I . (4.29) κ2 Combining (4.28) and (4.29) we end up getting

 √ −1/2 √  p I + J ≤ C ln I + I 4.2 Two-dimensional Cauchy problems 159 as expected. ut

Corollary 4.3. Let M > 0. There exists C = C(M,Ω,γ) so that, for any real-valued 2 function u ∈ C (Ω) satisfying ∆u = 0 in Ω and kukC2(Ω) ≤ M, we have

Z 1/2 Z 1/2! |∇u|2dx ≤ CΨ (u2 + |∇u|2)dσ . (4.30) Ω γ

Proof. We have from Green’s formula Z Z Z 2 0 = ∆uudx = − |∇u| dx + ∂ν uudσ. Ω Ω Γ Whence Z Z Z Z 2 2 2 2 2 |∇u| dx = ∂ν uudσ ≤ 2 (u + (∂ν u) )dσ ≤ 2 (u + |∇u| )dσ. Ω Γ Γ Γ Therefore (4.30) follows from (4.24). ut

Lemma 4.3. There exists a constant C = C(Ω) > 0 so that, for any real-valued function u ∈ H1(Ω), we have

Z Z Z  u2dx ≤ C |∇u|2dx + u2dσ . (4.31) Ω Ω Γ Proof. We proceed by contradiction. We assume then that, for each integer k ≥ 1, 1 there exists uk ∈ H (Ω) so that Z Z Z  2 2 2 ukdx > k |∇uk| dx + ukdσ . (4.32) Ω Ω Γ

If vk = uk/kukkL2(Ω) then (4.32) gives Z Z 2 2 1 |∇vk| dx + vkdσ < . Ω Γ k

1 2 Then the sequence (vk) is bounded in H (Ω) and L (Γ ). Subtracting a subsequence 1 if necessary, we may assume that vk converges to v, weakly in H (Ω), strongly in L2(Ω) and weakly in L2(Γ ). Using the lower semi-continuity of a norm with respect to the weak topology, we obtain

Z Z Z Z  2 2 2 2 |∇v| dx + v dσ ≤ liminf |∇vk| dx + vkdσ = 0. Ω Γ Ω Γ Hence v = 0. But 1 = kvkkL2(Ω) → kvkL2(Ω) = 1. This leads to the expected contradiction. ut 160 4 Classical inequalities, Cauchy problems and unique continuation

The following corollary is a consequence of Corollary 4.3, Lemma 4.3 and the identity 2 2 2 |∇u| = (∂τ u) + (∂ν u) . Corollary 4.4. Let M > 0. There exists a constant C = C(M,Ω,γ) so that, for any 2 real-valued function u ∈ C (Ω) satisfying ∆u = 0 in Ω and kukC2(Ω) ≤ M, we have   kukH1(Ω) ≤ CΨ kukH1(γ) + k∂ν ukL2(γ) .

This result is nothing but a logarithmic stability of the Cauchy problem for har- monic functions with data on γ. This corollary contains obviously the uniqueness of continuation from the Cauchy data on γ.

Corollary 4.5. Let u ∈ C2(Ω) be a real-valued function satisfying ∆u = 0 in Ω. If u = ∂ν u = 0 on γ then u is identically equal to zero.

4.3 A Carleman inequality for a family of operators

n Let Ω be a bounded domain of R with Lipschitz boundary Γ . Let I be an arbitrary set and consider the family of operators

Lt = div(At ∇·), t ∈ I ,

i j where for each t ∈ I the matrix At = (at ) is a symmetric with coefficients in W 1,∞(Ω). We assume that there exist κ > 0 and κ ≥ 1 so that

−1 2 2 n κ |ξ| ≤ At (x)ξ · ξ ≤ κ|ξ| , x ∈ Ω, ξ ∈ R , t ∈ I , (4.33)

and n n i j 2 n ∑ ∑ ∂kat (x)ξiξ j ≤ κ|ξ| , x ∈ Ω, ξ ∈ R , t ∈ I . (4.34) k=1 i, j=1 Pick 0 ≤ ψ ∈ C2(Ω) without critical points in Ω and let ϕ = eλψ .

Theorem 4.7. (Carleman inequality) There exist three positive constants C, λ0 and τ0, only depending on ψ, Ω, κ and κ, so that Z C λ 4τ3ϕ3v2 + λ 2τϕ|∇v|2e2τϕ dx Ω Z Z 2 2τϕ 3 3 3 2 2 2τϕ ≤ (Lt v) e dx + λ τ ϕ v + λτϕ|∇v| e dσ, (4.35) Ω Γ

2 for all v ∈ H (Ω), t ∈ I , λ ≥ λ0 and τ ≥ τ0. 4.3 A Carleman inequality for a family of operators 161

Proof. Since the dependance of the constants will be uniform with respect to t ∈ I , we drop for simplicity the subscript t in Lt and its coefficients. Let Φ = e−τϕ and w ∈ H2(Ω). Then straightforward computations give

−1 Pw = [Φ LΦ]w = P1w + P2w + cw, where

P1w = aw + div(A∇w),

P2w = B · ∇w + bw, with

2 2 2 2 a = a(x,λ,τ) = λ τ ϕ |∇ψ|A, B = B(x,λ,τ) = −2λτϕA∇ψ, 2 2 b = b(x,λ,τ) = −2λ τϕ|∇ψ|A, 2 2 c = c(x,λ,τ) = −λτϕdiv(A∇ψ) + λ τϕ|∇ψ|A.

Here p |∇ψ|A = A∇ψ · ∇ψ. We obtain by making integrations by parts Z 1 Z aw(B · ∇w)dx = a(B · ∇w2)dx Ω 2 Ω 1 Z 1 Z = − div(aB)w2dx + a(B · ν)w2dσ (4.36) 2 Ω 2 Γ and Z Z Z div(A∇w)(B · ∇w)dx = − A∇w · ∇(B · ∇w)dx + (B · ∇w)(A∇w · ν)dσ Ω Ω Γ Z = − B0∇w · A∇wdx Ω Z Z − ∇2wB · A∇wdx + (B · ∇w)(A∇w · ν)dσ. (4.37) Ω Γ

0 2 2 Here B = (∂ jBi) is the Jacobian matrix of B and ∇ w = (∂i jw) is the Hessian matrix of w. But Z Z 2 ik ik 2 Bi∂i jwa ∂kwdx = − Bia ∂ikw∂ jwdx Ω Ω Z Z h iki jk − ∂i Bia ∂kw∂ jwdx + Biνia ∂kw∂ jwdσ. Ω Γ Therefore 162 4 Classical inequalities, Cauchy problems and unique continuation Z 1 Z ∇2wB · A∇wdx = − div(B)A + A˜∇w · ∇wdx Ω 2 Ω Z 1 2 + |∇w|A(B · ν)dσ, (4.38) 2 Γ with A˜ = (a˜i j),a ˜i j = B · ∇ai j. It follows from (4.37) and (4.38) Z 1 Z div(A∇w)B · ∇wdx = −2AB0 + div(B)A + A˜∇w · ∇wdx Ω 2 Ω Z Z 1 2 + (B · ∇w)(A∇w · ν)dσ − |∇w|A(B · ν)dσ. (4.39) Γ 2 Γ A new integration by parts yields Z Z Z Z 2 div(A∇w)bwdx = − b|∇w|Adx − w∇b · A∇wdx + bw(A∇w · ν)dσ. Ω Ω Ω Γ This and the following inequality Z Z Z 2 −1 2 2 2 2 − w∇b · A∇wdx ≥ − (λ ϕ) |∇b|Aw dx − λ ϕ|∇w|Adx Ω Ω Ω imply Z Z Z 2 2 2 −1 2 2 div(A∇w)bwdx ≥ − (b + λ ϕ)|∇w|Adx − (λ ϕ) |∇b|Aw dx Ω Ω Ω Z + bw(A∇w · ν)dσ. (4.40) Γ Now a combination of (4.36), (4.39) and (4.40) leads Z Z Z Z Z 2 2 2 P1wP2wdx − c w dx ≥ f w dx + F∇w · ∇wdx + g(w)dσ, (4.41) Ω Ω Ω Ω Γ where 1 f = − div(aB) + ab − (λ 2ϕ)−1|∇b|2 − c2, 2 A 1  F = −AB0 + div(B)A + A˜ − (b + λ 2ϕ)A, 2 1 1 g(w) = aw2(B · ν) − |∇w|2 (B · ν) + (B · ∇w)(A∇w · ν) + bw(A∇w · ν). 2 2 A We deduce, by using the elementary inequality (p − q)2 ≥ p2/2 − q2, p > 0, q > 0, that 4.4 Three-ball inequalities 163

2 2 kPwk2 ≥ (kP1w + P2wk2 − kcwk2) 1 ≥ kP w + P wk2 − kcwk2 2 1 2 2 2 Z Z 2 2 ≥ P1wP2wdx − c w dx. Ω Ω In light of (4.41), we obtain Z Z Z 2 2 kPwk2 ≥ f w dx + F∇w · ∇wdx + g(w)dσ. (4.42) Ω Ω Γ

By straightforward computations, there exist four positive constants C0, C1, λ0 and τ0, only depending on ψ, Ω, κ and κ, such that, for all λ ≥ λ0 and τ ≥ τ0,

4 3 3 f ≥ C0λ τ ϕ , 2 2 n Fξ · ξ ≥ C0λ τϕ|ξ| , for any ξ ∈ R , 3 3 3 2 2 |g(w)| ≤ C1 λ τ ϕ w + λτϕ|∇w| .

Hence Z Z C (λ 4τ3ϕ3w2 + λ 2τϕ|∇w|2)dx ≤ (Pw)2dx Ω Ω Z + (λ 3τ3ϕ3w2 + λτϕ|∇w|2)dσ. Γ

Finally, w = Φ−1v, v ∈ H2(Ω), in the previous inequality gives (4.35) in straight- forward manner. ut Remark 4.1. Observe that Theorem 4.7 holds for complex-valued v ∈ H2(Ω). In that 2 2 2 case we have to substitute v and Lt v respectively by |v| and |Lt v| .

4.4 Three-ball inequalities

The following Caccioppoli’s type inequality will be useful in the sequel. The nota- tions and the assumptions are those of the preceding section. Lemma 4.4. Let 0 < k < `. There exists a constant C > 0, only depending on Ω, k, `, κ and κ, so that, for any x ∈ Ω, 0 < ρ < dist(x,Γ )/` and u ∈ H1(Ω) satisfying 2 Lt u ∈ L (Ω) in Ω, for some t ∈ I , we have Z Z Z 2 1 2 2 C |∇u| dy ≤ 2 u dy + (Lt u) dy. (4.43) B(x,kρ) ρ B(x,`ρ) B(x,`ρ)

Proof. We give the proof for k = 1 and ` = 2. That for arbitrary k and ` is similar. 1 2 Let x ∈ Ω, 0 < ρ < dist(x,Γ )/2, t ∈ I and u ∈ H (Ω) satisfying Lt u ∈ L (Ω). Then 164 4 Classical inequalities, Cauchy problems and unique continuation Z n Z i j 1 ∑ at ∂iu∂ jvdy = Lt uvdy for any v ∈ C0 (Ω). (4.44) Ω i, j=1 Ω ∞ Pick χ ∈ C0 (B(x,2ρ)) so that 0 ≤ χ ≤ 1, χ = 1 in a neighborhood of B(x,ρ) and |∂ γ χ| ≤ cr−|γ| for |γ| ≤ 2, where c is a constant not depending on ρ. Therefore, identity (4.44) with v = χu gives

n n Z i j Z i j Z χ ∑ at ∂iu∂ judy = − u ∑ at ∂iu∂ jχdy + χuLt udy Ω i; j=1 Ω i, j=1 Ω Z n Z 1 i j 2 = − ∑ at ∂iu ∂ jχdy + χuLt udy 2 Ω i, j=1 Ω Z n Z 1 2  i j  = u ∑ ∂i at ∂ jχ dy + χuLt udy. 2 Ω i j=1 Ω

But Z n Z i j 2 χ ∑ at ∂iu∂ judy ≥ κ χ|∇u| dy. Ω i, j=1 Ω Whence Z Z Z 2 1 2 2 C |∇u| dy ≤ 2 u dy + (Lt u) dy. B(x,ρ) ρ B(x,2ρ) B(x,2ρ) This is the expected inequality. ut Consider L = div(A∇·), where A = (ai j) is a symmetric matrix with W 1,∞(Ω) entries satisfying: there exist κ > 0 and κ ≥ 1 so that

−1 2 2 n κ |ξ| ≤ A(x)ξ · ξ ≤ κ|ξ| , x ∈ Ω, ξ ∈ R , (4.45) and d d i j 2 n ∑ ∑ ∂ka (x)ξiξ j ≤ κ|ξ| , x ∈ Ω, ξ ∈ R . (4.46) k=1 i, j=1 Theorem 4.8. Let 0 < k < ` < m. There exist C > 0 and 0 < γ < 1, only depending on Ω, k, `, m, κ and κ, so that, for any v ∈ H1(Ω) satisfying Lv ∈ L2(Ω) in Ω, y ∈ Ω and 0 < r < dist(y,Γ )/m, we have

 γ 1−γ Ckvk 2 ≤ kvk 2 + kLvk 2 kvk . L (B(y,`r)) L (B(y,kr)) L (B(y,mr)) L2(B(y,mr))

Proof. As in the preceding lemma we give the proof when k = 3/2, ` = 2 and m = 7/2. The proof of arbitrary k, ` and m is similar. Let v ∈ H1(Ω) satisfying Lv ∈ L2(Ω) and set B(s) = B(0,s), s > 0. Fix y ∈ Ω and 0 < r < ry = 2dist(y,Γ )/7 (≤ 2diam(Ω)/7). 4.4 Three-ball inequalities 165

Let w(x) = v(rx + y), x ∈ B(7/2). Straightforward computations show

2 Lrw = div(Ar∇w) = r Lv(rx + y) in B(7/2), (4.47) where i j i j i j Ar(x) = (ar (x)), ar (x) = a (rx + y).

It is not hard to see that the family (Ar) satisfies (4.45) and (4.46) uniformly with respect to r ∈ (0,ry). Set

n n U = {x ∈ R ; 1/2 < |x| < 3}, K = {x ∈ R ; 1 ≤ |x| ≤ 5/2}.

∞ and pick χ ∈ C0 (U) satisfying 0 ≤ χ ≤ 1 and χ = 1 in a neighborhood of K . We get by applying Theorem 4.7 to χw, with Ω is substituted by U, that for any λ ≥ λ0 and any τ ≥ τ0, Z C λ 4τ3ϕ3w2 + λ 2τϕ|∇w|2e2τϕ dx B(2)\B(1) Z 2 2τϕ ≤ (Lr(χw)) e dx. (4.48) B(3)

We have Lr(χw) = χLrw + Qr(w) with

i j i j i j 2 Qr(w) = ∂ jχar ∂iw + ∂ j(ar w)∂iw + ar ∂i jχw,

supp(Qrw)) ⊂ {1/2 ≤ |x| ≤ 1} ∪ {5/2 ≤ |x| ≤ 3} and 2 2 2 (Qrw) ≤ Λ(w + |∇w| ), where the constant Λ is independent of r. Therefore, fixing λ and changing τ0 if necessary, (4.47) implies for τ ≥ τ0 Z Z C w2 + |∇w|2e2τϕ dx ≤ w2 + |∇w|2e2τϕ dx B(2) B(1) Z Z 2 2τϕ 2 2 2τϕ + (Lrw) e dx + w + |∇w| e dx. (4.49) B(3) {5/2≤|x|≤3}

We get by taking ψ(x) = 9 − |x|2 in (4.49), which is without critical points in U, that for τ ≥ τ0 166 4 Classical inequalities, Cauchy problems and unique continuation Z Z Z  2 2 ατ 2 2 2 C w + |∇w| dx ≤ e w + |∇w| dx + (Lrw) dx (4.50) B(2) B(1) B(3) Z + e−βτ w2 + |∇w|2dx, B(3) where     α = e9λ − e5λ , β = e5λ − e11λ/4 .

On the other hand, we have by Caccioppoli’s inequality (4.43) Z Z Z 2 2 2 C |∇w| dx ≤ w dx + (Lrw) dx, (4.51) B(1) B(3/2) B(3/2) Z Z Z 2 2 2 C |∇w| dx ≤ w dx + (Lrw) dx (4.52) B(3) B(7/2) B(7/2)

Inequalities (4.51) and (4.52) in (4.50) yield

Z Z Z  Z 2 ατ 2 2 −βτ 2 C w dx ≤ e w dx + (Lrw) dx + e w dx. (4.53) B(2) B(3/2) B(7/2) B(7/2)

We introduce the temporary notations Z Z 2 2 P = w dx + (Lrw) dx, B(3/2) B(7/2) Z Q = C w2dx, B(2) Z R = w2dx. B(7/2)

Then (4.53) becomes ατ −βτ Q ≤ e P + e R, τ ≥ τ0. (4.54) Let ln(R/P) τ = . 1 α + β

If τ1 ≥ τ0 then τ = τ1 in (4.54) yields

α β Q ≤ 2P α+β R α+β . (4.55)

(α+β)τ If τ1 < τ0, we have R < e 0 P and then

α β α β Q ≤ R = R α+β R α+β ≤ eατ0 P α+β R α+β . (4.56)

Summing up, we find that in any case one of inequalities (4.55) and (4.56) holds. That is in terms the original notations 4.4 Three-ball inequalities 167

 γ 1−γ Ckwk 2 ≤ kwk 2 + kL wk 2 kwk , (4.57) L (B(2)) L (B(3/2)) r L (B(7/2)) L2(B(7/2)) with α γ = . α + β We derive in a straightforward manner from (4.47) and (4.57) that

 γ 1−γ Ckvk 2 ≤ kvk 2 + kLvk 2 kvk . L (B(y,2r)) L (B(y,3r/2)) L (B(y,7r/2)) L2(B(y,7r/2))

This is the expected inequality. ut

Prior to establishing the three-sphere inequality for the gradient we prove a gen- eralized Poincare-Wirtinger´ type inequality. For this purpose, let O be an arbitrary n open bounded subset of R . Define, for f ∈ L2(O) and E ⊂ O Lebesgue-measurable with non zero Lebesgue measure |E|,

1 Z ME ( f ) = f (x)dx. |E| E

Proposition 4.3. There exits a constant C > 0, only depending on O so that, for any f ∈ H1(O) and any Lebesgue-measurable set E with non zero Lebesgue measure, we have |O|1/2 k f − ME ( f )k 2 ≤ C k∇ f k 2 n . (4.58) L (O) |E|1/2 L (O,R ) Proof. A simple application of Cauchy-Schwarz inequality gives

|O|1/2 kME ( f )k 2 ≤ k f k 2 . (4.59) L (O) |E|1/2 L (O)

Inequality (4.59) with E = O and f substituted by f − ME ( f ) yields

kMO ( f − ME ( f ))kL2(O) ≤ k f − ME ( f )kL2(O). (4.60) On the other hand, by the classical Poincare-Wirtinger’s´ inequality (see Exercise 1.19) there exists a constant C, only depending on O, so that

k f − M ( f )k 2 ≤ Ck f k 2 n . O L (O) ∇ L (O,R ) (4.61)

Now, as ME (MO ( f )) = MO ( f ), we have

f − ME ( f ) = f − MO ( f ) − ME ( f − MO ( f )).

We then obtain in light of (4.59) ! |O|1/2 k f − ME ( f )k 2 ≤ 1 + k f − M ( f )k 2 L (O) |E|1/2 O L (O) 168 4 Classical inequalities, Cauchy problems and unique continuation implying |O|1/2 k f − ME ( f )k 2 ≤ 2 k f − M ( f )k 2 . L (O) |E|1/2 O L (O) Whence |O|1/2 k f − ME ( f )k 2 ≤ 2C k∇ f k 2 n . L (O) |E|1/2 L (O,R ) The proof is then complete. ut

Theorem 4.9. Let 0 < k < ` < m. There exist C > 0 and 0 < γ < 1, only depending on Ω, k, `, m, κ and κ, so that, for any v ∈ H1(Ω) satisfying Lv ∈ L2(Ω), y ∈ D and 0 < r < dist(y,Γ )/m, we have

 γ 1−γ Ck∇vk 2 n ≤ k∇vk 2 n + kLvk 2 k∇vk . L (B(y,`r),R ) L (B(y,kr),R ) L (B(y,mr)) L2(B(y,mr),Rn) Proof. We keep the same notations as in the preceding proof. We take k = 1, ` = 2 and m = 3. The proof for arbitrary 0 < k < ` < m is the same. An application of the generalized Poincare-Wirtinger’s´ inequality in Proposition 4.3 then yields 1 Z ρ = w(x)dx, |B(1)| B(1)

Z Z (w − ρ)2dx ≤ C |∇w|2dx, (4.62) B(1) B(1) Z Z (w − ρ)2dx ≤ C |∇w|2dx. (4.63) B(3) B(3)

On the other hand (4.50), in which w is substituted by w − ρ, gives

Z Z Z  2 ατ 2 2 2 C |∇w| dx ≤ e (w − ρ) + |∇w| dx + (Lrw) dx (4.64) B(2) B(1) B(3) Z + e−βτ (w − ρ)2 + |∇w|2dx. B(3)

In view of (4.62) and (4.63) in (4.64), we get

Z Z Z  Z 2 ατ 2 2 −βτ 2 C |∇w| dx ≤ e |∇w| dx + (Lrw) dx + e |∇w| dx. B(2) B(1) B(3) B(3)

The rest of the proof is similar to that of Theorem 4.8. ut

4.5 Stability of the Cauchy problem

The following lemma will useful in the sequel. 4.5 Stability of the Cauchy problem 169

Lemma 4.5. Let (ηk) be a sequence of real numbers satisfying 0 < ηk ≤ 1, k ∈ N, and γ ηk+1 ≤ c(ηk + b) , k ∈ N, for some constants 0 < γ < 1, b > 0 and c ≥ 1. Then

γk ηk ≤ C(η0 + b) , (4.65) where C = (2c)1/(1−γ).

Proof. Note first that (4.65) is trivially satisfied when η0 + b ≥ 1. Assume then that η0 + b < 1. As γ γ b < cb < c(ηk + b) , k ∈ N, we obtain γ ηk+1 + b ≤ 2c(ηk + b) . (4.66)

If τk = ηk + b then (4.66) can rewritten as follows

γ τk ≤ 2cτk , k ∈ N. An induction in k yields

1+γ+...+γk−1 γk 1/(1−γ) γk ηk ≤ (2c) τ0 ≤ (2c) (η0 + b) . The proof is then complete. ut Note that, as Ω is Lipschitz, it has the uniform cone property. Whence, there n− exist R > 0 and θ ∈]0,π/2[ so that to anyx ˜ ∈ Γ corresponds ξ = ξ(x˜) ∈ S 1 with the property that

n C (x˜) = {x ∈ R ; 0 < |x − xe| < R, (x − x˜) · ξ > |x − x˜|cosθ} ⊂ Ω.

Proposition 4.4. Let 0 < α ≤ 1. There exist ω b Ω, depending only on Ω and three constants C > 0, c > 0 and β > 0, depending only on Ω, κ, κ and α, so that for any 0 < ε < 1: (1) for any u ∈ H1(Ω) ∩C0,α (Ω) with Lu ∈ L2(Ω), we have

c/ε   β   CkukL∞(Γ ) ≤ e kukL2(ω) + kLukL2(Ω) + ε [u]α + kukL2(Ω) ,

(2) for any u ∈ C1,α (Ω) with Lu ∈ L2(Ω), we have

c/ε   β   Ck uk ∞ n ≤ e k uk 2 n + kLuk 2 + [ u] + k uk 2 n . ∇ L (Γ ,R ) ∇ L (ω,R ) L (Ω) ε ∇ α ∇ L (Ω,R )

Here [∇u]α = max1≤i≤n[∂iu]α . Proof. Fixx ˜ ∈ Γ and let ξ = ξ(x˜) be as in the definition of the uniform cone prop- erty. Let x0 = x˜+ (R/2)ξ, d0 = |x0 − x˜| and ρ0 = d0 sinθ/3. Note that B(x0,3ρ0) ⊂ C (x˜). 170 4 Classical inequalities, Cauchy problems and unique continuation

Define the sequence of balls (B(xk,3ρk)) as follows   xk+1 = xk − αkξ, ρk+1 = µρk,  δk+1 = µδk, where δk = |xk − xe|, ρk = κδk, αk = (1 − µ)δk, with κ = sinθ/3, µ = 1 − κ.

This definition guarantees that, for each k, B(xk,3ρk) ⊂ C (x˜) and

B(xk+1,ρk+1) ⊂ B(xk,2ρk). (4.67)

Let u ∈ H1(Ω) ∩C0,α (Ω) with u 6= 0 and Lu ∈ L2(Ω). From Theorem 4.8, we have

1−γ  γ kuk 2 ≤ Ckuk 2 kuk 2 + kLuk 2 L (B(xk,2ρk)) L (B(xk,3ρk)) L (B(xk,ρk)) L (B(xk,3ρk)) and then

1−γ  γ kuk 2 ≤ Ckuk kuk 2 + kLuk 2 L (B(xk,2ρk)) L2(Ω)) L (B(xk,ρk)) L (Ω)

But B(xk+1,ρk+1) ⊂ B(xk,2ρk). Hence,

1−γ  γ kuk 2 ≤ Ckuk kuk 2 + kLuk 2 , L (B(xk+1,ρk+1)) L2(Ω) L (B(xk,ρk)) L (Ω) or equivalently !γ kukL2(B(x , )) kuk 2 kLuk 2 k+1 ρk+1 ≤ C L (B(xk,ρk)) + L (Ω) . kukL2(Ω) kukL2(Ω) kukL2(Ω)

Substituting if necessary C by max(C,1), we may assume that C ≥ 1. Lemma 4.5 then yields

!γk kuk 2 kuk 2 kLuk 2 L (B(xk,ρk)) ≤ C L (B(x0,ρ0)) + L (Ω) . kukL2(Ω) kukL2(Ω) kukL2(Ω)

This inequality can be rewritten in the following form

k  γ 1−γk kuk 2 ≤ C kuk 2 + kLuk 2 kuk . (4.68) L (B(xk,ρk)) L (B(x0,ρ0)) L (Ω) L2(Ω)

An application of Young’s inequality, for ε > 0, gives 4.5 Stability of the Cauchy problem 171

−1/γk   1/(1−γk) Ckuk 2 ≤ ε kuk 2 + kLuk 2 + ε kuk 2 . L (B(xk,ρk)) L (B(x0,ρ0)) L (Ω) L (Ω) (4.69) We have, by using the Holder¨ continuity of u,

α |u(x˜)| ≤ [u]α |x˜− x| + |u(x)|, x ∈ B(xk,ρk).

Whence Z Z n−1 n 2 2 2α 2 |S |ρk |u(x˜)| ≤ 2[u]α |x˜− x| dx + 2 |u(x)| dx, B(xk,ρk) B(xk,ρk) or equivalently

 Z Z  2 n−1 −1 −n 2 2α 2 |u(x˜)| ≤ 2|S | ρk [u]α |x˜− x| dx + |u(x)| dx . B(xk,ρk) B(xk,ρk)

k As δk = µ δ0, we have

k |x˜− x| ≤ |x˜− xk| + |xk − x| ≤ δk + ρk = (1 + κ)δk = (1 + κ)µ δ0. Therefore

|u(x˜)|2 (4.70)  2 α α 2αk n−1 −1 −n −nk 2  ≤ 2 [u] (1 + ) δ µ + | | ( d0) µ kuk 2 . α κ 0 S κ L (B(xk,ρk)) Let [ ω = B(x0(x˜),ρ0) x˜∈Γ and introduce the following temporary notations

M = [u]α + kukL2(Ω),

N = kukL2(ω) + kLukL2(Ω). Then (4.70) yields

αk −nk/2 C|u(x)| ≤ Mµ + µ kuk 2 . (4.71) e L (B(xk,ρk)) A combination of (4.69) and (4.71) entails

−nk/2 −1/γk  αk −nk/2 1/(1−γk) CkukL∞(Γ ) ≤ µ ε N + µ + µ ε M, ε > 0.

k In this inequality we take ε > 0 in such a way that µαk = µ−nk/2ε1/(1−γ ). That is k ε = µ(n/2+α)k(1−γ ). We obtain

αk− k ( n +α) γk 2 αk CkukL∞(Γ ) ≤ µ N + µ M. 172 4 Classical inequalities, Cauchy problems and unique continuation

For t > 0, let k be the integer so that k ≤ t < k+1. Bearing in mind that 0 < µ,γ < 1, we deduce by straightforward computations from the preceding inequality

−ect αt CkukL∞(Γ ) ≤ µ N + µ M.

Take ect = 1/ε, we end up getting

c/ε β CkukL∞(Γ ) ≤ e N + ε M, 0 < ε < 1, which is the expected inequality in (1). We omit the proof of (2) which is quite similar of that of (1). We have only to apply Theorem 4.9 instead of Theorem 4.8. ut Proposition 4.5. Let ω b Ω and ω˜ b Ω be non empty. There exist C > 0 and β > 0, only depending on Ω, κ, κ, ω and ω˜ , so that, for any u ∈ H1(Ω) satisfying Lu ∈ L2(Ω) and ε > 0, we have

β −1   CkukL2(ω˜ ) ≤ ε kukL2(Ω) + ε kukL2(ω) + kLukL2(Ω) , (4.72)

β −1   Ck uk 2 n ≤ k uk 2 n + k uk 2 n + kLuk 2 . ∇ L (ω˜ ,R ) ε ∇ L (Ω,R ) ε ∇ L (ω,R ) L (Ω) (4.73)

Proof. We limit ourselves to the proof of (4.72). That of (4.73) is similar. Fix x0 ∈ ω and x ∈ ω˜ . There exists a sequence of balls B(x j,r), r > 0, j = 0,...,N, so that  B(x0,r) ⊂ ω,   B(x j+1,r) ⊂ B(x j,2r), j = 0,...,N − 1, x ∈ B(xN,r),   B(x j,3r) ⊂ Ω, j = 0,...,N.

We give in the end of this proof the construction of such sequence of balls. We get from Theorem 4.8

1−γ  γ kuk 2 ≤ Ckuk 2 kuk 2 + kLuk 2 , 1 ≤ j ≤ N, L (B(x j,2r)) L (B(x j,3r)) L (B(x j,r)) L (Ω) for some constants C > 0 and 0 < γ < 1, only depending on Ω, κ and κ. We obtain by proceeding as in the proof of Proposition 4.4

N 1−γN  γ kuk 2 ≤ Ckuk 2 kuk 2 + kLuk 2 . L (B(xN ,2r)) L (B(x j,3r)) L (B(x0,r)) L (Ω)

Combined with Young’s inequality this estimate yields

β −1   Ckuk 2 ≤ kuk 2 + kuk 2 + kLuk 2 , L (B(xN ,2r)) ε L (Ω) ε L (ω) L (Ω) where γN β = . 1 − γN 4.5 Stability of the Cauchy problem 173

As ω˜ is compact, it can be covered by a finite number of balls B(xN,r) that we 1 ` denote by B(xN,r),...,B(xN,r). Hence

` kuk 2 ≤ kuk 2 . L (ω˜ ) ∑ L (B(xN ,r)) i=1 Whence

β −1   CkukL2(ω˜ ) ≤ ε kukL2(Ω) + ε kukL2(ω) + kLukL2(Ω) ,

We complete the proof by showing how we construct the sequence of balls B(x j,r). Let γ : [0,1] → Ω be a continuous path joining x0 to x. That is γ is a con- tinuous function so that γ(0) = x0 and γ(1) = x. Fix r > 0 so that B(x0,r) ⊂ ω and n 3r < dist(γ([0,1]),R \ Ω). Let t0 = 0 and tk+1 = inf{t ∈ [tk,1]; γ(t) 6∈ B(γ(tk),r)}, k ≥ 0. We claim that there exists an integer N ≥ 1 so that γ(1) ∈ B(xN,r). If this claim does not hold, we would have γ(1) 6∈ B(γ(tk),r), for any k ≥ 0. Now, as the sequence (tk) is non decreasing and bounded from above by 1 it converges to tˆ ≤ 1. In particular, there exists an integer k0 ≥ 1 so that γ(tk) ∈ B(γ(tˆ),r/2), k ≥ k0. But this contradicts the fact that |γ(tk+1) − γ(tk)| = r, k ≥ 0. Let xk = γ(tk), 0 ≤ k ≤ N. Then x = γ(1) ∈ B(xN,r), and since

n 3r < dist(γ([0,1]),R \ Ω), we have B(xk,3r) ⊂ Ω. Finally, if |y − xk+1| < r then

|y − xk| ≤ |y − xk+1| + |xk+1 − xk| < 2r.

In other words, B(xk+1,r) ⊂ B(xk,2r). ut

Proposition 4.6. Let Γ0 be a non empty open subset of Γ . There exist ω0 b Ω, de- pending only on Ω and Γ0, and two constants C > 0 and γ > 0, depending only on Ω, κ and κ, so that, for any u ∈ H1(Ω) satisfying Lu ∈ L2(Ω) and ε > 0, we have

γ −1   Ckuk 1 ≤ ε kuk 1 + ε kuk 2 + k∇uk 2 n + kLuk 2 . H (ω0) H (Ω) L (Γ0) L (Γ0,R ) L (Ω)

Proof. Letx ˜ ∈ Γ0 be arbitrarily fixed and let R > 0 so that B(x˜,R) ∩Γ ⊂ Γ0. Pick x0 n in the interior of R \Ω sufficiently close tox ˜ is such a way that ρ = dist(x0,K) < R, where K = B(x˜,R) ∩Γ0 (think to the fact that Ω is on one side of its boundary). Fix then r > 0 in order to satisfy B(x0,ρ + r) ∩Γ ⊂ Γ0 and B(x0,ρ + θr) ∩ Ω 6= /0,for some 0 < θ < 1. Define (ρ + r)2 ψ(x) = ln 2 . |x − x0| Then 2 2 |∇ψ(x)| = ≥ , x ∈ B(x0,ρ + r) ∩ Ω. |x − x0| ρ + r 174 4 Classical inequalities, Cauchy problems and unique continuation

∞ α Pick χ ∈ C0 (B(x0,ρ + r)), χ = 1 in B(x0,(1 + θ)r/2) and |∂ χ| ≤ κ, |α| ≤ 2, for some constant κ. Let u ∈ H1(Ω) satisfying Lu ∈ L2(Ω). As in proof of Theorem 4.8, we have L(χu) = χLu + Q(u) with

Q(u)2 ≤ C u2 + |∇u|2 and supp(Q) ⊂ B(x0,ρ + r) \ B(x0,(1 + θ)r/2) := D.

It follows, from Theorem 4.7 applied to v = χu in the domain Ω ∩ B(x0,ρ + r), where λ ≥ λ0 is fixed and τ ≥ τ0, that

Z Z Z C e2τϕ u2dx ≤ e2τϕ (u2 + |∇u|2)dx + e2τϕ (Lu)2dx B(x0,ρ+θr)∩Ω D∩Ω B(x0,ρ+r)∩Ω Z + e2τϕ (u2 + |∇u|2)dσ. B(x0,ρ+r)∩Γ But (ρ+r)2 2λ λ ln 2 (ρ + r) ϕ(x) = e |x−x0| = . 2λ |x − x0| Whence Z Z Ceτϕ0 u2dx ≤ eτϕ1 (u2 + |∇u|2)dx (4.74) B(x0,ρ+θr)∩Ω D∩Ω Z Z + eτϕ2 (Lu)2dx + eτϕ2 (u2 + |∇u|2)dσ, B(x0,ρ+r) B(x0,ρ+r)∩Γ where

2(ρ + r)2λ 2(ρ + r)2λ 2(ρ + r)2λ ϕ0 = , ϕ1 = , ϕ2 = . (ρ + θr)2λ (ρ + (1 + θ)r/2)2λ ρ2λ Let 2rλ(1 − θ)(ρ + r)2λ 4λθr(ρ + d)2λ α = and β = . (ρ + (1 + θ)r/2)2λ+1 ρ2λ+1 Elementary computations show that

ϕ0 − ϕ1 ≥ α and ϕ2 − ϕ0 ≤ β.

These inequalities in (4.74) yield Z Z C u2dx ≤ e−ατ (u2 + |∇u|2)dx (4.75) B(x0,ρ+θr)∩Ω D∩Ω Z Z + eβτ (Lu)2dx + eβτ (u2 + |∇u|2)dσ. B(x0,ρ+r)∩Ω B(x0,ρ+r)∩Γ 4.5 Stability of the Cauchy problem 175

Let ω0 b ω1 b B(x0 + ρ + θr) ∩ Ω. Then Caccioppoli’s inequality gives Z Z Z C |∇u|2dx ≤ u2dx + (Lu)2dx. (4.76) ω0 ω1 ω1 Using (4.76) in (4.75) we obtain Z Z C (u2 + |∇u|2)dx ≤ e−ατ (u2 + |∇u|2)dx ω0 Ω Z Z + eβτ (Lu)2dx + eβτ (u2 + |∇u|2)dσ. Ω Γ0 We complete the proof similarly to that of Theorem 4.8. ut We shall need the following lemma. Lemma 4.6. There exists a constant C > 0 so that, for any u ∈ H1(Ω) with Lu ∈ L2(Ω), we have CkukH1(Ω) ≤ kLukL2(Ω) + kukH1/2(Γ ). (4.77) 1 1 Proof. Fix u ∈ H (Ω) and let F ∈ H (Ω) so that F|Γ = u|Γ and kFkH1(Ω) = kukH1/2(Γ ) (by Exercise 1.20, F exists and it is unique). According to Lax-Milgram’s Lemma the variational problem Z Z Z 1 A∇v · ∇wdx = − A∇F · wdx − Luwdx, w ∈ H0 (Ω) (4.78) Ω Ω Ω

1 has a unique solution v ∈ H0 (Ω). We obtain by taking w = v in (4.78) Z Z Z Z κ |∇v|2dx ≤ A∇v∇v = A∇F · ∇v − Luwdx. Ω Ω Ω Combined with Poincare’s´ inequality this estimate yields in a straightforward man- ner   kvkH1(Ω) ≤ C kFkH1(Ω) + kLukL2(Ω) , (4.79) for some constant C = C(κ,Ω). On the other hand, we get by using one more time (4.78), whereu ˜ = v + F,

1 L(u − u˜) = 0 in Ω and u − u˜ ∈ H0 (Ω) which leads immediately tou ˜ = u. Therefore in light of kFkH1(Ω) = kukH1/2(Γ ) and (4.79) inequality (4.77) follows. ut

Theorem 4.10. Let Γ0 be an open subset of Γ and 0 < α ≤ 1. There exist C > 0, 1,α c > 0 and β > 0, only depending on Ω, κ, κ, α and Γ0, so that, for any u ∈ C (Ω) satisfying Lu ∈ L2(Ω), we have

β c/ε   Ckuk 1 ≤ ε kuk 1,α +e kuk 2 + k∇uk 2 n + kLuk 2 (4.80) H (Ω) C (Ω) L (Γ0) L (Γ0,R ) L (Ω) 176 4 Classical inequalities, Cauchy problems and unique continuation for every 0 < ε < 1.

Proof. Let u ∈ C1,α (Ω) satisfying Lu ∈ L2(Ω). From Lemma 4.6, we have

CkukH1(Ω) ≤ kLukL2(Ω) + kukH1/2(Γ ).

In this proof, C > 0 and c > 0 are generic constants only depending on Ω, κ, κ, α and Γ0. By Proposition 4.4, and noting that W 1,∞(Γ ) is continuously embedded in 1/2 H (Γ ), we find β > 0 and ω b Ω so that

β c/ε   CkukH1(Ω) ≤ ε kukC1,α (Ω) + e kukH1(ω) + kLukL2(Ω) , 0 < ε < 1. (4.81)

On the other hand by Proposition 4.6 there exist ω0 b Ω and γ > 0 so that, for any ε1 > 0, we have

γ Ckuk 1 ≤ ε kuk 1,α (4.82) H (ω0) 1 C (Ω) −1   + ε kuk 2 + k∇uk 2 n + kLuk 2 . 1 L (Γ0) L (Γ0,R ) L (Ω)

But by Proposition 4.5 there is δ > 0 such that

δ −1   Ckuk 1 ≤ ε kuk 1,α + ε kuk 1 + kLuk 2 , ε > 0. (4.83) H (ω) 2 C (Ω) 2 H (ω0) L (Ω) 2

Estimate (4.82) in (4.83) gives

δ −1 γ CkukH1(ω) ≤ (ε2 + ε2 ε1 )kukC1,α (Ω) −1 −1   −1 + ε ε kuk 2 + k∇uk 2 n + kLuk 2 + ε kLuk 2 . 1 2 L (Γ0) L (Γ0,R ) L (Ω) 2 L (Ω)

(γ+1)/δ ε1 = ε2 in this estimate yields, where ρ = (γ + δ + 1)/δ,

δ CkukH1(ω) ≤ ε2 kukC1,α (Ω) −ρ   −1 + ε kuk 2 + k∇uk 2 n + kLuk 2 + ε kLuk 2 , 2 L (Γ0) L (Γ0,R ) L (D) 2 L (Ω) which in combination with (4.81) entails

β δ c/ε CkukH1(Ω) ≤ (ε + ε2 e )kukC1,α (Ω) −ρ c/ε   −1 c/ε + ε e kuk 2 + k∇uk 2 n + kLuk 2 + (ε + 1)e kLuk 2 . 2 L (Γ0) L (Γ0,R ) L (Ω) 2 L (Ω)

Therefore 4.5 Stability of the Cauchy problem 177

β δ c/ε CkukH1(Ω) ≤ (ε + ε2 e )kukC1,α (Ω)  −ρ −1  c/ε   + ε + ε + 1 e kuk 2 + k∇uk 2 n + kLuk 2 . 2 2 L (Γ0) L (Γ0,R ) L (Ω)

−2c/(εδ) We end up getting the expected inequality by taking ε2 = e . ut Theorem 4.10 can be used to get logarithmic stability estimate for the Cauchy problem.

Corollary 4.6. Let Γ0 be an open subset of Γ and 0 < α ≤ 1. There exist C > 0, 1,α β > 0, only depending on Ω, κ, κ, α and Γ0, so that, for any u ∈ C (Ω) satisfying u 6= 0 and Lu ∈ L2(Ω), we have ! kuk 1 kuk 2 + k∇uk 2 n + kLuk 2 H (Ω) L (Γ0) L (Γ0,R ) L (Ω) ≤ CΦβ . kukC1,α (Ω) kukC1,α (Ω)

−β Here Φβ (s) = |lns| + s, s > 0, and Φβ (0) = 0. This corollary is a direct consequence of Theorem 4.10 and the following lemma.

Lemma 4.7. Let α > 0, β > 0, c > 0, a > 0 and s > 0 be given constants. There exists a constant C > 0, only depending on α, β, c > 0, a and s, so that, for any a ∈ (0,a] and b > 0, the relation

a ≤ s−α + ecsbβ , s ≥ s, (4.84)

implies a ≤ C |lnb|−α + b. (4.85)

Proof. Let b = s−α e−cs. Assume that b ≤ b. Then, since the mapping s → s−α e−cs −α −cs0 is decreasing, there exists s0 ≥ s so that s0 e = b. In particular,

−1 α cs0 Cs0 b = s0 e ≤ e ,

or equivalently −1 −1 s0 ≤ C|lnb| . (4.86) On the other hand, if b ≥ b then a a ≤ b (4.87) b

Whence, (4.84) with s = s0, (4.86) and (4.87) yield (4.85). We can prove similarly to Corollary 4.6 the following consequence of Theorem 4.10.

Corollary 4.7. Let Γ0 be an open subset of Γ , 0 < α ≤ 1 and fix M > 0. There exist C > 0, β > 0, only depending on Ω, κ, κ, α and Γ0, C depending also on M, so 1,α 2 that, for any u ∈ C (Ω) satisfying kukC1,α (Ω) ≤ M and Lu ∈ L (Ω), we have 178 4 Classical inequalities, Cauchy problems and unique continuation   kuk 1 ≤ CΦ kuk 2 + k∇uk 2 n + kLuk 2 . H (Ω) β L (Γ0) L (Γ0,R ) L (Ω)

Here Φβ is as in Corollary 4.6.

4.6 Exercises and problems

4.1. Let u ∈ H (B(0,3)) satisfying u(0) 6= 0. (a) Let v = u2. Check that ∆v ≥ 0 and deduce from it that there exists a constant C > 0, only depending on n, so that

kvkL∞(B(0,r)) ≤ CkvkL2(B(0,2r)), 0 < r ≤ 1.

(b) Let 0 < ε < 1. (i) Show that there exists Cu,ε , depending on u and ε, so that

kukL∞(B(0,r)) ≤ Cu,ε kukL2(B(0,2r)), ε ≤ r ≤ 1.

(ii) Deduce then that there exists C˜u,ε , depending on u and ε, so that ˜ kukL∞(B(0,r)) ≤ Cu,ε kukL2(B(0,r)), ε ≤ r ≤ 1.

(c) Prove that there exists a constant Cu > 0, depending on u, so that

kukL∞(B(0,r)) ≤ CukukL2(B(0,r)), 0 < r ≤ 1. Hint: use the second mean-value identity.

n ∞ 4.2. Let Ω, D1 and D2 be three bounded domains of R of class C with Di b Ω, i = 1,2. Denote the boundary of Ω by Γ . Let ϕ ∈ C∞(Γ ) non identically equal 2 ∞ ∞ to zero. For i = 1, 2 let ui ∈ H (Ω) ∩C (Di) ∩C (Ω \ Di) be the solution of the boundary value problem  −∆u + χDi u = 0 in Ω, u = ϕ in Γ ,

where χDi is the characteristic function of Di, i = 1,2. We make the following assumptions: (A) Ω0 = Ω\D1 ∪ D2, D0 = D1 ∩ D2 are connected and S = ∂Ω0 ∩ ∂D0 has nonempty interior. (B) There exists γ a non empty open subset of Γ so that

∂ν u1 = ∂ν u2 on γ.

(a) Show that u1 = u2 in Ω0 and then u1 = u2 in D0. 2 (b) (i) Assume that ω = D2\D1 6= /0.Check that u = u2 − u1 ∈ H0 (ω) and ∆u = u2 4.6 Exercises and problems 179 in ω. Then show that u satisfies

−∆ 2u + ∆u = 0 in ω.

(ii) Deduce that u = 0 in ω and conclude that ω = /0. (c) Prove that D1 = D2. n 4.3. Let B denotes the unit ball of R . In this exercise, σ ∈ C2(B) and β ∈ C(B) satisfy σ0 ≤ σ ≤ σ1, |β| ≤ β0, where 0 < σ0 ≤ σ1 and β0 > 0 are fixed constants. Let u ∈ C2(B) so that

−div(σ∇u) + βu = 0 in B.

Define for 0 < r < 1 Z H(r) = σ(x)u2(x)dS(x), S(r) Z D(r) = σ(x)|∇u(x)|2 + β(x)u2(x) dx. B(r)

Here B(r) (resp. S(r)) is the ball (resp. sphere) of center 0 and radius r. (a) Prove that n − 1 H0(r) = H(r) + H˜ (r) + 2D(r), (4.88) r n − 2 D0(r) = D(r) + D˜ (r) + 2H(r) + Dˆ (r) + Hˆ (r), (4.89) r where Z H˜ (r) = u2(x)∇σ(x) · ν(x)dS(x), S(r) Z 2 H(r) = σ(x)(∂ν u(x)) dS(x), S(r) Z Hˆ (r) = β(x)u2(x)dx, S(r) Z D˜ (r) = |∇u(x)|2∇σ(x) · xdx, B(r) Z n − 2 Z Dˆ (r) = −2 β(x)u(x)x · ∇u(x)dx − β(x)u2(x)dx. B(r) r B(r)

(b) Let, for 0 < r < 1, Z K(r) = σ(x)u2(x)dx. B(r) (i) Show that if β ≥ 0 then 180 4 Classical inequalities, Cauchy problems and unique continuation

eσ1/σ0 K(r) ≤ H(r). n

(ii) Assume that σ = 1 (in that case we can take σ0 = σ1 = 1). Demonstrate then that   −11/2 K(r) ≤ rH(r), 0 < r < r0 = min 1, (n − 1)β0 . Recall that the frequency function N is defined by

rD(r) N(r) = H(r)

and the following identity holds

N0(r) 1 D0(r) H0(r) = + − . (4.90) N(r) r D(r) H(r)

(c) (i) Assume that β = 0. Prove that, for 0 < r < 1 and 0 < r ≤ r,

N(r) ≤ e2σ1/σ0 N(r).

ii) Show that, under the assumption that β ≥ 0 or σ = 1, we have

N(r) ≤ C max(N(r0),1),

where the constant C > 0 only depends on Ω, σ1/σ0 and β0. Hint: we can establish a preliminary result. Set

I = {r ∈ (0,δ); N(r) > max(N(r0),1)}

and observe that I is a countable union of open intervals:

∞ [ I = (ri,si). i=1 Show then that

Hˆ (r) Dˆ (r) and D(r) D(r)

are bounded on each (ri,si) by a constant independent on i.

, n 4.4. Let Ω be a C1 1 bounded domain of R with boundary Γ . We admit the follow- ing theorem which is contained in [5, Theorem 9.15 and Lemma 9.17]. Theorem 4.11. Let 1 < p < ∞. p 2,p 1,p (1) For any f ∈ L (Ω), there exists a unique u ∈ W (Ω) ∩W0 (Ω) satisfying −∆u = f in Ω. (2) There exists C > 0, depending on Ω and p, so that, for any u ∈ W 2,p(Ω) ∩ 1,p W0 (Ω), we have 4.6 Exercises and problems 181

kukW 2,p(Ω) ≤ Ck∆ukLp(Ω). (4.91) (a) Let A be the unbounded operator defined on L2(Ω) by Au = −∆u, u ∈ D(A) = 1 2 H0 (Ω) ∩ H (Ω). Fix 0 < α < 1, λ ∈ σ(A) and φ ∈ D(A) an eigenfunction associ- ated to λ. (i) Assume that n < 4. Prove that φ ∈ W 2,p(Ω), for any 1 < p < ∞, and

2 kφkW 2,p(Ω) ≤ Cλ kφkL2(Ω), (4.92) where the constant C only depends on Ω and n. Show that φ ∈ C1,α (Ω) and deduce from (4.92) that

2 kφkC1,α (Ω) ≤ Cλ kφkL2(Ω), where the constant C depends only on Ω, n and α. 2,q0 2n (ii) Consider the case 4 ≤ n < 8. Prove that φ ∈W (Ω), with q0 = n−4 if 4 < n < 8 and q0 = 2p/(2 − p) for an arbitrary fixed 1 < p < 2 if n = 4, and

2 kφkW 2,q0 (Ω) ≤ Cλ kφkL2(Ω), where the constant C only depends on Ω and n. Proceed then as in (i) in order to obtain 3 kφkC1,α (Ω) ≤ Cλ kφkL2(Ω), the constant C depends only on Ω, n and α. (iii) Prove that, for any n ≥ 1, we have φ ∈ C1,α (Ω) and there exists an non negative integer m = m(n) so that

m kφkC1,α (Ω) ≤ Cλ kφkL2(Ω), the constant C only depends on Ω, n and α. Hint: write n = 4k + `, k ∈ N0, ` ∈ {0,1,2,3}. m (b) (i) Let D be a Lipschitz bounded domain of R , m ≥ 1, E b D and 0 < α ≤ 1. Demonstrate that there exist three constants β > 0, c > 0 and C > 0, only depending on D, E and α, so that for any v ∈ C1,α (D) ∩ H2(D) and 0 < ε < 1 we have

β c/ε   CkvkH1(D) ≤ ε kukC1,α (D) + e kvkL2(E) + k∆vkL2(D) . (4.93)

(ii) Let ω b Ω and 0 < α ≤ 1. Prove that there exist β > 0, c > 0 and C > 0, only depending on Ω, ω and α, so that, for any λ > 0, u ∈ C1,α (Ω) ∩ H2(Ω) and 0 < ε < 1, we have that √ −2 λ β c/ε   Ce kukL2(Ω) ≤ ε kukC1,α (Ω) + e kukL2(ω) + k(∆ + λ)ukL2(Ω) . (4.94)

Hint: apply (i) to v(x,t) = u(x)eλt , (x,t) ∈ D = Ω × (0,1). c) (i) We use the same notations as in (a). If m is the integer in (a) (iii) then show that 182 4 Classical inequalities, Cauchy problems and unique continuation √ c −2 λ m β ε Ce kφkL2(Ω) ≤ λ ε kφkL2(Ω) + e kφkL2(ω),0 < ε < 1. (4.95)

The constants C, c and β are the same as in (ii). (ii) Conclude that there exists a constant κ > 0, only depending on n, Ω and ω, so that √ κ λ kφk 2 e−e ≤ L (ω) . kφkL2(Ω)

, n 4.5. Let Ω be a C1 1 bounded domain of R with boundary Γ . We recall the follow- ing simple version of an interpolation inequality due to G. Lebeau and L. Robbiano [6]

Theorem 4.12. Let ω b Ω and D = Ω ×(0,1). There exist two constants C > 0 and 2 1 β ∈ (0,1), only depending on n, Ω and ω so that, for v = v(x,t) ∈ H (D) ∩ H0 (D), we have that

1−β  β kvk 1 ≤ Ckvk k vk 2 + k v( ,·)k 2 . H (Ω×(1/4,3/4)) H1(D) ∆ L (D) ∂t 0 L (ω) (4.96)

Henceforth, ω b Ω, C is a generic constant only depending on n, Ω and ω, and β is as in Theorem 4.12. 1 a) Let u ∈ H0 (Ω) and λ > 0. Prove that

√ 3 λ 1−β  β kuk 1 ≤ Ce 4 kuk k( + )uk 2 + kuk 2 . H (Ω) H1(Ω) ∆ λ L (Ω) L (ω) √ Hint: apply Theorem 4.12 to v(x,t) = u(x)e λt , (x,t) ∈ D = Ω × (0,1). b) Let A be the unbounded operator, defined on L2(Ω), by Au = −∆u, u ∈ D(A) = 1 2 H0 (Ω) ∩ H (Ω). Fix λ ∈ σ(A) and φ ∈ D(A) an eigenfunction associated to λ. Show that √ kφk 2 e−κ λ ≤ L (ω) , kukL2(Ω) with κ = 11/(4β) − 2.

n 4.6. Let Ω is a bounded domain of R . a) (i) In this exercise we use the same assumptions and notations as in Section 4.4. Prove the following variant of Theorem 4.7.

Theorem 4.13. Let 0 < k < ` < m and Λ > 0. There exist C > 0 and 0 < γ < 1, that can depend only on Ω, Λ, k, `, m, κ and κ, so that, for any v ∈ H1(Ω) satisfying Lv ∈ L2(Ω) in Ω together with

(Lv)2 ≤ Λ v2 + |∇v|2 in Ω, y ∈ Ω and 0 < r < dist(y,Γ )/m, we have

γ 1−γ Ckvk 2 ≤ kvk kvk . L (B(y,`r)) L2(B(y,kr)) L2(B(y,mr)) 4.6 Exercises and problems 183

(ii) Establish the following result

Proposition 4.7. Let ω b Ω and ω˜ ⊂ Ω be non empty. There exist C > 0 and β > 0, that can depend on ω and ω˜ , so that, for any u ∈ H2(Ω) satisfying

(Lu)2 ≤ Λ u2 + |∇u|2 in Ω, we have

β −1 CkukL2(ω˜ ) ≤ ε kukL2(Ω) + ε kukL2(ω), β −1 Ck uk 2 n ≤ k uk 2 n + k uk 2 n ∇ L (ω˜ ,R ) ε ∇ L (Ω,R ) ε ∇ L (ω,R ) for any ε > 0. b) (Calderon’s´ theorem) Let g : R → R be continuous and satisfies |g(s)| ≤ c|s|, s ∈ R, for some constant c > 0. Let u ∈ H2(Ω) satisfying Lu = g(u) and there exist a nonempty ω b Ω so that u = 0 in ω. Prove that u is identically equal to zero. 184 4 Classical inequalities, Cauchy problems and unique continuation References

1. Brummelhuis, R.:Three-spheres theorem for second order elliptic equations. J. Anal. Math. 65 (1995), 179-206. 2. Choulli, M.: Applications of elliptic Carleman inequalities to Cauchy and inverse problems. Springer, Berlin, 2016. 3. Garofalo, N., Lin, F.-H. : Monotonicity properties of variational integrals, Ap weights and unique continuation. Indiana Univ. Math. J. 35 (2) (1986), 245-268. 4. Garofalo, N., Lin, F.-H. : Unique continuation for elliptic operators: a geometric-variational approach. Commun. Pure Appl. Math. 40 (3) (1997), 347-366. 5. Gilbarg D. and Trudinger N. S. : Elliptic partial differential equations of second order, 2nd ed. Springer, Berlin, 1983. 6. Lebeau, G., Robbiano, L. : Controleˆ exact de l’equation´ de la chaleur. Commun. Partial Dif- ferent. Equat. 20 (1995) (1-2), 335-356. Solutions of Exercises and Problems

Exercices and Problems of Chapter 1

1.1 If u ∈ Lq(Ω), we get by applying Holder’s¨ inequality

Z Z p/q Z 1−p/q Z p/q |u|pdx ≤ |u|p(q/p)dx dx = |u|qdx |Ω|1−p/q. Ω Ω Ω Ω

Hence u ∈ Lp(Ω) and 1/p−1/q kukp ≤ |Ω| kukq. 1.2 We proceed by induction in k. The case k = 2 corresponds to classical Holder’s¨ inequality. Assume then that the inequality holds for some integer k ≥ 2 and let 1 ≤ p1,... ,1 ≤ pk, 1 ≤ pk+1 so that 1 1 1 + ... + = 1. p1 pk pk+1 The assumption that the inequality holds for k entails

Z k+1 k−1 ∏ |u j|dx ≤ ∏ ku jkp j kukuk+1kq, (5.1) Ω j=1 j=1 where q is defined by 1 1 1 = + . q pk pk+1

On the other hand, since pk+1/q is the conjugate exponent of pk/q, the classical Holder’s¨ inequality yields

ku u kq = k|u u |qk ≤ k|u |qk k|u |qk . k k+1 q k k+1 1 k pk/q k+1 pk+1/q That is

185 186 Solutions of Exercises and Problems

kukuk+1kq ≤ kukkpk kuk+1kpk+1 . This in (5.1) give the expected inequality. 1.3 We get from Holder’s¨ inequality

q q(1−λ) qλ kukq = k|u| |u| k1 Z q(1−λ)/p Z q/r ≤ |u|q(1−λ)[p/q(1−λ)]dx |u|qλ(r/qλ)dx , Ω Ω where we used that r/(qλ) is the conjugate exponent of p/[q(1 − λ)]. Whence u ∈ Lq(Ω) and

1−λ λ kukq ≤ kukp kukr .

1.4 For each integer j ≥ 1, set

n Kj = {x ∈ ω; dist(x,R \ ω) ≥ 1/ j and |x| ≤ j}.

We have ω = ∪ jKj, and as Kj is compact, we can covert it by finite number of

ωi, say Kj ⊂ ∪i∈Ij ωi, with Ij ⊂ I finite. Hence J = ∪ jIj is countable and we have ω = ∪i∈Jωi. We know that f = 0 in ωi \ Ai with Ai of zero measure for each i. Therefore, f = 0 in ω \ (∪i∈JAi). That is f = 0 a.e. in ω because ∪i∈JAi is of zero measure. 1.5 (a) The case p = ∞ is obvious. Next, we consider the case p = 1. Set

h(x,y) = f (x − y)g(y).

Then we have Z Z n |h(x,y)|dx = |g(y)| | f (x − y)|dx = k f k1|g(y)| < ∞ a.e. y ∈ R Rn Rn and Z Z dy |h(x,y)|dx = k f k1kgk1 < ∞. Rn Rn n n We conclude by applying Tonelli’s theorem that h ∈ L1(R × R ). Then, in light of Fubini’s theorem, we obtain Z n |h(x,y)|dy < ∞ a.e. x ∈ R Rn and Z Z dx |h(x,y)|dy ≤ k f k1kgk1 < ∞. Rn Rn This completes the proof for the case p = 1. We proceed now to the proof of the case 1 < p < ∞. From the case p = 1, y 7→ p n n 1/p p n | f (x−y)||g(y)| is integrable in R , a.e. x ∈ R . That is | f (x−y)| |g(y)| ∈ Ly (R ) Solutions of Exercises and Problems 187

n /p0 p0 n a.e. x ∈ R . Since | f (x − y)|1 ∈ L (R ), we deduce by using Holder’s¨ inequality that

1/p 1/p0 1 n n | f (x − y)||g(y)| = | f (x − y)| |g(y)|| f (x − y)| ∈ Ly(R ) a.e. x ∈ R and

Z Z 1/p p 1/p0 1 n n | f (x−y)||g(y)|dy ≤ | f (x − y)||g(y)| dy k f k1 ∈ Ly(R ) a.e. x ∈ R . Rn Rn In other words,

p p p/p0 n |( f ∗ g)(x)| ≤ (| f | ∗ |g| )(x)k f k1 a.e. x ∈ R .

p n The result for the case p = 1 enables us to deduce that f ∗ g ∈ L (R ) and

p p p/p0 k f ∗ gkp ≤ k f k1kgkpk f k1 . Thus k f ∗ gkp ≤ k f k1kgkp. n (b) Fix x ∈ R so that y 7→ f (x − y)g(y) is integrable. Then we have Z Z ( f ∗ g)(x) = f (x − y)g(y)dy = f (x − y)g(y)dy. Rn (x−supp( f ))∩supp(g) If x 6∈ supp( f ) + supp(g) then (x − supp( f )) ∩ supp(g) = /0and ( f ∗ g)(x) = 0. Whence n ( f ∗ g)(x) = 0 a.e. in R \ (supp( f ) + supp(g)). In particular, n ( f ∗ g)(x) = 0 a.e. in R \ supp( f ) + supp(g) and hence supp( f ∗ g) ⊂ supp( f ) + supp(g). 1.6 First, note that f (x) = xα belongs to L2(]0,1[) if and only if α > −1/2. Ac- cording to the definition of weak derivatives, g = f 0 means that

Z 1 Z 1 g(x)ϕ(x)dx = − xα ϕ0(x) for any ϕ ∈ D(]0,1[). 0 0

Let ϕ ∈ D(]0,1[) with supp(ϕ) ⊂]a,b[⊂]0,1[ where 0 < a < b < 1. Then

Z 1 Z b Z b Z 1 xα ϕ0(x) = xα ϕ0(x)dx = − αxα−1ϕ(x)dx = αxα−1ϕ(x)dx. 0 a a 0

Whence g(x) = αxα−1 and then g ∈ L2(]0,1[) if and only if α > 1/2.

1.7 (a) Introduce the notations Ii = { j; 1 ≤ j ≤ k, j 6= i and Ω j ∩ Ωi 6= /0} and, for j j j j ∈ Ii, set Γi = Ω j ∩Ωi. We denote by νi the unit normal vector field on Γi directed 188 Solutions of Exercises and Problems

1 from Ωi to Ω j. Let f ∈ Cpie(Ω,(Ωi)1≤i≤k) and ϕ ∈ D(Ω). Green’s formula on each Ω gives, with f = f | , i i Ωi

Z k Z f ∂`ϕdx = ∑ fi∂`ϕdx Ω i=1 Ωi k Z k Z = − ∑ ∂` fiϕdx + ∑ fiϕν`dσ i=1 Ωi i=1 Γi k Z k Z = − f dx + f ( j) d . ∑ ∂` iϕ ∑ ∑ j iϕ νi ` σ Ω Γ i=1 i i=1 j∈Ii i

If J = {(i, j); 1 ≤ i, j ≤ k, i 6= j and j ∈ Ii}, we deduce

Z k Z Z f dx = − f dx + ( f ( j) + f ( i) ) d . ∂`ϕ ∑ ∂` iϕ ∑ j i νi ` j ν j ` ϕ σ Ω i=1 Ωi (i, j)∈J Γi

j i But νi = −ν j. Hence

Z k Z Z f dx = − f dx + ( f − f ) ( j) d . (5.2) ∂`ϕ ∑ ∂` iϕ ∑ j i j ϕ νi ` σ Ω i=1 Ωi (i, j)∈J Γi

If in addition f ∈ C Ω then

Z k Z f ∂`ϕdx = − ∑ ∂` fiϕdx. Ω i=1 Ωi

2 Hence g` defined by g`|Ωi = ∂` fi in Ωi, 1 ≤ i ≤ k, is in L (Ω) because ∂` fi ∈  C Ωi and we have in the weak sense ∂` f = g`, 1 ≤ ` ≤ k. 1 (b) Let f ∈ Cpie(Ω,(Ωi)1≤i≤k). Assume that there exists, for fixed 1 ≤ ` ≤ k, g` ∈ 2 L (Ω) so that ∂` f = g` in the weak sense, i.e.

Z Z k Z f ∂`ϕdx = − g`ϕdx = − ∑ g`ϕdx for any ϕ ∈ D(Ω). Ω Ω i=1 Ωi This and (5.2) entail

k Z Z (g − f ) dx + ( f − f ) ( j) d = 0. (5.3) ∑ ` ∂` i ϕ ∑ j i j ϕ νi ` σ i=1 Ωi (i, j)∈J Γi

We deduce, by choosing ϕ to be the extension by 0 of a function in D(Ωi), Z (g` − ∂` fi)ϕdx = 0 for any ϕ ∈ D(Ωi). Ωi Solutions of Exercises and Problems 189

Thus, g`|Ωi = ∂` fi by the cancellation theorem. We come back to (5.3) to conclude that Z ( f − f ) ( j) d = 0 for any ∈ ( ) and 1 ≤ ` ≤ n. (5.4) ∑ j i j ϕ νi ` σ ϕ D Ω (i, j)∈J Γi

This implies that fi = f j for any (i, j) ∈ J. Otherwise, we would find (i, j) de J, j 1 ≤ ` ≤ n and Br ⊂ Ω a ball centred at a point in Γi so that Br does not intersect ` j j any other Γk and ( fi − f j)(νi )` does not vanish and is of constant sign in Br ∩Γi . If we choose ϕ ∈ D(Ω) satisfying 0 ≤ ϕ ≤ 1, ϕ = 1 in Br/2 and suppϕ ⊂ Br in (5.4) we obtain Z ( f − f )(ν j) dσ = 0. j i j i ` Γi ∩Br/2 This yields the expected contradiction. 1.8 (a) For α > 0, we have

2 α α ln|x| u(x) = |ln|x|| = . 2 Therefore x ∇u(x) = −α |ln|x||α−1 , x 6= 0 |x|2 and hence 2 Z Z α|ln|x||α−1  |∇u|2dx = dx. B1/2 B1/2 |x| We obtain by passing to polar coordinates

Z Z 1/2 (lnr)2(α−1) Z +∞ |∇u|2dx = 2πα2 dr = 2πα2 s2(α−1)ds, B1/2 0 r ln2 where we make the change of variable s = −lnr in order to get the last integral. We 1 conclude that u ∈ H (B1/2) whenever the last integral is convergent. This is the case if 2(α − 1) < −1 or equivalently α < 1/2. Finally, if α > 0 then u is unbounded. (b) Let 0 < β < (n − 2)/2 and u(x) = |x|−β . Note that u is unbounded because β > 0. On the other hand, we have

∇u(x) = −β|x|−(β+2)x, x 6= 0.

n− n If S 1 denotes the unit sphere of R , we obtain by passing to spherical coordinates Z Z Z 1 2 2 −2(β+1) 2 n−1 n−1−2(β+1) |∇u| dx = β |x| dx = β |S | r dr. B1 B1 0

1 Thus u ∈ H (B1) if n − 2β − 3 > −1 or equivalently β < (n − 2)/2. 190 Solutions of Exercises and Problems

1.9 Let 1 ≤ i, j ≤ n. α α−2 (a) We have ∂i(|x| ) = α|x| xi for any x 6= 0 as a consequence of the formula ∂i|x| = xi/|x|. We obtain by passing to spherical coordinates

Z Z Z 1 |x|α pdx = dσ(ω) rn−1+pα dr B Sn−1 0 and Z Z Z 1 α p p n−1+p(α−1) |∂i(|x| )| dx = |ωi| dσ(ω) r dr. B Sn−1 0 Therefore |x|α ∈ W 1,p(B) if and only if n + p(α − 1) > 0. We have similarly

Z Z Z +∞ |x|α pdx = dσ(ω) rn−1+pα dr Rn\B Sn−1 1 and Z Z Z +∞ α p p n−1+p(α−1) |∂i(|x| )| dx = |ωi| dσ(ω) r dr. Rn\B Sn−1 1 ,p n We deduce that |x|α ∈ W 1 (R \ B) if and only if n + pα < 0. (b) We have   x δi j xix j ∂ i = − . j |x| |x| |x|3 Whence Z p Z Z 1 xi p n−1 dx = |ωi| dσ(ω) r dr ≤ |B| B |x| Sn−1 0 and Z   p Z Z 1 xi p n−1−p ∂ j dx = |δi j − ωiω j| dσ(ω) r dr. B |x| Sn−1 0 ,p n In consequence, x/|x| ∈ W 1 (B,R ) if and only if p < n. 1.10 (a) We assume, without loss of generality, that x ≤ y. Then we have

Z y 2 u(x)2 + u(y)2 − 2u(x)u(y) = (u(x) − u(y))2 = u0(t)dt x and by Cauchy-Schwatz’s inequality

Z y 2 Z y Z b u0(t)dt ≤ (y − x) u0(t)2dt ≤ (b − a) u0(t)2dt. x x a

Hence Z b u(x)2 + u(y)2 − 2u(x)u(y) ≤ (b − a) u0(t)2dt. a (b) We obtain by integrating with respect to x Solutions of Exercises and Problems 191

Z b Z b Z b u(x)2dx + (b − a)u(y)2 − 2u(y) u(x)dx ≤ (b − a)2 u0(x)2dx. a a a We then integrate with respect to y to get

Z b Z b 2 Z b 2(b − a) u(x)2dx − 2 u(x)dx ≤ (b − a)3 u0(x)2dx. a a a

Hence the result follows. 0 p− 1.11 (a) Clearly, G is of C1 and G (t) = p|t| 1 for any t ∈ R. Whence, w is of class C1 and w0 = p|v|p−1v0. As v has a compact support and G(0) = 0, we obtain that w has also a compact support. Assume that 1 < p. Then Z |w(x)| ≤ p|v|p−1|v0|dt. R Then Holder’s¨ inequality yields

0 Z 1/p Z 1/p p p 0 p p/p0 0 |v(x)| ≤ p |v| |v | ≤ pkvkp kv kp R R and hence 1/p 1/p0 0 1/p |v(x)| ≤ p kvkp kv kp . Since p ∈ [0,+∞[→ p1/p attains its maximum at p = e, the last inequality gives

1/e 1/p0 0 1/p |v(x)| ≤ e kvkp kv kp .

We get by applyingYoung’s inequality1

 0  1/e kvkp kv kp 1/e |v(x)| ≤ e + ≤ e kvk 1,p . p0 p W (R)

The expected inequality is obvious when p = 1. In fact we have simply in that case 0 |v(x)| ≤ kv k1. 1/e 1,p 1 1,p (b) Set c = e . Let u ∈ W (R) and (uk) ∈ Cc (R) converging to u in W (R). By (a) we have, for every k and `,

ku − u k ∞ ≤ cku u k 1,p . ` k L (R) ` k W (R)

1 Young’s inequality. Let a and b be two positive reals numbers, p > 1 and p0 the conjugate exponent of p. Then 0 ap bp ab ≤ + . p p0 192 Solutions of Exercises and Problems

∞ ∞ Whence (uk) is a Cauchy sequence in L (R). Therefore it converges to u in L (R). The expected result is obtained by passing to the limit, as k → +∞, in kukkL∞(R) ≤ cku k 1,p k W (R). 1.12 As in the proof of Lemma 1.9, we have

Z +∞ 0 q 0 q−1 0 |u(x ,0)| ≤ q |u(x ,xn)| |∂nu(x ,xn)|dxn. 0 We obtain then by applying Fubini’s theorem Z Z 0 q 0 q−1 |u(x ,0)| dx ≤ q |u(x)| |∂nu(x)|dx. Rn−1 Rn We end up getting by using Holder’s¨ inequality

0 Z Z 1/p 0 q 0 (q−1)p0 |u(x ,0)| dx ≤ qk∂nukp |u(x)| dx . Rn−1 Rn The result then follows by noting that a simple computation yields (q − 1)p0 = p∗. 1.13 We have

Z Z 1 Z xβ Z 1 |v|2dxdy = dx x2α dy = x2α+β dx. Ω 0 0 0

2 Whence v ∈ L (Ω) if and only if 2α + β > −1. Observing that ∂yv = 0 and ∂xv = αxα−1, we prove similarly that v ∈ H1(Ω) if and only if 2(α − 1) + β > −1 or equivalently 2α + β > 1. On the other hand,

Z Z 1 |v|2dσ = 1 + x2α (1 + β 2x2β−1)1/2dx. ∂Ω 0

Since β > 2, (1+β 2x2β−1)1/2 is bounded in [0,1]. We then deduce that v ∈ L2(∂Ω) if and only if 2α > −1. Now, as β > 2, there exists α so that 1 − β < 2α < −1. In that case, v ∈ H1(Ω) and v 6∈ L2(∂Ω). This result shows that for the domain Ω the trace operator

2 C(Ω) → L (∂Ω) : u → u|∂Ω does not admit a bounded extension to H1(Ω). 1.14 (a) Let, for ε > 0,

h 2 α/2 i 2 α/2−1 2 2 α/2 ∂i (|x| + ε) xi = α(|x| + ε) xi + (|x| + ε) .

Hence h i div (|x|2 + ε)α/2x = α(|x|2 + ε)α/2−1|x|2 + n(|x|2 + ε)α/2. Solutions of Exercises and Problems 193

1 n It is not hard to check that, in Lloc(R ),

(|x|2 + ε)α/2 → |x|α and div[(|x|2 + ε)α/2x] → (α + n)|x|α .

By the closing lemma, we have div(|x|α x) = (α + n)|x|α in the weak sense. For n u ∈ D(R ), we get from the divergence theorem Z div[|u|p|x|α x]dx = 0. Rn

Thus Z Z (α + n) |u|p|x|α dx = −p x · ∇u|u|p−1|x|α dx. Rn Rn This and Holder’s¨ inequality entail Z p Z |u|p|x|α dx ≤ |x · Du||u|p−1|x|α dx Rn α + n Rn 1−1/p 1/p p Z 0  Z  ≤ |u|(p−1)p |x|α dx |x · ∇u|p|x|α dx α + n Rn Rn p Z 1−1/p Z 1/p ≤ |u|p|x|α dx |x · ∇u|p|x|α dx , α + n Rn Rn and hence the result follows. ,p n n ,p n (b) Let u ∈ W 1 (R ). In light of the fact that D(R ) is dense in W 1 (R ), there n 1,p n exists a sequence (um) in D(R ) converging to u in W (R ). We find, by applying (a) with α = −p,

um1 um2 p − ≤ k∇um1 − ∇um2 kp. |x| |x| p n − p

p n p n (um/|x|) is then a Cauchy sequence in L (R ). As um → u in L (R ), um/|x| → u/|x| p n in L (R ). We have also

um p ≤ k∇umkp. |x| p n − p

Passing to the limit, as m goes to ∞, we end up getting

u p ≤ k∇ukp. |x| p n − p

n ,p n 1.15 (a) As in the preceding proof, using the density of D(R ) in W 1 (R ), we n 1,p n obtain that there exists a sequence (vm) in D(R ) converging in W (R ) to v.A slight modification of the proof of Proposition 1.6 enables us to get

∂i(u ∗ vm) = u ∗ ∂ivm, 1 ≤ i ≤ n. 194 Solutions of Exercises and Problems

On the other hand, ku∗vm −u∗vkp ≤ kuk1kvm −vkp and ku∗∂ivm −u∗∂ivkp ≤ kuk1k∂ivm −∂ivkp.

p n Whence, u ∗ vm converges to u ∗ v in L (R ) and ∂i(u ∗ vm) converges to u ∗ ∂iv in p n 1,p n L (R ). The closing lemma then yields u ∗ v ∈ W (R ) and ∂i(u ∗ v) = u ∗ ∂iv, 1 ≤ i ≤ n. (b) (i) We have

supp(ρm ∗ ϕu − ρm ∗ u) = supp(ρm ∗ (1 − ϕ)u).

But  1  supp(ρ ∗ (1 − ϕ)u) ⊂ B 0, + supp((1 − ϕ)u) m m supp((1 − ϕ)u) ⊂ supp(1 − ϕ).

In consequence,

 1  supp(ρ ∗ ϕu − ρ ∗ u) ⊂ B 0, + supp(1 − ϕ) ⊂ n \ ω m m m R provided that m is sufficiently large. We obtain from (a)

p n ∂i(ρm ∗ ϕu) → ∂i(ϕu), in L (R ).

As ϕ has compact support in Ω, we can check that

∂iϕu = ∂i(ϕu) = ∂iϕu + ϕ∂iu and then p n ∂i(ρm ∗ ϕu) → Diϕu + ϕDiu in L (R ). In particular, p ∂i(ρm ∗ ϕu) → ∂iu in L (ω), and since ρm ∗ ϕu = ρm ∗ u in ω, we deduce

p ∂i(ρm ∗ u) → ∂iu in L (ω).

(ii) If (θm) is a truncation sequence then um = θm(ρm ∗ u) satisfies the required conditions. (c) From Friedrichs’s theorem and its proof, there exist two sequences (um) and n (vm) in D(R ) so that Solutions of Exercises and Problems 195

p um → u, vm → v in L (Ω) and a.e. in Ω, p ∇um → ∇u, ∇vm → ∇v in L (ω), for any ω b Ω, kumk∞ ≤ kuk∞, kvmk∞ ≤ kvk∞.

We have, for any ϕ ∈ D(Ω), Z Z umvm∂iϕdx = − (∂iumvm + um∂ivm)ϕdx, 1 ≤ i ≤ n. Ω Ω In light of the dominated convergence theorem, we can pass to the limit, when m goes to ∞. We get Z Z uv∂iϕdx = − (∂iuv + u∂iv)ϕdx, 1 ≤ i ≤ n, Ω Ω for any ϕ ∈ D(Ω), and the expected result follows. 1.16 (a) Fix ω an open set so that supp(u) ⊂ ω b Ω and pick ϕ ∈ D(Ω) so that ϕ = 1 in supp(u). According to Friedrichs’s theorem (see Exercise 1.15), there exists n p p n a sequence (φm) in D(R ) such that φm → u in L (Ω) and ∇φm → ∇u in L (ω,R ). 1,p 1,p 1,p Whence, ϕφm → ϕu in W (Ω), ϕu ∈ W0 (Ω) and then u ∈ W0 (Ω). 1,p (b) (i) We have um ∈ W (Ω) by Proposition 1.10. On the other hand, we easily 1,p check, with the aid of the dominated convergence theorem, that um → u in W (Ω). As G(mu) = 0 if |u| ≤ m, we then get

supp(um) ⊂ {x ∈ Ω; |u(x)| > 1/m}.

1,p Now, as u = 0 on Γ , um has compact support in Ω. Therefore, um ∈ W0 (Ω) by (a). (ii) Let (θm) be the truncation sequence in the proof of Theorem 1.10. By (i), 1,p 1,p 1,p θmu ∈ W0 (Ω), and as θmu → u in W (Ω), we conclude that u ∈ W0 (Ω). (c) We saw in (b) that if u ∈ W 1,p(Ω) ∩ C Ω is such that u = 0 on Γ then 1,p 1,p  u ∈ W0 (Ω). Conversely, if u ∈ W0 (Ω) ∩C Ω then, according to the definition 1,p 1,p of W0 (Ω), u is the limit in W (Ω) of a sequence of elements in D(Ω). Using 1,p p that the trace operator γ0 is bounded from W (Ω) into L (Γ ) and that γ0um = 0, we obtain γ0u = 0. R x 0 1.17 (a) We get, by using ϕ(x) = 0 ϕ (t)dt and applying then Cauchy-Schwarz’s inequality,

Z x Z 1/2  1 |ϕ(x)|2 ≤ x |ϕ0(t)|2 ≤ x |ϕ0(t)|2, x ∈ 0, . 0 0 2

R 1 0 Similarly, Cauchy-Schwarz’s inequality applied to the identity ϕ(x) = x ϕ (t)dt yields

Z 1 Z 1 1  |ϕ(x)|2 ≤ (1 − x) |ϕ0(t)|2 ≤ (1 − x) |ϕ0(t)|2, x ∈ ,1 . 1 x 2 2 196 Solutions of Exercises and Problems

As Z 1/2 Z 1 1 xdx = (1 − x)dx = , 0 1/2 8 1 the last two inequalities and the density of D(]0,1[) in H0 (]0,1[) entail

Z 1 Z 1 2 1 0 2 1 |u(x)| dx ≤ |u (x)| dx for any u ∈ H0 (]0,1[). 0 8 0 (b) Let u be a solution of (1.13). Multiply each side of the first equation of (1.13) by ϕ ∈ D(]0,1[) to derive that

Z 1 Z 1 Z 1 − u00(x)ϕ(x)dx − k u(x)ϕ(x)dx = f (x)ϕ(x)dx. 0 0 0 But, from the definition of derivatives in the weak sense, we have

Z 1 Z 1 − u00(x)ϕ(x)dx = u0(x)ϕ0(x)dx. 0 0 Hence Z 1 Z 1 Z 1 u0(x)ϕ0(x)dx − k u(x)ϕ(x)dx = f (x)ϕ(x)dx. (5.5) 0 0 0

Let now u1 and u2 be two solutions of (1.13). As (5.5) is satisfied for both u1 and u2, we deduce, for any ϕ ∈ D(]0,1[), that

Z 1 Z 1 0 0 u (x)ϕ (x)dx − k u(x)ϕ(x)dx = 0, with u = u1 − u2. 0 0

1 Once again, by the density of D(]0,1[) in H0 (]0,1[), we can choose ϕ = uk, where 1 (uk) is a sequence in D(]0,1[) converging in H0 (]0,1[) to u. After passing to the limit, as k goes to ∞, we get

Z 1 Z 1 |u0(x)|2dx − k |u(x)|2dx = 0. 0 0 On the other hand, the definition of C yields

Z 1 Z 1 C |u0(x)|2dx ≥ |u(x)|2dx. 0 0 Thus Z 1 Z 1 Ck |u(x)|2dx ≥ |u(x)|2dx 0 0 and hence u = 0 if Ck < 1. (c) The non trivial solutions, for k 6= 0, of the boundary value problem

u00(x) + ku(x) = 0, x ∈]0,1[ and u(0) = u(1) = 0 Solutions of Exercises and Problems 197 √ are of the form u(x) = sin( kx), with k = n2π2. In particular, if k = π2 and if u is a solution (1.13) then u + sin(πx) is also a solution of (1.13) according to (b). But this holds only if kC ≥ 1 or equivalently π2C ≥ 1. We already know from (a) that C ≤ 1/8. In conclusion, we proved that

1 1 ≤ C ≤ . π2 8

1.18 (a) (i) Let x ∈ I and (xm) be a sequence in I converging to x. Let J ⊂ I a 2 compact interval containing x and xm, for each m. We check that χ]x,xm[g converges a.e. to 0 (indeed if t 6= x is such that |g(t)| < ∞, then t 6∈]x,xm[ for sufficiently large

m) and |χ]x,xm[g| ≤ |g| a.e.. An application of the dominated convergence theorem gives Z f (xm) − f (x) = χ]x,xm[gdt → 0. (ii) As

Z Z c Z c Z b Z x f ϕ0dx = − dx g(t)ϕ0(x)dt + dx g(t)ϕ0(x), I a x c c we get from Fubini’s theorem

Z Z c Z t Z b Z b f ϕ0dx = − g(t)dt ϕ0(x)dx + g(t)dt ϕ0(x)dx I a a c t Z c Z b Z = − g(t)ϕ(t)dt − g(t)ϕ(t) = − gϕdx. a c I

R x 0  (b) Set u = c u (t)dt, where c ∈ I is arbitrarily fixed. By (a) (i), u ∈ C I , and by (a) (ii), we have Z Z Z − u0ϕdx = uϕ0dx = − u0ϕdx for any ϕ ∈ D(I). I I I

That is Z (u − u)0ϕdx = 0 for any ϕ ∈ D(I) I and hence u − u = k a.e. in I by the closing lemma, where k is a constant. The functionu ˜ = u + k satisfies then the required properties. (c) Let u be an element of B, the unit ball of W 1,p(I). Using that u has a continuous representative by (b), we can write Z y u(y) − u(x) = u0(x)dx. x Apply Holder’s¨ inequality to the right hand side of this identity in order to deduce that

2 Here ]x,xm[ is the interval with endpoints x and xm. 198 Solutions of Exercises and Problems

0 1/p0 1/p0 |u(y) − u(x)| ≤ ku kLp(I)|x − y| ≤ |x − y| , for any x,y ∈ I.

In other words, we proved that B is equi-continuous and therefore it is relatively compact in C I by Arzela-Ascoli’s theorem. 1.19 Observing that ∇(u − u) = ∇u, it is not hard to see that we are reduced to prove the following result: there exists a constant C > 0 so that, for any u ∈ V = 1 R {v ∈ H (Ω); Ω vdx = 0}, we have

kuk 2 ≤ Ck uk 2 n . L (Ω) ∇ L (Ω,R )

V is a closed subspace of H1(Ω) that we endow with norm of H1(Ω). If the above inequality does not hold, we would find a sequence (uk) in V so that

ku k 2 > kk u k 2 n . k L (Ω) ∇ k L (Ω,R )

Note that we can always assume that kukkH1(Ω) = 1 for each k. Therefore, Sub- tracting a subsequence if necessary, we can also assume that uk converges strongly 2 1 2 in L (Ω) and weakly in H (Ω). But, as ∇uk converge strongly to 0 in L (Ω), we deduce that ∇u = 0 by the closing lemma. Whence u is a.e. equal to a constant c. This and the fact that Z Z 0 = ukdx → udx Ω Ω

entail that c = 0 and then u = 0. In particular, kukkH1(Ω) → 0 which contradicts kukkH1(Ω) = 1, for each k. 1/2 1 1.20 Pick v ∈ H (Γ ) and let Kv be the closed convex set of H (Ω) given by

1 Kv = {v ∈ H (Ω); γ0(u) = v}.

1 According to the projection theorem, there exists a unique PKv (0) ∈ H (Ω) so that

k0 − PKv (0)kH1(Ω) = min k0 − ukH1(Ω). u∈Kv That is

kuvkH1(Ω) = min kukH1(Ω) = kvkH1/2(Γ ), u∈Kv

where uv = PKv (0).

Exercises and Problems of Chapter 2

2.1 Let E and F be two infinite dimensional Banach spaces. If A ∈ K (E,F) ad- mitted an inverse A−1 ∈ L (F,E) then AA−1 = I would be compact. But this is impossible by Riesz’s theorem. Solutions of Exercises and Problems 199

2.2 The operator A is bounded since, for any x ∈ `2, we have

kAxk2 = (a x )2 ≤ sup a2 x2 ≤ C2kxk2 . `2 ∑ m m m ∑ m `2 m≥1 m≥1 m≥1

` 2 Assume that limm→+∞ am = 0. Let (x ) be a sequence in the closed unit ball of ` and set, for each `, y` = Ax`. To prove that A is compact we construct a subsequence of (y`) converging in `2. Let y`,0 = y`. By induction in k, if y`,k is constructed then, as  `,k  ϕm,k(`),k the sequence ym is bounded, there exists ym a convergent subsequence ` `  `,k `,k+1  ϕm,k(`),k of ym . We set then y = ym . We extract a diagonal subsequence by ` m ` `,` ` setting z = y . We claim that (z ) is a Cauchy sequence in `2. Indeed, for ε > 0, as am tends to 0, there exists an integer p so that for any m > p, we have that |am| < ε. Therefore, for each `, we have

2 2  `  2  `  2 ∑ zm ≤ ε ∑ xm ≤ ε . m≥p m

`  On the other hand, since zm ` is convergent, there exits `0 such that

` `0 2 2 0 ∑ |zm − zm| ≤ ε for any `,` ≥ `0. m≤p

A combination of these two inequalities then yields

` `0 2 2 0 ∑|zm − zm| ≤ 3ε for any `,` ≥ `0. m

` Thus (z ) is a Cauchy sequence in `2. Using that `2 is complete we end up getting that (z`) is a convergent subsequence of (yk). Conversely, assume that the sequence (am) does not converge to 0. Hence there exits a constant C > 0 so that, for any integer m, we find an integer k > m with the property that |a | > C. Define the sequences (x`) in `2 and (k ) in so that x` = δ m, k ` N m k` ` ` ` |ak` | > C, and the sequence (k`) is increasing. Set y = Ax . Then the sequence (x ) in bounded in `2, while (y`) does not admit any convergent sub-sequence. Indeed, 0 ` `0 for any ` 6= ` , we have ky − y k`2 > 2C. In other words, the operator A is non compact. 2.3 We have Z 1 2 2 2 2 2 2 2 2 kA f kH = (x + 1) f (x) dx ≤ kx + 1kL∞(0,1)k f kH = 4k f kH 0 and then A is bounded. It is clear that A is self-adjoint and

Z 1 (A f , f ) = (x2 + 1) f (x)2dx > 0 for any f ∈ H, f 6= 0. 0 200 Solutions of Exercises and Problems

That is A is positive. If f ∈ H is an eigenvector of A corresponding to the eigenvalue λ then, for any g ∈ H, we have

Z 1 Z 1 (x2 + 1) f (x)g(x)dx = (A f ,g) = λ( f ,g) = λ f (x)g(x)dx 0 0 and hence Z 1 (x2 + 1 − λ) f (x)g(x)dx = 0. 0 2 2 2 Choosing g = (x +1−λ) f , we get that k(x +1−λ) f kH = 0. Consequently, (x + 1 − λ) f = 0 a.e. in (0,1) implying that f = 0 a.e. in (0,1). This contradicts the fact that f is an eigenvector. Whence A does not admit any eigenvalue. To prove that A−λI is invertible, if g ∈ H we seek f ∈ H satisfying (A−λI) f = g. That is, we want to find f ∈ H such that

(x2 + 1 − λ) f (x) = g(x) a.e. in (0,1).

Therefore, if A − λI is invertible then necessarily f = (A − λI)−1g is given by

f (x) = (x2 + 1 − λ)−1g(x) a.e. in (0,1).

The inverse of (x2 + 1 − λ) is well defined except at the endpoints. Hence, f (x) is well defined a.e. x ∈ (0,1). 2 If λ 6∈ [1,2], then m(λ) = min[0,1] |x + 1 − λ| > 0. Hence A − λI is invertible with bounded inverse:

−1 −1 k(A − λI) gkH ≤ m(λ) kgkH .

For λ ∈ [1,2], if (A − λI) was invertible then (x2 + 1 − λ)−1 would be an element of H. This can not be true because otherwise, since 1 1 = √ √ , x2 + 1 − λ (x − λ − 1)(x + λ − 1) we would have 1 1 p ∼ √ √ as x → λ − 1. x2 + 1 − λ 2 λ − 1(x − λ − 1) Hence Z 1 1 2 2 dx = +∞. 0 (x + 1 − λ) This leads to the expected contradiction.

2.4 Assume that A is compact and let (xm) be a sequence in E converging weakly to x. We show that the only limit point for the strong topology of the sequence (Axm) is Ax. Indeed, if Axψ(m) → y then Solutions of Exercises and Problems 201

ϕ,Axψ(m) = ϕ ◦ A,xψ(m) → hϕ ◦ A,xi = hϕ,Axi,

0 for any ϕ ∈ F , where we used that (xm) converges weakly to x. Therefore, hϕ,Ax− yi = 0 for any ϕ ∈ F0, and hence y = Ax. On the other hand, we know that any weakly convergent sequence are bounded. Then (xm) is bounded, and since A is 3 compact, (Axm) admits Ax as limit point. Whence (Axm) converges to Ax. Conversely, as E is reflexive, the closed unit ball is compact for the weak topol- ogy. To prove that the image by A of the closed unit ball of E is relatively compact, it is sufficient to show that for any arbitrary sequence (xm) of E so that kxmk ≤ 1, the sequence (Axm) has a convergent subsequence. As (xm) belongs to the closed unit ball, it has a limit point for the weak topology and therefore a convergent sub- sequence because the weak topology is metrizable. That is we have a subsequence  xψ(m) converging weakly to x ∈ E. Using the assumption on A, we deduce that Axψ(m) converges strongly to Ax. In other words, (Axm) admits a limit point and hence A is compact. 2.5 We have by applying Cauchy-Schwarz’s inequality

Z 1/2 Z 1/2 |A f (y)| ≤ |k(x,y)|2dµ(x) | f (x)|2dµ(x) . X X

Hence Z Z Z  Z |A f (y)|2dν(y) ≤ |k(x,y)|2dµ(x) dν(y) | f (x)|2dµ(x). Y Y X X

2 R 2 As k ∈ L (X ×Y, µ × ν), we get from Fubini’s theorem that X |k(x,y)| dµ(x) is finite a.e. y ∈ Y and

Z Z  Z |k(x,y)|2dµ(x) dν(y) = |k(x,y)|2dµ(x) ⊗ dν(y). Y X X×Y

Thus kA f kL2(Y) ≤ kkkL2(X×Y)k f kL2(X). Since A is clearly a linear map, we conclude that A is bounded linear operator from 2 2 L (X) into L (Y) and its norm is less or equal to kkkL2(X×Y). When L2(X) (reflexive) is separable, it is sufficient, by the preceding exercise, to 2 show that if ( fn) converges weakly to f in L (X) then (A fn) converges strongly to 2 A f in L (Y). Substituting fn by fn − f , we may assume that ( fn) converges weakly to 0. For such a sequence (A fn(y)) converges to 0 = A f (y), for any y so that x 7→ k(x,y) ∈ L2(X), i.e. we have Z p 2 2 |A fn(y)| ≤ k fnkL2(X) K(y) with K(y) = |k(x,y)| dµ(x) ∈ L (Y). X

3 Recall that in a compact space a sequence admitting a unique limit point is convergent. 202 Solutions of Exercises and Problems

But, the sequence ( fn) being weakly convergent, it is therefore bounded. We apply R 2 then the dominated convergence theorem to get that Y |A f (y)| dν(y) tends to 0. In 2 other words, (A fn) converges strongly to 0 in L (Y) and hence A is compact. 2.6 Assume that u is a solution of (2.97). Multiply each side of the first equation of (2.97) by v ∈ C1(Ω) and then integrate over Ω. Apply then the divergence theorem to the resulting identity to obtain Z Z Z ∇u · ∇vdx − ∂ν uvdσ = f vdx. Ω Γ Ω

As ∂ν u = 0 on Γ , we deduce Z Z ∇u · ∇vdx = f vdx for any v ∈ C1(Ω). (5.6) Ω Ω Conversely, if u is a solution of (5.6) then, again by the divergence theorem, Z Z 1 (−∆u − f )vdx + ∂ν uvdσ = 0 for any v ∈ C (Ω), Ω Γ from which we deduce first that −∆u = f in Ω and then ∂ν u = 0 on Γ . Choosing v = 1 in (5.6), we obtain that if there exists a solution of class C2 then necessarily Z f dx = 0. Ω

1 2.7 We multiply each side of the first equation of (2.98) by v ∈ H0 (Ω) and then we integrate over Ω. The divergence theorem then enables us to obtain the following 1 variational problem: find u ∈ H0 (Ω) satisfying

1 a(u,v) = Φ(v), for any v ∈ H0 (Ω), with Z a(u,v) = (∇u · ∇v +V · ∇uv)dx Ω and Z Φ(v) = f vdx. Ω It is not hard to check that a and Φ are continuous. In order to apply Lax-Milgram’s lemma, we need to prove that a is coercive. We have Z 1 Z 1 Z V · ∇uudx = div(u2V)dx = u2V · ν = 0. Ω 2 Ω 2 Γ

Whence Z a(u,u) = ∇u · ∇udx Ω 1 and hence a is coercive in H0 (Ω). Solutions of Exercises and Problems 203

2.8 (a) We proceed by contradiction. Assume then that, for each m, there exists 1 vm ∈ H (Ω) so that

kv k 2 > m(kv k 2 + k v k 2 n ). m L (Ω) m L (Γ ) ∇ m L (Ω,R )

Substituting kvmkL2(Ω) by vm/kvmkL2(Ω), we may assume that kvmkL2(Ω) = 1. Then 1 (vm) is bounded in H (Ω) and, hence by Rellich’s theorem, there exists (vp) a sub- 2 2 sequence of (vm) converging strongly in L (Ω) to v ∈ L (Ω). Additionally, ∇vp n converges to 0 in L2(Ω,R ). In light of the closing lemma, we get that v ∈ H1(Ω) and ∇v = 0. Thus v is a.e. equal to a constant in Ω. On the other hand, as the trace 1 2 operator w ∈ H (Ω) 7→ w|Γ ∈ L (Γ ) is bounded we deduce that v = 0 on Γ and consequently v = 0 in Ω. But this contradicts the fact that kvkL2(Ω) = 1. (b) We easily obtain, for v ∈ H1(Ω), Z Z Z ∇u · ∇vdx − ∂ν uvdσ = f vdx. Ω Γ Ω

But −∂ν u = u−g on Γ . That is we have a variational problem in the form a(u,v) = Φ(v), where Z Z a(u,v) = ∇u · ∇vdx + uvdσ Ω Γ and Z Z Φ(v) = f vdx + gvdσ. Ω Γ The existence and uniqueness of a solution of the variational problem is obtained by applying Lax-Milgram’s Lemma. It is straightforward to check that a and Φ are continuous. While the coercivity of a follows from (a) because

a(u,u) = k uk 2 n + kuk 2 . ∇ L (Ω,R ) L (Γ ) 2.9 If ϕ (of class C∞) is an eigenfunction then ϕ is a solution of the ordinary differential equation, with λ > 0,

ϕ00 + λϕ = 0.

The solutions of this equation are of the form √ √ ϕ = Asin( λx) + Bcos( λx). √ The boundary conditions ϕ(0) = ϕ(1) = 0 imply B = 0 and λ = kπ for some k ∈ Z. Therefore, the eigenfunctions of the Laplace operator with Dirichlet boundary condition are given by ϕk = sin(kπx), k ≥ 1, each ϕk corresponding to the eigenvalue 2 2 λk = k π . 2 1 R 1 0 0 Apply Theorem 1.9 to H = L (0,1), V = H0 (0,1) and a(u,v) = 0 u v dx to 2 deduce that the sequence ϕk forms an orthonormal basis of L (0,1) and the sequence 1 (ϕk/(kπ)) forms an orthonormal basis of H0 (0,1) for the scalar product ( f ,g) = 204 Solutions of Exercises and Problems

R 1 0 0 2 2 0 f g dx. Whence, ∑ak sin(kπx) converges in L (0,1) if and only if ∑ak < ∞ and 1 2 2 in H0 (0,1) if and only if ∑k ak < ∞.

2.10 Let (ϕk), ϕk = sin(kπx) for each k, be the sequence of eigenfunctions of the Laplace operator in ]0,1[ under Dirichlet boundary condition. For 1 ≤ p ≤ n and k ≥ 1, set ϕp,k(x) = ϕk (x/`p). Finally, define, for k1 ≥ 1,...,kn ≥ 1,

ψk1,...,kn (x1,...,xn) = ϕ1,k1 (x1)...ϕn,kn (xn).

We can easily check that ψk1,...,kn is an eigenfunction for the Laplace operator under Dirichlet boundary condition corresponding to the eigenvalue

n  2 kiπ λk1,...,kn = ∑ . i=1 `i

 2 To complete the proof we have to show that ψk1,...,kn forms a basis in L (Ω), i.e. if w ∈ L2(Ω) is so that

(w,ψk1,...,kn ) = 0 for all k1 ≥ 1,...,kn ≥ 1, (5.7) then w = 0. We proceed by induction in the dimension n. The result is true for n = 1 by the preceding exercise. Assume then that the result holds in dimension n − 1. 2 Introduce the function y ∈ L (]0,`n[) defined by Z 0 0 y(xn) = w(x ,xn) ϕk (xi)dx , 0 ∏ i Ω i

0 0 with Ω =]0,`1[×...×]0,`n−1[ and x = (x1,...,xn−1). By (5.7), for each k ≥ 1,

Z `n y(xn)ϕn,k(xn)dxn = 0. 0

2 As (ϕn,k)k forms a basis in L (]0,`n[), y(xn) = 0 a.e. in ]0,`n[. Hence, for a.e. xn ∈ 0 0 2 0 ]0,`n[, wxn (x ) = w(x ,xn) ∈ L (Ω ) is so that Z 0 0 wxn (x ) ϕk (xi)dx = 0 0 ∏ i Ω i

 R 2  Ω |∇u| dx 1 λ1 = min R 2 ; u ∈ H0 (Ω), u 6= 0 . Ω u dx 2.12 We first prove that V is a Hilbert space. It is clear that h·,·i defines a scalar product on V. It remains to show that V is complete for the norm associated to this scalar product. This norm is denoted by k · kV . Proceeding by contradiction, we can Solutions of Exercises and Problems 205 easily show that there exists a constant C > 0 so that, for any u ∈ V,

Z Z Z  u2dx ≤ C |∇u|2dx + u2dx , B2 B2 B2\B1 where Bi is the ball centered at 0 with radius i = 1,2. Then

kuk 1 n ≤ Ckuk . H (R ) V

Therefore, if (um) is a Cauchy sequence in V then it is also a Cauchy sequence in 1 n 1 n 1 n H (R ). Thus, there exists u ∈ H (R ) so that um converges to u in H (R ). Also as 2 n 2 n 2 n (|x|um) is a Cauchy sequence in L (R ) it converges in L (R ) to some v ∈ L (R ). n On the other hand, |x|um converges weakly to |x|u. Indeed, for ϕ ∈ D(R ), Z Z Z lim |x|umϕdx = |x|uϕdx = vϕdx m→+∞ Rn Rn Rn from which we deduce that v = |x|u and um converges to u in V. 2 n We now prove that V is compactly imbedded in L (R ). Let (um) be a bounded 1 sequence in V, kumkV ≤ M. Bearing in mind that H (B) is compactly imbedded in 2 n L (B), B an arbitrary open ball of R , we conclude that (um) admits a convergent 2 subsequence denoted again by (um) converging to u in L (B). Moreover, |x|u ∈ n L2(R ). Whence Z Z Z 2 2 1 2 2 (u − um) dx < (u − um) dx + 2 |x| (u − um) dx Rn |x|R Z 2 2M < (u − um) dx + 2 . |x|

Then Z 2 2M limsup (u − um) dx ≤ 2 for any R > 0. m→+∞ Rn R 2 n Therefore (um) converges to u in L (R ). That is, we proved that V in compactly n embedded in L2(R ). Next, we observe that the bilinear form Z a(u,v) = (∇u · ∇v + Q(x)uv)dx Rn is clearly continuous and coercive on V. Apply then Theorem 3.9 to conclude that n there exists a Hilbertian basis of L2(R ) consisting in eigenfunctions, corresponding to a sequence of positive eigenvalues converging to infinity. 2.13 Writing u(x,y) = v(r,θ), we get 1 1 ∆u(x,y) = ∂ (r∂ v(r,θ)) + ∂ 2 v(r,θ). r r r r2 θ 2 206 Solutions of Exercises and Problems

If we seek v in the form v(r,θ) = f (r)g(θ), we find, after making straightforward computations, that f and g are the respective solutions of the equations

−g00(θ) = λg(θ), 0 < θ < β, g(0) = g(β) = 0, (5.8) and r2 f 00(r) + r f 0(r) − λ f (r) = 0, 0 < r < 1. (5.9) By Exercise 2.9, (5.8) admits as solutions

kπ  g (θ) = sin θ , 0 ≤ θ ≤ β, k ≥ 1, k β and kπ 2 λ = , k ≥ 1. β For the equation (5.9), we look for solutions of the form f (r) = rγ 4. After some computations, we get two systems of solutions

kπ/β −kπ/β f+(r) = r , f−(r) = r .

Using the boundary conditions at θ = 0,β, we obtain two families of solutions

kπ  kπ  u = rkπ/β sin θ , u = r−kπ/β sin θ . +,k β −,k β

In light of the boundary condition at r = 1, we end up getting that there are only two possible solutions

 π   π  u = rπ/β sin θ , u = r−π/β sin θ . + β − β

Next we use the relations  sinθ   cosθ  ∂ u = cosθ∂ − ∂ v, ∂ u = sinθ∂ + ∂ v x r r θ y r r θ

±π/β−1 to deduce that ∂xu± et ∂yu± are of the form ψ±(θ)r with ψ± a function ∞ 1 1 of class C . In consequence, we get that u+ ∈ H (Ω) and u− 6∈ H (Ω) because rπ/β−1 ∈ L2((0,1),rdr) and r−π/β−1 6∈ L2((0,1),rdr). Thus the only solution be- 1 longing to H (Ω) is u = u+. 2 2 2 π/β−2 We can similarly check that ∂xxu, ∂xyu, ∂yyu are of the form φ(θ)r where φ is a function of class C∞. Therefore, u ∈ H2(Ω) if and only if rπ/β−2 ∈ L2((0,1),rdr), or equivalently u ∈ H2(Ω) if and only if β < π.

4 We can also observe that (5.9) is an Euler’s equation and solve it by setting s = lnr and h(s) = f (r). Solutions of Exercises and Problems 207

−1 2.14 (a) (i) V is closed because V = t1 {0} and t1 is bounded. (ii) If the inequality does not hold, we would find a sequence (wn) in V so that 1 kw k 2 = 1 and k∇w k 2 n ≤ . n L (Ω) n L (Ω,R ) n

1 1 In particular, (wk) would be a bounded sequence in H (Ω). As H (Ω) is compactly 2 imbedded in L (Ω), (wk) would admit a subsequence, still denoted by (wk), con- 2 2 L ( ) w ∈ L ( ) k w k 2 n ≤ /k verging in Ω to Ω . On the other hand, since ∇ k L (Ω,R ) 1 , 2 n ∇wk tends to 0 in L (Ω,R ). In light of the closing lemma, we deduce that w ∈ H1(Ω) and ∇w = 0 a.e. in Ω. Hence w is equal a.e. to a constant. Now, t1 being bounded, we get that 0 = t1wn → 0 = t1w. Thus w = 0, contradicting 1 = kwkkL2(Ω) → 1 = kwkL2(Ω). (b) If u ∈ V ∩ H2(Ω) is a solution (2.103) and if v ∈ V then by the divergence theorem Z Z Z Z Z − f vdx = ∆uvdx = − ∇u · ∇vdx + ∂ν uvdσ + ∂ν uvdσ. Ω Ω Ω Γ1 Γ2

As ∂ν u|Γ2 = 0 and v|Γ1 = 0, we conclude that Z Z f vdx = ∇u · ∇vdx. Ω Ω That is u is a solution of (2.104). Conversely, assume u ∈ V ∩ H2(Ω) is a solution of (2.104). Then choosing v ∈ D(Ω) in (2.103), and applying again the divergence theorem, we easily obtain Z (−∆u − f )v = 0 for any v ∈ D(Ω). Ω

Hence −∆u = f a.e. in Ω. Next, we take in (2.104) v ∈ {w ∈ D(Ω); w|Γ1 = 0}. We get, by using once again the divergence theorem, Z ∂ν uvdσ = 0 for any v ∈ D(Γ2). Γ2

Whence Z 2 Dν uvdσ = 0 for any v ∈ L (Γ2) Γ2 2 by the density of D(Γ2) in L (Γ2). In consequence, we obtain that ∂ν u|Γ2 = 0 and u is a solution of (2.103). (c) Set Z a(u,v) = ∇u · ∇vdx, u,v ∈ V. Ω 208 Solutions of Exercises and Problems

R By (a) (i), a defines an equivalent scalar product on V. As v ∈ V → Ω f vdx belongs to V 0, Riesz-Frechet’s´ representation theorem enables us to deduce that (2.104) ad- mits a unique solution u ∈ V. (d) Is immediate from Theorem 2.2 applied to a, V and H = L2(Ω). 1 (e) Let Vm (resp. Wm) be the set consisting in all subspaces of V (resp. H0 (Ω)) of dimension m. Since Wm ⊂ Vm we have according to the min-max’s formula that

R 2 R 2 Ω |∇v| dx Ω |∇v| dx max R 2 ≥ min max R 2 = µm for any Fm ∈ Wm. v∈Fm, v6=0 Ω v dx Em∈Vm v∈Em, v6=0 Ω v dx Whence R 2 Ω |∇v| dx λm = min max R 2 ≥ µm. Em∈Wm v∈Em, v6=0 Ω u dx 2.15 (a) As u − m(4r) and M(4r) − u are two non negative solutions of Lu = 0 in B(4r), we can apply Theorem 2.24 for both. We obtain, for p = 1, Z (u − m(4r)) ≤ Crn(m(r) − m(4r)), (5.10) B(2r) Z (M(4r) − u) ≤ Crn(M(4r) − M(r)), (5.11) B(2r) where the constant C only depends on the L∞-norm of the coefficients of λ −1L, n and r0. We add side by side inequalities (5.10) and (5.11) to get

M(4r) − m(4r) ≤ C [(M(4r) − m(4r)) − (M(r) − m(r))].

That is ω(4r) ≤ C(ω(4r) − ω(r)). Hence ω(r) ≤ γω(4r), with γ = (C − 1)/C. (b) We obtain by iterating the last inequality  r  ω ≤ γkω(4r), k ≥ 0 is an integer. (5.12) 4k−1

Fix now 0 < r ≤ r0 and let k be the integer so that r r 0 < r ≤ 0 . 4k 4k−1 We obtain from (5.12)  r  ω(r) ≤ ω 0 ≤ γkω(4r ), 4k−1 0 Solutions of Exercises and Problems 209 where we used that ω is a non decreasing function. But

ln(r /r) 0 < k. ln4 Thus, r lnγ/ln4 γk ≤ eln(r0/r)lnγ/ln4 = 0 r α −α because γ ≤ 1. Therefore, ω(r) ≤ Mr with α = −lnγ/ln4 and M = ω(4r0)r0 . 2.16 We have φ  u   u   u  L (u,ϕ ) = θ 0 a∇u · ∇u + θ a∇u · ∇φ + θ φd · ∇u ε ε ε ε ε  u   u   u   u  + uθ c · ∇φ + duθ φ + φ θ 0 c · ∇u, ε ε ε ε where c = (ci) and d = (di). As

φ  u  θ 0 a∇u · ∇u ≥ 0 ε ε and u is a sub-solution of Lu = f in Ω, we deduce Z Z L˜(u,ϕε )dx ≤ f ϕε dx, (5.13) Ω Ω where  u   u   u  L˜(u,ϕ ) = θ a∇u · ∇φ + θ φd · ∇u + uθ c · ∇φ ε ε ε ε  u   u   u  + duθ φ + φ θ 0 c · ∇u. ε ε ε When ε → 0, we have  u  θ → χ + µχ a.e. in Ω, ε {u>0} {u=0} u  u  θ 0 → 0 a.e. in Ω. ε ε On the other hand,  u  u  u  0 ≤ θ ≤ 1, θ 0 ≤ kθ 0k ε ε ε ∞ and + ∂iuχ{u>0} = ∂iu a.e. in Ω, ∂iu = 0 a.e. in {u = 0}. According to the dominated convergence theorem, we can pass to the limit when ε → 0 in (5.13). We obtain 210 Solutions of Exercises and Problems Z Z + L (u ,φ)dx ≤ f (χ{u>0} + µχ{u=0})φdx, Ω Ω as expected. 2.17 (a) We obtain, by taking v = u in (2.106), Z Z Z |∇u|2dx = F(u)udx + f udx Ω Ω Ω Z Z Z = (F(u) − F(0))udx + F(0)udx + f udx. Ω Ω Ω But (F(u) − F(0))u ≤ 0. Hence Z Z Z |∇u|2dx ≤ F(0)udx + f udx. Ω Ω Ω Cauchy-Schwarz’s inequality then yields

2  1/2  k∇uk ≤ |F(0)||Ω| + k f k 2 kuk 2 . L2(Ω,Rn) L (Ω) L (Ω)

In light of Poincare’s´ inequality, there exists µ = µ(Ω) so that

kuk 2 ≤ k uk 2 n L (Ω) µ ∇ L (Ω,R ) and hence 1 k uk 2 n ≤ (|F( )|| | 2 + k f k 2 ) = C. ∇ L (Ω,R ) µ 0 Ω L (Ω)

(b) Let u1 and u2 be two variational solutions. Then Z Z Z ∇u1 · ∇(u1 − u2)dx = F(u1)(u1 − u2)dx + f (u1 − u2), Ω Ω Ω Z Z Z ∇u2 · ∇(u1 − u2)dx = F(u2)(u1 − u2)dx + f (u1 − u2). Ω Ω Ω Subtracting side by side these two identities, we get Z Z 2 |∇u1 − ∇u2| dx = (F(u1) − F(u2))(u1 − u2)dx. Ω Ω

But (F(u1) − F(u2))(u1 − u2) ≤ 0, because F is non decreasing. Whence ∇(u1 − 1 u2) = 0 in Ω. Therefore, since u1 − u2 ∈ H0 (Ω), we deduce that u1 = u2 in Ω. (c) Let B be the unit ball of L2(Ω). For (λ,w) ∈ [0,1] × B, we easily check, similarly to (a), that u = T(λ,w) satisfies   k uk 2 n ≤ kF(w)k 2 + k f k 2 ∇ L (Ω,R ) µλ L (Ω) L (Ω)   ≤ µ kF(w)kL2(Ω) + k f kL2(Ω) .

But Solutions of Exercises and Problems 211

1/2 1/2 kF(w)kL2(Ω) ≤ a|Ω| + bkwkL2(Ω) ≤ a|Ω| + b. Then  1/2  k uk 2 n ≤ a| | + b + k f k 2 . ∇ L (Ω,R ) µ Ω L (Ω) 1 1 In other words, T([0,1] × B) is bounded in H0 (Ω). Since H0 (Ω) is compactly imbedded in L2(Ω), it follows that T is compact. In order to show that T satisfies the assumptions Leray-Schauder’s theorem, it is sufficient to check that the set

{u ∈ L2(Ω); u = T(λ,u) for some λ ∈ [0,1]} is bounded in L2(Ω). In view of Poincare’s´ inequality, it is enough to prove that this 1 set is bounded in H0 (Ω). Repeating the estimates in (a), in which we substitute F and f respectively by λF and λ f , we deduce, for u = T(λ,u),

2 1/2 k∇uk ≤ µλ(|F(0)||Ω| + k f k 2 ) L2(Ω,Rn) L (Ω)  1/2  ≤ µ |F(0)||Ω| + k f kL2(Ω) = C.

Exercises and Problems of Chapter 3

3.1 Let 0 < α ≤ 1. We first note that, as a consequence of the following identities

k(T−h − I) f k∞ = kT−h(I − Th) f k∞ = k(I − Th) f k∞. we obtain k(I − T ) f k [ f ] = h ∞ . α sup α h>0 h On the other hand, we have

2 2 Th − 2I + T−h = T−h(Th − 2Th + I) = T−h(Th − I) .

Whence k(T − I)2 f k [ f ]∗ = h ∞ . α sup α h>0 h

As k(Th − I)gk∞ ≤ 2kgk∞, we conclude that

k(T − I)2 f k k(T − I)(T − I) f k [ f ]∗ = h ∞ = h h ∞ α sup α sup α h>0 h h>0 h k(T − I) f k ≤ h ∞ = [ f ] . 2sup α 2 α h>0 h Assume now that 0 < α < 1. Using the following two identities 212 Solutions of Exercises and Problems 1 T − I = (T 2 − I) − (T − I)2, T 2 = T , h 2 h h h 2h we get 1 k(T − I) f k = k(T − I) f − (T − I)2 f k h ∞ 2 2h h ∞ 1 1 ≤ k(T − I) f k + k(T − I)2 f k . 2 2h ∞ 2 h ∞ Hence, for h > 0,

k(T − I) f k k(T − I) f k k(T − I)2 f k h ∞ ≤ 2α−1 2h ∞ + h ∞ hα (2h)−α 2h−α α−1 −1 ∗ ≤ 2 [ f ]α + 2 [ f ]α . Whence α−1 −1 ∗ [ f ]α ≤ 2 [ f ]α + 2 [ f ]α . As 0 < α < 1, we have 2α−1 < 1. Therefore

2−1 2 [ f ] ≤ C[ f ]∗ with C = = . α α 1 − 2α−1 2 − 2α

h (ε)i 3.2 (a) (i) We first prove that u ≤ Uα . We have α Z (ε) (ε) u (x + h) − u (x) ≤ |u(x + h − εy) − u(x − εy)|ϕ(y)dy Rn Z α α ≤ h Uα ϕ(y)dy = h Uα . Rn Hence the result follows. n Let k ≥ 1 be an integer and ` ∈ N such that |`| = k. Then Z x − y ∂ `u(ε)(x) = ε−n∂ ` u(y)ϕ( )dy Rn ε Z x − y = ε−n−|`| u(y)∂ `ϕ( )dy Rn ε Z Z = ε−|`| u(x − εy)∂ `ϕ(y)dy − ε−|`| u(x)∂ `ϕ(y)dy, Rn Rn where we used that R ∂ `ϕ(y)dy = 0 (see Lemma 1.5). We proceed as above to de- ` (ε) α−k duce from the last term that |∂ u | ≤ Cε Uα with Z C = C(ϕ,k) = max |∂ `ϕ(y)|dy. |`|=k Rn (ii) We get by taking the derivative under the integral Solutions of Exercises and Problems 213 Z Z (ε) −n−1 x − y −n−2 x − y ∂ε u (x) = −nε u(y)ϕ( )dy − ε u(y)∇ϕ( ) · (x − y)dy Rn ε Rn ε Z Z = −nε−1 u(x − εy)ϕ(y)dy − ε−1 u(x − εy)∇ϕ(y) · ydy Rn Rn Z = −nε−1 u(x − εy)ϕ(y)dy + nε−1u(x) Rn Z − nε−1u(x) − ε−1 u(x − εy)∇ϕ(y) · ydy Rn Z = −nε−1 [u(x − εy) − u(x)]ϕ(y)dy Rn Z − ε−1 [u(x − εy) − u(x)]∇ϕ(y) · ydy, Rn where we used Z ∇ϕ(y) · ydy = −n. (5.14) Rn

(ε) α−1 The last term in these identities gives ∇ε u ≤ Cε , where Z C = C(n,ϕ) = n + |∇ϕ(y) · y|dy. Rn Note that (5.14) can be simply established by using the divergence theorem: Z Z ∇ϕ(y) · ydy = ∇ϕ(y) · ydy Rn B(0,1) Z Z = − ϕ(y)div(y)dy + ϕ(y)y · ν B(0,1) ∂B(0,1) Z = −n ϕ(y)dy B(0,1) = −n.

−1 (ε) (ε) ∞ (b) As suppϕ(ε ·) ⊂ (−ε,ε), u and v are in Cc (−2,2), 0 ≤ ε ≤ 1. We decompose w as follows

Z 1 (0) (0) (1) (1) (ε) (ε) w(x) = u ∗ v(x) = u ∗ v = u ∗ v − ∂ε [u ∗ v (x)]dε 0 (1) (1) = u ∗ v − w1(x) − w2(x), where Z 1 Z 1 (ε) (ε) (ε) (ε) w1(x) = ∂ε u ∗ v (x)dε, w2(x) = u ∗ ∂ε v dε. 0 0 (i) Assume 0 < α + β < 1. As u(1) ∗ v(1) belongs to C∞(−2,2), it is sufficient to prove that [wi]0,α+β < ∞, for i = 1,2. We prove [w1]0,α+β < ∞. By interchanging the roles of u and v we get also that [w2]0,α+β < ∞. We are then reduced to prove that there exits a constant C > 0 so that 214 Solutions of Exercises and Problems

α+β |w1(x + h) − w1(x)| ≤ Ch for any h > 0.

Since this result is obvious if h ≥ 1, it is enough to consider the case 0 < h ≤ 1. Recall that we defined the operator Th by Th f (x) = f (x + h). We have

Z 1 (ε) (ε) w1(x + h) − w1(x) = (Th − I)w1(x) = ∂ε u ∗ (Th − I)v (x)dε. 0 Clearly,

(ε) β (ε) β (Th − I)v (x) ≤ h [v ]0,β ≤ h Vβ . On the other hand, according to the mean-value theorem, we have

(ε) (ε) β−1 (Th − I)v (x) ≤ hk∂xv k∞ ≤ Chε Vβ ,

(ε) α−1 and as |∂ε u | ≤ Cε Uα , we get

Z h Z 1 (ε) (ε) |w1(x + h) − w1(x)| ≤ + |Dε u | ∗ |(Th − I)v |(x)dε 0 h  Z h Z ∞  β α−1 α+β−2 ≤ CUαVβ h ε dε + h ε dε 0 h  1 1  ≤ CU V + hα+β . α β α 1 − (α + β)

1 (ii) Consider now the case 1 < α + β < 2. We show that w = u ∗ v ∈ Cc (R) and 0 [w ]0,α+β−1 < ∞. As before, it sufficient to give the proof with w1 instead of w. To this end, we introduce the following approximation of w1 :

Z 1 (ε) (ε) w1,δ (x) = ∂ε u ∗ v (x)dε, 0 < δ < 1. δ We have from the estimates in (a)

Z δ (ε) (ε) α |w1(x) − w1,δ (x)| = ∂ε u ∗ v (x)dε ≤ Cδ , 0 where the constant C is independent on δ. Hence w1,δ converges uniformly in R to w1, when δ tends to 0. But

Z 1 (ε) (ε) ∂xw1,δ = ∂ε u ∗ ∂xv (x)dε δ and hence, for some constant C independent on δ, Solutions of Exercises and Problems 215

Z δ2 (ε) (ε) ∂xw1,δ1 − ∂xw1,δ2 = ∂ε u ∗ ∂xv (x)dε| δ1 Z δ2 ≤ C| εα+β−2dε| δ1 α+β−1 α+β−1 ≤ C|δ1 − δ2 |.

It follows that (∂xw1,δ ) is a uniform Cauchy sequence in R (note that α +β −1 > 0). 1 In consequence, w1 ∈ Cc (R) and ∂xw1 = limδ→0 ∂xw1,δ . For w1,δ , we have

Z 1 (ε) (ε) w1,δ (x + h) − w1,δ (x) = (Th − I)w1,δ (x) = ∂ε u ∗ (Th − I)∂xv (x)dε. δ

(ε) Similarly to (i), we give two estimates for the term (Th − I)v (x). In the second estimate we use the mean-value theorem. These estimates are the following ones

(ε) (ε) β−1 (Th − I)∂xv (x) ≤ 2k∂xv k∞ ≤ CVβ ε ,

(ε) 2 (ε) β−2 (Th − I)∂xv (x) ≤ hk∂x v k∞ ≤ ChVβ ε .

Fix h ∈ (0,1) and set ` = max(h,δ). Then

Z ` Z 1 (ε) (ε) ∂xw1,δ (x + h) − ∂xw1,δ (x) ≤ + ∂ε u ∗ (Th − I)∂xv (x)dε δ ` Z ` Z 1  α+β−2 α+β−3 ≤ CUαVβ ε dε + h ε dε δ ` Z ` Z +∞  α+β−2 α+β−3 ≤ CUαVβ ε dε + h ε dε δ h  1 1  ≤ CU V + hα+β−1. α β α + β − 1 2 − α − β

Passing to the limit in the first term of these inequalities, when δ goes to 0, we get   1 1 α+β−1 |∂xw (x + h) − ∂xw (x)| ≤ CU V + h . 1 1 α β α + β − 1 2 − α − β

This completes the proof of (ii).

3.3 Let us first prove by contradiction that λk < 0. Indeed, if λk ≥ 0 then Luk = λkuk ≥ 0. Hence supuk = supuk = 0, Ωk ∂Ωk by the maximum principle. But this contradicts the fact that uk > 0 in Ωk. We now prove, again by using a contradiction, that λ1 < λ2. Assume then that λ1 ≥ λ2. As 2 u2 > 0 in Ω, we have that v = u1/u2 belongs to C (Ω) ∩C(Ω) and it is a solution 216 Solutions of Exercises and Problems of the equation ˜ i j 2 ˜i Lv = ∑a˜ ∂i jv + ∑b ∂iv + cv˜ = 0 in Ω1, i, j i with i j i j ˜i i j a˜ = a u2, b = ∑a ∂ ju2, c˜ = (λ2 − λ1)u2. j

Sincec ˜ ≤ 0 and v = 0 on ∂Ω1, we obtain v = 0 in Ω1, which contradicts the fact that v > 0 in Ω1 and completes the proof. 3.4 We obtain from Poincare’s´ inequality Z Z 2 2 u dx ≤ C0(n) |∇u| dx B1 B1 and we get from the divergence theorem Z Z |∇u|2dx = − ∆uudx. B1 B1 These two inequalities together with Cauchy-Schwarz’s inequality imply

Z Z Z 1/2 Z 1/2 2 2 2 u dx ≤ −C0(n) ∆uudx ≤ C0(n) (∆u) dx u dx B1 B1 B1 B1 and hence the result follows.

3.5 For x ∈ ∂B1(0), we have ν = ν(x) = x. In consequence,

a u(x +tν) = u((1 +t)x) = (1 +t) u(x), x ∈ ∂B1(0).

Hence u(x +tν) − u(x) ∂ν u(x) = lim = au(x), x ∈ ∂B1(0). t→0 t

In a similar manner, we have also ∂ν v(x) = bv(x) if x ∈ ∂B1(0). We get by applying the divergence theorem Z Z Z 0 = [v∆u − u∆v]dx = [uDν v − Dν uv]ds = (b − a)uvds. B1(0) ∂B1(0) ∂B1(0) We deduce, as a 6= b, Z uvds(x) = 0. ∂B1(0) 3.6 (a) A simple change of variable implies

1 Z x f (x −t) u(x ,x ) = 2 1 dt. 1 2 2 2 π R t + x2 Then Solutions of Exercises and Problems 217 1 Z x | f (x −t) − f (y −t)| |u(x ,x ) − u(y ,x )| ≤ 2 1 1 dt 1 2 1 2 2 2 π R t + x2 and hence  1 Z x  |u(x ,x ) − u(y ,x )| ≤ 2 dt [ f ] |x − y |α . 1 2 1 2 2 2 α 1 1 π R t + x2 But 1 Z x 1 Z 1 2 dt = ds = 1. 2 2 2 π R t + x2 π R s + 1 Therefore α |u(x1,x2) − u(y1,x2)| ≤ [ f ]α |x1 − y1| . (5.15)

(b) The change of variable t = x1 + sx2 yields

1 Z f (x1 + sx2) u(x1,x2) = 2 ds π R s + 1 and consequently

1 Z | f (y1 + sx2) − f (y1 + sy2)| |u(y1,x2) − u(y1,y2)| ≤ 2 ds. π R s + 1 α α As | f (y1 + sx2) − f (y1 + sy2)| ≤ [ f ]α |s| |x2 − y2| , we find  Z α  1 |s| α |u(y1,x2) − u(y1,y2)| ≤ 2 ds [ f ]α |x2 − y2| . (5.16) π R s + 1

2 (c) Let x = (x1,x2), y = (y1,y2) ∈ R+. Then

|u(x) − u(y)| = |u(x1,x2) − u(y1,y2)|

≤ |u(x1,x2) − u(y1,x2)| + |u(y1,x2) − u(y1,y2)|.

This (5.15) and (5.16) entail

α |u(x) − u(y)| ≤ C[ f ]α |x − y| , with 1 Z |s|α C = 1 + 2 ds. π R s + 1 That is [u]α ≤ C[ f ]α . 3.7 Fix p ∈ (0,1). (a) We have

q−2 2 q−2 q−4 2 ∂iv = cq|x| xi and ∂i v = cq|x| + cq(q − 2)|x| xi and 218 Solutions of Exercises and Problems

∆v = cq(n + q − 2)|x|q−2 = cq(n + q − 2)|x|qp. It is then sufficient to take c = [q(n + q − 2)]1/(p−1) in order to get ∆v = vp. (b) The equality

maxu(x) = maxu(x) for any r > 0 |x|≤r |x|=1 follows readily from the maximum principle because ∆u ≥ 0. (c) Assume that there exists r > 0 such that u < v in ∂Br(0). As

∆u = up > vp = ∆v in Ω, we conclude that u ≤ v in Ω, which is impossible. Therefore u ≥ v in ∂Br(0) for all r > 0. That is we proved the following estimate

maxu(x) ≥ cr2/(1−p) for all r > 0. |x|≤r

3.8 (a) Let x be such that B2r(x) ⊂ B2(0). By Lemma 3.11, we have that if y ∈ Br(x) then dy|∂iu(y)| ≤ C sup u, B2r(x) with a constant C = C(n), where dy = dist(y,∂B2r(x)). As dy ≥ r, we conclude

r|∂iu(y)| ≤ C sup u. B2r(x) In consequence, C sup |∇u| ≤ 0 sup (u) for any B (x) ⊂ B (0), r 2r 2 Br(x) B2r(x)  with a constant C0 = C0(n). We easily deduce from the last estimate take r = 1/4

|∇u(x)| ≤ C1 sup u for any x ∈ B1(0) (5.17) B1/2(x) with a constant C1 = C1(n). (b) We have according to Harnack’s inequality (Theorem 2.8)

sup u ≤ C2 inf u ≤ C2u(x) for any x ∈ B1(0), (5.18) B (x) B1/2(x) 1/2 where C2 = C2(n) is a constant. (c) A combination of (5.17) and (5.18) entail

|∇u(x)| ≤ C1C2u(x), for any x ∈ B1(0). Solutions of Exercises and Problems 219

In other words, we proved

|∇u(x) |∇(lnu(x))| = ≤ C for any x ∈ B (0), u(x) 1 with C = C1C2. 1 2 3.9 Let u ∈ H0 (Ω) ∩ H (Ω). We have by the divergence theorem Z Z |∇u|2 = − ∆uu. Ω Ω Therefore, we get by using the convexity inequality |ab| ≤ a2/2 + b2/2

Z Z √   Z Z   1 2 1 2 ∆uu ≤ 2ε∆u √ u ≤ ε (∆u) + u . Ω Ω 2ε Ω 4ε Ω Hence Z Z Z 1 Z |∇u|2 = − ∆uu ≤ ε (∆u)2 + u2. Ω Ω Ω 4ε Ω 3.10 We have by applying the mean-value theorem 1 Z −∂nu(x0) = ∂n(M − u)(x0) = n ∂n(M − u)dx. ωnr Br Whence, we obtain from the divergence theorem, applied to to the last term, 1 Z 1 Z −∂nu(x0) = n (M − u)νndσ(x) ≤ n (M − u)dσ(x). ωnr ∂Br ωnr ∂Br Then an application of the mean value theorem to the last term gives

n−1 1 Z nωnr n −∂nu(x0) ≤ n (M − u)dσ(x) = n (M − u(x0)) = (M − u(x0)). ωnr ∂Br ωnr r Hence the result follows by letting r → 1. n 3.11 Let x ∈ R and r > 0. As u is harmonic we can apply Lemma 3.11. We obtain in particular, for any k ∈ N,

maxrk|∂ `u(x)| ≤ C sup |u|, |`|=k B(x,r) with a constant C = C(n,k). This and the assumption on u imply

maxrk|∂ `u(x)| ≤ Crα |`|=k and hence max|∂ `u(x)| ≤ Cr−k+α . |`|=k 220 Solutions of Exercises and Problems

When −k + α < 0, we obtain, by letting in this inequality r → +∞, that ∂ `u(x) = 0 for all |`| = k. In other words, u is a polynomial of degree less or equal to [α], the integer part of α.

−ρr2 −ρr2 3.12 (a) (i) We have ∂ie = −2ρxie . Therefore

2 2 2 −ρr2 ∆v = ∑∂i v = (4r ρ − 2nα)e . i

0 But r = |x − y| ≥ |y − x0| − |x − x0| ≥ R − R = δ in D. Whence

2 2 2 −αr2 ∆v = ∑∂i ≥ (4δ ρ − 2nρ)e in D. i Thus, ∆v > 0 in D provided that ρ is chosen sufficiently large. Then, for ε > 0, we have ∆(u − u(x0) + εv) = ∆u + ε∆v > 0 in D.

(ii) It is not hard to check that u−u(x0)+εv ≤ 0 on ∂D if ε is chosen sufficiently large. We get then, by applying the maximum principle, that w = u−u(x0)+εv ≤ 0 in D and, as w(x0) = 0, we deduce that w attains its maximum at x0. Therefore ∂ν w(x0) ≥ 0. In consequence,

−ρR2 ∂ν u = ∂ν w − ε∂ν v = ∂ν w + 2Rρεe > 0.

(b) Set M = maxu and make the assumption that the (closed) set F = {x ∈ Ω; u(x) = M} is nonempty. As u is non constant Ω \ F is also non empty. By Lemma 3.14 there exists a ball B so that B ⊂ Ω, B ∩ F = /0and ∂B ∩ F 6= /0.We get then, by applying Hopf’s lemma with B and y ∈ ∂B ∩ F, that ∂ν u(y) > 0. But this contradicts ∂iu(y) = 0, 1 ≤ i ≤ n, and consequently y ∈ Ω and u attains its maximum at y. 3.13 (a) Consider an even solution U. Then U0(0) = 0, and as U00 < 0, we get U0 < 0 in (0,1). Whence U satisfies

U00 − KU0 + 1 = 0 in (0,1).

The solution of this equation is of the form t U(t) = αeKt + β + . K Using that U0(0) = 0, U(1) = 0 and the fact that U is even, we deduce

1 U(t) = (eK − K − eK|t| + K|t|), t ∈ [−1,1]. K2 This solution is clearly of class C2 because Solutions of Exercises and Problems 221

x2 |x| j e|x| − |x| = 1 + + ∑ . 2! j≥3 j!

(b) Since Lu = u00 + pu0 = −1 < 0, the maximum principle entails that u attains its minimum in [−1,1] at x = ±1. Hence u ≥ 0. On the other hand,

L(U − u) ≤ (−K|U0| + pU0) ≤ 0.

Then the maximum principle enables us to assert that U −u attains its null minimum at x = ±1 and hence u ≤ U. If U1 and U2 are two solutions of the non linear equation, we apply the preceding results with p = Ksgn(U1) and U = U2. We conclude that U1 ≤ U2. Interchanging the roles of U1 and U2, we obtain U2 ≤ U1 and therefore U1 = U2. 3.14 We extend by reflexion u to an odd function that we still denote by u. This 2 2 2 extension belongs to C (R ), satisfies ∆u = 0 in R and |u(x)| ≤ c1 +c2|x|. If Br(x) denotes the ball of radius r and center x then by Lemma 3.11, we have

2 2 −2 max|∂i ju(x)| ≤ max sup |∂i ju| ≤ Cr sup |u| i, j i, j Br(x) B2r(x) −2 ≤ Cr (c1 + 2c2r), for all r > 0.

2 Letting r → +∞, we deduce ∂i ju = 0, for each i and j. In consequence, u is of the form u(x1,x2) = a + bx1 + cx2. Using that u = 0 on ∂Ω, we end up getting u = 0. 3.15 (a) As Γ (x) → +∞ when x → 0, we get, for any ε > 0, that there exists δ > 0 so that |u − v| ≤ εΓ sur Bδ \{0}.

(b) In light of the fact that v = u sur ∂Br, we obtain from (a)

v− = v − εΓ ≤ u ≤ v+ = v + εΓ , sur ∂(Br \ Bδ ).

We then deduce, by applying the maximum principle, that v− ≤ u ≤ v+ on Br \ Bδ . That is |u − v| ≤ εΓ , on Br \ Bδ . (c) We make successively in the last inequality δ → 0 and then ε → 0. We get u = v in Br \{0}. In other words, v is the extension of u in the whole B. 3.16 (a) We easily check that

2 2 2 ∂11v = [4µ(µ − 1)x1 − 2µ(1 − x1)]w, 2 2 ∂12v = −2µλx1(1 − x1)tanh(λx2)w, 2 2 2 2 ∂22v = λ (1 − x1) w 222 Solutions of Exercises and Problems

2 µ−2 with w = (1 − x1) cosh(λx2). Under the notations

2 t = tanh(λx2), ξ1 = −2tµx1, ξ2 = λ(1 − x1), we have

2 2 2 2 2 2 2 ∂11v = [ξ1 + 4µ (1 −t )x1 − 4µx1 − 2µ(1 − x1)]w, 2 ∂12v = ξ1ξ2w, 2 2 ∂22v = ξ2 w.

Hence

 2 2 2 2 2  Lv = a(ξ1,ξ2) · (ξ1,ξ2) + 4µ (1 −t )x1a11 + [−4µx1 − 2µ(1 − x1)]a11 w.

We obtain from the ellipticity condition

 2 2 2 2 2 −1 2 −1 2  Lv ≥ ν(ξ1 + ξ2 ) + 4νµ (1 −t )x1 − 4ν µx1 − 2ν µ(1 − x1) w  2 2 2 2 2 2 2 2 2 −1 2 −1 2  ≥ 4νµ t x1 + νλ (1 − x1) + 4νµ (1 −t )x1 − 4ν µx1 − 2ν µ(1 − x1) w  2 2 2 2 2 −1 2 −1 2  ≥ 4νµ x1 + νλ (1 − x1) − 4ν µx1 − 2ν µ(1 − x1) w.

2 −1 2 2 −1 Set r = 1 − x1 and R(µ,λ,r) = 4µ(νµ − ν )(1 − r) + νλ r − 2ν µr. Then there exists µ∗ = µ∗(ν) sufficiently large in such a way that 1 R(µ∗,λ,r) ≥ R(µ∗,0,r) ≥ 0, for any 0 ≤ r ≤ and λ. 2 Furthermore, there exists λ ∗ = λ ∗(ν) so that 1 R(µ∗,λ ∗,r) ≥ 0, for any ≤ r ≤ 1. 2 ∗ ∗ We then choose µ = µ and λ = λ . We obtain Lv ≥ R(µ,λ,r)w ≥ 0 in (−1,1)×R. (b) We prove that

−(1+α) Mr ≤ 2 M2r for 0 < 2r < R (5.19) entails 2r 1+α M ≤ M for 0 < 2r < R. (5.20) r R R For all 0 < r ≤ R, there exists a non negative integer k so that R/2 < 2kr ≤ R. If k = 0, we have R < 2r and (5.20) is obvious. If k ≥ 1, we obtain by iterating (5.19)

−(1+α) −2(1+α) −k(1+α) −k(1+α) Mr ≤ 2 M2r ≤ 2 M22r ≤ ...2 M2kr ≤ 2 MR. (5.21) Solutions of Exercises and Problems 223

In light of the inequality R/2 < 2kr, we deduce that 2−k < 2r/R and hence 2−k(1+α) ≤ (2r/R)1+α . We obtain the expected result by combining the last inequality and the right hand side of the third inequality in (5.21). 0 (c) Let w = U(x1,x2)−cM4v(x1 −x2), (x1,x2) ∈ Ω . We have Lw ≤ 0 in Ω4, and since w ≥ 0 on ∂Ω 0, we get w ≥ 0 in Ω 0 by applying the maximum principle. In particular, we have

−1 2 M4 ± u(2,x2) = U(2,x2) ≥ cM4 for |x2| ≤ 2.

−1 −1−α −1−α For α = α(ν) given by 2 − c = 2 , we obtain then that 2 M4 ± u ≥ 0 on −1−α ∂Ω2 (Note that u = 0 in {|x2| = x1}). The maximum principle yields ±u ≤ 2 M4 in Ω2 and hence −1−α M2 = sup|u| ≤ 2 M4. Ω2 3.17 (a) Since L(M − u) = 0, there exists by Harnack’s inequality (Theorem 2.8) C = C(n,ν,K,r) so that

sup(M − u) ≤ C inf(M − u) ≤ C(M − u(x0)). B Br r

b) Assume that M1 > 0 and fix k ≥ 1 an integer. Pick then r > k|z|. We apply (a) to v and M = M1 in an arbitrary B2r = B(x0,2r). We obtain

sup(M1 − v) ≤ C1(M1 − v(x0)), Br

where C1 = C1(n,ν,K) is a constant. Choose now x0 in such a way that C1(M1 − v(x0)) ≤ M1/2. This choice entails 1 v ≥ M in B . 2 1 r

Using x0 + jz ∈ Br, 0 ≤ j ≤ k, we obtain

k u(x0 + (k + 1)z) − u(x0) = ∑[u(x0 + ( j + 1)z) − u(x0 + jz)] j=0 k k + 1 = ∑ v(x0 + jz) ≥ M1. j=0 2

The right hand side of the last inequality tends to +∞, when k → +∞, contradicting that u is bounded. Therefore M1 ≤ 0. In the preceding results substituting z by −z we can see that we have also

sup[u(x − z) − u(x)] ≤ 0. Rn But 224 Solutions of Exercises and Problems

sup[u(x − z) − u(x)] = sup[u(x) − u(x + v)] = sup(−v) = −infv = −m . n 1 Rn Rn Rn R

Thus m1 ≥ 0 entailing that M1 = m1 = 0. (c) As u is periodic, we have

M = supu = u(x0), Rn n for some√ x0 ∈ Q = [0,1) . Let Br the ball of center 0 and radius r. Since Q ⊂ Br for r ≥ n, we can apply again (a) to deduce

0 ≤ M − u ≤ C(M − u(x0)) = 0 in Br.

Hence u = M in Q ⊂ Br and, using again that u is periodic, we conclude that u = M n in R .

3.18 (a) In light of the analyticity of u in B1, we find a ball Bδ = B(0,δ) in such a way that u coincide in Bδ with its Taylor series at the origin, i.e.

u(x) = ∑ Pk(x), x ∈ Bδ , k≥0 where, for each k, Pk is homogenous polynomial of degree k. Whence

0 = ∆u(x) = ∑ ∆Pk(x), x ∈ Bδ . k≥0

This entails that ∆Pk = 0 for each k. (b) (We provide a direct proof. However we can also adapt the proof of Exercice −1 3.5) For r ∈ (0,1), the exterior normal vector at x ∈ ∂Br is nothing but ν = r x. k From Pk(λx) = λ P(x), k ≥ 0, we get

−1 −1 k Pk(x + λν) = P((1 + λr )x) = (1 + λr ) Pk(x) and consequently d k ∂ν Pk(x) = P(x + λν) = Pk(x). dλ λ=0 r We obtain then, by applying the divergence’s theorem with k,` ≥ 0, Z Z ` − k Z 0 = (Pk∆P` − P`∆Pk)dx = (Pk∂ν P` − P`∂ν Pk)dσ = PkP`dσ. Br ∂Br r ∂Br R P P d = k 6= ` Then ∂Br k ` σ 0 if and hence

Z Z 1 Z PkP`dx = dr PkP`dσ = 0, if k 6= `. B1 0 ∂Br Solutions of Exercises and Problems 225

(c) (i) The existence of harmonic polynomial of degree k so that hk = pk in ∂Br is guaranteed by Lemma 3.12. By the maximum principle we have (u is also harmonic)

sup|u − hk| = sup|u − hk| = sup|u − pk| ≤ sup|u − pk| → 0 when k → +∞, Br ∂Br ∂Br Br and then ku − h k 2 → k → + . k L (Br) 0 when ∞ (5.22) 2 (ii) As u − Sk is orthogonal in L (Br) to Sk − hk ∈ Ek, we find

2 2 2 ku − h k 2 = ku − S k 2 + kS − h k 2 . k L (Br) k L (Br) k k L (Br) This and (5.22) imply

ku − S k 2 ≤ ku − h k 2 → k → + . k L (Br) k L (Br) 0 when ∞

n (d) Fix 0 < r < R < 1 and let ϕ ∈ D(R ) so that Z n ϕ ≥ 0 in R , ϕ = 0 for |x| ≥ 1, ϕ(x)dx = 1.

ε −n −1 Note that we can choose ϕ = ϕ0(|x|). For 0 < ε < R−r, define ϕ (x) = ε ϕ(ε x). We have, according to the properties of harmonic functions in Lemma 3.11, Z (ε) ε ε u(x) = u (x) = u ∗ ϕ (x) = u(y)ϕ (x − y)dy, x ∈ Br, BR and a similar formula holds for Sk. We deduce by applying Cauchy-Schwarz’s in- equality

Z ε ε sup|u − S | = sup (u(y) − S (y))ϕ (x − y)dy ≤ kϕ k 2 n ku − S k 2 . k k L (R ) k L (BR) Br Br BR

We end up getting that ku − S k 2 → 0, as k → + , implies k L (BR) ∞

sup|u − Sk| → 0 as k → +∞. BR

Exercises and Problems of Chapter 4

4.1 (a) We have ∆v = 2u∆u + 2|∇u|2 = 2|∇u|2 ≥ 0. We obtain, by applying Harnak’s inequality for sub-solutions (Theorem 2.20), that

kvkL∞(B(0,r)) ≤ CkvkL2(B(0,2r)). 226 Solutions of Exercises and Problems

(b) Form (a), we get

2 2 kukL∞(B(0,r)) = kvkL∞(B(0,r)) ≤ Cku kL2(B(0,2r)) ≤ CkukL∞(B(0,2))kukL2(B(0,2r)). Hence kukL∞(B(0,r)) ≤ Cu,ε kukL2(B(0,2r)), 1/2 ≤ r ≤ 1, (5.23) where Cu is given by 2 kukL∞(B(0,2)) Cu = C . kukL∞(B(0,ε))

Note that according the uniqueness of continuation kukL∞(B(0,ε)) 6= 0. (c) In light of (5.23), the doubling inequality entails

kukL∞(B(0,r)) ≤ CukukL2(B(0,r)), ε ≤ r ≤ 1, (5.24)

(d) Fix ε > 0 so that kukL∞(B(0,ε)) ≤ 2|u(0)|. (5.25) By the second mean-value identity n Z u(0) = udx |B(0,ε)| B(0,ε) and hence n |u(0)| ≤ kuk 2 . |B(0,ε)|1/2 L (B(0,ε)) This in (5.25) yields 2n kuk ∞ ≤ kuk 2 . (5.26) L (B(0,ε)) |B(0,ε)|1/2 L (B(0,ε))

A combination of (5.24) and (5.26) gives the expected inequality.

4.2 (a) For u = u1 − u2, we have

∆u = 0 in Ω0, u = 0 on Γ , ∂ν u = 0 on γ.

We get, by applying Corollary 2.5, that u = 0 in Ω0. As −∆u+u = 0 in D0 and u = ∂ν u = 0 in S, we obtain, by using again Corollary 2.5, that u = 0 in D0. 2 (b) (i) By (a), u = 0 in Ω0 ∪D0. In particular, u = ∂ν u on ∂ω. Whence u ∈ H0 (ω). We have −∆u2 + u2 = 0 and ∆u1 = 0 in ω. In consequence,

∆u = u2 in ω.

∞ Now, as u,u2 ∈ C (ω), we can apply ∆ to each member of the last identity. We obtain 2 ∆ u = ∆u2 = ∆u2 − ∆u1 = ∆u in ω. Solutions of Exercises and Problems 227

(ii) From the divergence theorem, we get Z Z Z Z Z 2 2 ∆uudx = − ∆ uudx = (∆u) dx − ∂ν ∆uudσ(x) + ∆u∂ν udσ(x) ω ω ω ∂ω ∂ω Z = (∆u)2dx, ω Z Z Z Z 2 2 ∆uudx = − |∇u| dx + ∂ν uudσ(x) = − |∇u| dx. ω ω ∂ω ω

Hence Z Z (∆u)2dx + |∇u|2dx = 0. ω ω 2 Therefore ∇u = 0 in ω. Thus u = 0 in ω because u ∈ H0 (ω). If ω 6= /0then u = 0 in ω would entail that u2 = 0 in ω and hence u2 = 0 in Ω by Theorem 2.16. In consequence, we would have ϕ = 0 which is impossible. (c) By (b), we can not have neither D2 \ D1 6= /0nor D1 \ D2 6= /0.We end up getting that D1 = D2. 4.3 (a) A simple change of variable yields Z H(r) = σ(ry)u2(ry)rn−1dS(y). S(1)

Hence n − 1 Z H0(r) = H(r) + ∇(σu2)(ry) · yrn−1dS(y) r S(1) n − 1 Z Z = H(r) + u2∇σ(ry) · yrn−1dS(y) + σ∇(u2)(ry) · yrn−1dS(y) r S(1) S(1) n − 1 Z Z = H(r) + u2∇σ(x) · ν(x)dS(x) + σ(x)∇(u2)(x) · ν(x)dS(x) r S(r) S(r) n − 1 Z = H(r) + H˜ (r) + σ∇(u2)(x) · ν(x)dS(x). r S(r)

Identity (4.88) will follow if we prove that Z 2D(r) = σ∇(u2)(x) · ν(x)dS(x). S(r)

Since div(σ∇u) = βu, we get

div(σ∇(u2)) = 2udiv(σ∇u) + 2σ|∇u|2 = 2σ|∇u|2 + 2βu2.

We obtain by applying the divergence theorem Z Z 2D(r) = div(σ(x)∇(u2)(x))dx = σ(x)∇(u2)(x) · ν(x)dS(x). (5.27) B(r) S(r) 228 Solutions of Exercises and Problems

By a change of variable, we have Z r Z D(r) = σ(ty)|∇u(ty)|2 + β(ty)u2(ty) tn−1dS(y)dt. 0 S(1)

Hence Z D0(r) = σ(ry)|∇u(ty)|2 + β(ry)u2(ry) rn−1dS(y) S(1) Z Z = σ(x)|∇u(x)|2dS(x) + β(x)u2(x)dS(x) S(r) S(r) 1 Z Z = σ(x)|∇u(x)|2x · ν(x)dS(x) + V(x)u2(x)dS(x). r S(r) S(r)

Then an application of the divergence theorem gives 1 Z Z D0(r) = div(σ(x)|∇u(x)|2x)dx + β(x)u2(x)dS(x). r B(r) S(r)

Therefore 1 Z 1 Z D0(r) = |∇u(x)|2div(σ(x)x)dx + σ(x)x · ∇(|∇u(x)|2)dx r B(r) r B(r) Z + β(x)u2(x)dS(x) S(r) implying Z 0 n 1 1 2 D (r) = D(r) + De(r) + σ(x)x · ∇(|∇u(x)| )dx + Hˆ (r). (5.28) r r r B(r)

On the other hand, Z Z 2 2 σ(x)x j∂ j(∂iu(x)) dx = 2 σ(x)x j∂i ju∂iu(x)dx B(r) B(r) Z = −2 ∂i [∂iu(x)σ(x)x j]∂ ju(x)dx B(r) Z + 2 σ(x)∂iu(x)x j∂ ju(x)νi(x)dS(x) S(r) Z 2 = −2 ∂ii u(x)σ(x)x j∂ ju(x)dx B(r) Z − 2 ∂iu(x)∂ ju(x)∂i [σ(x)x j]dx B(r) Z + 2 σ(x)∂iu(x)x j∂ ju(x)νi(x)dS(x). S(r)

Thus, taking into account that σ∆u = −∇σ · ∇u + βu, Solutions of Exercises and Problems 229 Z Z σ(x)x · ∇(|∇u(x)|2)dx = −2 σ(x)|∇u(x)|2dx B(r) B(r) Z − 2 β(x)u(x)x · ∇u(x)dx B(r) Z 2 + 2r σ(x)(∂ν u(x)) dS(x). S(r)

This identity in (5.28) yields n − 2 1 Z D0(r) = D(r) + D˜ (r) − 2 β(x)u(x)x · ∇u(x)dx r r B(r) n − 2 Z − β(x)u2(x)dx + 2H(r) + Hˆ (r). r B(r)

That is we proved (4.89). (b) (i) Assume that β ≥ 0. Since 1 Z H(r) = σ(x)u2(x)x · ν(x)dS(x), r S(r) we get by applying the divergence theorem 1 Z H(r) = divσ(x)u2(x)xdx. (5.29) r B(r)

Hence 1 1 Z H0(r) = − H(r) + divσ(x)u2(x)xdS(x) r r S(r) Z Z n − 1 2 = H(r) + ∂ν σ(x)u (x)dS(x) + 2 σ(x)∂ν u(x)u(x)dS(x). r S(r) S(r)

But Z Z Z 2 σ(x)∂ν u(x)u(x)dS(x) = div(σ(x)∇u(x))u + σ(x)|∇u| dx S(r) B(r) B(r) Z = σ(x)|∇u(x)|2 + β(x)u2(x) dx = D(r). B(r)

Therefore Z 0 n − 1 2 H (r) = H(r) + 2D(r) + ∂ν σ(x)u (x)dS(x) r S(r) Z 2 σ1 ≥ ∂ν σ(x)u (x)dS(x) ≥ − H(r), S(r) σ0 where we used that H(r) ≥ 0 and D(r) ≥ 0. 230 Solutions of Exercises and Problems

rκ˜ Consequently, r → e H(r) is non decreasing, where κ˜ = σ1/σ0. We obtain from this Z r Z r Z r rn H(t)tn−1dt ≤ etκ˜ H(t)tn−1dt ≤ erκ˜ H(r)tn−1dt ≤ erκ˜ H(r). 0 0 0 n As Z r K(r) = H(t)dt, 0 we end up getting eκ˜ K(r) ≤ H(r). n (ii) Assume that σ = 1. Using Green’s formula, we get in a straightforward man- ner Z Z Z ∆(u2)(x)(r2 − |x|2)dx = −2n u2(x)dx + 2r u2(x)dS(x). (5.30) B(r) B(r) S(r)

But ∆(u2) = 2u∆u + 2|∇u|2 = 2βu2 + 2|∇u|2. Thus Z Z ∆(u2)(x)(r2 − |x|2)dx = 2 β(x)u2(x) + 2|∇u(x)|2 (r2 − |x|2)dx B(r) B(r) (5.31) Z ≥ 2 β(x)u2(x)(r2 − |x|2)dx. B(r)

(5.31) in (5.30) yields Z rH(r) ≥ (n − β(x)(r2 − |x|2))u2(x)dx. (5.32) B(r)

Since 2 2 2 n − β(x)(r − |x| ) ≥ n − β0r0 = 1, we obtain Z 2 rH(r) ≥ u (x)dx = K(r), 0 < r ≤ r0. B(r) (c) (i) Assume that β = 0. By formulas (4.88) and (4.89) and identity (4.90), we have

N0(r) D˜ (r) He(r) H(r) D(r) = − + 2 − 2 (5.33) N(r) D(r) H(r) D(,r) H(r) D˜ (r) H˜ (r) H(r)H(r) − D(r)2 = − + 2 . D(r) H(r) D(r)H(r)

But, in light of (5.27), we have Solutions of Exercises and Problems 231 Z D(r) = σ(x)u(x)∂ν u(x)dS(x). S(r)

We find by applying Cauchy-Schwarz’s inequality

Z Z  2 2 2 D(r) ≤ σ(x)u (x)dS(x) σ(x)(∂ν u) (x)dS(x) = H(r)H(r). S(r) S(r) (5.34) This and (5.33) lead N0(r) D˜ (r) H˜ (r) ≥ − . (5.35) N(r) D(r) H(r) On the other hand, k∇σk∞ σ1 H˜ (r) ≤ H(r) ≤ H(r), (5.36) σ0 σ0 and similarly σ1 D˜ (r) ≤ dD(r). (5.37) σ0 We get from (5.35), (5.36) and (5.37)

N0(r) ≥ −κ, N(r) that is to say (eκrN(r))0 ≥ 0. Consequently N(r) ≤ eκ(r−r)N(r) ≤ eκrN(r). (ii) We first note that (5.34) remains true without any condition. On the other hand, D˜ = 0 when σ = 1 and (5.37) is clearly satisfied if β ≥ 0. Therefore, we need only to estimate the terms Hˆ (r)/D(r) and Dˆ (r)/D(r). Set I = {r ∈ (0,δ); N(r) > max(N(r0),1)}. Observe that H(r) 6= 0 for any r ∈ (0,1). Otherwise, we would have H(r) = 0 for some r ∈ (0,1) and therefore u would be identically equal to zero by the unique continuation property. Hence N is continuous and I is an open subset of R. Con- sequently, I is a countable union of open intervals:

∞ [ I = (ri,si). i=1

Let c = eσ1/σ0 /n if β ≥ 0 and c = 1 if σ = 1. On each (ri,si), in light of the fact that N(r) > 1, we have H(r) < rD(r). Hence

Hˆ (r) H(r) ≤ β0 ≤ β0c. (5.38) D(r) D(r) 232 Solutions of Exercises and Problems

Also,   Z n − 2 Dˆ (r) ≤ β0 2r |u(x)||∇u(x)|dx + K(r) B(r) r  Z Z  2 2 n − 2 ≤ β0 r u (x)dx + r |∇u(x)| dx + K(r) B(r) B(r) r  Z  2 n − 2 ≤ β0 rD(r) − r β(x)u dx + K(r) B(r) r   n − 2  ≤ β rD(r) + β + K(r) . 0 0 r

From (b), we get     n − 2 Dˆ (r) ≤ β0 rD(r) + β0 + c H(r) . r

Combined with H(r) < rD(r), this estimate yields

Dˆ (r) ≤ β0 (1 + β0 + c(n − 2)). (5.39) D(r) In light of the previous comments, (5.38) and (5.39) we have

N0(r) ≥ −c0, N(r)

0 the constant c only depends on Ω, σ1/σ0 and β0. Hence

0 0 c s j c r0 N(r) ≤ e N(s j) ≤ e max(N(r0),1), r ∈ (r j,s j).

Then N(r) ≤ C max(N(r0),1), r ∈ I , for some constant C > 0, only depending on Ω, σ1/σ0 and β0. The proof is com- pleted by noting that N ≤ max(N(r0),1) on (0,r0) \ I . 4.4 (a) (i) Since n < 4, H2(Ω) is continuously imbedded in Lp(Ω) for 1 ≤ p < ∞. By Theorem 4.11, we have φ ∈ W 2,p(Ω) and

kφkW 2,p(Ω) ≤ CλkφkLp(Ω)

≤ CλkukH2(Ω)k 2 ≤ Cλ kφkL2(Ω).

2 Here we used the following H a priori estimate: kwkH2(Ω) ≤ CΩ k∆ukL2(Ω) if w ∈ H2(Ω). Choose p > 1 so that 2 − n/p = 1 + α, i.e. p = n/(1 − α). In that case, W 2,p(Ω) is Solutions of Exercises and Problems 233 continuously imbedded in C1,α (Ω). Therefore

2 kφkC1,α (Ω) ≤ Cλ kφkL2(Ω).

2 q (ii) Assume that 4 ≤ n < 8. As H (Ω) is continuously imbedded in L 0 (Ω), q0 = 2n/(n − 4), 4 < n < 8. Also, as H2(Ω) is continuously embedded in W 2,p(Ω), 1 < 2 q p < 2, we deduce that H (Ω) is continuously imbedded in L 0 (Ω), q0 = 2p/(2− p), for some fixed 1 < p < 2, when n = 4. Then it follows from Theorem 4.11 that φ ∈ W 2,q0 (Ω) and

q kφkW 2,q0 (Ω) ≤ CλkφkL 0 (Ω) 2 ≤ Cλ kφkL2(Ω).

But 2 − n/q0 = 4 − n/2 > 0 if 4 < n < 8 and 2 − n/q0 = 4(p − 1)/p if n = 4. Hence, W 2,q0 (Ω) is continuously imbedded in Lp(Ω) for 1 < p < ∞. We deduce by repeating the argument in (i) that φ ∈ C1,α (Ω) and

3 kφkC1,α (Ω) ≤ Cλ kφkL2(Ω). (iii) In light of (ii), we can make an induction argument to deduce that if n = 4 j +`, with m ≥ 1 and ` ∈ {0,1,2,3}, then

2+ j kφkC1,α (Ω) ≤ Cλ kφkL2(Ω). That is we have m(n) kφkC1,α (Ω) ≤ Cλ kφkL2(Ω). where m(n) − 2 is the unique non negative integer j so that n/4 − j ∈ [0,1). (b) (i) Similarly to the proof of (4.81), there exists E0 ⊂ D so that for 0 < ε < 1 and v ∈ H2(D)

β c/ε   Ckvk 1 ≤ ε kvk 1,α + e kvk 1 + k∆vk 2 . H (D) C (D) H (E0) L (D)

Combined with Caccioppoli’s inequality this inequality yields, where F0 c E0,

β c/ε   Ckvk 1 ≤ ε kuk 1,α + e kvk 2 + k∆vk 2 . (5.40) H (D) C (D) L (F0) L (D)

On the other hand, we have from Proposition 4.5

γ −1   Ckvk 2 ≤ ε kvk 2 + ε kuk 2 + k∆vk 2 , ε > 0. L (F0) 1 L (D) 1 L (E) L (Ω) 1

−c/(γε) β/γ Taking in this inequality ε1 = e ε , we find

−c/ε β c/(γε) −β/γ   Ckvk 2 ≤ e ε kvk 2 + e ε kuk 2 + k∆vk 2 . L (F0) L (D) L (E) L (Ω) 234 Solutions of Exercises and Problems

This and (5.40) yields the expected inequality.√ (ii) We apply (i) with v(x,t) = u(x)e λt , (x,t) ∈ D = Ω × (0,1) and E = ω × (0,1). Noting that √ √ λt λ λ 1 ≤ ke kL2(0,1) ≤ e and kvkC1,α (D) ≤ κe kukC1,α (Ω) for some universal constant κ, we deduce from (4.93) that (4.94) holds for v. (c) (i) Follows immediately from (4.94) with u = φ and the estimate in (a) (iii). (ii) It is straightforward to check that (4.95) implies the following inequality √ Ce−k λ ≤ εβ + ec/ε ℵ, 0 < ε < 1, (5.41) where k = 2(1 + m) and kφkL2( ) ℵ = ω . kφkL2(Ω) If ℵ < e−c, we find 0 < ε < 1 so that εβ e−c/ε = ℵ. This particular choice of ε in (5.41) yields √ 1 Ce−k λ ≤ . (−lnℵ)β The expected inequality then follows. When ℵ ≥ e−c the expected inequality is obviously satisfied. 4.5 (a) Using the following inequalities √ √ λ/4 λt λt λ e ≤ ke kL2(1/4,3/4), ke kL2(0,1) ≤ e , √ we easily get, by applying (4.96) to v(x,t) = u(x)e λt ,

√ β 3 λ/4 1−β   kuk 1 ≤ Ce kuk k( + )uk 2 + kuk 2 . (5.42) H (Ω) H1(Ω) ∆ λ L (Ω) L (ω) b) We obtain by taking u = φ in (5.42)

√ 1−β 3 λ 1−β β k k 2 ≤ C e 4 k k k k , φ L (Ω) λ φ L2(Ω) φ L2(ω)

This inequality implies the expected one in a straightforward manner. 4.6 (a) (i) The proof is obtained by slight modifications of that of Theorem 4.8. In the sequel the notations are those of the proof of Theorem 4.8. We have Lr(χw) = χLrw + Qr(w) and 2 2 2 (χLrw) ≤ Λ˜ w + |∇w| , the constant Λ˜ only depends on Ω and Λ. Before obtaining an inequality similar to (4.48) when using Carleman inequality in Theorem 4.7, we absorb Λ˜ w2 + |∇w|2 by the left hand side by modifying λ0 and τ0 if necessary. In that case, we have an Solutions of Exercises and Problems 235 inequality similar to (4.48) in which Lr(χw) is substituted by Qr(w). The rest of the proof is quite similar to that of Theorem 4.8. (ii) We mimic the proof of Proposition 4.5 by using Theorem 4.13 instead of Theorem 4.8. (b) Follows immediately from Proposition 4.7 since (Lu)2 = g(u)2 ≤ c2u2.

Appendix A Building a fundamental solution by the parametrix method

We aim in this appendix to construct a fundamental solution of a general elliptic operators. We followed the paper by Kalf [1] where he constructed a fundamental solution using a method introduced by E. E. Levi [3]. This method consists in build- ing a fundamental solution as a perturbation of the canonical parametrix, which is roughly speaking a fundamental solution corresponding to constant coefficients el- liptic operator. The problem is then reduced to solve an integral equation with a weakly singular kernel. Proceeding in this way, the main difficulty is to guarantee an orthogonality relation appearing if one wants to use Fredholm’s alternative. We overcome this difficulty by deforming the right hand side of the integral equation in order to comply with the orthogonality condition appearing is Fredholm’s alterna- tive. The paper by Kalf [1] contains many historical comments and remarks starting from the founding paper by E. E. Levi. Some technical results we used in this appendix are borrowed from the books of R. Kress [2] and H. Triebel [4].

A.1 Functions defined by singular integrals

We start with the following technical lemma.

Lemma A.1. Let 0 ≤ αi < n, i = 0,1. There exists a constant C > 0, depending on n, Ω, α0 and α1, so that, for any x,y ∈ Ω with x 6= y, we have  1 if α0 + α1 > n,  −n+α0+α1  |x − y| Z dz  ≤ C (A.1) |x − z|α0 |z − y|α1 |ln|x − y|| + 1 if α0 + α1 = n, Ω    1 if α0 + α1 < n.

237 238 A Building a fundamental solution by the parametrix method

Proof. Let x,y ∈ Ω with x 6= y. Write z = x + |x − y|η. Hence

|x − z| = |x − y||η| and

x − y |z − y| = |z − x + x − y| = ||x − y|η + x − y| = |x − y| η + . |x − y|

x−y If e = |x−y| then the last identities yield

|z − y| = |x − y||η + e|.

We have, where d = 2diam(Ω),

Z dz 1 Z dη ≤   . |x − z|α0 |z − y|α1 |x − y|−n+α0+α1 d | |α0 | − e|α1 Ω B 0, |x−y| η η

If η ∈ B(e,1/2) then |η| ≥ |e| − |η − e| ≥ 1/2. Whence

Z dη Z dη ωn ≤ α1 ≤ −n+α0+α1 . α α 2 α 2 B(e,1/2) |η| 0 |η − e| 1 B(e,1/2) |η| 0 n

Also,

α Z dη Z dη ωn2 0 ≤ α0 ≤ n−α1 . α α 2 α 3 B(0,2)\B(e,1/2) |η| 0 |η − e| 1 B(e,3) |η − e| 1 n

 d  For η ∈ B 0, |x−y| \ B(0,2) we have |e| = 1 ≤ |η| − |e| ≤ |η − e| and hence

|η| ≤ |η − e| + |e| ≤ 2|η − e|.

Thus Z dη Z dη   ≤   d | |α0 | − e|α1 d | |α0+α1 B 0, |x−y| \B(0,2) η η B 0, |x−y| \B(0,2) η Z d |x−y| n−1−α −α ≤ ωn r 0 1 dr. 2 Since  lnd − ln2 − ln|x − y| if α0 + α1 = n, d  Z |x−y|  rn−1−α0−α1 dr = 1  dn−α0−α1  2  − n−α0−α1 + 6= n,  n−α −α 2 if α0 α1  n − α0 − α1 |x − y| 0 1 we obtain A.1 Functions defined by singular integrals 239

 |lnd − ln2| + |ln|x − y|| if α + α = n,  0 1   n− − d  d α0 α1 Z |x−y|  −n+α0+α1 n−1−α0−α1 |x − y| if α0 + α1 < n, r dr ≤ n − α0 − α1 2   n− −  2 α0 α1  if α0 + α1 > n. −n + α0 + α1 Putting together all these inequalities, we get the expected result. ut

We use hereafter the notations

D = {(x,x); x ∈ Ω}, Σ = Ω × Ω \ D.

0 Theorem A.1. (i) Let fi ∈ C (Σ), i = 0,1, satisfying

−n+βi | fi(x,y)| ≤ c|x − y| , (x,y) ∈ Σ, (A.2) for some constants c > 0 and βi ∈ (0,n), i = 0,1. Then f given by Z f (x,y) = f0(x,z) f1(z,y), (x,y) ∈ Σ. Ω belongs to C0(Σ) and

 −n+β0+β1  |x − y| if β0 + β1 < n, | f (x,y)| ≤ C (A.3)  |ln|x − y|| if β0 + β1 = n, and f can be extended by continuity in Ω × Ω when β0 + β1 > n. 1 (ii) Suppose in addition of the assumptions in (i) that f0(·,y) ∈ C (Ω \{y}) for all y ∈ Ω and −n+β |∇x f0(x,y)| ≤ κ|x − y| , for some constants κ > 0 and β > 0. Then, for any y ∈ Ω, we have f (·,y) ∈ C1(Ω \ {y}) and Z ∇x f (x,y) = ∇x f0(x,z) f1(z,y)dz, (x,y) ∈ Ω × Ω, x 6= y. Ω

(iii) Assume additionally to the assumptions in (i) that f0 satisfies the following estimate: there exists c > 0 and δ ∈]0,1] so that

δ −n −n | f0(x1,y) − f0(x2,y)| ≤ c|x1 − x2| |x1 − y| + |x2 − y| , for any x1,x2,y ∈ Ω satisfying |y − x1| ≥ 2|x1 − x2|. If µ0 = min(β0,β1) and µ1 = min(δ, µ0), then there exits C > 0 and so that 240 A Building a fundamental solution by the parametrix method

µ1 −n+µ0 −n+µ0  | f (x1,y) − f (x2,y)| ≤ C|x1 − x2| |x1 − y| + |x2 − y| , for any x1,x2,y ∈ Ω, y 6= x j, j = 1,2.

Proof. (i) Fix (x0,y0) ∈ Σ. If δ = |x0 − y0| > 0, pick then 0 < η ≤ δ/4 and ε > 0. For (x,y) ∈ Σ, we write

f (x,y) − f (x0,y0) = I0 + I1, with Z I0 = [ f0(x,z) − f0(x0,z)] f1(z,y)dz, Ω Z I1 = f0(x0,z)[ f1(z,y) − f1(z,y0)]dz. Ω

Suppose first that β0 + β1 < n. We split I0 into two terms I0 = J0 + J1, where Z J0 = [ f0(x,z) − f0(x0,z)] f1(z,y)dz, Ω∩B(x0,η) Z J1 = [ f0(x,z) − f0(x0,z)] f1(z,y)dz. Ω\B(x0,η)

Assume that |x−x0| < η and |y−y0| < δ/4. For z ∈ B(x0,η), we have |x−z| < 2η and |z − y| ≥ |x0 − y0| − |y0 − y| − |z − x0| ≥ δ/2.

Whence, where z ∈ B(x0,η),   2 −n+β0 −n+β0 −n+β1 |[ f0(x,z) − f0(x0,z)] f1(z,y)| ≤ c |x − z| + |x0 − z| |z − y|

δ −n+β1   ≤ c2 |x − z|−n+β0 + |x − z|−n+β0 2 0 and consequently

δ −n+β1 Z   2 −n+β0 −n+β0 |J0| ≤ c |x − z| + |x0 − z| dz. (A.4) 2 B(x0,η)

But B(x0,η) ⊂ B(x,2η). Hence (A.4) yields

δ −n+β1 Z Z  2 −n+β0 −n+β0 |J0| ≤ c |x − z| dz + |x0 − z| dz , 2 B(x,2η) B(x0,η) from which we deduce that there exists η0 so that, for any 0 < η ≤ η0, we have

−n+ ω c2(2β0 + 1) δ  β1 ε n β0 |J0| ≤ η ≤ . (A.5) β0 2 4 A.1 Functions defined by singular integrals 241   Let b > 0 so that Ω ⊂ B(x0,b). As f0 is uniformly continuous in Ω ∩ B(x0,η/2) ×   Ω \ B(x0,η) , there exits η1 ≤ η/2 so that ,for any |x − x0| ≤ η1 and z ∈ Ω \ B(x0,η), we have ε | f (x,z) − f (x ,z)| ≤ , 0 0 0 4ℵ where cω  5δ β1 ℵ = n b + . β1 4

If |x − x0| ≤ η1 then cε Z −n+β1 |J1| ≤ |z − y| dz (A.6) 4ℵ B(x0,b) cε Z ≤ |z − y|−n+β1 dz ≤ ε/4. 4ℵ B(y,b+5δ/4)

For η = min(η0,η1), we get by combining (A.5) and (A.6)

δ |I | ≤ ε/2, |x − x | ≤ η |y − y | < . 0 0 0 4 ∗ Proceeding similarly to I1, we obtain that there exists η so that

∗ ∗ | f (x,y) − f (x0,y0)| ≤ ε, |x − x0| ≤ η |y − y0| ≤ η .

We now consider the case β0 + β1 > n. Fix η > 0. Let (x,y) ∈ Ω × Ω, x 6= y so that |x − x0| ≤ η and |x − y| ≤ η. Then, for z ∈ B(x0,η), we have

|z − y| ≤ |z − x0| + |x0 − x| + |x − y| ≤ 3η and hence |z − y| 3η ≤ = t. |x − y| |x − y| Note that |x − y| ≤ η entails 3 ≤ t. With x − y u = , |x − y| the substitution z = y + |x − y|w yields Z I = |x − z|−n+β0 |y − z|−n+β1 dz B(x0,η) Z ≤ |x − y|−n+β0+β1 |u − w|−n+β0 |w|−n+β1 dw. B(0,t)

We decompose the last integral into three terms 242 A Building a fundamental solution by the parametrix method Z Z Z Z {...} = {...} + {...} + {...}. B(0,t) B(0,1/2) B(0,2)\B(0,1/2) B(0,t)\B(0,2)

The first term in right hand side of this identity is estimated as follows Z Z |u − w|−n+β0 |w|−n+β1 dw ≤ 2n−β0 |w|−n+β1 dw B(0,1/2) B(0,1/2) 2−n+β0+β1 ω = n . β1 We have similarly for the second term Z Z |u − w|−n+β0 |w|−n+β1 dw ≤ 2n−β1 |u − w|−n+β0 dw B(0,2)\B(0,1/2) B(0,2)\B(0,1/2) 2−n+β1 3β0 ω = n . β0 While for the third term we find Z Z |u − w|−n+β0 |w|−n+β1 dw ≤ 2n−β1 2n−β0 |w|−n+β0+β1 dw B(0,t)\B(0,2) B(0,t)\B(0,2) 2n+β0t−n+β0+β1 ω = n β0 + β1 − n 2n+β0 (2η)−n+β0+β1 ω = n . −n+β +β (β0 + β1 − n)|x − y| 0 1 We find by collecting all these inequalities

−n+β0+β1 I ≤ Cη , |x − x0| ≤ η/2, |y − x0| ≤ η/2, where the constant C only depends on n, β0 and β1. This shows that in particular I0 → 0 when (x,y) → (x0,x0). On the other hand, since (A.6) still holds when δ/4 is substituted by |y − x0|, we prove analogously that I1 → 0 when (x,y) → (x0,x0). We proceed now to the proof of (ii). Fix x0 ∈ Ω \{y}. Let δ = |x0 − y| and n 0 < η ≤ δ/4. Denote the canonical basis of R by (e1,...,en). For |t| ≤ η we have Z f (x0 +tei,y) − f (x0,y) −t ∂xi f0(x0,z) f1(z,y)dz = I0 + I1, Ω with Z I0 = [ f0(x0 +tei,z) − f0(x0,z) −t∂xi f0(x0,z)] f1(z,y)dz, Ω∩B(x0,η) Z I1 = [ f0(x0 +tei,z) − f0(x0,z) −t∂xi f0(x0,z)] f1(z,y)dz. Ω\B(x0,η)

If z ∈ B(x0,η) then A.1 Functions defined by singular integrals 243

Z 1 f (x0 +tei,z) − f (x0,z) −t∂xi f0(x0,z) = t [∂xi f (x0 + stei,z) − ∂xi f0(x0,z)]ds. 0 Whence

| f0(x0 +tei,z) − f0(x0,z) −t∂xi f0(x0,z)| Z 1 ≤ |t| (|∂xi f0(x0 + stei,z)| + |∂xi f0(x0,z)|)ds 0 Z 1  −n+β −n+β  ≤ |t|κ |(x0 + stei) − z| + |x0 − z| ds. 0

Noting that B(x0,η) ⊂ B((x0 + tsei,2η) and |z − y| ≥ η/2, we get similarly to I1 that there exits η0 ≤ δ/4 so that, for any |t| ≤ η0, we have

|I0| ≤ |t|ε/2.

On the other hand, using the continuity of ∂xi f , we can mimic the proof used for estimating I1. We find η1 > 0 so that, for |t| ≤ η1, we have

|I1| ≤ |t|ε/2.

In light of the last two inequalities, we can assert that ∂xi f (x0,y) exists and Z ∂xi f (x0,y) = ∂xi f0(x0,z) f1(z,y)dz. Ω The proof of (ii) is then complete. Next, we proceed to the proof of (iii). Let x1,x2 ∈ Ω, x1 6= x2, d = |x1 − x2| and y ∈ Ω \{x1,x2}. Then Z Z  f (x1,y) − f (x2,y) = + [ f0(x1,z) − f0(x2,z)] f1(z,y)dz. Ω\B(x1,2d) Ω∩B(x1,2d) We deduce from this identity

δ | f (x1,y) − f (x2,y)| ≤ C(|x1 − x2| J0 + J1, (A.7) with Z  −n −n J0 = |x1 − z| + |x2 − z| | f1(z,y)|dz, Ω\B(x1,2d) Z h i −n+β0 −n+β0 J1 = |x1 − z| + |x2 − z| | f1(z,y)|dz. Ω∩B(x1,2d) Define 244 A Building a fundamental solution by the parametrix method

Λ0 = {z ∈ Ω \ B(x1,2d); 2|y − z| ≥ |x1 − y|},

Λ1 = {z ∈ Ω \ B(x1,2d); 2|y − z| < |x1 − y|}.

Let R = diam(Ω). Then Z Z −n −n −n+β1 |x1 − z| | f1(z,y)| ≤ c |x1 − z| |z − y| Λ0 Λ0 Z R dr n−β1 −n+β1 ≤ c2 ωn|x1 − y| 2d r  R  ≤ c2n−β1 ω |x − y|−n+β1 ln . n 1 2d

On the other hand, since 2|z − x1| > |x1 − y| for z ∈ Λ1, we get Z Z −n −n −n+β1 |x1 − z| | f1(z,y)|dz ≤ c |x1 − z| |z − y| Λ1 Λ1 Z n −n −n+β1 ≤ c2 |x − y1| |z − y| dz B(y,|x1−y|/2) 2n+β1 ω n −n+β1 ≤ c |x1 − y| β1 and hence Z −n −n+β1 |x1 − z| | f1(z,y)|dz ≤ C|x1 − y| . Ω\B(x1,2d) We have similarly Z −n −n+β1 |x2 − z| | f1(z,y)|dz ≤ C|x1 − y| . Ω\B(x1,2d)

Whence   −n+β1 −n+β1 J0 ≤ C |x1 − y| + |x2 − y| . (A.8)

We now estimate J1. Define for this purpose

Σ0 = {z ∈ Ω ∩ B(x1,2d); 2|y − z| ≥ |x1 − y|},

Σ1 = {z ∈ Ω ∩ B(x1,2d); 2|y − z| < |x1 − y|}.

We have Z Z −n+β0 −n+β0 −n+β1 |x1 − z| | f1(z,y)| ≤ c |x1 − z| |z − y| dz Σ0 Σ0 Z n−β1 −n+β1 −n+β0 ≤ c2 |x1 − y| |x1 − z| dz B(x1,2d) −n+β1 β0 β0 −n+β1 ≤ C|x1 − y| d = C|x1 − x2| |x1 − y| .

As before, using 2|z − x1| > |x1 − y| and |y − z| ≤ 2d for z ∈ Σ1, we obtain A.2 Weakly singular integral operators 245 Z Z −n+β0 −n+β0 −n+β1 |x1 − z| | f1(z,y)| ≤ c |x1 − z| |z − y| dz Σ1 Σ1 Z n−β0 −n+β0 −n+β0 ≤ c2 |x1 − y| |y − z| dz B(y,2d)

−n+β0 β1 ≤ C|x1 − y| |x1 − x2| .

Doing the same with x1 substituted by x2 and noting that B(x1,2d) ⊂ B(x2,3d), we end up getting

µ0 −n+µ0 −n+µ0  J1 ≤ C|x1 − x2| |x1 − y| + |x2 − y| . (A.9)

The expected inequality follows by combining (A.7), (A.8) and (A.9). ut

A.2 Weakly singular integral operators

Let K : Ω ×Ω → C be a measurable function. Consider the integral operator acting on L2(Ω) as follows Z (A f )(x) = K(x,y) f (y)dy. (A.10) Ω The function K is usually called the kernel of the operator A.

Lemma A.2. Assume that K, the kernel of the operator A given by (A.10), have the property that for any f ∈ L2(Ω) it holds Z Z |K(x,y)|| f (y)|dy < ∞, |K(y,x)|| f (y)|dy < ∞ a.e. x ∈ Ω Ω Ω and Z Z |K(·,y)|| f (y)|dy ∈ L1(Ω), |K(y,·)|| f (y)|dy ∈ L1(Ω). Ω Ω Suppose furthermore that A ∈ B(L2(Ω)). Then the adjoint of A is given by Z (A∗g)(x) = K(y,x)g(y)dy. Ω Proof. Let φ ∈ D(Ω). In light of the assumptions on K, an application of Fubini’s theorem allows us to get

Z Z  Z Z  (Aφ|g) = K(x,y)φ(y)dy g(x)dx = K(x,y)g(x)dx φ(y)dy. Ω Ω Ω Ω

But Z (Aφ|g) = (φ|A∗g) = φ(y)(A∗g)(y)dy. Ω Hence 246 A Building a fundamental solution by the parametrix method Z Z Z  φ(y)(A∗g)(y)dy = K(x,y)g(x)dx φ(y)dy. Ω Ω Ω The result then follows by applying the cancellation theorem. ut

If the kernel K of the operator A given by (A.10) is of the form

B(x,y) K(x,y) = , |x − y|α for some complex-valued function B ∈ L∞(Ω × Ω) and 0 < α < n, we say that A is a weakly singular integral operator. The following lemma will be useful in sequel. Lemma A.3. Let 0 < α < n. Then there exists a constant C > 0, depending on n, Ω and α, so that Z dx ≤ C. sup α x∈Ω Ω |x − y| Proof. Choose R > 0 in such a way that Ω ⊂ B(y,R) for any y ∈ Ω. Then we get by passing to spherical coordinates

Z dx Z dx Z R ω Rn−α ≤ = rn−α−1dr = n α α ωn Ω |x − y| B(y,R) |x − y| 0 n − α and hence Z dx ω Rn−α ≤ n . sup α x∈Ω Ω |x − y| n − α The proof is then complete. ut

Theorem A.2. Any weakly singular integral operator on L2(Ω) is compact.

Proof. Let A a be weakly singular integral operator. Then there exists 0 < α < n and B ∈ L∞(Ω × Ω) so that, for any f ∈ L2(Ω), we have

Z B(x,y) (A f )(x) = f (y)dy x ∈ . α a.e. Ω Ω |x − y|

In a first step we prove that A is bounded. Pick f ∈ L2(Ω). Then according to Lemma A.3 we obtain Z Z |B(x,y)| Z | f (y)|dxdy ≤ CkBk | f (y)|dy ≤ C| |1/2kBk k f k , α ∞ Ω ∞ 2 Ω Ω |x − y| Ω where C is the constant in Lemma A.3. Therefore, with reference to Fubini’s theorem, we get that the integrals

Z B(x,y) Z |B(x,y)| Z |B(y,x)| f (y)dy, | f (y)|dy | f (y)|dy α α and α (A.11) Ω |x − y| Ω |x − y| Ω |x − y| A.2 Weakly singular integral operators 247 exist for a.e. x ∈ Ω. Also, as

Z Z | f (y)|2 Z  Z 1  Z dxdy = | f (y)|2 dx dy ≤ C | f (y)|2dy < , α α ∞ Ω Ω |x − y| Ω Ω |x − y| Ω the integral Z | f (y)|2 dy α (A.12) Ω |x − y| exists for a.e. x ∈ Ω. This follows again from Fubini’s theorem. Whence, for x ∈ Ω so that the integrals in (A.11) and (A.12) exist, we get by applying Cauchy- Schwarz’s inequality and Lemma A.3

Z B(x,y) 2 Z 1 | f (y)| f (y)dy ≤ C dy α / / (A.13) Ω |x − y| Ω |x − y|α 2 |x − y|α 2 Z | f (y)|2 ≤ C dy. α Ω |x − y| Here and until the end of this proof, C denotes a generic constant only depending on n, Ω, α and kBk∞. Inequality (A.13) being valid for a.e. x ∈ Ω, we can integrate over Ω with respect to x. We find Z Z | f (y)|2 kA f k2 ≤C dxdy 2 α Ω Ω |x − y| Z Z 1  ≤ C | f (y)|2 dx dy α Ω Ω |x − y| 2 ≤ Ck f k2 and hence A ∈ B(L2(Ω)). We now prove that A is compact. We split A into two integral operators A = Aε + Rε , ε > 0 is given, where the operators Aε and Rε have as respective kernels

Kε (x,y) = K(x,y)χ(|x − y|), Lε (x,y) = B(x,y)(1 − χε (|x − y|).

Here χε is the characteristic function of the interval [ε,+∞). 2 Since Kε ∈ L (Ω × Ω), Aε is compact (see Exercise 2.5). On the other hand, similarly to the proof of Lemma A.3, we have Z 1 dy ≤ C n−α . sup α ε x∈Ω B(x,ε) |x − y| In light of this estimate, we can carry out the same calculation as for A in order to get, for a.e. x ∈ Ω, 248 A Building a fundamental solution by the parametrix method

Z B(x,y) Z 1 Z | f (y)|2 |(R f )(x)| ≤ | f (y)|dy ≤ C dy dy ε α α α Ω∩B(x,ε) |x − y| B(x,ε) |x − y| Ω |x − y| Z | f (y)|2 ≤ C n−α dy, ε α Ω |x − y| from which we deduce, as we have done for A,

n−α kRε f k2 ≤ Cε k f k2 and hence n−α kRε kB(L2(Ω)) ≤ Cε . Thus

kA − Aε kB(L2(Ω)) → 0 as ε → 0. In light of Theorem 2.3, the compactness of A follows then readily. ut

As a straightforward consequence of Theorem A.10 and Theorem 2.8 we have the following result. Theorem A.3. Let A be a weakly singular operator of the form (A.10). Then σ(A) = {0}, or else σ(A)\{0} is finite, or else σ(A)\{0} consists in a sequence converging to 0. Let A be a weakly singular operator with kernel K, λ 6= 0 and g ∈ L2(Ω). Con- sider then the Fredholm integral equation of the second kind Z K(x,y) f (y)dy − λ f (x) = g(x), a.e. x ∈ Ω. (A.14) Ω The result we state now is a direct consequence of Fredholm’s alternative. Theorem A.4. Let A be a weakly singular operator with kernel K, λ 6= 0 and g ∈ L2(Ω). Then the integral equation (A.14) has a unique solution f ∈ L2(Ω), or else the homogenous equation Z K(y,x)h(y)dy − λh(x) = 0 a.e. x ∈ Ω Ω has exactly p linearly independent solutions h1,...,hp. In that case, (A.14) is solv- able if and only if g satisfies the following orthogonality relations Z g(x)h j(x)dx = 0, j = 1,..., p. Ω Next, when in the kernel K of the weakly singular integral operator A is so that B ∈ C(Ω ×Ω \D)∩L∞(Ω ×Ω) then we are going to show that A acts as a compact operator on C(Ω). Note that, for each f ∈ C(Ω), we have

−α |K(x,y) f (y)| ≤ kBkL∞(Ω×Ω)k f kC(Ω)|x − y| , x,y ∈ Ω, x 6= y. A.2 Weakly singular integral operators 249

Therefore, as an improper integral, Z (A f )(x) = K(x,y) f (y)dy Ω exits for any x ∈ Ω. Theorem A.5. The weakly singular operator A : C(Ω) → C(Ω) is compact. Proof. Pick χ ∈ C∞([0,∞)) satisfying 0 ≤ χ ≤ 1, χ(t) = 0 for t ≤ 1/2 and χ(t) = 1 for t ≥ 1. Define then, for j ≥ 1,

 χ( j|x − y|)K(x,y) if x 6= y, K (x,y) = j 0 if x = y and denote by A j the integral operator with kernel Kj. It is clear that Kj is continuous and there exists a constant C > 0 so that, for each j, we have Z −α A f (x) − A j f (x) ≤ Ck f kC(Ω) |x − y| Ω∩B(x,1/ j) −α ≤ Ck f kC(Ω) j for each x ∈ Ω. Whence A f ∈ C(Ω) as the uniform limit of the sequence continuous functions (A j f ). Moreover

−α kA − A jkB(C(Ω)) ≤ C j . The proof will be completed by showing that an integral operator with continuous kernel is compact. We then consider A as an integral operator with kernel K ∈C(Ω × Ω). Let ε > 0. Since K is uniformly continuous, there exists η > 0 so that, for any x,y,z ∈ Ω satisfying |x − z| ≤ η, we have

|K(x,y) − K(z,y)| ≤ ε/|Ω|.

Thus, for an arbitrary f ∈ C(Ω) with k f kC(Ω) ≤ 1,

|A f (x) − A f (z)| ≤ ε, provided that |x − z| ≤ η. In other words, F = {A f ; f ∈ C(Ω), k f kC(Ω) ≤ 1} is relatively compact by Arzela-Ascoli’s theorem and the result follows. ut Considering C Ω,C Ω as dual system with respect to the usual scalar prod- uct of L2(Ω), we get that Theorem A.4 is also valid for weakly singular operators acting on C Ω. We refer to [2, Chapter 4] for a general Fredholm’s alternative for dual systems.

Proposition A.1. For i = 0,1, let Ai be a weakly singular integral operator with kernel Ki satisfying 250 A Building a fundamental solution by the parametrix method

Bi(x,y) Ki(x,y) = , x,y ∈ Ω x 6= y, |x − y|αi

∞ with Bi ∈ L (Ω ×Ω). Then A = A0A1 is likewise a weakly singular integral operator with kernel Z B (x,z)B (z,y) K(x,y) = 0 1 dz. α α Ω |x − z| 0 |z − y| 1

Furthermore, there exists a constant C > 0 only depending on n, Ω, α0, α1, kB0k∞ and kB1k∞ so that, for any x,y ∈ Ω, we have  1 if α0 + α1 > n,  −n+α0+α1  |x − y|  |K(x,y)| ≤ C (A.15) |ln|x − y|| + 1 if α0 + α1 = n,    1 if α0 + α1 < n.

Proof. Let f ∈ L2(Ω). According to (A.1), for a.e. x ∈ Ω, the integral

Z Z |B (x,z)B (z,y)| Z Z 1 | f (y)| 0 1 dzdy ≤ C | f (y)| dzdy α α α α Ω Ω |x − z| 0 |z − y| 1 Ω Ω |x − z| 0 |z − y| 1 exists. Hence, for a.e. x ∈ Ω, B (x,z)B (z,y) f (y) 0 1 |x − z|α0 |z − y|α1 is integrable in Ω × Ω with respect to (y,z). For such a point x ∈ Ω, it follows from Fubini’s theorem Z B (x,z) Z B (z,y) (A A f )(x) = 0 1 f (y)dydz 0 1 α α Ω |x − z| 0 Ω |z − y| 1 Z Z B (x,z)B (z,y) = f (y) 0 1 dzdy. α α Ω Ω |x − z| 0 |z − y| 1

We complete the proof by using Lemma A.1 and noting that, when α0 + α1 = n, −ε |K(x,y)| ≤ Cε |x − y| for any 0 < ε < n. ut

∞ Let K be as in the preceding proof and assume that Bi ∈ C(Ω ×Ω \D)∩L (Ω × Ω), i = 0,1, then by Theorem A.1, K ∈ C(Ω × Ω \ D). This and estimate (A.15) show that K is the kernel of a weakly integral operator acting on C(Ω). A.3 Canonical parametrix 251 A.3 Canonical parametrix

n Let Ω be a bounded domain of R of class C2. In that case, for any j + β < k + α ≤ 2 with 0 ≤ α,β ≤ 1 and positive integers j and k, we know that C j,β (Ω) is continuously imbedded in Ck,α (Ω) (see [5, Lemma 6.35, page 135]). i j n2 Consider A = (a ) ∈ C(Ω,R ) satisfying the ellipticity condition

−1 2 2 n µ |ξ| ≤ (A(x)ξ|ξ) ≤ µ|ξ| , for all x ∈ Ω and ξ ∈ R , (A.16)

n where µ ≥ 1 is a constant and (·|·) denotes the Euclidian scalar product of R . Let bi, 1 ≤ i ≤ n, and c belong to C(Ω), and consider the non-divergence form operator defined by

(Lu)(x) = (A(x)∇u(x)|∇u(x)) + (B(x)|∇u(x)) + c(x), u ∈ C2(Ω).

Here B = (b1,...,bn). Define d(x) = pdet(A). Then (A.16) yields in straightforward manner that

µ−n/2 ≤ d(x) ≤ µn/2 for all x ∈ Ω. (A.17)

n For x ∈ R and y ∈ Ω, put

1/2 ρ(x,y) = A−1(y)(x − y)|x − y , which in light of (A.16) satisfies

µ−1|x − y| ≤ ρ(x,y) ≤ µ|x − y|. (A.18)

−1 If A = (ai j) then clearly

1 n ∂xi ρ(x,y) = ∑ ai j(y)(x j − y j), x 6= y. (A.19) ρ(x,y) j=1

Consider the function defined, for t > 0, by  − lnt if n = 2,  2π Fn(t) = 2−n  t if n ≥ 3.  (n−2)ωn

n−1 Here ωn = |S |. n Define the function H, for x ∈ R and y ∈ Ω with x 6= y, by F (ρ(x,y)) H(x,y) = n . d(y)

In light of (A.17) and (A.18), we have 252 A Building a fundamental solution by the parametrix method

1 µn/2 |x − y|2−n ≤ H(x,y) ≤ |x − y|2−n, x 6= y, (A.20) n/2 (n − 2)ωnµ (n − 2)ωn if n ≥ 3 and 1 µ − ln(µ|x − y|) ≤ H(x,y) ≤ − lnµ−1|x − y|, x 6= y, (A.21) 2πµ 2π if n = 2 and 0 < µ|x − y| < 1. Using (A.19), we get

1 n ∂xi H(x,y) = − n ∑ ai j(y)(x j − y j), x 6= y, (A.22) ωnd(y)ρ (x,y) j=1 from which we deduce 1 2 H(x,y) = − a (y) (A.23) ∂xix j n i j ωnd(y)ρ (x,y) n n n + 2+n ∑ aik(y)(xk − yk) ∑ a j`(y)(x` − y`). ωnd(y)ρ (x,y) k=1 `=1 A straightforward computation yields

n n n i j 2 ∑ a (y) ∑ aik(y)(xk − yk) ∑ a j`(y)(x` − y`) = ρ (x,y). i, j=1 k=1 `=1

It follows from this identity that H(·,y) is the solution of the equation

n L0H(x,y) = ai j(y) 2 H(x,y) = 0. (A.24) x ∑ ∂xix j i, j=1

Define also n L˜ 0H(x,y) = ai j(x) 2 H(x,y). x ∑ ∂xix j i, j=1 Henceforth, Σ is as in Section A.1.

Lemma A.4. (i) There exists a constant C0, only depending on µ and n, so that

−n+1 |∂xi H(x,y)| ≤ C0|x − y| , 1 ≤ i ≤ n, x 6= y. (A.25)

(ii) Assume in addition that ai j ∈ C0,α (Ω), 1 ≤ i, j ≤ n and

i j max [a ]α ≤ Λ, 1≤i, j≤n for some constants 0 < α ≤ 1 and Λ > 0. Then there exists a constant C1, only depending on n, µ and Λ, so that A.3 Canonical parametrix 253

˜ 0 −n+α LxH(x,y) ≤ C1|x − y| , (x,y) ∈ Σ. (A.26)

Proof. (i) is immediate from (A.16), (A.18) and (A.22). To prove (ii), we note that from (A.24) we have

˜ 0 ˜ 0 0 LxH(x,y) = LxH(x,y) − LxH(x,y) n = ai j(x) − ai j(y) 2 H(x,y). ∑ ∂xix j i, j=1

Hence n L˜ 0H(x,y) ≤ Λ|x − y|α ∂ 2 H(x,y) . x ∑ xix j i, j=1 This together with (A.23) entail the expected inequality. ut We have as an immediate consequence of this lemma: Corollary A.1. Assume in addition that ai j ∈ C0,α (Ω), 1 ≤ i, j ≤ n and

i j i max [a ]α + max |b (x)| + |c(x)| ≤ Λ, x ∈ Ω, 1≤i, j≤n 1≤i≤n for some constant 0 < α ≤ 1 and Λ > 0. Then there exists a constant C, only de- pending on n, diam(Ω), µ and Λ, so that

−n+α |LxH(x,y)| ≤ C|x − y| , (x,y) ∈ Σ. (A.27)

Lemma A.5. Let 0 < α ≤ 1, Λ > 0 and assume that ai j ∈ C1(Ω)(⊂ C0,α (Ω)), 1 ≤ i, j ≤ n and i j max ka kC1( ) ≤ Λ. 1≤i, j≤n Ω Then there exists a constant C > 0, only depending on n, diam(Ω), α, Λ and Ω, so that

−n+α+1 ∂xk H(x,y) + ∂yk H(y,x) ≤ C|x − y| , (x,y) ∈ Σ, (A.28)

∂ 2 H(x,y) + ∂ 2 H(y,x) ≤ C|x − y|−n+α , (x,y) ∈ Σ. (A.29) x jxk x jyk

Proof. We have by (A.22)

∂xk H(x,y) + ∂yk H(y,x) 1 n 1 n = − n ∑ ak`(y)(x` − y`) + n ∑ ak`(x)(y` − x`) ωnd(y)ρ (x,y) `=1 ωnd(x)ρ (y,x) `=1 n   1 ak`(x) ak`(y) = n ∑ − (x` − y`) ωnρ (x,y) `=1 d(x) d(y) 1 n  1 1  + ∑ ak`(x)(y` − x`) n − n . ωnd(x) `=1 ρ (y,x) ρ (x,y) 254 A Building a fundamental solution by the parametrix method

0,α Noting that ak`, 1 ≤ k,` ≤ n, belong also to C (Ω), the first term in the right hand side of the last identity is clearly estimated by C|x − y|−n+α+1. To complete the proof of (A.28), we establish the estimate

1 1 −n+ − ≤ C|x − y| α , x 6= y. ρn(y,x) ρn(x,y)

Invoking the mean-value theorem, we find θ ∈ (0,1) so that n ρn(x,y) − ρn(y,x) = A−1(y) − A−1(x)(x − y)|x − y 2 n/2−1 × θ A−1(y)(x − y)|x − y + (1 − θ)A−1(x)(x − y)|x − y .

Inequality (A.28) then follows. We have from the preceding calculations

2 H(x,y) + 2 H(y,x) = I + I + I , ∂x jxk ∂x jyk 1 2 3 with   1 ak j(x) ak j(y) I1 = n − ωnρ (x,y) d(x) d(y)   ak j(x) 1 1 − n − n ωnd(x) ρ (y,x) ρ (x,y) n   1 ak`(x) + n ∑ ∂x j (x` − y`), ωnρ (y,x) `=1 d(x) n   n n ak`(x) ak`(y) I2 = − n+2 ∑ − (x` − y`) ∑ a ji(y)(xi − yi), ωnρ (x,y) `=1 d(x) d(y) i=1 1 n  1 1  I3 = ∑ ak`(x)(y` − x`)∂x j n − n ωnd(x) `=1 ρ (y,x) ρ (x,y)

−n+α It is straightforward to check that |I1 + I2| ≤ C|x − y| . To estimate I3, we first compute the term  1 1  ∂x − . j ρn(y,x) ρn(x,y) We have  1 1  ∂x − j ρn(y,x) ρn(x,y) n   a j`(y) a j`(x) = n ∑ n+2 − n+2 (x` − y`) `=1 ρ (x,y) ρ (y,x) n −1  − ∂x A (x)(x − y)|x − y . 2ρn+2(y,x) j A.3 Canonical parametrix 255

Splitting the first term on the right hand side into two ones, we can mimic the proof of (A.28) in order to estimate this term by C|x−y|−n−1+α . While the second term in −n the last inequality is clearly estimated by C|x − y| . Returning back to I3, we find that it is estimated by C|x − y|−n+α . The proof is then complete. ut Lemma A.6. Let 0 < α ≤ 1 and Λ > 0. Assume that ai j ∈ C0,1(Ω), bi, c ∈ C0,α (Ω) and i j i max ka kC0,1( ) + max kb kC0,α ( ) + kckC0,α ( ) ≤ Λ. 1≤i, j≤n Ω 1≤i≤n Ω Ω Then there exists a constant C > 0, depending on n, diam(Ω), µ α and λ, so that

α −n −n |LxH(x,z) − LxH(y,z)| ≤ |x − y| |x − z| + |y − z| , (A.30) for all x,y,z ∈ Ω satisfying |x − z| ≥ 2|x − y|. Proof. Recall that 1/2 ρ(x,z) = A−1(z)(x − z)|x − z and 1 n ∂xi ρ(x,z) = ∑ ai j(z)(x j − z j). ρ(x,z) j=1 Fix x,y ∈ Ω and s > 0. Define then

θ(t) = ρ(x +t(y − x),z)−s, t ∈ [0,1].

Since there exits τ ∈ (0,1) so that θ(1)−θ(0) = θ 0(τ), we get ,where w = x+τ(y− x), s ρ(y,z)−s − ρ(x,z)−s = − A−1(z)(w − z)|y − x. ρs+2(w,z) Assume that |x−z| ≥ 2|x−y|. Then |w−z| ≤ |x−y| < |x−z|/2 and hence |w−z| ≥ |z − x|/2. Therefore

−s −s −(s+1) ρ(y,z) − ρ(x,z) ≤ C|x − y||x − z| . (A.31)

Note that (A.31) with s = n − 2, n ≥ 3, and (A.20) yield

|c(y)H(y,z) − c(x)H(x,z)| ≤ C |x − y|α |x − z|−n+2 +C|x − y||x − z|−n+1 and hence |c(y)H(y,z) − c(x)H(x,z)| ≤ C|x − y|α |x − z|−n. (A.32) We have the same inequality for n = 2. From (A.22), we have

1 n ∂xi H(x,z) = − n ∑ ai j(z)(x j − z j). ωnd(z)ρ (x,z) j=1

Whence 256 A Building a fundamental solution by the parametrix method

n 1 −n −n  ∂xi H(y,z) − ∂xi H(x,z) = ρ (x,z) − ρ (y,z) ∑ ai j(z)(y j − z j) ωnd(z) j=1 1 n + n ∑ ai j(z)(y j − x j). ωnd(z)ρ (x,z) j=1

As |y − z| ≤ 3|x − z|/2, we obtain, by applying (A.25) and (A.31) with s = n,

−n |∂xi H(y,z) − ∂xi H(x,z)| ≤ C|x − y||x − z| . (A.33)

It follows from (A.25) and (A.33)

i i −n α −n+1 b (y)∂xi H(y,z) − b (x)∂xi H(x,z) ≤ C |x − y||x − z| + |x − y| |x − z| and consequently

i i α −n b (y)∂xi H(y,z) − b (x)∂xi H(x,z) ≤ C|x − y| |x − z| . (A.34)

Next, using n ai j(z) 2 H(x,z) = 0, ∑ ∂xix j i, j=1 one obtains n n ai j(x) 2 H(x,z) − ai j(y) 2 H(y,z) = ∑ ∂xix j ∑ ∂xix j i, j=1 i, j=1 n ai j(x) − ai j(y) 2 H(x,z) ∑ ∂xix j i, j=1 n   + ai j(y) − ai j(z) 2 H(x,z) − 2 H(y,z) . ∑ ∂xix j ∂xix j i, j=1

In light of (A.23), we can proceed as in the preceding lemma. With the aid of (A.31) and the identity

(xk − zk)(x` − z`) − (yk − zk)(y` − z`) = (xk − yk)(x` − z`) − (yk − zk)(x` − y`), we get

n n ai j(x)∂ 2 H(x,z) − ai j(y)∂ 2 H(y,z) ≤ C|x − y||x − z|−n. (A.35) ∑ xix j ∑ xix j i, j=1 i, j=1

The expected inequality is obtained by putting together (A.32), (A.34) and (A.35). ut A.3 Canonical parametrix 257

Let us now give the precise definition of a parametrix and a fundamental solution. To this end, we assume from now on that, for some fixed 0 < α ≤ 1 and Λ > 0, the following assumptions fulfill.

ai j ∈ C1(Ω), (A.36) bi,c ∈ C0,α (Ω), (A.37) n j jk j 1 B = ∑ ∂xk a − b ∈ C (Ω), 1 ≤ j ≤ n, (A.38) k=1 and i j i max ka kC1( ) + max kb kC0,α ( ) + kckC0,α ( ) ≤ Λ. (A.39) 1≤i, j≤n Ω 1≤i≤n Ω Ω . Assume moreover that the ellipticity condition (A.16) holds. Under all these assumptions, we can compute the adjoint of L. We find

∗  jk  2 L v = ∑ ∂x j a (x)∂xk v + [div(B) + c]v, v ∈ C (Ω). j,k=1

Here B = (B1,...,Bn).

Definition A.1. A function P : Ω × Ω \ D → R with P(·,y) ∈ C2(Ω \{y}) is called a parametrix or a Levi function for L relative to Ω if, for any y ∈ Ω, P(·,y) ∈ L1(Ω), 1 LxP(·,y) ∈ L (Ω) and, for any ϕ ∈ D(Ω), we have Z ∗ [−P(x,y)L ϕ(x) + LxP(x,y)ϕ(x)]dx = ϕ(y), y ∈ Ω. Ω A parametrix P satisfying, for any y ∈ Ω,

LxP(x,y) = 0, x ∈ Ω \{y}, is called a fundamental solution for L relative to Ω.

Proposition A.2. H(x,y) is a parametrix for L relative to Ω.

Proof. From the definition of H we easily see that H(·,y) ∈ C2(Ω \{y}), for any 1 1 y ∈ Ω. The fact that H(·,y) ∈ L (Ω) (resp. LxP(·,y) ∈ L (Ω)), for any y ∈ Ω, is immediate from (A.20) (resp. Corollary A.1) and Lemma A.3. Fix y ∈ Ω and ε > 0 sufficiently small so that B(y,ε) b Ω. Let ϕ ∈ D(Ω). Starting from the identity, where u,v are arbitrary in C2(Ω \ B(y,ε)),

n ∗ h jk  i uL v − Luv = ∑ ∂x j a u∂xk v − ∂xk uv + cuv , j,k=1 we get by applying Gauss’s theorem 258 A Building a fundamental solution by the parametrix method Z ∗ [−H(x,y)L ϕ(x) + LxH(x,y)ϕ(x)]dx Ω\B(y,ε) Z n h jk  i = ∑ a H(x,y)∂xk ϕ − ∂xk H(x,y)ϕ + cH(x,y)ϕ ν jdσ(x) ∂B(y,ε) j,k=1 Z n jk = − ∑ a ∂xk H(x,y)ϕ(x)ν j(x)dσ(x) + o(1) ∂B(y,ε) j,k=1 Z n jk = − ∑ a ∂xk H(x,y)ϕ(y)ν j(x)dσ(x) + o(1) ∂B(y,ε) j,k=1

We will show in the next section that Z n jk ∑ a ∂xk H(x,y)ν j(x)dσ(x) = 1, ∂B(y,ε) j,k=1 which yields in a straightforward manner the expected identity. ut

The parametrix H(x,y) constructed in this section is usually called the canonical parametrix.

A.4 Fundamental solution

n In this section, Ω is a bounded domain of R of class C2 and assume that assump- tions (A.36) to (A.39) together with (A.16) hold. Denote by H(x,y) the canonical parametrix constructed in the previous section and let K = LxH(x,y). Consider then A the weakly singular integral operator with the kernel K(x,y) acting on C(Ω). Introduce N = {φ ∈ N(I − A∗); supp(φ) ⊂ Ω}.

Let P be the orthogonal projection on N and L = L|D(Ω). Lemma A.7. We have that N = R(PL ). Proof. Write N = R(PL ) ⊕ R(PL )⊥. For all ϕ ∈ D(Ω) and ψ ∈ R(PL )⊥, we have Z 0 = (ψ|PL ϕ) = (Pψ|L ϕ) = (ψ|L ϕ) = ψLϕdx. Ω In other words, ψ is a weak solution of L∗ψ = 0. Since ψ has a compact support, according to Theorem 4.5, ψ is identically equal to zero. Therefore R(PL )⊥ = {0} and hence the result follows. ut A.4 Fundamental solution 259

By Lemma A.7, if (φ1,...φp) is a fixed basis of N then φ j = PL ϕ j = Lϕ j, ϕ j ∈ D(Ω), 1 ≤ j ≤ p. Set p R0(x,y) = − ∑ ϕ j(x)φ j(y) j=1 and p `0(x,y) = − ∑ (Lϕ j)(x)φ j(y). j=1 Then Z p Z `0(x,y)φk(x)dx = − ∑ φ j(y) (Lϕ j)(x)φk(x)dx = −φk(y), Ω j=1 Ω where we used that Lϕ j = φ j. In other words, the following orthogonality relation holds Z φ(y) + `0(x,y)φk(x)dx = 0, φ ∈ N , y ∈ Ω. (A.40) Ω

Fix Ω0 b Ω, set  p  C = ∪ j=1supp(φ j) ∪ Ω0.

⊥ and let ψ1,...ψm be an orthonormal basis of N .

Lemma A.8. There exist x1,...xm ∈ Ω \ C so that

det(ψ`(xk)) 6= 0. (A.41)

Proof. We use an induction in m. Note first that when m = 1, if ψ1(x) = 0 for any x ∈ Ω \ C , then ψ1 ∈ N which is impossible. Let (A.41) holds for m and we going to check that it holds also for m + 1. We argue by contradiction by assuming that   ψ1(x1) ... ψm(x1) ψm+1(x1)  . . . .   . . . .  det  = 0, x ∈ Ω \ C .  ψ1(xm) ... ψm(xm) ψm+1(xm)  ψ1(x) ... ψm(x) ψm+1(x)

m+1 m+1 Whence there exists d ∈ R , d 6= 0, so that ψ = ∑`=1 d`ψ` vanishes in Ω \ C . In particular, ψ ∈ N which is impossible since ψ is non identically equal to zero. ut

We fix in the sequel x1,...xm ∈ Ω \C satisfying (A.41). Therefore, for any y ∈ Ω, n there exists a unique ( f1(y),... fm(y)) ∈ R satisfying m Z ∑ ψk(x`) fk(y) = ψk(y) = ψk(y) + `0(y,x)ψk(x)dx, 1 ≤ k ≤ m. (A.42) `=1 Ω 260 A Building a fundamental solution by the parametrix method

Note that the second equality follows from

p `0(x,y) = − ∑ φ j(x)φ j(y). j=1

By Cramer’s method for solving linear systems, we can easily check that fk ∈ C(Ω). ⊥ ∗ In light of (A.40), (A.42), using that N ⊕ N = N(I − A ) and φ(x`) = 0 for φ ∈ N , ` = 1,...,m, we get

Z m ∗ φ(y) + `0(x,y)φk(x)dx = ∑ ψk(x`) fk(y), φ ∈ N(I − A ), y ∈ Ω. (A.43) Ω `=1

But Z ψk(x`) = K(x,x`)ψk(x)dx. Ω Thus, if m `(x,y) = `0(x,y) − ∑ K(x,x`) fk(y), `=1 then (A.43) is equivalent to the following orthogonality relation Z φ(y) + `(x,y)φ(x)dx = 0, φ ∈ N(I − A∗), y ∈ Ω. (A.44) Ω Define m R(x,y) = R0(x,y) − ∑ H(y,x`) fk(y). `=1 Clearly LxR(x,y) = `(x,y). (A.45)

By induction in j ≥ 1, define K1(x,y) = K(x,y) and Z Kj+1(x,y) = K(x,z)Kj(z,y)dz. Ω

j Then it is straightforward to check that Kj is the kernel of A . Moreover, we get by applying Theorem A.1 that Kj ∈ C(Ω × Ω) provided that j is sufficiently large. We fix then j so that Kj ∈ C(Ω × Ω) and we set Z f (x,y) = Kj(x,z)[K(z,y) + `(z,y)]dz. Ω In light of the properties of the canonical parametrix, we deduce, once again from Theorem A.1, that f ∈ C(Ω × Ω). Furthermore, for φ ∈ N(I − A∗), and bearing in mind that φ = (A∗) jφ, we have A.4 Fundamental solution 261 Z Z f (x,y)φ(x)dx = (A∗) j(z)[K(z,y) + `(z,y)]dz Ω Ω Z = φ(z)[K(z,y) + `(z,y)]dz. Ω Z = φ(y) + φ(z)`(z,y)dz. Ω This and (A.44) entail Z f (x,y)φ(x)dx = 0, φ ∈ N(I − A∗), y ∈ Ω. Ω This orthogonality relation at hand, we can apply Fredholm’s alternative to deduce that the integral equation Z g(x,y) − K(x,z)g(z,y)dz = f (x,y) (A.46) Ω has a unique solution g(·,y) ∈ C(Ω) orthogonal to N(I − A). Lemma A.9. We have g ∈ C(Ω × Ω). Proof. We claim that there exists C > 0 so that, for any y,z ∈ Ω, we have

max|g(x,y) − g(x,z)| ≤ C max| f (x,y) − f (x,z)| x∈Ω x∈Ω from which the result follows. We proceed by contradiction. So if our claim does not hold we would find two sequences (y j) and (z j) in Ω so that

τ j = max|g(x,y j) − g(x,z j)| > j max| f (x,y j) − f (x,z j)| = jt j, j ≥ 1. x∈Ω x∈Ω

In particular, the sequence of functions ( f j) given by

f (x,y j) − f (x,z j) f j(x) = , x ∈ Ω, τ j converges uniformly in Ω to 0. Define then two sequences of functions (u j) and (v j) by

g(x,y j) − g(x,z j) u j(x) = , x ∈ Ω, τ j Z v j(x) = K(x,z)u j(z)dz = (Au j)(x), x ∈ Ω. Ω

As ku jkC(Ω) = 1 and A is compact, subtracting a subsequence if necessary, we may assume that v j = Au j converges to v in C(Ω). But

u j − v j = f j 262 A Building a fundamental solution by the parametrix method by (A.46). Whence, u j converges also to u = v in C(Ω) and hence u = Au or equiv- ⊥ alently u ∈ N(I − A). On the other hand, we know that u j ∈ N(I − A) . Therefore Z 0 = u j(x)u(x)dx, j ≥ 1, Ω that yields Z 0 = lim u j(x)u(x)dx = kukL2(Ω). j→∞ Ω

We end up getting the expected contradiction by noting that 1 = ku jkC(Ω) → kukC(Ω) = 1. ut Define

j−1 Z G(x,y) = g(x,y) + K(x,y) + `(x,y) + ∑ Ks(x,z)[K(z,y) + `(z,y)]dz. s=1 Ω   We note that the term ` belongs to C Ω \{x1,...,xm} × Ω while the other terms are in C Ω × Ω \ D. Moreover, we can check that G is a solution of integral equation Z G(x,y) − K(x,z)G(z,y)dz = K(x,y) + `(x,y). (A.47) Ω

Fix y ∈ Ω and let x0 = y. Let η sufficiently small in such a way that B(xi,η) ∩ B(xk,η) = /0,for 0 ≤ i, j ≤ m, i 6= j and B(xi,η) ⊂ Ω \ Ω0, j = 1,...m. We decom- pose G as follows m  0 1 G = ∑ Gk + Gk , i=0 with

0 G0(x,y) = g(x,y) + K(x,y) + `0(x,y), j−1 Z 1 G0(x,y) = ∑ Ks(x,z)[K(z,y) + `0(z,y)]dz, s=1 Ω 0 Gi (x,y) = −K(x,xi) fi(y), j = 1,...,m, j−1 Z 1 Gi (x,y) = − ∑ Ks(x,z)K(z,xi) fi(y)dz, j = 1,...,m. s=1 Ω Define j Λη = Ω \ B(xi,η), j = 0,...,m. 0,α 0 Noting that the coefficients of L are in C (Ω), it not hard to check that Gi (·,y) 0,α i belongs to C (Λη ), i = 0,...m. On the other hand, in view of Lemma A.6 and the estimate −n+α |K(z,xi)| ≤ C|x − xi| , i = 0,...,m, A.4 Fundamental solution 263

1 0,β we deduce by applying (iii) of Theorem A.1 that Gi (·,y) belongs to C (Λη ), for 0 1 0,β any β ∈ (0,α), and consequently Gi(·,y) = Gi (·,y) + Gi (·,y) ∈ C (Λη ) for i = 0,...m. Consider the function Z Γk(x,y) = H(x,z)Gk(z,y)dz, k = 0,...m. Ω

According to Theorem A.1 (i), Γk belongs to C(Ω0 × Ω0 \ D) and, by Lemma A.4 1 and Theorem A.1 (ii), Γi(·,y) ∈ C (Ω0 \{y}) with Z ∇xΓ0(x,y) = ∇xH(x,z)Gk(z,y)dz. Ω We need to take partial derivatives of both sides of this identity. For doing that, we first rewrite this identity in a different form. Fix y ∈ Ω and set Ωη = Ω \B(y,η). Note that if x 6= y and η is sufficiently small x 6∈ B(y,η). We write Z ∂x j Γk(x,y) = [∂x j H(x,z) + ∂z j H(z,x)]Gk(z,y)dz Ωη Z Z − ∂z j H(z,x)Gk(z,y)dz + ∂x j H(x,z)Gk(z,y)dz. Ωη B(xk,η) We have in mind to apply the following Michlin’s theorem

n Theorem A.6. Let Ω be a bounded open subset of R and w ∈ C(Σ) of the form  x − y  w(x,y) = |x − y|−n+1Ψ x, , (x,y) ∈ Σ, |x − y|

n− , with Ψ ∈ C1(Ω × S 1). Let β ∈ (0,1) and u ∈ C0 β (Ω). Then Z v(x) = w(x,y)u(y)dy Ω belongs to C1(Ω) and we have, for x ∈ Ω, Z Z ∇v(x) = lim ∇xw(x,y)u(y)dy + u(x) ξΨ(x,ξ)dσ(ξ). ε→0 Ω\B(x,ε) Sn−1 Proceeding as for the first order derivative, we get by substituting H(x,y) by

∂x j H(x,z) + ∂z j H(z,x), Z Z [ H(x,z)+ H(z,x)]G (z,y)dz = [ 2 H(x,z)+ H(z,x)]G (z,y)dz. ∂xi ∂x j ∂z j k ∂xix j ∂xiz j k Ω Ω Further, as 1 n ∂z j H(z,x) = − n ∑ a jk(x)(zk − xk) ωnd(x)ρ (z,x) k=1 264 A Building a fundamental solution by the parametrix method and 1/2 ρ(z,x) = A−1(z − x)|z − x , we have  z − x  ∂ H(z,x) = |x − y|−n+1Ψ x, , z j j |z − x| with n a jk(x) −1 −n/2 Ψj(x,ξ) = − ∑ ξk A (x)ξ|ξ . k=1 ωnd(x) 0,β k k Since Gk(·,y) ∈ C (Λη ) we can apply Theorem A.6 with Ω substituted by Λη . We get Z Z 2 ∂x ∂z H(z,x)Gk(z,y)dz = lim ∂ H(z,x)g(z,y)dz i k j k xiz j Λη ε→0 Λη \B(x,ε) Z + Gk(x,y) ξiΨj(x,ξ)dσ(ξ). Sn−1 We have n n i j i j a jk(x) −1 −n/2 ∑ a (x)ξiΨj(x,ξ) = −∑a (x) ∑ ξiξk A (x)ξ|ξ i, j=1 i, j k=1 ωnd(x) 1 −n/2 = − A−1(x)ξ|ξ . ωnd(x)

It can be proved1 that

1 Z −n/2 A−1(x)ξ|ξ dσ(ξ) = 1. ωnd(x) Sn−1 Thus n Z ∑ ai j(x)∂xi ∂z j H(z,x)Gk(z,y)dz i, j=1 Ωη Z n = a (x) 2 H(z,x)G (z,y)dz − G (x,y). ∑ i j ∂xiz j k k Ωη i, j=1

Finally, we have Z Z H(z,x)G (z,y)dz = 2 H(z,x)G (z,y)dz. ∂xi ∂z j k ∂xiz j k B(xk,η) B(xk,η) Assembling all these calculations, we end up getting Z LxΓk(x,y) = LxH(x,z)Gk(z,y)dz − Gk(x,y), x ∈ Ω0 \{y}. (A.48) Ω 1 See [1, Appendix 2, page 289]. A.4 Fundamental solution 265

Define Z Γ (x,y) = H(x,z)G(z,y)dz. Ω 2 Then Γ ∈ C (Ω0 × Ω0 \ D) and (A.48) yields Z LxΓ (x,y) = LxH(x,z)G(z,y)dz − G(x,y), x ∈ Ω0 \{y}. (A.49) Ω Consider the function Z F(x,y) = H(x,y) + H(x,y)G(x,y) + R(x,y). Ω

Using (A.23), (A.49) and the fact that LxR(x,y) = `(x,y), we find, for any y ∈ Ω0,

LxF(x,y) = 0, x ∈ Ω0 \{y}.

In light of Proposition A.2, the definition of R and the properties of H and G col- lected above, we can state the following ultimate result

Theorem A.7. F is a fundamental solution of L relative to Ω0 satisfying: for any β ∈ (0,α) if n = 2 and β = α if n ≥ 3, we find a constant C > 0, only depending on n, Ω, α and Λ, so that, for any x,y ∈ Ω 0 with x 6= y, we have

|F(x,y) − H(x,y)| ≤ C|x − y|−n+2+β , −n+1+β |∇xF(x,y) − ∇xH(x,y)| ≤ C|x − y| . 266 A Building a fundamental solution by the parametrix method References

1. Kalf H. : On E. E. Levi’s method of constructing a fundamental solution for second-order elliptic equations. Rend. Circ. Mat. Palermo 41 (2) (1992), 251-294. 2. Kress, R. : Linear integral equations. Third edition. Springer, New York, 2014. 3. Levi E.E. : I problemi dei valori al contorno per le equazioni lineari totalmente ellittiche alle derivate parziali. Memorie Mat. Fis. Soc. Ital. Scienze (detta dei XL) 16 (3) (1909), 3-113. Corrections in: , Opere, Vol. II, 207-343, Edizioni Cremonese, Roma, 1960. 4. Triebel, H. : Higher analysis. Johann Ambrosius Barth Verlag GmbH, Leipzig, 1992. Index

Ascoli-Arzela’s theorem, 37 Interpolation inequality, 9

Banach’s fixed point theorem, 47 Jensen’s inequality, 94 Banach’s theorem, 52 John-Nirenberg’s theorem, 94

Caccioppoli’s inequality, 163 Lebesgue’s dominated convergence theorem, 7 Caffarelli-Kohn-Nirenberg’s theorem, 40 Leray-Schauder’s fixed point theorem, 103 Calderon’s´ theorem, 183 Cancellation theorem, 13 Mean-value identities, 153 Carleman inequality, 74, 157, 160 Michlin’s theorem, 263 Change of variable formula, 21 Min-max principle, 59 Closing lemma, 15 Morrey inequality, 33 Commutator, 155 Moser’s iterative scheme, 93 Comparison principle, 117 Partition of unity, 14 Density theorem, 29 Perron’s method, 134 Divergence theorem, 30 Poincare’s´ inequality, 37 Doubling inequality, 149 Poincare-Wirtinger´ inequality, 43 Du Bois-Reymond lemma, 16 Projection theorem, 45

Extension by reflexion, 23 Rayleigh quotient, 58 Extension theorem, 28 Rellich-Kondrachov imbedding theorem, 36 Exterior sphere property, 120 Riesz’s lemma, 51 Riesz’s representation theorem, 18 Fredholm’s alternative, 50 Riesz-Frechet’s´ representation theorem, 46 Frequency function, 150 Riesz-Frechet-Kolmogorov’s´ theorem, 9 Friedrichs’s theorem, 41 Fubini’s theorem, 8 Sequence of mollifiers, 10 Sobolev imbedding theorem, 34 Generalized Holder’s¨ inequality, 8 Sobolev inequality, 31 Generalized Poincare-Wirtinger’s´ inequality, Stampacchia’s truncation method, 72 167 Strong maximum principle, 97, 140, 154 Strong unique continuation property, 152 Holder¨ regularity of weak solutions, 101 Sub-harmonic, 121 Holder’s¨ inequality, 8 Sub-solution, 82 Hahn-Banach’s extension theorem, 19 sub-solution, 117 Harmonic, 121 Super-harmonic, 121

267 268 Index

Super-solution, 82 Truncation sequence, 20 super-solution, 117 Support of measurable function, 39 Uniform cone property, 169 Three-ball inequality, 147, 164 Three-ball inequality for the gradient, 168 Weak form of Kato’s inequality, 102 Tonelli’s theorem, 39 Weak maximum principle, 116 Topological supplement, 50 Weak solution, 82 Trace inequality, 25 Trace theorem, 29 Young’s inequality, 191